Modren's ABC Chemistry Part - 1 Class 11
Modren's ABC Chemistry Part - 1 Class 11
33.93 mm JD-1
abc
.
ed
CHEMISTRY
The book in your hands is strictly based upon the syllabi prescribed by C.B.S.E.,
OF
New Delhi and Educational Boards of other Indian states. It has been written
rv
according to N.C.E.R.T. pattern keeping in view the changing trends of different
examinations. Thus this book has been very popular among the teachers and
se
students all over India and praised for its clear presentation, effective approach of
solving numerical problems and attractive figures.
re
CHEMISTRY
KEY FEATURES OF THE BOOK ARE
l Simple language and easily reproducible diagrams.
ts
l Large variety of SOLVED NUMERICAL PROBLEMS.
l Additional numerical problems under the heading PRACTICE PROBLEMS
gh
for self assessment and practice.
l Tough & Tricky PROBLEMS with answers to accelerate the potential for
ri
solving Numerical Problems.
l REVISION EXERCISES in the form of 'Very Short Answer Type', 'Short
ll
Answer Type' and 'Long Answer Type Questions' with HINTS and
SOLUTIONS to some questions is also included.
.A
l CONCEPTUAL QUESTIONS are solved at the end of each chapter.
l Additional information is given under 'Learning Plus','Key Note' and
rs
'Watch Out'.
l For Quick Revision of the chapter objective type questions in the form of
he
'True or False', 'Complete the Missing Links' and 'Choose the Correct
Alternative', under the heading Memory Test along with their answers, are
given at the end of each unit.
lis
l COMPETITION FILE which includes 'Additional Information', 'Higher Order
PART-I
CLASS-XI
Thinking Skills' (HOTS), 'Brain-Twisting Questions' (with answers) and
ub
'Multiple Choice Questions', is also given. This will prepare students for
various competitive entrance examinations for entry to Medical, IIT and
other Engineering colleges.
P
l COMPLETE coverage of previous years’ questions from all the board
examinations and competitive examinations such as AIPMT/NEET, JEE
n
(Main), JEE (Advance) and other State Boards.
er
Dr. S.P. Jauhar
In a NUTSHELL the book provides Excellent Guidance to students for
Boards' examinations as well as for competitive examinations for entrance
CLASS-XI
od
to professional colleges.
A THOROUGH & SINCERE STUDY OF THE UNIQUE & UNMATCHED BOOK WILL
ENCOURAGE THE STUDENTS AND PROVE A GREAT HELP IN ACHIEVING THEIR
PART-I
M
TARGET
Dr. S.P. Jauhar
©
MODERN PUBLISHERS STRICTLY ACCORDING TO THE NEW SYLLABUS
1904D0303A0048 An ISO 9001:2015 Certified Company INDIA'S FIRST SMART BOOK
©
M
od
er
n
P
ub
lis
he
rs
.A
ll
ri
gh
ts
re
se
rv
ed
.
q New Delhi: MBD House, Gulab Bhawan, 6, Bahadur Shah Zafar Marg Ph. 61312345, 23318301
q Mumbai: A-683, T.T.C. Industrial Area, M.I.D.C. Off. Thane-Belapur Road, Navi Mumbai Ph. 32996410, 27780821, 8691053365
q Chennai: No. 26 B/2 SIDCO Estate, North Phase, Pataravakkam Ambattur Industrial Estate, Ambattur Ph. 26359376, 26242350
q Chennai: Plot No. 3018, Old Y Block, 3rd Street, 12th Main Road, Anna Nagar West, Chennai Ph. 23741471
.
q Kolkata: Satyam Building, 46-D, Rafi Ahmed Kidwai Marg
ed
Ph. 22296863, 22161670
q Jalandhar City: MBD House, Railway Road Ph. 2458388, 2459046, 2455663
q Bengaluru: 124/31, 1st Main, Industrial Town (Near Chowdeshwari Kalyan Mantap),
rv
West of Chord Road, Rajajinagar Ph. 23103329, 23104667
q Hyderabad: Plot No. 41, Road No. 6, IDA, Mallapur, Uppal Mandal, Near Bharat Electronics Ltd. Ph: 9985820001
q Ernakulam: Surabhi Building, South Janatha Road, Palarivattom Ph. 2338107, 2347371
se
q Nagpur: Plot No. 231, Lendra Park, Behind Krims Hospital, Ramdaspeth Ph. 0712-2447306, 2447330, 7410017938
q Gandhinagar: B-114/115, Sector-25, GIDC, Gandhinagar Ph.079-29655442, 7600024563, 7600024542
q Cuttack: Badambadi, Link Road
re
Ph. 2367277, 2367279, 2313013
q Guwahati: Chancellor Commercial, Hem Baruah Road, Paan Bazar Ph. 7637014051
q Lucknow: 173/15, Dr. B. N. Verma Road, Old 30 Kutchery Road Ph. 4010992, 4010993
q Patna: Ist Floor, Annapurna Complex, Naya Tola Ph. 2678732, 2678994
ts
q Bhopal: Plot No. 137, 138, 139, Sector-I, Special Industrial Area, Govindpura Ph. 2581540, 2601535
q Goa: H. No. 1644, Plot No. 100, Kranti Nagar H.B. Colony, Nr. Nana Nani Park, Porvorim Ph. 0832-2413982, 7028912261
gh
q Jaipur: C-66A, In front of Malpani Hospital, Road No. 1, V.K. Industrial Area, Sikar Road Ph. 4050309, 4020168
q Raipur: Behind Aligarh Safe Steel Industries, Vidhan Sabha Road, Avanti Bai Chowk, Lodhi Para Pandri Ph. 2445370, 4052529
q Karnal: Plot No. 203, Sector-3, HSIDC, Near Namaste Chowk, Opp. New World Ph. 2220006, 2220009
ri
q Shimla (H.P.): C-89, Sector-I, New Shimla-9 Ph. 2670221, 2670816
q Jammu (J&K): MBD, Khasra No. 417, Industrial Area, Gangyal, Near SICOP Office Ph. 9419104035
q Ranchi (Jharkhand):
q Sahibabad (U.P.): B-9 & 10, Site IV, Industrial Area
ll Ph. 7260813703 (BM), 7260813710 (ASM), 7260813715 (ASO)
Ph. 3100045, 2896939
.A
q Dehradun (Uttarakhand): Plot No. 37, Bhagirathipuram, Niranjanpur, GMS Road Ph. 2520360, 2107214
DELHI LOCAL OFFICES
q Delhi (Shakarpur) : MB 161, Street No. 4 Ph. 22546557, 22518122
rs
q Delhi (Daryaganj): MBD House, 4587/15, Opp. Times of India Ph. 23245676
q Delhi (Patparganj): Plot No. 225, Industrial Area Ph. 22149691, 22147073
he
lis
duplicates,
n
with Python
er
od
M MODERN MODERN MODERN MODERN MODERN MODERN MODERN
MODERN MODERN MODERN MODERN MODERN MODERN MODERN MODERN MODERN MODERN MODERN MODERN MODERN MODERN MODERN MODERN MODERN MODERN MODERN MODERN MODERN MODERN MODERN MODERN MODERN MODERN MODERN MODERN MODERN MODERN MODERN MODERN MODERN MODERN MODERN MODERN MODERN MODERN MODERN MODERN MODERN MODERN MODERN MODERN MODERN MODERN MODERN MODERN MODERN MODERN MODERN MODERN MODERN MODERN MODERN MODERN MODERN MODERN MODERN MODERN MODERN MODERN MODERN MODERN MODERN MODERN MODERN MODERN MODERN MODERN MODERN MODERN MODERN MODERN MODERN MODERN MODERN MODERN MODERN MODERN MODERN MODERN MODERN MODERN MODERN MODERN MODERN MODERN MODERN MODERN MODERN MODERN MODERN MODERN
© All rights reserved. No part of this publication may be reproduced, stored in a retrieval system, or transmitted in any form or by any means,
electronic, mechanical, photocopying, recording, or otherwise without the prior written permission of the publisher. Any breach will entail legal
action and prosecution without further notice.
©
MODERN MODERN MODERN MODERN MODERN MODERN MODERN MODERN MODERN MODERN MODERN MODERN MODERN MODERN MODERN MODERN MODERN MODERN MODERN MODERN MODERN MODERN MODERN MODERN MODERN MODERN MODERN MODERN MODERN MODERN MODERN MODERN MODERN MODERN MODERN MODERN MODERN MODERN MODERN MODERN MODERN MODERN MODERN MODERN MODERN MODERN MODERN MODERN MODERN MODERN MODERN MODERN MODERN MODERN MODERN MODERN MODERN MODERN MODERN MODERN MODERN MODERN MODERN MODERN MODERN MODERN MODERN MODERN MODERN MODERN MODERN MODERN MODERN MODERN MODERN MODERN MODERN MODERN MODERN MODERN MODERN MODERN MODERN MODERN MODERN MODERN MODERN MODERN MODERN MODERN MODERN MODERN MODERN MODERN
.
students. The book has been prepared strictly according to the new syllabus proposed by C.B.S.E. New Delhi and
ed
Education Board of other Indian States and N.C.E.R.T. Text Book.
Salient Features of the Book
✰ Text matter has been planned giving emphasis on fundamental concepts in a simple, clear and systematic method.
rv
✰ The text has been presented in an interesting style with a large number of Illustrative examples and Numerical
problems.
✰ Additional information about the topic is given in Key Note, Key Facts, Facts File, Learning Plus, In focus,
se
Competition Plus to provide stimulus to find more about the subject.
✰ A number of Problems and Short answer questions have been given under the heading Practice Problems promoting
re
problem solving skills in students.
✰ Alarge number of Conceptual Questions (Solved) are given in each chapter.
✰ Advanced Level Problems with solutions are added to accelerate the potential of the students for SOLVING
NUMERICAL PROBLEMS for JEE (Advanced).
ts
✰ All the numerical problems of Practice Problems are completely solved in Solution File at the end of each chapter.
✰ Large number of Numerical Problems from I.I.T., Roorkee, M.L.N.R. and other Competitive Examinations have been
gh
given in Competition File.
✰ A variety of Multiple Choice Questions from different competitive examinations have been added to make the book
useful for the preparation of competitive examinations in Competition File.
ri
✰ MCQs with more than one correct answers and MCQs based on given comprehension/passage, Matrix Match Type
Questions and Integer Type or Numerical Value Type Questions according to latest IIT pattern are also given.
✰ In NCERT File, all the Textbook Exercises and Exemplar Problems are fully solved.
ll
✰ At the end of each chapter, Revision Exercises are given according to Latest Examination Pattern covering Objective
.A
and Subjective Questions.
✰ A number of Higher Order Thinking Skills (HOTS) and Advanced Level Questions with answers are included at the end
of each chapter.
✰ The definition of Key Terms &Laws and QUICK CHAPTER ROUND UP are given at the end under the heading Chapter
rs
Summary.
✰ UNIT PRACTICE TESTS with Hints & Solutions are given at the end of each chapter to help the students to check their
he
performance after covering the chapter. A Mock Test with solutions according to CBSE pattern is given at the end of the
book.
I am extremely thankful to many teachers and students who have been sending their valuable suggestions and comments
lis
for the improvement of the book. I am greatly indebted to them. I wish to acknowledge my sincere thanks to Dr. (Mrs.)
Sheenu Jauhar for her untiring efforts and valuable contribution in critical revision of the book and updating Objective
Questions. I also feel pleasure to express my thanks to Mr. Arun Kamboj (G.S.S. School, Tohana), Dr. G. J. P. Singh
ub
(Chemistry Deptt., P.U., Chandigarh), Dr. Sajeev Soni (S.D. College, Chandigarh), Dr. A.N. Sharma (Govt. College,
Hamirpur), Ms. Parveen (Faridkot), Mrs. Jagmohan Kaur (Govt. Model S.S. School Sec. 35 Chandigarh), Mrs. Anju Goel
(Govt. S. S. School Sec. 16, Chandigarh), Dr. G.S. Arora, Mrs. Sunita Saroha (D.A.V. S.S. School, Sec. 15, Chandigarh), Mr.
P
Gaurav Chakraborty, Mrs. Amita Sharma (Saraswati Institute, Chandigarh), Mrs. Seema (Seema Chemistry classes), Mr.
Sudhanshu Jaitley (Chandigarh), Mr. Deepak Mishra (H.O.D. Shri Gauri Shankar Inter College, Ferozabad), Mr. Praveen
Kumar (S.V.M. Senior Sec. School, Kosi Kalan, Mathura), Mr. Vikas Chander (Saint Mary S.S. School Gurdaspur), Shri
n
Rakesh Jassotia, (Sr. Lecturer, Govt. G.H.S.S. Nagari PAROLE, Kathua) and Mr. Vaibhav Yavlekar (Ujjain. I am also highly
er
thankful to Mr. Vinod Kumar Jangra (G.S.S.S. Akbarpur Baroto, Sonepat), Mr. Chander Mohan Kumar (Retd. P.G.T.,
Chemistry, Sacred Heart Convent School, Sarabha Nagar, Ludhiana) and Dr. Anjul Rajput (Dhampur) for thoroughly
checking of the book and providing valuable suggestions. I am also thankful to Simran Kaur for her sincere and dedicated
od
editorial work. Finally, I am happy to express my sincerest thanks and indebtness to our dynamic and versatile publisher
and his efficient staff for making the project successful. I am also thankful to Mr. Manik Juneja, Director — Content and
Production, Mr. B.S. Rawat, Mr. S.K. Sikka, Mr. Ravinder Pathania and L.B. Mishra who have taken great pains in
M
ed
.
©
M
1
od
er
n
P
ub
lis
he
rs
.A
ll
ri
gh
ts
re
se
rv
ed
.
C M
Y K
.
Aluminium Al 13 26.9
ed
Americium Am 95 243 [Rn]86 5f 7 7s2
Antimony Sb 51 121.75 [Kr]36 4d10 5s2 5p3
Argon Ar 18 39.94 [Ne]10 3s2 3p6
rv
Arsenic As 33 74.92 [Ar]18 3d10 4s2 4p3
Astatine At 85 210 [Xe]54 4f14 5d10 6s2 6p5
Barium Ba 56 137.3 [Xe]54 6s2
se
Berkelium Bk 97 247 [Rn]86 5f9 7s2
Beryllium Be 4 9.01 [He]2 2s2
re
Bismuth Bi 83 208.98 [Xe]54 4f14 5d10 6s2 6p3
Boron B 5 10.81 [He]54 2s2 2p1
Bromine Br 35 79.90 [Ar]18 3d10 4s2 4p5
Cadmium Cd 48 112.40 [Kr]36 4d10 5s2
ts
Calcium Ca 20 40.02 [Ar]18 4s2
Californium Cf 98 251 [Rn]86 5f10 7s2
gh
Carbon C 6 12 [He]2 2s2 2p2
Cerium Ce 58 140.12 [Xe]54 4f1 5d1 6s2
Cesium Cs 55 132.90 [Xe]54 6s1
ri
Chlorine Cl 17 35.45 [Ne]10 3s2 3p5
Chromium Cr 24 51.99 [Ar]18 3d5 4s1
Cobalt
Copernicium
Co
Cn
27
112
ll 58.93
285
[Ar]18 3d7 4s2
[Rn]86 5f14 6d10 7s2
.A
Copper Cu 29 63.54 [Ar]18 3d10 4s1
Curium Cm 96 245 [Rn]86 5f7 6d1 7s2
Dysprosium Dy 66 162.50 [Xe]54 4f10 6s2
rs
Helium He 2 4 1s2
Holmium Ho 67 164.93 [Xe]54 4f11 6s2
er
Hydrogen H 1 1 1s1
Indium In 49 114.82 [Kr]36 4d10 5s2 5pl
od
C M
Y K
C M
Y K
.
Moscovium Mc 115 289 [Rn]86 5f14 6d10 7s2 7p3
ed
Neodymium Nd 60 144.24 [Xe]54 4f4 6s2
Neon Ne 10 20.18 [He]2 2s2 2p6
Neptunium Np 93 237 [Rn]86 5f4 6d1 7s2
rv
Nickel Ni 28 58.71 [Ar]18 3d8 4s2
Nihonium Nh 113 286 [Rn]86 5f14 6d10 7s2 7p1
se
Niobium Nb 41 92.90 [Kr]36 4d4 5s1
Nitrogen N 7 14 [He]2 2s2 2p3
Nobelium No 102 254 [Rn]56 5f14 7s2
re
Oganesson Og 118 294 [Rn]86 5f14 6d10 7s2 7p6
Osmium Os 76 190.2 [Xe]54 4f14 5d6 6s2
Oxygen O 8 15.99 [He]2 2s2 2p4
ts
Palladium Pd 46 106.4 [Kr]36 4d10
Phosphorus P 15 30.97 [Ne]10 3s2 3p3
Platinum Pt 78 195.09 [Xe]54 4f14 5d9 6s1
gh
Plutonium Pu 94 244 [Rn]86 5f6 7s2
Polonium Po 84 210 [Xe]54 4f14 5d10 6s2 6p4
Potassium K 19 39.102 [Ar]18 4s1
ri
Praseodymium Pr 59 140.907 [Xe]54 4f3 6s2
Promethium Pm 61 145 [Xe]54 4f5 6s2
Protactinium
Radium
Pa
Ra
91
88
ll
231
226
[Rn]86 5f2 6d1 7s2
[Rn]86 7s2
.A
Radon Rn 86 222 [Xe]54 4f14 5d10 6s2 6p6
Rhenium Re 75 186.2 [Xe]54 4f14 5d5 6s2
Rhodium Rh 45 102.90 [Kr]36 4d8 5s1
rs
C M
Y K
©
M
od
er
n
P
ub
lis
he
rs
.A
ll
ri
gh
ts
re
se
FIRST PRIZE
rv
ed
.
.
ed
Award
of
rv
Excellence
se
re
ts
gh
ri
ll
.A
First Prize
rs
he
To help the students to visualize the concepts more easily and enhance understanding – a digital revolution:
Watch chemistry come alive.
ub
P
n
er
od
M
©
©
M
od
er
A NOTE TO THE STUDENTS n
P
ub
lis
he
rs
ts
re
se
rv
ed
.
SYLLABUS
CHEMISTRY
CLASS XI (THEORY) Total Periods (Theory 160 + Practical 60)
.
Time : 3 Hours Total Marks : 70
ed
Unit No. Title No. of Periods Marks
Unit I Some Basic Concepts of Chemistry 12
rv
11
Unit II Structure of Atom 14
se
Unit III Classification of Elements and Periodicity in Properties 08 04
re
Unit V States of Matter : Gases and Liquids 12
21
Unit VI Chemical Thermodynamics 16
ts
Unit VII Equilibrium 14
gh
Unit VIII Redox Reactions 06
Unit IX Hydrogen 08
16
Unit X s-Block Elements 10
ri
Unit XI p-Block Elements 14
Unit XII
ll
Organic Chemistry : Some Basic Principles and Techniques 14
.A
Unit XIII Hydrocarbons 12 18
Unit XIV Environmental Chemistry 06
rs
Total 160 70
he
and molecules.
Atomic and molecular masses, mole concept and molar mass, percentage composition, empirical and
ub
Bohr’s model and its limitations, concept of shells and subshells, dual nature of matter and light,
de Broglie’s relationship, Heisenberg uncertainty principle, concept of orbitals, quantum numbers,
n
shapes of s, p and d orbitals, rules for filling electrons in orbitals – Aufbau principle, Pauli’s exclusion
er
principle and Hund’s rule, electronic configuration of atoms, stability of half-filled and completely
filled orbitals.
od
atomic radii, ionic radii, inert gas radii, ionization enthalpy, electron gain enthalpy, electronegativity,
valency. Nomenclature of elements with atomic number greater than 100.
©
.
ed
Liquid state : Vapour pressure, viscosity and surface tension (qualitative idea only, no mathematical
derivations).
Unit VI : Chemical Thermodynamics (16 Periods)
rv
Concepts of systems and types of systems, surroundings, work, heat, energy, extensive and intensive
se
properties, state functions. First law of thermodynamics – internal energy and enthalpy, heat capacity
and specific heat, measurement of DU and DH, Hess’s law of constant heat summation, enthalpy of
bond dissociation, combustion, formation, atomization, sublimation, phase transition, ionization,
re
solution and dilution. Second law of thermodynamics (brief introduction).
Introduction of entropy as a state function, Gibb’s energy change for spontaneous and non-
ts
spontaneous processes, criteria for equilibrium. Third law of thermodynamics (brief introduction).
Unit VII: Equilibrium (14 Periods)
gh
Equilibrium in physical and chemical processes, dynamic nature of equilibrium, law of mass action,
equilibrium constant, factors affecting equilibrium – Le Chatelier’s principle; ionic equilibrium –
ri
ionization of acids and bases, strong and weak electrolytes, degree of ionization, ionization of
poly basic acids, acid strength, concept of pH. Henderson equation, hydrolysis of salts (elementary
ll
idea), buffer solutions, solubility product, common ion effect (with illustrative examples).
.A
Unit VIII : Redox Reactions (06 Periods)
Concept of oxidation and reduction, redox reactions, oxidation number, balancing redox reactions in
rs
terms of loss and gain of electrons and change in oxidation number, applications of redox
reactions.
he
hydrogen, hydrides-ionic covalent and interstitial; physical and chemical properties of water, heavy
water, hydrogen peroxide-preparation, reactions and structure and use; hydrogen as a fuel.
ub
Unit X : s-Block Elements (Alkali and Alkaline Earth Metals) (10 Periods)
Group 1 and Group 2 elements : General introduction, electronic configuration, occurrence,
P
anomalous properties of the first element of each group, diagonal relationship, trends in the variation
of properties (such as ionization enthalpy, atomic and ionic radii), trends in chemical reactivity with
n
oxygen, water, hydrogen and halogens, uses. Preparation and properties of some important
er
Calcium oxide and Calcium carbonate and their industrial uses, Biological importance of Magnesium
and Calcium.
M
.
ed
General introduction, methods of purification, qualitative and quantitative analysis, classification and
IUPAC nomenclature of organic compounds. Electronic displacements in a covalent
bond: inductive effect, electromeric effect, resonance and hyper-conjugation. Homolytic and
rv
heterolytic fission of a covalent bond: free radicals, carbocations, carbanions, electrophiles and
se
nucleophiles, types of organic reactions.
Unit XIII : Hydrocarbons (12 Periods)
re
Classification of Hydrocarbons
Aliphatic Hydrocarbons:
Alkanes – Nomenclature, isomerism, conformation (ethane only), physical properties, chemical
ts
reactions including free radical mechanism of halogenation, combustion and pyrolysis.
Alkenes – Nomenclature, structure of double bond (ethene), geometrical isomerism, physical
gh
properties, methods of preparation, chemical reactions: addition of hydrogen, halogen, water,
hydrogen halides (Markownikov’s addition and peroxide effect), ozonolysis, oxidation, mechanism of
ri
electrophilic addition.
Alkynes – Nomenclature, structure of triple bond (ethyne), physical properties, methods of
ll
preparation, chemical reactions: acidic character of alkynes, addition reaction of hydrogen, halogens,
.A
hydrogen halides and water.
Aromatic hydrocarbons – Introduction, IUPAC nomenclature, benzene: resonance, aromaticity,
chemical properties, mechanism of electrophilic substitution. Nitration, sulphonation, halogenation,
rs
Friedel Craft’s alkylation and acylation, directive influence of a functional group in mono-substituted
benzene. Carcinogenicity and toxicity.
he
atmospheric pollutants, acid rain, ozone and its reactions, effects of depletion of ozone layer,
greenhouse effect and global warming – pollution due to industrial wastes, green chemistry as an
ub
alternative tool for reducing pollution, strategies for control of environmental pollution.
P
n
er
od
M
©
.
ed
rv
se
1. Some Basic Concepts of Chemistry 1/1 – 1/140
re
2. Structure of Atom 2/1 – 2/120
ts
3. Classification of Elements and Periodicity in Properties 3/1 – 3/70
gh
5. States of Matter : Gases and Liquids 5/1 – 5/106
ri
6. Thermodynamics 6/1 – 6/132
7. Equilibrium
ll 7/1 – 7/173
.A
Additional Questions
(According to the Latest CBSE Question Paper Designed by CBSE) 1 – 34
he
Appendices 1–6
lis
.
1
ed
rv
22.4 L
at STP
se
re
Volume
1
32g Mole
ts
S Pa
ss rtic
Ma les
gh
SOME BASIC Gram atom or
ri
6.022 ´ 1023
CONCEPTS
Gram mole
58.5g
NaCl
ll
OF CHEMISTRY
.A
Chemistry is the study of the materials that make up the universe and the changes which these materials undergo. The
rs
study deals with the composition, structure and properties of matter. These aspects can be best understood in terms of
he
the constituents of matter: atoms and molecules. In fact, chemistry is called the science of atoms and molecules.
Chemistry is defined as:
the branch of science which deals with the study of composition, properties and structures of matter
lis
and the changes which the matter undergoes under different conditions and the laws which govern
these changes.
Chemistry plays a central role in science and is often interlinked with other branches of science. It also plays an
ub
OBJECTIVES
Building on..... Assessing..... Preparing for Competition.....
n
Hints & Solutions for Practice Questions with Answers109 Medical Entrance 119
Problems 79 JEE (Main) & Other State Boards’
CHAPTER SUMMARY & QUICK UNIT PRACTICE TEST 141
M
Solutions 92 Objective Questions 127
NCERT Exemplar Problems with
Answers/Hints 96
1/1
2
od
er
n
P
ub
lis
he
rs
.A
ll
ri
gh
1/14/2020, 4:44 PM
ts
re
se
Fig. 1 Applications of chemistry in different fields.
rv
MODERN’S abc + OF CHEMISTRY–XI
ed
.
SOME BASIC CONCEPTS OF CHEMISTRY 1/3
IMPORTANCE AND SCOPE OF CHEMISTRY tranquilizers (for treatment of stress and mental
In chemistry, we study the composition of diseases), antimicrobials (to cure infections),
materials to find out what they are made of. The antimalarials (to treat malaria), etc. Antibiotics
marvellous thing about chemistry is that all the matter such as penicillin, amoxycillin, streptomycin and
in the universe is made up of tiny smallest particles broad spectrum antibiotics such as tetracycline,
.
called atoms or molecules. It is very interesting that chloramphenicol, etc., have cured a variety of diseases
ed
only about 118 types of atoms make up whole of the due to harmful micro-organisms. Antifertility drugs
matter in the universe. We also learn how their have been world-wide used for birth control methods.
composition affects their characteristics and behaviour Chemistry has given us a large number of life
rv
so that we can plan to make new materials with saving drugs.
properties of our interests. For this, we have to learn
se
Life saving drugs like cisplatin and taxol for cancer
how substances undergo changes in composition and therapy and AZT (azidothymidine) used for helping
properties. These changes in the language of chemistry AIDS victims are latest contributions of chemistry
are called chemical reactions. Thus, studying
re
in medicines.
chemistry is essential for us to understand better the
world in which we live. Chemistry for our comforts, pleasure and
luxuries
ts
Since there is a large variety of substances in the
universe, the scope of chemistry is immense. Whether Chemistry has also pioneer contribution towards
we are concerned with living systems in biology and our comforts, pleasure and luxuries.
gh
medicine, with materials such as iron, steel and concrete ● It has given us building materials, synthetic
as in engineering or with manufacture of computer fibres and variety of articles of domestic use.
chips, we deal with chemistry. Progress in modern
ri
● Chemical industries manufacturing fertilizers,
society is completely based on advances in chemistry. acids, alkalies, salts, dyes, polymers, drugs, soaps,
In today’s technological age, the importance of detergents, metals, alloys and other organic and
ll
chemistry is increasing. Chemical principles are playing inorganic chemicals including new materials have
.A
key role in diverse areas such as weather patterns, improved our national economy.
functioning of brain and operation of computers.
Chemistry for giving us new materials
It is difficult today to imagine our life In recent years, chemistry has given us new
rs
our everyday life. The important applications of Chemistry has also increased the striking power
chemistry in different fields are shown in Fig. 1 and of a country in war times. It is responsible for the
are discussed below: discovery of highly explosive substances such as
ub
Chemistry for meeting our basic needs and dynamite, TNT (trinitrotoluene), nitroglycerine,
necessities of life. poisonous gases such as mustard gas, phosgene,
lewisite and many deadly weapons such as atom bomb
P
necessities of life.
In recent years, chemistry has solved with a fair
er
● Chemistry has also helped for better health Still many goals are there for the chemists to achieve
and sanitation. The epidemics such as cholera, small successfully. Environmental problems, management
pox, plague have now become things of the past. of Green house gases (like methane, carbon dioxide,
©
.
The presence of matter can be felt by one or more
ed
of our senses. Everything around us, for example, book,
pen, table, sugar cubes, iron rod, water, air are
composed of matter because they have mass and they
rv
occupy space.
Matter can be classified in two ways :
se
A : Physical classification of matter
B : Chemical classification of matter.
A : Physical Classification of Matter
re
Depending upon the physical state of matter, it Fig. 2. Arrangement of particles in solid, liquid and
can be classified into three states, namely, solid, liquid gaseous state.
and gaseous state. These three states of matter are interconvertible
ts
(i) Solid state. A solid has a definite shape and by changing the conditions of temperature and
definite volume. Thus, solids are generally hard and
pressure. On heating, a solid changes to a liquid and
gh
rigid. For example, wood, table, copper rod, common
salt, etc. on further heating the liquid changes to the gaseous
(ii) Liquid state. A liquid has a definite volume (or vapour) state. In the reverse process, a gas on
ri
but not definite shape. A liquid takes the shape of the cooling liquefies to the liquid and liquid on further
container in which it is placed. For example, water, cooling freezes to the solid.
milk, oil, etc.
ll heat
SOLID heat
LIQUID
GAS
(iii) Gaseous state. A gas neither possesses a
.A
cool cool
definite volume nor a definite shape. It occupies the
whole of the volume of the vessel in which it is placed. B. Chemical Classification of Matter
For example, air, oxygen, hydrogen, carbon dioxide, etc. The chemical classification of matter is based upon
rs
Particle Nature of Matter its composition. Different substances differ from each
As we know matter is made up of particles. In terms other in their constituents composition. On the basis
he
of particle concept of matter, in solids, the particles of chemical composition, matter can be classified as :
are closely packed and the empty spaces between the A : Pure substances
particles are very small. Due to close packing of the
lis
B : Mixtures
particles, they can only vibrate about their fixed
positions i.e. can have only vibratory motion. Due to A. Pure substances
fixed positions of the particles in solids, they have Pure substances consist of single type of
ub
highly ordered arrangement and this gives definite particles. All the constituent particles of a pure
regular shape to the crystals. The regular ordered substance are same in their chemical nature. For
arrangement of particles in solids is called lattice.
P
are relatively weak. However, these are not weak composition. Similarly, glucose contains carbon,
enough to allow the particles to separate from one hydrogen and oxygen in a fixed ratio and thus behaves
another. Thus, liquids have definite volume but as a pure substance. The constituents of pure
M
donot have fixed shape. They take the shape of the substances cannot be separated by simple physical
container. methods (like filtration, evaporation, distillation,
©
On the other hand, in gases, the particles are very sublimation, mechanical separation, etc.). These can
loosely packed and empty spaces between them are
only be separated by chemical or electrochemical
very large. As a result, the attractive forces between
the particles are very very small so that their methods.
movement is easy and fast. They can move to each Pure substances can be further classified into
corner of the vessel and therefore gases donot have elements and compounds.
definite shape and definite volume.
.
from simpler substances by ordinary
ed
physical or chemical methods. (b) Organic compounds. These are the
The common examples of elements are hydrogen, compounds which are obtained from living sources
rv
oxygen, nitrogen, sulphur, iron, lead, gold, mercury, such as plants and animals. All these contain carbon.
etc. There are about 118 elements known at present. For example, carbohydrates, oils, fats, waxes, proteins,
Out of these, 92 have so far been found to occur etc.
se
naturally and the remaining have been prepared in
the laboratory. All the elements do not occur in the B. Mixtures
Mixtures contain more than one kind of pure form
re
crust of earth in equal proportions. About twenty
elements make up 99% of the earth’s crust. The most of matter, known as substance. It is a simple
abundant elements in the earth’s crust are oxygen, combination of two or more substances in which the
silicon, aluminium, iron, calcium, sodium, potassium,
ts
constituent substances retain their identities. The
hydrogen, chlorine, carbon, etc. composition of the mixture may be varied to any
Elements are further classified into following types :
gh
extent. The substances present in a mixture are called
(a) Metals. These elements are generally solids and its components. Therefore, a mixture may contain
possess characteristics such as bright lustre, hardness and two or more substances in any ratio. Thus, a mixture
ri
ability to conduct electricity and heat. These are generally may be defined as:
malleable (can be beaten into thin sheets) and ductile (can
be drawn into wires). Some common examples of metals ll a combination of two or more elements or
are copper, iron, silver, gold, aluminium, etc. About 80% compounds in any proportion so that the
.A
of the known elements are metals. components do not lose their identity.
(b) Non-metals. These elements are generally
Many of the substances present around us are
non lustrous, brittle and poor conductors of heat and
rs
electricity. The common examples of non-metals are mixtures. For example, sugar solution in water, air,
carbon, hydrogen, oxygen, nitrogen, etc. tea, brass (an alloy of copper and zinc), soft drink, soil,
etc., are all mixtures.
he
a pure substance containing two or more even under a powerful microscope. Homogeneous
than two elements combined together in a mixtures are also called solutions. Some examples of
n
definite proportion by mass and which can homogeneous mixtures are air, gasoline, sea water, brass,
be decomposed into its constituent elements etc. Air is a homogeneous mixture of a number of gases
er
by suitable chemical methods. such as oxygen, nitrogen, carbon dioxide, water vapour,
A compound always contains the same elements etc. The composition of air is the same everywhere.
od
Similarly, carbon dioxide always contains carbon throughout. These mixtures consist of two or more
and oxygen in the ratio of 3 : 8. parts (called phases) which have different
compositions. These have visible boundaries of
©
The properties of the compounds are totally separation between the different constituents and can
different from the elements from which these are
be easily seen even with naked eye. For example, a
formed. For example, hydrogen is combustible while
oxygen is supporter of combustion but water mixture of iron filings, common salt and sulphur gives
(a compound of hydrogen and oxygen) is normally used a heterogeneous mixture in which the components can
for extinguishing fire. be seen lying side by side very easily.
Compounds Mixtures
1. In a compound, two are more elements are 1. In a mixture, two or more elements or compounds
.
combined chemically. just mix together.
ed
2. The compounds contains two or more elements in 2. The components of a mixture may be present in
a fixed ratio by mass. Its composition is always fixed. any ratio. Its composition is variable.
rv
3. A compound has a definite formula. 3. A mixture does not have a definite formula.
4. A compound is always homogeneous i.e., has the 4. A mixture may be homogeneous or heterogeneous.
se
same composition throughout.
5. A chemical reaction takes place and therefore, the 5. No chemical reaction takes place and therefore,
formation of a compound takes place with the formation of mixture is not accompained by
re
absorption or evolution of energy. any energy change.
6. The properties of a compound are entirely different 6. A mixture shows the properties of its
from those of its constituents. constituents.
ts
7. A compound cannot be separated into its 7. A mixture can be separated into its constituents
constituents by ordinary physical methods. These by physical methods (like filtration, evaporation,
gh
can be separated by chemical or electrochemical distillation, sublimation, mechanical separation
reactions. etc.)
ri
8. A compound has a fixed melting point, boilling 8. A mixture does not have fixed melting point,
point, etc. boiling point, etc.
which can be measured or observed without combustibility and reactions with other elements and
changing the identity or composition of the compounds.
substance. Many physical properties of matter such as mass,
For example, colour, odour, melting point, boiling length, volume, temperature, pressure, density, etc.,
point, density, etc. On the other hand, are quantitative in nature. Therefore, these properties
are also called physical quantities.
.
of a box as 5700 g or 5.7 kg. The simple numerical figure in Table 1. These base units pertain to seven
ed
5700 or 5.7 does not convey any meaningful information. fundamental scientific quantities and all other units
Thus, it is essential to include units with every can be derived from these.
experimental value. A unit is defined as Table. 1. Seven Base SI Units.
rv
the standard of reference chosen to measure
any physical quantity. Physical Symbol for Name of Symbol
Quantity quantity Unit
se
Naturally, we must have a convenient system of units
for assigning numerical values to the measured or Length l metre m
calculated values. Fortunately, it turns out that it is Mass m kilogram kg
Time t
re
sufficient to define some basic units such as units of second s
Thermodynamic T kelvin K
mass, length and time. Since these are independent
temperature
units and cannot be derived from any other unit, these Electric current I ampere A
ts
are also called fundamental units. Luminous intensity Iv candela cd
The units for other quantities can be derived from Amount of substance n mole mol
these units and hence are called derived units.
gh
It may be noted that the unit of temperature
There are many different systems of units. For according to SI system is Kelvin but still celsius scale
the first time in 1791, a study committee of the French (°C) of temperature is commonly used in our daily life.
ri
Academy of Science devised a system called “the
These two units are related as :
metric system” which became popular in the scientific
community throughout the world. The fundamental Kelvin temperature (K) = °C + 273.15
ll
units of metric system are the grams for mass, the Similarly, Angstrom (Å) is commonly used as a
.A
metre for length and the litre for the volume. In India, unit of length in chemistry. It is almost of the same
the metric system was adopted in 1957. The metric size as the size of an atom. It is equal to 10–10 m.
system is a decimal system and the different units for 1 Å = 10–10 m
rs
a physical quantity are related in powers of ten. The However, it must be kept in mind that it is not
different powers are generally indicated by a prefix SI unit. In SI units, we may use nanometre (nm) or
attached to the unit. picometre (pm) :
he
1 nm = 10–9 m
SI Units
1 pm = 10–12 m
In recent years, the scientists have generally The definitions of the SI base units are given in
lis
Measures in France.
Time second The second is the duration of 9, 192, 631, 770 periods of the radiation
corresponding to the transition between the two hyperfine levels of the
od
placed one metre apart in vacuum, would produce between these conductors
a force equal to 2 × 10–7 newton per metre of length.
Thermodynamic kelvin The kelvin is the unit of thermodynamic temperature and equals exactly
©
PREFIXES
The SI units of some of the physical quantities are either too small or too large. To change the order of
magnitude, these are expressed by using prefixes before the name of the base units. The various prefixes are listed
in Table 3.
Table 3. List of common prefixes used in SI system.
.
ed
Multiple Prefix Symbol Multiple Prefix Symbol
10−1 deci d 101 deca da
10−2 centi c 102 hecto h
rv
10−3 milli m 103 kilo k
10−6 micro μ 106 mega M
10−9 nano n 109 giga G
se
10−12 pico p 1012 tera T
10−15 femto f 1015 peta P
10−18 atto a 1018 exa E
re
10–21 zepto z 1021 zetta Z
10–24 yocto y 1024 yotta Y
ts
SOME COMMONLY USED QUANTITIES balance commonly used in laboratories is shown in
Let us learn about some common quantities which Fig. 3 (a). These days, many types of electronic balances
gh
we often come across. are available (Fig. 3 b). The analytical balances usually
1. Mass and weight. weigh to ± 0.0001 g, although some balances are
available which can weigh to ± 0.00001 g.
Mass of a substance is the amount of matter
ri
present in it. The SI unit of mass is kilogram, (kg). Because
kilogram is too large unit for many purposes, its
Weight is the force exerted on an object by the pull
fraction, gram, g, is commonly used and 1 kg = 1000
ll
of gravity. The mass of a substance is constant and is
g. The smaller units such as milligram, mg (1mg =
independent of its location. For example, your body
.A
10–3 g), microgram, μg (1 μg = 10–3 mg or = 10–6 g) are
has same mass whether you are on Earth or on the
also commonly used.
moon. However, weight of an object may change from
one place to another due to change in gravity. The mass 1 kg = 103 g = 106 mg = 109 μg
rs
Rider carrier
lis
Rider carrier
knob
ub
P
Beam
n
er
Pan
od
Levelling screw
M
such as litre, L and millilitre, mL. Litre is equal in size SI unit of mass
to dm3 and millilitre is equal in size to cm3. SI unit of density =
SI unit of volume
1L = 1dm3 and 1mL = 1 cm3
and 1L = 1000 mL = 1000 cm3 = 1dm3 kg
= or kg m–3
These relationships can be understood from Fig. 4. m3
.
This unit is quite large and it is commonly
ed
expressed as g cm–3 for solids and g mL–1 for liquids,
where mass is expressed in gram and volume in cm3
rv
or mL. As we know that volume of a substance changes
when heated or cooled, therefore, densities depend
se
upon temperature. Hence, while reporting a density,
the temperature must also be mentioned.
4. Temperature
re
1 cm
Volume : 1 cm3 Temperature measures the degree of hotness or
or 1 mL coldness of an object. There are three common scales
ts
1 cm
to measure temperature
10 cm = 1 dm
1. The SI scale or Kelvin scale measured in
gh
Volume = 1000 cm3 or 1000 mL
= 1 dm3 or 1 L Kelvin (K)
Fig. 4. Comparison between the relative sizes of two
units of volume. 2. Celsius scale measured in degree
ri
Celsius (°C)
The volume of liquids or solutions are commonly
measured in the laboratory by graduated cylinder, ll 3. Fahrenheit scale measured in degrees
burette or pipette. A volumetric flask is used to prepare Fahrenheit (°F)
.A
a known volume of a solution. Volumetric flasks of
different volumes are available. These devices for The Fahrenheit scale has been commonly used in
measuring volume are shown in Fig. 5. A precise United States and is slowly been replaced by Celsius
rs
volume of a liquid can also be measured with the help scale and Kelvin scale. The thermometers with celsius
of graduated syringe. scale are calibrated from 0° to 100° where these two
temperatures are the freezing point and boiling point
he
of water respectively.
0
mL 5
The Fahrenheit scale is represented between 32°
lis
10
100
90 15 to 212° which represent freezing point and boiling
80 20 point of water respectively. There are 100 degrees
ub
30
60 degrees between them in Fahrenheit giving rise to
50 35
exact relationship as
P
40 40
30 45
20 50 180° F = 100°C or 1.8°F = 1°C
n
1 litre
Graduated
Pipette Burette Volumetric flask 9
cylinder °F = (°C) + 32°
od
5
Fig. 5. Some volume measuring devices.
So to convert Celsius to Fahrenheit, we can use
M
5
relates the mass of an object to its volume. It is simply
the mass of an object divided by its volume. The and to convert Fahrenheit to Celsius, we can use
units of density may be obtained as : the formula,
Mass 5
Density = °C = [°F – 32°]
Volume 9
.
degree sign. On this scale, the freezing point of water
ed
is – 273.15 K and boiling point of water is 373.15 K. Volume = (1.0 m) × (2.0 m) × (3.0 m) = 6.0 m3
The coldest possible temperature 0 K becomes The other units may also be built up in this way
–273.15°C and it is also sometimes called absolute from the definition of the physical quantity. For
rv
zero. example,
The Kelvin scale is related to Celsius scale as :
se
(i) Area. The units of area can be easily derived
K = °C + 273.15 as :
or K = °C + 273 (for simplicity) Area = Length × Breadth
re
so that = m × m = m2
Temperature in Kelvin = Temperature in °C + 273.15 (Units of length are m)
ts
Temperature in °C = Temperature in K – 273.15 (ii) Density. The density of a substance is mass
per unit volume, i.e.
For example, the human body temperature and
gh
room temperature on these temperature scales are : Mass of sample kg
Density = = 3
Human body temperature = 98.6°F = 37°C = 310.15 K Volume of sample m
ri
Room temperature = 77°F = 25°C = 298.15K
= kg m–3
The three types of thermometers using different
scales are shown in Fig. 6.
ll (iii) Velocity. It is the change in distance per unit
time, i.e.
.A
Distance m
Velocity = =
Time s
rs
= m s–1
he
Acceleration =
Time
(m s−1 )
ub
=
(s)
Celsius Fahrenheit Kelvin
= m s–2
P
Fig. 6. Thermometers using different temperature scales. (v) Force. It is mass multiplied by acceleration.
Force = Mass × Acceleration
n
Area
derived from the seven base units. These are called
derived units because these are derived from the But force has the derived units as kg m s–2.
©
.
Volume Length cube — m3
ed
Density Mass per unit volume — kg/m3 or kg m–3
Speed Distance travelled per unit time — m/s or m s–1
rv
Acceleration Speed change per unit time — m/s × 1/s or m s–2
se
Force Mass × acceleration Newton* N = kg m s–2
Pressure Force per unit area Pascal Pa = kg m–1 s–2
re
Energy, work Force × distance travelled Joule** J = kg m s–2 × m or kg m2 s–2
Power Energy/Time Watt W = kg m2 s–3 or Js–1
Electric charge Current × time Coulomb C = As
ts
or potential difference
gh
Electric resistance Potential difference/current Ohm Ω = VA–1
Electrical conductance Reciprocal of resistance Ohm–1 or S = AV–1
Siemen
ri
Frequency Cycles/sec Hertz Hz = s–1
ll
* Newton is defined as the force which gives an acceleration of 1 m s–2 to a mass of 1 kg; so that
.A
f = ma = 1 (1 kg) (1 m s–2) = 1 kg m s–2 = 1 N
** Joule is the work done when a displacement of 1 metre takes place by a force of 1 newton so that
w = f × d = (1N) × (1 m) = (1 kg m s–2) × (1m) = 1 kg m2 s–2 = 1 J
rs
KEY NOTES
• Exponents also operate on prefixes. e.g.,
SOME IMPORTANT NOTES ON THE USE OF 1 cm2 = (10–2 m)2 = 10–4 m2
he
SI UNITS (∵ 1 cm = 10–2 m)
In order to avoid confusion in the SI units, it is It is not equal to 10–2 m2
lis
(∵ 1 mm = 10–3 m)
between, e.g. metre kelvin may be written It is not equal to 10–3 m3
as : It may be noted that some of the units are named
P
metre Kelvin = m.K or m K after the name of the scientist who did pioneering
It should not be written as mK which stands work in the related field. For example, the unit of
n
for milli kelvin. energy is joule abbreviated as ‘J’ and the unit of
er
• Words and symbols should not be used in electric charge is coulomb abbreviated as C. The
mixed forms. Thus, it is not proper to use symbols of the units which bear the names of the
J per mole. It should be written either as joule
od
volume; thermometer measures temperature, etc. In other words, precision gives the extent of
Every scientific measurement is limited by the agreement of the individual values among themselves
reliability of the measuring instrument and the skill i.e., between the repeated measurements of the same
of the observer. In each measurement, uncertainty quantity. This means that smaller the difference
should be reported carefully. If we repeat a particular between the individual values of repeated
measurement, we usually do not obtain the same result
.
measurements, the greater is the precision. For
ed
because each measurement is subject to experimental example, the analysis of an element gave the following
error. Therefore, different measured values vary results by two methods.
slightly from one another. To express the results of
Method A : %Cl = 20.60 ± 0.05
rv
different measurements two terms ; accuracy and
precision are commonly used. Method B : %Cl = 20.46 ± 0.15
Thus, method A gives more precise data because
se
Accuracy and Precision
the results differ only by ± 0.05%.
The term accuracy denotes the closeness of an
experimental value or the mean value of a set of The terms accuracy and precision can be illustrated
re
measurements to the true value. Thus, by the following example. Consider the results obtained
accuracy is a measure of the difference between by four students for the % of iron in a sample. These are
the experimental value or the mean value of a given in Table 5 and are shown graphically in Fig. 7. The
ts
set of measurements and the true value. true value is 20.50 mg of iron.
If the mean value of different measurements is Table 5. Results obtained by four students for
gh
close to the true value, the measurement is said to be the percentage of iron in a sample.
accurate. Smaller the difference between the mean
Student Student Student Student
value and the true value, the larger is the accuracy.
ri
Suppose a sample of a compound contains 20.30% of A B C D
chloride. The chemical analysis of the sample by one 20.50 20.75 20.20 20.20
method (say method A) gives mean value of 20.60% ll
chloride while another method (say method B) gives 20.48 20.65 20.25 20.40
.A
mean value of 20.46% chloride. Then, 20.52 20.40 20.15 20.00
Difference between the true value and mean value 20.50 20.28 20.20 20.60
in method A = 20.60 – 20.30 = 0.30% Mean = 20.50 20.52 20.20 20.30
rs
Difference between the true value and mean value Deviation = ± 0.02 ± 0.24 ± 0.05 ± 0.30
in method B = 20.46 – 20.30 = 0.16%
The results obtained by student A have good
he
precision of the measurements is calculated. have good accuracy because the mean value (20.52) is
Precision refers to how closely two or more close to the true value. Student C has obtained results
measurements of the same quantity agree with one of good precision because the measurements are close
P
another. Thus, together (± 0.05) but have poor accuracy because the
precision is expressed as the difference between mean value (20.20) is far from the true value. The
n
a measured value and the arithmetic mean value results of student D have poor precision because the
for a series of measurements. measurements are scattered and have poor accuracy
er
because the mean value (20.30) is far from the true 0.0001 48 72
value. ↑
From the above observation, it can be concluded so that exponent n = – 4 and we can write
that accurate results are generally precise but precise 0.00014872 = 1.4872 × 10–4
results need not be accurate. In other words, good Thus, in general, number is written in scientific
.
precision does not assure good accuracy. notation as :
ed
number with a single number of places decimal
Exponential Number or Scientific Notation. non-zero digit ⎯⎯ point was moved
In Chemistry, we come across very large and very
⎯⎯
rv
→ + moved left
small numbers. It is very tedious to write down such N × 10n – moved right
numbers in the ordinary way. For example, it is not
se
convenient to write Avogadro constant as 602, 213, Arithmetics Using Scientific Notation, Addition
700,000,000,000,000,000. These numbers are usually and Subtraction
re
expressed in a simple way known as exponential form To add or subtract numbers in scientific notation,
or scientific notation. For example, the number 246.38 the exponent (n) must be the same in both numbers.
may be expressed as : If the exponent is not same in both numbers, it has to
ts
246.38 = 2.4638 × 10 × 10 or = 2.4638 × 102 be made same before adding or subtracting. For
where 10 2 means 10 × 10 and 2 is the power or example, suppose we want to add 4.236 × 104 and
gh
exponent to which 10 is raised. 3.582 × 103. We must first express both numbers so
In general, in scientific notation, a number may that they have the same exponent, n. So we can
transform 3.582 × 103 to 0.3582 × 104 and then add
ri
be expressed as :
N × 10n 4.236 × 104 + 0.3582 × 104 = (4.236 + 0.3582) × 104
where n is an exponent having positive or negative
ll = 4.5942 × 104
values and N is a single non-zero digit and lies between Similarly, to subtract these numbers,
.A
1 to 10. N is called digit term and n is called exponent. 4.236 × 104 – 3.582 × 103
For example, 1487.2 may be written as: 4.236 × 104 – 0.3582 × 104 = (4.236 – 0.3582) × 104
rs
left until there is only one non zero digit before the
decimal point. If the decimal point is moved x places, Before adding or subtracting scientific numbers,
the exponent n = x. For example, in transforming it is important to make both numbers to the same
lis
1487.2
↑ To multiply two numbers in scientific notation, we
Here exponent, n = 3 and we can write make use of the relation :
P
602, 213,700,000,000,000,000,000 = 6.022137 × 1023 (2.4 × 103) × (5.6 × 105) = (2.4 × 5.6) × 103+5
Similarly, 0.00014872 may be written as: 13.44 × 108 = 1.344 × 109
od
the number in the numerator. For example, 2.005 has four significant figures
4.74 × 1012
FG 4.74 IJ × 1012–20 3.0023 has five significant figures
6.82 × 1020
= H 6.82 K (iii) The zeros preceding to the first non-zero number
= 0.695 × 10–8 = 6.95 × 10–9 (i.e., to the left of the first non-zero number) are
2.7 × 10 −3 not significant. Such zeros indicate the position
⎛ 2.7 ⎞
.
= ⎜ –3–4
⎟ × 10 of decimal point. For example,
ed
4
5.5 × 10 ⎝ 5.5 ⎠
0.324 has three significant figures
= 0.4909 × 10–7 = 4.909 × 10–8
0.0052 has two significant figures
Significant Figures
rv
0.0003 has one significant figure
The uncertainty in the experimental or the
(iv) All zeros placed at the end or to the right of a
calculated values is mainly due to the skill and
se
number are significant provided they are on the
accuracy of the observer and limitation of the
right side of the decimal point. In fact, these
measuring instrument as explained below:
represent the accuracy or the precision of the
(i) Skill and accuracy of the observer. Suppose
re
measuring scale. For example,
three students A, B and C measure the volume of the
liquid in a cylinder. They report the following values : 0.0200 has three significant figures
Result of A = 23.4 mL ; Result of B = 23.5 mL ; 243.0 has four significant figures
ts
Result of C = 23.6 mL 243.00 has five significant figures
243.000 has six significant figures
If the correct volume of the liquid is 23.5 mL, it
gh
means that the student B has measured the volume KEY NOTE
of the liquid correctly while the students A and C have • The results obtained by counting are exact
made some error.
ri
numbers i.e., numbers without any
(ii) Limitations of the measuring instrument. uncertainty. These exact numbers have infinite
Limitation of the measuring instrument is an important ll number of significant figures. For example, in
factor which leads to uncertainty in measurement. For 2 balls or 20 eggs, there are infinite number of
.A
example, suppose the mass of an object has been significant figures because these are exact
determined to be 9.265 g. If the accuracy of the analytical numbers and may be represented by writing
balance used is 0.001 g, this means that the actual infinite number of zeros after placing a decimal
mass of the object is 9.265 + 0.001 g, i.e., it may be
rs
is uncertain and the uncertainty would be +1 in the the right side of decimal point. For example,
last digit. All measured quantities are reported in same 0.300 has three significant figures. But zeros
way by writing the certain digits and the last uncertain at the end of a number without decimal point
lis
digit. This is done in terms of significant figures. are ambiguous. For example, 2500 may have
The significant figures in a number are all two, three or four significant figures depending
the certain digits plus one uncertain digit.
ub
Thus, the number of significant figures conveys Exponential Numbers and Significant Figures
the information that except the last digit, all other In exponential notations, the numerical portion
digits are known with certainty. (all digits) represents the number of significant figures.
n
Rules for Determining the Number of For example, a number, 0.000054 is expressed as
er
number of significant figures in a particular number : the number of significant figures in the Avogadro’s
(i) All non-zero digits are significant. For example, number, 6.023 × 1023 is four.
3.132 has four significant figures Now, if we want to write five thousand with three
M
6.2316 has five significant figures. It cannot be written as 5000 because it has four
The decimal place does not determine the and not three significant figures.
number of significant figures. It must be remembered that the decimal point
(ii) Zeros between two non zero digits are significant. does not count towards the number of significant figures.
For example, For example, consider the number 54023. It has
3.01 has three significant figures five significant figures. We may write this number in
.
figures (i.e., 5) regardless of the position of the decimal
Example 4.
ed
point. The different numbers simply represent different
ways (or units) of expressing the measurement. Calculate the number of significant figures in the
following
The above rules for calculating significant figures
rv
are illustrated below : (i) 0.0025 (ii) 208 (iii) 5005
Significant (iv) 126,000 (v) 500.0 (vi) 2.0034
se
2040 4 significant figures
0.000
Not significant
Solution:
123.56 : 5 significant figures
re
Number of significant figures :
5.407 : 4 significant figures (zero is (i) 0.0025 = 2
significant) (ii) 208 = 3
ts
900.0 : 4 significant figures (all zeros are (iii) 5005 = 4
significant) (iv) 126000 = 3 (last three zeros are not significant)
gh
0.0230 : 3 significant figures (only last zero (v) 500.0 = 4
is significant) (vi) 2.0034 = 5
Calculations Involving Significant Figures
ri
The results of various measurements are to be
Example 1. added, subtracted, multiplied or divided, in most of
ll
the experiments to get desired results. But the
State the number of significant figures in each of
.A
different numbers do not have the same precision in
the following numbers :
measurement. In such cases, the final result cannot
(i) 207.35 (ii) 0.00368 (iii) 653
be more precise than the least precise number involved
(iv) 3.653 × 104 (v) 0.378.
rs
(iv) 3.653 × 104. It has four significant figures. The upto the same number of decimal places as are
number has been expressed in scientific notation. present in the term having the least number of
decimal places.
ub
(i) 0.0048 (ii) 234,000 least decimal places is 2.1. Therefore, the final result
(iii) 8008 (iv) 500.0 after adding should also have digits only up to one
n
Solution:
7.23
(i) 0.0048 = 4.8 × 10–3 2.1 ←⎯ has only one decimal place
0.312
od
7.21 Since 7.21 has two noted that in calculations, the exact numbers
12.141 decimal places, therefore, will not have any effect on the number of
0.0028 answer should be reported significant figures. The number of significant
19.3538 upto two decimal places. figures in a calculation result depends only on the
Correct answer = 19.35 numbers of significant figures in quantities having
.
(b) Subtraction of numbers. The subtraction of uncertainties. For example, we want to calculate the
ed
numbers is carried out in the same way as the addition. total mass of 8 pencils when each pencil has a mass
For example : of 3.0 g. The calculation is 3.0 × 8 = 24
25.4630 Since 3.0 has two significant figures, the result
rv
− 24.21 ←⎯⎯ has two decimal places should be 24 having two significant figures. The
number 8 is exact and does not determine the
se
1.2530 number of significant figures.
Correct answer = 1.25 (upto two decimal places)
It may be noted that if a calculation involves a
re
Some other examples are :
18.4215 5.2748 number of steps, the result should contain the
− 6.0 − 5.2721 same number of significant figures as that of
the least precise number involved, other than
ts
12.4215 0.0027
the exact numbers.
Since 6.0 has only Correct
gh
one digit after decimal answer = 0.0027 Retention of Significant Figures–Rounding off
Correct answer = 12.4 Figures
Rule 2. In multiplication or division, the final In solving problems, the final result often contains
ri
result should be reported up to the same number more digits than the number of significant figures. To
of significant figures as are present in the term retain the required number of significant figures,
ll
with the least number of significant figures. rounding off procedure is applied.
.A
(c) Multiplication of numbers. If we multiply Rules for Rounding Off
5.1028 (having five significant figures) with 1.30 (i) If the digit coming after the desired number of
(having three significant figures), the value comes out significant figures happens to be more than 5,
rs
to be 6.63364. But according to the rule, it must be the preceding number is increased by 1. For
reported with three significant figures. Therefore, the example, if we have to remove 6 in 1.386, we
he
(d) Division of numbers. If we divide 5.2765 (iii) If the digit coming after the desired number of
(having five significant figures) by 1.25 (having three significant figures happens to be 5, then the
significant figures), the result comes out to be 4.2212. preceding number is increased by 1 only in case it
P
But according to the rule, it must be reported with three happens to be odd. In case of even number, the
significant figures. Thus, the correct answer is 4.22. preceding number remains unchanged For
n
= 4.2212
Thus, to express the results to three significant
Correct answer = 4.22 (3 significant figures) figures
8.312 is rounded off to 8.31
M
KEY NOTE
8.316 is rounded off to 8.32
Calculations involving exact numbers
Exact numbers arise when we count items 8.375 is rounded off to 8.38 [rule (iii)]
©
or sometimes when we define a unit. For 8.365 is rounded off to 8.36 [rule (iii)]
example, when we say 10 pencils, we exactly mean It must be remembered that if the problem
10, not 9.9 or 10.1. Also when we say that there are involves more than one step, the ‘rounding off ’ must
12 inches in a foot, we mean exactly 12. It may be be done only in the final answer. The intermediate
steps of the calculations remain unchanged.
.
Express the results of the following calculations to
ed
(iii) 0.007837 (iv) 1.78986 × 103
the appropriate number of significant figures :
(v) 60000
Solution: 3.24 × 0.08666 (1.36 × 10 −4 ) (0.5)
(i) (ii)
rv
(i) 5.607892 = 5.608 (ii) 32.392800 = 32.39 5.006 2.6
(iii) 0.007837 = 0.007837 (iii) 0.582 + 324.65 (iv) 2.64 × 103 + 3.27 × 102
(iv) 1.78986 × 103 = 1.790 × 103 (v) 943 × 0.00345 + 101.
se
(v) 6.000 × 104 Solution:
Example 6. 3.24 × 0.08666
(i) = 0.05608 = 0.0561
re
Express the following up to three significant places : 5.006
In the calculation, the number of significant figures in
(a) the height of a man, 5 feet 9 inches in
the term 3.24 is 3, therefore, the result should have 3
centimetres (1 inch = 2.54 cm)
significant figures. Therefore, the correct answer is 0.0561.
ts
(b) one millionth of one. The number after 0 is 8 and therefore it is rounded off to 1.
(c) four thousand
(d) decimal equivalent of 2/3 (1.36 × 10 −4 ) (0.5)
(ii) = 0.2615 × 10–4
gh
Solution: 2.6
= 0.3 × 10–4.
(a) 5 ft 9 inches = 69 inch = 69 × 2.54 = 175.26 cm.
With three significant figures answer is 175 cm. The answer should have one significant figure because
ri
0.5 has one significant figure.
1
(b) One millionth of one is . It can be written with (iii) 0.582
106
3 significant figures as 1.00 × 10–6.
ll + 324.65 (Second place of decimal)
(c) four thousand = 4.00 × 103 325.232 = 325.23
.A
2 Since 324.65 contains digits up to second place of decimal,
(d) = 0.667.
3 therefore, the answer must be reported up to second place.
Example 7. (iv) 2.64 × 103 + 3.27 × 102
rs
Calculate to proper significant figures : or 2.64 × 103 + 0.327 × 103 = 2.967 × 103 = 2.97 × 103
(a) 12.6 × 11.2 (b) 108/7.2 Since 2.64 has two digits after decimal place, the answer
he
0.5785
Example 10.
er
(ii) 5 × 5.364
The mass of a piece of paper is 0.02 g and the
(iii) 0.0125 + 0.7864 + 0.0215 mass of a solid substance and the piece of paper
Solution:
od
Volume of solid = 2.16 cm3 7. Perform the following calculations upto proper
Mass 20.02 number of significant figures :
Density = = = 9.268 (i) (1.20 × 10–6) + (6.00 × 10–5) = ?
Volume 2.16
Since 2.16 has three significant figures, the answer (ii) (2.164 × 105)1/2 = ?
should also contain three significant figures. (iii) (9.13 × 10–2) (7.006 × 10–3) = ?
∴ Density up to proper number of significant figures (iv) 4.00 × 10–2 + 3.26 × 10–3 + 1 × 10–6 = ?
d.
8. Calculate the number of significant figures up to
= 9.27 g/cm3.
which the following results will be expressed :
Example 11.
ve
2.36 × 0.07251 (28.2 − 21.2) (1.79 × 106 )
Perform the following calculations and express (i) (ii)
the result to proper number of significant figures : 2.130 1.62
9. Round up the following upto three significant
er
(i) 144.3 m2 + (2.54 m × 8.4 m)
figures :
(ii) (4.05 × 102 mL) – (0.0225 × 102 mL)
(i) 34.216 (ii) 10.4107
(iii) (3.50 × 102 cm) (4.00 × 106 cm)
s
(iii) 0.04597 (iv) 2808
Solution: 10. If the speed of light is 3.0 × 108 m s–1, calculate the
re
(i) (144.3 m2) + (2.54 m × 8.4 m) distance covered by the light in 2.00 ns.
2.54 m × 8.4 m = 21.336 m2 or 21 m2 (upto 2
significant figures)
144.3 m2
s
21.0 m2 1. (a) Three (b) Two (c) Four (d) One (e) Three
ht
—––——— 2. (i) 6.02 × 10 23 (ii) 6.00 × 10 3 (iii) 32.4
165.3 m2 or 165 m2 (iv) 5.60 (v) 0.667 (vi) 1.63 × 104
(ii) (4.05 × 10 mL) − (0.0225 × 102 mL)
2
3. 4.0 × 102 g
g
4.05 × 102 mL
− 0.0225 × 102 mL 4. (i) 9.00 × 105 (ii) 2.14 × 10–6
ri
————————— (iii) 4.07 × 105 (iv) 1.00 × 10–6
4.0275 × 102 mL or 4.03 × 102 mL 5. (a) 15 (b) 2.6 × 104 (c) 1 × 10–7 (d) 0.0700 (e) 1.23
(upto second decimal ll 6. (i) 1.4 m2 (ii) 17 cm3 (iii) 4.1 m
place as in 4.05)
7. (i) 6.12 × 10–5 (ii) 4.652 × 102
A
(iii) (3.50 × 102 cm) × (4.00 × 106 cm) (iii) 6.40 × 10–4 (iv) 4.33 × 10–2
= 14.0 × 108 cm2(upto 3 significant figures)
8. (i) Three (ii) Two
.
10. 0.60m
he
1. Calculate the number of significant figures in the Hints & Solutions on page 79
following :
(a) 1.00 × 106 (b) 0.0050 (c) 1.234 DIMENSIONAL ANALYSIS
lis
(i) 6.023 × 1023 (ii) 6000 (iii) 32.362400 called factor label method or unit factor method
(iv) 5.6034 (v) decimal equivalent of 2/3 or also called dimensional analysis. This is based on
(vi) 1.6276 × 104 the relationship between different units that expresses
P
3. The density of ice is 0.921 g cm–3. Calculate the mass the same quantity. For example, suppose we want to
of a cubic block of ice which is 76 mm on each side. convert 25.6 metres to centimetres. We know that
n
100 cm
(i) 900035 (ii) 0.000002136 or 1=
1m
(iii) 406721 (iv) 0.000001
100 cm/1 m is called unit conversion factor or
od
(c)
e1.35 × 10 j (0.4)
−6
(d)
3.25 × 0.08621
of multiplication by these factors is to multiply by 1.
Now, multiply the given quantity by the
4.002 conversion factor, retaining the units of the physical
©
5.6
(e) (1.0042 – 0.0034) (1.23) quantity and that of the conversion factor in such a
6. Calculate way that all units cancel out leaving behind only the
(i) area of a square whose side is 1.2 m required units i.e., numerator should have that part
(ii) volume of a sphere whose radius is 1.6 cm which is required in the desired result.
(iii) length of a rectangle having area 10.25 m2 and
100 cm
breadth 2.5 m. Thus, 25.6 m = 25.6 m × = 2560 cm.
1m
• It may be remembered that we choose the Let us convert litre atmosphere to joule (the SI
conversion factor that has metres in the unit of energy). Let us convert 1 L atm to joules. This
denominator. can be done by multiplying with two unit factors as :
The advantage of this method is that if the F 10 m I
−3 3
equation is set up correctly, only then all the units 1 L = 10–3 m3 or 1 = GH L JK
.
will get cancelled except the required unit. If we do not 1 atm = 101.325 × 103 Pa
ed
get desired unit, then an error must have been made. 101.325 × 103 Pa
Let us consider another example. Suppose we or 1 =
atm
rv
want to convert 1.50 km3 of water into volume in litres. Applying these unit factors :
We know that
F 10 m I
–3 3
1L = 1 dm3 GH 1L JK
se
1 L atm = 1 L atm ×
So, we want to convert km3 to dm3 (litres). This
can be done in two steps as : ⎛ 101.325 × 103 Pa ⎞
× ⎜⎜ ⎟⎟
re
(i) Convert km3 to m3 ⎝ atm ⎠
(ii) Convert m3 to dm3 = 101.325 Pa m3
For converting km3 to m3, the unit factor is N
ts
1 km = 103 m Now, Pa =
m2
⎛ 103 m ⎞ N FG IJ
gh
so that 1 = ⎜⎜
⎝ 1 km ⎠
⎟⎟ ∴ 101.325 Pa m3 = 101.325
m2
m3
H K
3
⎛ 103 m ⎞ = 101.325 Nm
ri
∴ 3
1.50 km = 1.50 km × ⎜⎜ 3
⎟⎟
⎝ 1 km ⎠ = 101.325 J [∵ Nm = J]
= 1.50 × 10 m 9 3
ll Thus, 1 L atm = 101.325 J
Now 1 m = 10 dm Thus, we can easily apply several conversion
.A
10 dm factors in one step. For example, suppose we want to
or 1=
1m convert 2.50 miles into centimetres. The relationship
Applying unit factor, between various units are :
rs
⎛ 10 dm ⎞
3
1 mile = 1760 yards, 1 yard = 3 ft, 1 ft = 12 inches
Volume of water =1.50 × 109 m3 × ⎜ ⎟ and 1 inch = 2.54 cm.
he
⎝ 1m ⎠
12
= 1.50 × 10 dm 3 The unit factors are :
Both the steps can be performed in a single step as 1760 yards 3 ft 12 inches
1= ; 1= ;1= ;
lis
3 3
1 mile 1 yard 1 ft
⎛ 103 m ⎞ ⎛ 10 dm ⎞
Volume of water =1.50 km3 × ⎜⎜ ⎟⎟ × ⎜ ⎟ and 1 =
2.54 cm
1 km ⎠
⎝ ⎝ 1m ⎠
1 inch
ub
1760 yards
= 1.50 × 1012 dm3 2.50 miles × × × ×
Since cubic decimetre is equal to litres, the volume 1 mile 1 yard 1 ft 1 inch
of water is
n
Other units
1Å = 10–10 m 1 dyne = 10–5 N
1 atm = 760 mm or 760 torr 1 calorie = 4.184 J
= 101,325 Pa or Nm–2 1 erg = 10–7 J
= 1.013 × 106 dyne cm–2 1 eV = 1.6022 × 10–19 J
.
ed
1 bar = 105 Nm–2 = 105 Pa
1 mm or 1 torr = 133.322 Pa or Nm–2
rv
Solution:
(i) The S I unit of distance is metre (m)
se
Example 12. 1 mile = 1.60 kilometre = 1.60 × 1000 m
The density of vanadium is 5.96 g cm–3. Convert 1.60 × 1000 m 1.6 × 103 m
Unit factor = =
re
the density to SI units of kg m–3. 1 mile 1 mile
Solution: Density = 5.96 g cm−3 93 × 106 miles × 1.6 × 103 m
We know 1 kg = 1000 g ∴ 93 million miles =
1 mile
ts
1 kg = 93 × 1.6 × 109 m
∴ Unit factor =
1000 g = 148.8 × 109 m = 1.49 × 1011 m.
gh
1 m = 100 cm (ii) 5 feet 2 inches = 62 inches
100 cm
∴ Unit factor = 1 inch = 2.54 × 10−2 m
1m
2.54 × 10−2 m
ri
Applying unit factor for g and cm3, we get Unit factor =
FG IJ 3
1 inch
5.96 g 1 kg 100 cm
62 inches × 2.54 × 10−2 m
∴ 5.96 g cm–3 =
cm 3 ×
1000 g
×
1m H K ll 62 inches =
1 inch
.A
= 5.96 × 103 kg m–3
= 62 × 2.54 × 10−2 m
= 5960 kg/m3.
= 157.48 × 10−2 m = 1.57 m.
Example 13.
(iii) 1 mile = 1.60 km = 1.60 × 103 m
rs
Solution: We know
1 L = 1000 cm3 1 hr = 60 × 60 s = 3.6 × 103 s
1000 cm 3 3.6 × 103 s
or Unit factor = Unit factor =
lis
1L 1 hr
1 m = 100 cm 100 miles
∴ Speed =
1m hr
ub
Unit factor =
100 cm 100 miles 1.60 × 103 m 1hr
= × ×
⎛ 1000 cm3 ⎞ ⎛ 1m ⎞
3 hr 1 mile 3.6 × 103s
P
10 −10 m
Example 14. Unit factor =
er
1A °
Express each of the following in S I units :
° × 10 −10 m
(i) 93 million miles (this is the distance between 0.74 A
∴ 0.74 Å =
od
(iii) 100 miles per hour (this is the typical speed 46°C = 273.15 K + 46 K = 319.15 K
of Rajdhani Express). (vi) 1 pound = 454 × 10−3 kg
©
.
= 9.1 × 10–31 kg
ed
1 g = 15.4 grains ?
(ii) Mass of human DNA molecule = 1 fg
Solution:
1 fg = 10–15 g
Weight of ring = 0.600 carat diamond + 8.500 g gold
rv
Now 1 carat = 3.168 grains 10–15g
or Unit factor, 1 =
1 fg
3.168 grains
se
Unit factor = 1 g = 10–3 kg
1 carat
1 g = 15.4 grains
10–3 kg
re
1g or Unit factor, 1 =
Unit factor = 1g
15.4 grains
Weight of diamond = 0.600 carat
F I
ts
⎛ 10–15 g ⎞ 10–3 kg
= 0.600 carat ×
3.168 grains
×
1 gram ∴ 1 fg = 1 fg × ⎜⎜ ⎟⎟ × GH
1g JK
1 carat 15.4 grains ⎝ 1 fg ⎠
gh
= 0.123 g
= 1 × 10–18 kg
Total mass of ring = 8.500 + 0.123 = 8.623 g (b) (i) 1.4 Gm (diameter of Sun)
ri
Example 16. We know 1 Gm = 109 m
A tennis ball was observed to travel at a speed of ll 109 m
96 miles per hour. Calculate the speed of the ball or Unit factor, 1 =
1 Gm
.A
in metres per second.
Solution: F 10 m I = 1.4 × 10 m
9
Now l mile = 1.60 km = 1.60 × 103 m (ii) 40 Em (thickness of Milky Way Galaxy)
We know, 1 Em = 1018 m
he
1.60 × 103 m
Unit factor =
1 mile 1018 m
or Unit factor, 1 =
1 hr = 60 × 60 s = 3.6 × 103 s 1Em
lis
3.6 × 103 s F 10 m I = 40 × 10
18
Unit factor = ∴ 40 Em = 40 Em × GH 1 Em JK 18 m
ub
1 hr
hr
= 42.7 m s–1
11. The wavelength of a yellow line in spectrum of
Example 17. sodium atom is 5896 Å. Express it in nm.
od
(a) Convert the following in kilogram 12. How many cubic centimetres are there in 1 m3 ?
13. Convert
(i) 0.91 × 10–27 g (mass of electron)
(i) 4.86 kg L–1 to grams per millilitre
M
Solution: (a) (i) 0.91 × 10–27 g (mass of electron) wide and 50 mm deep ?
15. How many cubic centimetres are there in 100 L ?
1 g = 10–3 kg
16. Convert the following in kilogram : 5. Law of combining volumes (or Gay Lussac’s law
(i) 500 Mg (mass of jumbo jet loaded) of combining volumes).
(ii) 3.34 × 10–24 g (mass of hydrogen molecule) The first four laws of chemical combination deal
17. Express the following in the designated units : with mass relationships while the fifth law deals with
(i) 1.54 mm s–1 to pm μs–1 the volumes of the reacting gases.
.
(ii) 25 gL–1 to mg dL–1
ed
1. Law of Conservation of Mass
(iii) 25 L to m3
This law deals with the relation between the mass
(iv) 2.66 g cm–3 to μg μm–3
of reactants and the products during the chemical
rv
(v) 4.2 L h–2 to mL s–2 changes. It was postulated by a French chemist
18. Convert into metre : Antoine Lavoisier in 1789.
se
(i) 7 nm (diameter of small virus)
Law of conservation of mass states that
(ii) 41 pm (distance of nearest star)
during any physical or chemical change, the
re
19. How many seconds are there in 2 days ?
total mass of the products is equal to the
total mass of the reactants.
ts
20. Convert the following into base units :
In other words, matter can neither be created nor
(i) 28.7 pm
destroyed during any physical or chemical change.
gh
(ii) 15.15 μs
Therefore, this law is also known as law of
(iii) 25365 mg
indestructibility of matter.
ri
Experimental verification of the law of
11. 589.6 nm conservation of mass.
12. 106 cm3
ll The law can be verified with the help of Landolt’s
.A
13. (i) 4.86 g/mL (ii) 1.86 × 105 cm experiment. Landolt took the solutions of sodium
(iii) 0.692 μm, 6920 Å (iv) 9.2 × 10–6 L chloride (NaCl) and silver nitrate (AgNO3) separately
14. 4L in two limbs of a ‘H’ shaped tube (known as Landolt’s
rs
15. 105 cm3 tube). The tube was sealed and weighed. After
16. (i) 5.0 × 105 kg (ii) 3.34 × 10–27 kg weighing, the two solutions were mixed thoroughly
he
19. 172800 s
20. (i) 2.87 × 10–11m (ii) 1.515 × 10–5s After the reaction, the tube was again weighed. It
(iii) 2.5365 × 10–2 kg was observed that the weight remained practically
P
.
(i) By burning coal or candle
According to Einstein theory of relativity, mass
ed
and energy are interconvertible. The mass (m) and C + O2 ⎯⎯→ CO2
energy (E) are related as, E = mc2 where c is the (ii) By heating limestone (CaCO3)
velocity of light (3 × 108 m/sec). We know that CaCO3 ⎯⎯⎯ Heat
→ CaO + CO2
rv
chemical reactions are generally accompanied by (iii) By the action of dilute hydrochloric acid on
liberation of energy. Since energy and mass are marble pieces.
se
related to each other, this means that the energy CaCO3 + 2HCl ⎯⎯→ CaCl2 + CO2 + H2O
must be coming from the reactants. As a result,
there should be decrease in mass of the reactants. (iv) By heating sodium bicarbonate.
re
However, the mass changing into energy for 2NaHCO3 ⎯⎯⎯ Heat
→ Na2CO3 + CO2 + H2O
ordinary chemical reactions (according to relation It has been observed that each sample of carbon
m = E/c2) is extremely small because the value of c dioxide contains carbon and oxygen elements in the
ts
is very large and, therefore, there is no measurable ratio of 3 : 8 by weight.
change in mass during chemical processes. Similarly, pure water can be obtained from many
gh
However, in case of nuclear reactions and sources. Irrespective of the source, water always
radioactive disintegrations, the change in mass contains hydrogen and oxygen elements combined
is quite significant because tremendous amount of together in the ratio of 1 : 8 by weight.
ri
energy is released during these reactions. Experimental verification of the law. The law
Therefore, the law of conservation of mass does not can be easily verified in the laboratory. For example,
hold good. In these reactions, some mass gets copper oxide (CuO) can be prepared by the following
ll
converted into energy. In such cases, mass and methods :
.A
energy is totally conserved though mass and energy (i) by heating copper powder in oxygen :
are not separately conserved. Thus, law of 2Cu + O2 ⎯⎯⎯→ 2CuO
conservation of mass is modified and the modified Copper powder
rs
Copper nitrate
Proportions The weighed quantities of three samples of copper
The law of constant composition deals with the oxide as obtained above were reduced to copper
composition of various elements present in a compound.
ub
same elements combined together in the same noted. From the weight of copper left behind, the weight
definite proportion by weight. of oxygen in the samples of copper oxides was calculated.
n
Proust worked with two samples of cupric It was observed that in all the samples, the ratio
er
carbonate; one of which was naturally occurring cupric of copper and oxygen by weight has been found to be
carbonate and other was prepared in the laboratory. the same i.e., 4 : 1. This illustrates the law of constant
He found that the composition of the elements present composition.
od
in two samples of cupric carbonate was same as shown Limitations of Law of Constant Composition
below :
(i) The law of constant composition does not hold
M
cupric carbonate O is 12 : 32. But when CO2 is formed from C-14 isotope,
the ratio of C : O is 14 : 32. Thus, different isotopes of
Cupric carbonate 51.35 9.74 38.91 the same element give different mass ratio between
prepared in laboratory combining atoms.
(ii) The elements may combine in the same ratio
The above results show that irrespective of the
but the compounds formed may be different.
source, a given compound always contains same
For example, in the compounds C 2 H 5 OH and In water, 2 parts by mass of hydrogen combine
CH 3OCH 3 (both have same molecular formula with 16 parts by mass of oxygen as :
C2H6O, known as isomers) the ratio of C : H : O is Hydrogen + Oxygen ⎯⎯→ Water
24 : 6 : 16 or 12 : 3 : 8. Thus, the inverse of the law is not 2g 16 g 18g
correct. In hydrogen peroxide, 2 parts by mass of hydrogen
combines with 32 parts by mass of oxygen as :
.
3. Law of Multiple Proportions
ed
This law was proposed by Dalton in 1803. Law of Hydrogen + Oxygen ⎯⎯→ Hydrogen peroxide
multiple proportions states that 2g 32 g 34g
when two elements combine to form more than Therefore, the masses of oxgen (16 g, 32 g) which
rv
one compound, then the masses of one of the combine with fixed mass of hydrogen (2 parts) bear a
elements which combine with a fixed mass of simple ratio i.e. 16 : 32 or 1 : 2.
se
the other element are in a simple whole (ii) Nitrogen and oxygen combine to form five
number ratio. oxides namely nitrous oxide (N2O), nitric oxide (NO),
The law may be illustrated by the following nitrogen trioxide (N2O3), nitrogen tetroxide (N2O4) and
re
examples : nitrogen pentoxide (N2O5). The different weights of
(i) Hydrogen combines with oxygen to form oxygen which combine with the fixed weight of
two compounds namely water and hydrogen peroxide. nitrogen in all these oxides are calculated.
ts
Oxide Number of Number of Fixed weight Number of parts by weight
gh
parts by parts by of nitrogen of oxygen combining with
weight of weight of (14 parts) 14 parts by weight of
ri
nitrogen oxygen nitrogen
N2 O 28 16 ll 14 8
NO 14 16 14 16
.A
N2O3 28 48 14 24
N2O4 28 64 14 32
rs
N2O5 28 80 14 40
he
The ratio between the different weights of oxygen when two different elements combine separately
in different compounds which combine with the same with a fixed mass of a third element, the ratio
lis
weight of nitrogen (14 parts) is : in which they do so will be the same or some
8 : 16 : 24 : 32 : 40 simple multiple of the ratio in which they
or 1 : 2 : 3 : 4 : 5 combine with each other.
ub
This is a simple whole number and hence, supports In other words, the mass ratio of two elements A
the law. and B which combine with the fixed mass of C
P
Experimental Verification of Law of Multiple separately, is either the same or some simple whole
Proportions number multiple of the mass ratio in which A and B
Copper forms two oxides cuprous oxide (Cu2O) and
n
(CuO and Cu2O respectively) is heated in a current of Consider three elements sulphur, oxygen and
hydrogen. Both the oxides react with hydrogen hydrogen. Both sulphur and oxygen separately
od
producing metallic copper. From the weight of copper combine to form hydrogen sulphide (H2S) and water
obtained, the respective weights of oxygen in the two (H2O) respectively. They also combine with each other
compounds are obtained. Then, the different weights to form sulphur dioxide (SO2) as shown:
M
states that S O
32 SO2 32
According to the law, the ratio of weights of S and O which combine with the same weight of H will either be
same or a simple multiple of the ratio in which S and O combine with each other. This may be verified as :
In hydrogen sulphide (H2S), 2 parts by weight of hydrogen combine with 32 parts by weight of sulphur.
In water molecule (H2O), 2 parts by weight of hydrogen combine with 16 parts by weight of oxygen.
∴ The ratio of the weights of sulphur and oxygen which, combine separately with the fixed weight
(= 2 parts) of hydrogen is,
.
ed
32 : 16 or 2 : 1 ...(i)
Now, let us calculate the ratio of sulphur and oxygen in SO2.
In sulphur dioxide (SO2)
rv
Sulphur and oxygen combine in the ratio of 32 : 32 or 1 : 1 ...(ii)
The two ratios (i) and (ii) are related to each other as
se
2 1
: or 2 : 1
1 1
which are simple multiple of each other.
re
However, sulphur and oxygen also react to form sulphur trioxide (SO3) which is also in accordance with the law.
In sulphur trioxide (SO3)
Sulphur and oxygen combine in the weight ratio of 32 : 48 or 2 : 3 ...(iii)
ts
The two ratios (i) and (iii) are also related to each other as
2 2
gh
: or 3 : 1
1 3
which are simple multiple of each other.
5. Gay Lussac’s Law of Combining Volumes
ri
Gay Lussac performed a number of experiments on reactions involving gases and found that some regularity
exists between the volumes of the gaseous reactants and products. In 1808, he put forward a generalisation
known as Gay Lussac’s law of combining volumes. It states that ll
when gases react together or produced in a chemical reaction, they do so in a simple ratio by
.A
volume to one another and to the volumes of the products (if these are also gases) provided all
gases are at the same temperature and pressure.
This law may be illustrated by the following examples :
rs
he
lis
ub
It has been experimentally found that 1 volume of hydrogen reacts with 1 volume of chlorine to give
P
For example, 100 mL of hydrogen combine with 100 mL of chlorine to give 200 mL of hydrogen chloride.
od
Similarly, we observe that 2 volumes of hydrogen combine with 1 volume of oxygen to give 2 volumes of
water vapour as
M
©
Thus, the volumes of hydrogen and oxygen which In the second compound,
combine to form water is in the ratio : 2 : 1 : 2. Mass of carbon combining with 72.7 g of
2H2 (g) + O2 (g) ⎯⎯
→ 2H2O (g) oxygen = 27.3 g
2 volumes 1 volume 2 volumes. Mass of carbon combining with 1.0 g of
Similarly, it has been found that one volume of
.
27.3
ed
nitrogen combines with three volumes of hydrogen to oxygen = = 0.375 g
72.7
give two volumes of ammonia gas.
The ratio of mass of carbon combining with fixed mass
N2(g) + 3H2(g) ⎯ ⎯→ 2NH3 of oxygen (i.e., 1 g) is
rv
1 volume 3 volumes 2 volumes
0.75 : 0.375 or 2 : 1.
se
This is a simple ratio and therefore, illustrates the
law of multiple proportions.
Example 18. Example 20.
re
If 6.3 g of NaHCO3 are added to 15.0 g of CH3COOH 2.0 g of a metal burnt in oxygen gave 3.2 g of its
solution, the residue is found to weigh 18.0 g. oxide. 1.42 g of the same metal heated in steam
What is the mass of CO2 released in the reaction ? gave 2.27 g of its oxide. Which law is shown by
ts
Solution: this data?
NaHCO3 + CH3COOH ⎯ ⎯→ Solution: In the first compound,
gh
6.3 g 15.0 g 3.2 g of metal oxide contained 2.0 g of metal
CH3 COONa + H2 O + CO2 100 g of metal oxide contained metal
ri
xg 2.0
Residue =
× 100 = 62.5 g
18.0 g 3.2
Sum of the mass of reactants = Mass of NaHCO3 + ll ∴ % Metal in first compound = 62.5%
Mass of CH3COOH In the second compound,
.A
= 6.3 + 15.0 = 21.3 g 2.27 g of metal oxide contained metal = 1.42 g
Sum of the mass of products = Mass of residue +
Mass of CO2
100 g of metal oxide contained metal
rs
(18.0 + x) g 1.42
=
× 100 = 62.55 g
(where x is mass of CO2 released) 2.27
According to law of conservation of mass ∴% Metal in second compound = 62.55%
he
Mass of reactants = Mass of products Thus, the percentage of metal in metal oxide
21.3 = 18.0 + x obtained from two experiments is nearly same. Hence,
lis
or x = 21.3 – 18.0 = 3.3 g the above data illustrate the law of constant
∴ Mass of CO2 released = 3.3 g composition.
Example 19. Example 21.
ub
Carbon and oxygen are known to form two Phosphorus and chlorine form two compounds.
compounds. The carbon content in one of these The first compound contains 22.54% by mass of
P
compounds is 42.9% while in the other, it is 27.3%. phosphorus and 77.46% by mass of chlorine. In
Show that the data are in the agreement with the the second compound the percentages are 14.88
n
law of multiple proportions. for phosphorus and 85.12 for chlorine. Show that
er
Let us fix the mass of phosphorus as 1 g and find the (ii) Brown oxide
different masses of chlorine which combine with 1 g of Mass of oxygen which combines with 1.035 g of lead
phosphorus in two compounds. = 0.160 g
In the first compound, Mass of oxygen which combines with 1 g of lead
Mass of chlorine which combines with 22.54 g of 0.160
= = 0.15 g
.
phosphorus = 77.46 g 1.035
ed
The mass of chlorine which combines with 1 g of (iii) Red oxide
Mass of oxygen which combines with 1.61 g of lead
77.46
rv
phosphorus = = 3.44 g = 0.16 g
22.54
Mass of oxygen which combines with 1 g of lead
In the second compound,
se
Mass of chlorine which combines with 14.88 g of 0.16
= = 0.10 g
phosphorus = 85.12 g 1.61
re
Mass of chlorine which combines with 1 g of phosphorus The ratio of different masses of oxygen which combine
85.12 with same mass of lead (1 g) in these oxides is :
= = 5.72
14.88 0.075 : 0.15 : 0.10
ts
The ratio of the masses of chlorine which combines with 3 : 6 : 4
the fixed mass of phosphorus (1 g) in the two compounds is
gh
This is a simple ratio.
3.44 : 5.72
1 : 1.66 or 3 : 5 (approximately) Hence, the data illustrate the law of multiple
This is a simple whole number ratio. Therefore, the data proportions.
ri
is in agreement with the law of multiple proportions. Example 23.
Example 22. ll Two oxides of a metal contain 27.6% and 30% of
Three oxides of lead on analysis were found to oxygen respectively. If the formula of the first
.A
contain lead as under : compound is M3O4 , find the formula of the second
(i) 3.45 g of yellow oxide contains 3.21 g of lead. compound.
(ii) 1.195 g of brown oxide contains 1.035 g of
rs
Solution:
lead.
First oxide Second oxide
(iii) 1.77 g of red oxide contains 1.61 g of lead.
he
Solution: The amounts of lead and oxygen in three Formula of first oxide = M3O4
oxides are : Suppose the atomic weight of metal = x
(i) Yellow oxide : Mass of lead = 3.21 g
ub
∴ × 100 = 72.4
= 0.160 g 3 x + 64
er
= 0.16 g
Now in the second oxide, metal and oxygen are 70% and
Let us fix the mass of lead as 1 g and calculate the 30%. Therefore, their atomic ratio will be
different weights of oxygen which combine with 1 g of lead
M
in these oxides. M : O
56 16
Mass of oxygen which combines with 3.21 g of
1.25 : 1.875
lead = 0.24 g
or 1 : 1.5
Mass of oxygen which combines with 1 g of lead
or 2 : 3
0.24
= = 0.075 g
3.21 Therefore, formula of the compound = M2O3.
.
reciprocal proportions.
ed
nitrogen. Show that these data illustrate law of
Solution: reciprocal proportions.
In water (H2O) Solution:
rv
Mass of hydrogen = 11.11 g In water (H2O)
se
Mass of oxygen = 100 – 11.11 g Mass of hydrogen = 11.10 g
= 88.89 g Mass of oxygen = 88.90 g
In sulphur dioxide (SO2 )
re
In ammonia (NH3)
Mass of oxygen = 50 g Mass of hydrogen = 17.65 g
Mass of sulphur = 100 – 50 g = 50 g Mass of nitrogen = 82.35 g
ts
H
gh
NH3 H2 O
ri
ll N O
.A
N2O3
Let us fix the mass of oxygen as 1 g.
Now in H2O, Let us fix the mass of hydrogen as 1 g.
rs
In SO2, = 8.01 g
50 g of oxygen combine with sulphur = 50 g In NH3,
ub
0.125 : 1 or 1 : 8 ...(i)
Thus, the ratio of nitrogen and oxygen which combine
er
hydrogen sulphide is
5.89 : 94.11 or 1 : 16 ...(ii) Mass of oxygen = 63.15 g
The ratios (i) and (ii) are
©
.
in a current of hydrogen until constant weights were
observed that 2.2 g of CO 2 is released to the
ed
obtained. The water produced in each case was
atmosphere. The residue left is found to weigh 12.0
carefully collected and weighed. It was observed that
g. Show that these observations are in agreement
1 g of each oxide gave 0.1254 g and 0.2263 g of water
with the law of conservation of mass.
rv
respectively. Show that the data illustrate the law
Hint : Mass of reactants = 14.2 g : Mass of products of multiple proportion.
= 14.2 g
se
Hints & Solutions on page 79
22. Hydrogen peroxide and water contain 5.93% and
11.2% of hydrogen respectively. Show that the data DALTON ATOMIC THEORY
re
illustrates law of multiple proportions.
To provide theoretical justification to the laws of
Hint : The ratio of weights of oxygen combining chemical combination, John Dalton postulated a
with 1 g of hydrogen is 2 : 1.
ts
simple theory of matter. The basic postulates of the
23. Elements A and B combine to form two different theory are given below :
compounds as :
gh
(i) Matter is made up of extremely small indivisible
0.3 g of A + 0.4 g of B ⎯ ⎯→ 0.7 g of compound X and indestructible ultimate particles called
atoms.
ri
18 g of A + 48 g of B ⎯ ⎯→ 66 g of compound Y.
(ii) Atoms of the same element are identical in all
Show that the data illustrates the law of multiple ll respects i.e. in shape, size, mass and chemical
proportions. properties.
.A
24. Carbon combines with hydrogen to form compounds (iii) Atoms of different elements are different in all
having the following compositions :
respects and have different masses and chemical
Compound Carbon (%) Hydrogen (%) properties.
rs
proportions. molecules).
Hint : Ratio between weights of H which combine (vi) Atoms can neither be created nor destroyed during
with fixed weight of C is 4 : 2 : l. any physical or chemical change.
ub
25. 1.375 g CuO was reduced by hydrogen and 1.098 g (vii) Chemical reactions involve only combination,
Cu was obtained. In another experiment, 1.178 g of separation or rearrangement of atoms.
P
Cu was dissolved in nitric acid and the resulting Limitations of Dalton’s Atomic Theory
copper nitrate was converted into CuO by ignition.
Dalton’ s atomic theory gave a powerful initiative
n
(i) 0.32 g of sulphur on burning in air produced discoveries in the beginning of 20th century by Sir J.
224 ml of SO2 at N.T.P. J. Thomson, Rutherford, Neils Bohr and others, the
M
(ii) A metal sulphite reacts with a mineral acid to atomic theory was reviewed and modified. The main
give SO2 gas which contains 50% of sulphur and drawbacks of Dalton’s atomic theory are :
50% of oxygen.
(i) It could explain the laws of chemical combination
©
27. Copper sulphate crystals contain 25.45% Cu and by mass but failed to explain the law of gaseous
36.07% H2O. If the law of constant proportions is volumes.
true then calculate the weight of Cu required to
(ii) It could not explain why atoms of different
obtain 40 g of crystalline copper sulphate.
elements have different masses, sizes, valencies,
Ans. 10.18 g etc.
(iii) It could not explain how and why atoms of AVOGADRO’S HYPOTHESIS
different elements combine with each other to While trying to correlate Gay Lussac’s law of
form compound atoms or molecules. gaseous volumes and Dalton’s atomic theory, Berzelius,
(iv) It failed to explain the nature of forces that bind the Swedish scientist, put forward a generalisation,
atoms together in a molecule. in 1811 known as Berzelius hypothesis. According to
.
Berzelius hypothesis:
ed
(v) It gave no satisfactory explanation between the
equal volumes of all gases contain equal
ultimate particle of an element and that of a
number of atoms under similar conditions of
compound. temperature and pressure.
rv
Modern Atomic Theory However, when the Berzelius hypothesis was
The main modifications made in the Dalton’s applied to some chemical reactions it was observed
se
atomic theory as a result of new discoveries about atom that even a fraction of atoms was involved in some
are : cases. The idea of fraction of atoms is against the
concept of Dalton’s atomic theory :
re
1. Atom is no longer considered to be
indivisible. It has been found that an atom has a For example, consider the formation of
hydrochloric acid gas (HCl). It has been observed
complex structure and is made up of subatomic
experimentally that one volume of hydrogen combines
ts
particles such as electrons, protons and neutrons.
with one volume of chlorine to produce two volumes
2. Atoms of same elements may not be similar of hydrochloric acid gas. This may be expressed as:
gh
in all respects. Atoms of same element may possess Hydrogen + Chlorine ⎯→ Hydrochloric acid gas
different relative masses. For example, there are two 1 vol 1 vol 2 vol
different types of atoms of chlorine with atomic masses
ri
According to Berzelius hypothesis, equal volumes
35 a.m.u. and 37 a.m.u. respectively. Such atoms of will contain equal number of atoms. Let one volume
the same element which possess different atomic contains n atoms :
ll
masses are called isotopes. Hydrogen + Chlorine ⎯→ Hydrochloric acid gas
.A
3. Atoms of different elements may have or n atoms n atoms 2n compound atoms
similar one or more properties. There are certain or 1 atom 1 atom 2 compound atoms
atoms of different elements which possess same relative
rs
entirely different. Such atoms of different elements ½ atom of hydrogen and ½ atom of chlorine. This means
which possess same mass are called isobars. that fraction of atoms take part in chemical combination
lis
4. Atom is the smallest unit which takes part and therefore, atoms may undergo division during
in chemical reactions. Though an atom is composed chemical reactions. But this idea of fraction of atoms is
against the concept of Dalton’s atomic theory according
of subatomic particles yet it is the smallest particle
ub
combine may be fixed and integral but may Dalton’s atomic theory and Berzelius hypothesis,
not always be simple. For example, in sugar Avogadro suggested that matter consists of two kinds
n
molecule (C12H22O11), the ratio of C, H and O atoms is of ultimate particles. These are atoms and molecules.
12 : 22 : 11, which is not simple. According to him,
er
6. Atom of one element may be changed into the smallest particle of an element which may
atoms of other element. For example, atoms of or may not have independent existence and takes
od
nitrogen can be changed into oxygen atoms by part in a chemical reactions is called an atom.
interaction with α-rays. Similarly, uranium (235
92U) can The smallest particle of a substance (element
be converted into plutonium ( 239 Pu) through
M
under similar conditions of temperature and Now ½ molecule of hydrogen can exist because
pressure, equal volumes of all gases contain one molecule of hydrogen contains 2 atoms of hydrogen
equal number of molecules. and ½ molecule of hydrogen means one atom of
Mathematically, we can say hydrogen. Similarly, ½ molecule of chlorine contains
V∝N an atom of chlorine because chlorine is also a diatomic
.
where N is the number of molecules.
ed
molecule. Thus, one molecule of hydrogen chloride is
For example, if we enclose equal volumes of three formed from one atom of hydrogen and one atom of
gases hydrogen (H2), oxygen (O2) and chlorine (Cl2) in
chlorine. This is in agreement with Dalton’s atomic
different flasks of the same capacity under similar
rv
theory.
conditions of temperature and pressure, we find that
all the flasks have the same number of molecules. Applications of Avogadro’s Law
se
However, these molecules may differ in size and mass. 1. Calculation of Atomicity of Elementary
Gases
re
The number of atoms present in its one molecule
of the substance is called its atomicity. Avogadro’s
law helps in determining the atomicity of elementary
ts
gases such as hydrogen, oxygen, chlorine, etc. For
example,
gh
Atomicity of oxygen. The atomicity of oxygen
can be calculated by considering the reaction between
ri
hydrogen and oxygen to form water vapour. It has been
experimentally found that 2 volumes of hydrogen react
Fig. 10. Illustration of Avogadro hypothesis. ll
with 1 volume of oxygen to form 2 volumes of water
vapour.
.A
Gay-Lussac and Avogadro’s laws can be illustrated
as follows : Hydrogen + Oxygen ⎯→ Water vapour
2H2(g) + O2(g) ⎯→ 2H2O (g) 2 volumes 1 volume 2 volumes
rs
applied to different chemical reactions. Thus, one molecule of water contains one molecule
Formation of Hydrochloric acid gas of hydrogen (2 atoms) and ½ molecule of oxygen. The
molecular mass of water has been found to be 18 a.m.u.
P
Hydrogen + Chlorine ⎯→ Hydrogen chloride weight of one atom of oxygen. Thus, one atom is
1 vol 1 vol 2 vol present in half molecule of oxygen. Therefore,
od
.
(similar conditions) gas.
ed
According to Avogadro’s hypothesis, equal volumes 3. Determination of relationship between
of all gases under similar conditions of temperature mass and volume of gas.
rv
and pressure contain equal number of molecules. Let As discussed above,
the given volume of the gas and hydrogen contain Molar mass = 2 × Vapour density
se
n molecules at S.T.P. conditions. Mass of certain volume of gas at S.T.P.
Vapour density =2×
Mass of same volume of hydrogen at S.T.P.
re
Mass of n molecules of gas
= Mass of 1L of gas at S.T.P.
Mass of n molecules of hydrogen =2×
Mass of 1L of hydrogen at S.T.P.
ts
But mass of 1L of hydrogen gas is 0.089 g
Mass of 1 molecule of gas
=
Mass of 1 molecule of hydrogen Mass of 1L of gas at S.T.P.
gh
∴ Molar mass = 2 ×
0.089
Since 1 molecule of hydrogen contains 2 atoms of
hydrogen, 2
ri
= × Mass of 1L of gas at S.T.P.
Mass of 1 molecule of gas 0.089
Vapour density = ll = 22.4 × Mass of 1L of gas at S.T.P.
Mass of 2 atoms of hydrogen
= Mass of 22.4L of gas at S.T.P.
.A
But the ratio of the mass of one molecule of gas to
Thus, 22.4 L of any gas at S.T.P. weigh equal to
the mass of an atom of hydrogen is called molar mass
molar mass of gas expressed in grams. This is also
(discussed later).
called Gram Molecular Volume (G.M.V.).
rs
he
1
lis
Q.1. How many significant figures are there in each of the following numbers :
ub
⎛ 103 cm3 ⎞
Ans. 22.4 L × ⎜ = 22.4 × 103 cm3
L ⎟⎠
n
⎝
⎛ 10−6 m3 ⎞
er
3 3
= 22.4 × 10 cm × ⎜ ⎟
⎝ cm3 ⎠
= 22.4 × 10–3 m3
od
Q.3. What physical quantities are represented by the following units and what are their common
names ?
(i) kg m2 s–2 (ii) kg m s–2 (iii) dm3
M
Ans. (i) Energy, Joule (ii) Force, Newton (iii) Volume, Litre.
Q.4. The longest visible rays, at the end of the visible spectrum are 7.8 × 10–7 m in length. Express
©
.
ed
(a) tap water (b) air (c) soil (d) smoke (e) cloud
Ans. Tap water, air, cloud.
Q.6. Is the molar volume of CO2 same or different from CO ?
rv
Ans. Same.
Q.7. At what temperature have the Celsius and Fahrenheit reading the same numerical value ?
se
5
Ans. °C = (F − 32)
9
re
5
( x − 32)
x =
9
4x = – 160 ∴ x = – 40
ts
∴ – 40°C, –40°F.
Q.8. What is the difference between 0.006 and 6.00 × 10–3 g ?
Ans. 0.006 g contains one significant digit while 6.00 × 10–3 g contains 3 significant digits.
gh
Q.9. Classify the following substances into elements, compounds and mixtures :
(i) Milk (ii) 22 carat gold (iii) Iodized table salt (iv) Diamond (v) Smoke (vi) Steel (vii) Brass
ri
(viii) Dry ice (ix) Mercury (x) Air (xi) Aerated drinks (xii) Glucose (xiii) Petrol (xiv) Glass (xv) Wood
Ans. Elements : (iv), (ix)
Compounds : (viii), (xii)
ll
.A
Mixtures : (i), (ii), (iii), (v), (vi), (vii), (x), (xi), (xiii), (xiv), (xv)
Q.10. Given that density of water is 1 g mL–1. What is the density in SI units?
3
⎛ 1 kg ⎞ 1 ⎛ 100 cm ⎞
rs
Q.11. Is the law of constant composition true for all types of compounds? Explain why or why not?
Ans. Law of constant composition is not true for all types of compounds. It is true only for the compounds obtained
from one isotope. For example, carbon exists in two common isotopes, 12C and 14C. When CO2 is formed from 12C,
lis
the ratio of masses is 12 : 32 or 3 : 8, but when it is formed from 14C, the ratio will be 14 : 32 or 7 : 16, which is
not same as in first case.
Q.12. Which postulate of the Dalton’s atomic theory was modified after the discovery of isotopes ?
ub
Ans. According to postulates of the Dalton’s atomic theory, atoms of same element are identical in all respects i.e., in
shape, size, mass and chemical properties. However, after the discovery of isotopes, it was modified as : atoms
P
of same element may not be similar in all respects. For example, there are two types of atoms of chlorine with
atomic masses 35 amu and 37 amu respectively (called isotopes).
n
er
understand this, consider a piece of an element, say are capable of independent existence.
gold. If we break this into smaller and smaller pieces, All elements are composed of atoms. There are
M
it will become so small that we will not be able to see about 118 elements known and therefore, there are
it without a microscope. If we keep on breaking this 118 different types of atoms.
particle, we will ultimately reach a stage when this Molecule
©
particle cannot be further broken. This ultimate Just as the ultimate particle of an element is atom,
particle of an element is called an atom. Thus, the ultimate particle of a chemical compound is called
the smallest particle of an element which may a molecule. When two or more atoms of different
or may not have independent existence is elements combine, the molecule of a comound is
called an atom. formed. For example,
.
ed
ATOMIC AND MOLECULAR MASSES
molecule of Atomic Mass
rv
atom of atom of
oxygen oxygen oxygen (O2)
An atom of an element is so small particle that its
mass cannot be determined even with the help of most
se
The molecules are made up of atoms of the same sensitive balance. For example, by an indirect method,
or different elements and are capable of independent the absolute mass of hydrogen atom has been found
existence. Thus,
re
to be 1.6736 × 10–24 g. Such extremely small numbers
the smallest particle of a substance (element are very inconvenient for calculations. The difficulty
or compound) which is capable of was overcome by expressing atomic masses, as
independent existence is called a molecule.
ts
relative masses. i.e., with respect to the mass of an
Thus, a molecule contains two or more atoms. The atom of a standard substance.
properties of a substance are due to the properties of
gh
Earlier the chemists selected hydrogen atom as
its molecules. Molecules can be classified into two
the standard substance because it is the lightest of all
types :
known atoms. The atomic mass of hydrogen was taken
ri
(i) Homoatomic molecules. as one. Thus, the atomic mass of an element is the
These molecules are made up of the atoms of the number of times an atom of that element is heavier
same element. Most of the elementary gases consist of ll
than an atom of hydrogen. The relative atomic masses
homoatomic molecules. For example, hydrogen gas were referred to as the atomic weights. For example,
.A
consists of two atoms of hydrogen (H2). Similarly, it has been found that an atom of oxygen is 16 times
chlorine gas consists of two atoms of chlorine (Cl2). heavier than an atom of hydrogen. Thus, the atomic
Similarly, nitrogen and oxygen gases are N2 and O2,
mass of oxygen relative to hydrogen is 16. Later on,
rs
P
Thus, atomic mass of an element is defined as
Helium Hydrogen Ozone Phosphorus P4 the average relative mass of an atom of an
n
These molecules are made up of atoms of In other words, atomic mass is a number which
different elements. They are also classified as di, tri, expresses as to how many times an atom of the element
od
tetra...etc., depending upon the number of atoms is heavier than 1/12th of the mass of carbon atom (12C).
present. For example H 2O, HCl, CO2, NH 3, CH4, Therefore,
PCl5 are heteroatomic molecules.
M
Mass of an atom
The molecules of some common compounds are Atomic mass =
1
shown below : mass of a carbon atom (12C)
12
©
of all other atoms are given relative to this standard. Phosphorus P 31.0
Hence, one atomic mass unit is defined as Potassium K 39.0
the quantity of mass equal to 1/12th of the Silicon Si 28.1
mass of an atom of carbon (12C). Silver Ag 107.9
Mass of 1 amu has been calculated to be Sodium Na 23.0
.
1 amu = 1.66056 × 10–24 g Sulphur S 32.1
ed
Tin Sn 118.7
Mass of an atom of hydrogen = 1.6736 × 10–24 g
Zinc Zn 65.4
Thus, in terms of amu, the mass of hydrogen atom
Concept of Average Atomic Mass
rv
1.6736 × 10−24 g
= = 1.008 amu The word average has been used in the above
1.66056 × 10−24 g
definition and is very significant. It has been found
se
Thus, the atomic mass of hydrogen is that the majority of the elements occur in nature as
1.008 u. Similarly, the mass of oxygen – 16 (16O) atom mixtures of several isotopes. Isotopes are the different
is 15.995 u or 16 u; of nitrogen is 14 u and of sulphur
re
atoms of the same element possessing different atomic
is 32 u. This means that an atom of hydrogen is 1.008 masses but same atomic number. The atomic mass of
times heavier than 1/12th of the mass of carbon such an element is the average of the atomic masses
atom (12C); an atom of oxygen is 16 times heavier than
ts
1/ th of the mass of carbon atom (12C) and so on.
of the naturally occurring isotopes of the element
12 taking into account their relative abundance. This is
The atomic masses of some common elements are called average atomic mass.
gh
given in Table 7. It may be noted that only the
For example, carbon occurs as three isotopes with
approximate values of atomic masses which are
relative abundances and masses as :
normally used in the chemical calculations, are given.
ri
For example, atomic mass of hydrogen is 1.008 g but Isotope Relative Atomic
it is taken as 1. abundance (%) mass (amu)
Nowadays, atomic masses of the elements have
ll 12C 98.892 12
been determined accurately using an instrument
.A
13C 1.108 13.00335
called mass spectrometer.
14C 2 × 10–10 14.00317
Table 7. Atomic mass of a few common
elements referred to 12C = 12.0 Therefore, average atomic mass of carbon
rs
Boron B 10.8 35 × 3 + 37 × 1
Average atomic mass of chlorine = = 35.5 u
Bromine Br 79.9 3+1
Calcium Ca 40.1
P
22Ne 0.0922
Lithium Li 6.94
Manganese Mn 54.9 Average atomic mass of neon
©
.
than 1/12th of the mass of an atom of carbon. For
ed
36Ar 35.96755 0.337%
38Ar
example, a molecule of carbon dioxide is 44 atom.
37.96272 0.063%
40Ar
Therefore, the molecular mass of carbon dioxide (CO2)
39.9624 99.600%
is 44 a.m.u.
rv
Molecular mass from atomic masses. The
Solution: Average atomic mass of argon molecular mass may be calculated as the sum of the
se
35.96755 × 0.337 + 37.96272 × 0.063 + 39.9624 × 99.60 atomic masses of all the atoms in a molecule. For
= example,
100
re
= 39.948g mol–1 Molecular mass of methane (CH4)
Example 27. = At. mass of C + 4 × At. mass of H
Boron occurs in nature in the form of two isotopes = 12.001 + 4 × 1.008
ts
having atomic mass 10 and 11. What are the = 16.033 u
percentage abundances of two isotopes in a sample
For simplicity, we may take rounded values of
gh
of boron having average atomic mass 10.8 ? atomic masses of elements such as C = 12, H = 1,
Solution: Let the % abundance of 10B isotope = x O = 16, .................. so on.
Then, % abundance of 11B
isotope = 100 – x Molecular mass of carbon dioxide (CO2)
ri
x × 10 + (100 − x) × 11 = At. mass of C + 2 × At. mass of O
The average atomic mass =
100
But, average atomic mass = 10.8
ll = 12 + 2 × 16 = 44 u.
x × 10 + (100 − x) × 11 Similarly,
.A
∴ = 10.8
100 Molecular mass of H2SO4 = 2 × Atomic mass of H
or 10x + 1100 – 11x = 10.8 × 100 + Atomic mass of S + 4 × Atomic mass of O
–x = – 1100 + 1080 = 2 × 1 + 32 + 4 × 16
rs
or x = 20
= 98 u
Thus, percentage abundance; 10B = 20, 11B = 80
he
actual mass of the atom. may simply be taken as the molecular mass
Molecular Mass expressed in grams. However, this should not be
Like atoms, the actual masses of molecules are confused with the mass of one molecule of the
©
also very small and cannot be measured by actual substance in grams. For example, the mass of one
weighing. Like atomic masses, molecular masses are molecule of oxygen is only 5.32 × 10–23 g, whereas
expressed relative to the stable isotope of carbon (12C) the gram molecular mass, which is the mass in
having mass number 12. Thus, grams equal to molecular mass, is 32 g. The mass
Molecular mass of a substance may be defined of one molecule of a substance, is known as actual
as mass of the molecule.
LEARNING PLUS
Formula mass Example 28.
Calculate
Some substances such as sodium chloride,
(a) mass of 1.5 gram atoms of calcium (at. mass
potassium chloride (ionic compounds) do not have
= 40)
.
discrete molecules as their constituent units. In such
ed
(b) gram atoms in 12.8 g of oxygen (at. mass
compounds, positive (sodium) and negative (chloride) = 16).
ions are arranged in a three dimensional structure. Solution:
rv
In this structure, it has been observed that each Na+ (a) 1 gram atom of calcium = 40 g
ion is octahedrally surrounded by six Cl– ions and 1.5 gram atoms of calcium = 40 × 1.5 = 60 g
se
each Cl– ion is surrounded by six Na+ ions. The (b) 1 gram atom of oxygen = 16 g
structure of NaCl is shown below in Fig. 11. 16 g of oxygen = 1 gram atom
re
1
12.8 g of oxygen = × 12.8
16
= 0.8 gram atom
ts
Example 29.
(a) Calculate the gram molecular mass of sugar
gh
having molecular formula C12H22O11
(b) Calculate
ri
(i) the mass of 0.5 gram molecule of sugar and
(ii) Gram molecule of sugar in 547.2 gram.
ll Solution: (a) Molecular mass of sugar (C12H22O11)
Cl–
Na+ = 12 × at. mass of C + 22 × at. mass of H
.A
+ 11 × at. mass of O
= 12 × 12 + 22 × 1 + 11 × 16 = 342
Fig. 11. Structure of NaCl. ∴ Gram molecular mass of sugar = 342 g
rs
= 171 g
between various ions present. So, for these
(ii) 342 g of sugar = 1 gram molecule
compounds we use formula mass instead of
lis
2.6 gram molecules of SO2 = 64 × 2.6 = 166.4 g It may be noted that the exact value of Avogadro
(ii) Molecular mass of H2O = 2 × 1 + 16 = 18 number, NA is 6.022137 × 1023. However, in most of
∴ 18 g of water = 1 gram molecule the calculations the value 6.022 × 1023 entities/mol
1 for NA is generally used.
No. of gram molecules in 576 g = × 576 = 32
18
.
A curious question comes to our mind. Why did
ed
chemists choose the number 6.022 × 1023 for a mole ?
This seems to be very odd choice ? They would have
30. The relative abundance of various isotopes of silicon
selected simple number such as 103 (one thousand),
rv
is as Si (28) = 92.25%, Si (29) = 4.65% and Si (30)
= 3.10%. Calculate the average atomic mass of silicon. 106 (one million), 1010, 1020 or 1050. However, this
choice is based on the fact that chemists preferred the
se
31. Calculate the mass of (a) 1.6 gram atoms of oxygen
(b) 5.6 gram atoms of sulphur (c) 2.4 gram atoms of definition in terms of a quantity that can be measured
iodine. easily. We have learnt that atoms and molecules are
re
(Atomic masses : O = 16, S = 32, I = 127) so small that we cannot see them even with a powerful
32. Calculate the mass of microscope. Therefore, it is very difficult to count the
(i) 2.5 gram molecules of H2S, (ii) 3.6 gram molecules actual number of atoms or molecules in a given sample
ts
of glucose (C6H12O6)
of substance. Thus, weighing is easier than direct
33. Calculate the number of counting when the number of particles to be counted
gh
(i) gram atoms in 669.6 g of iron (at. mass = 55.8) is very large. A mole was, therefore, originally defined
(ii) gram molecules in 73.6 g of C2H5OH. in terms of mass. On this basis, chemists selected a
34. Which of the following has maximum mass
ri
standard number, which is equal to the number of
(a) 2.6 gram atoms of sulphur (b) 2.6 gram molecules atoms present in exactly 12.0 g of carbon (12C).
of sucrose (C12H22O11) (c) 2.6 g of iodine ll Thus, a mole is defined as
the amount of substance that contains as
.A
30. 28.11 u. many particles or entities (atoms, molecules
31. (a) 25.6 g (b) 179.2 g (c) 304.8 g or ions), as there are atoms in exactly 12 g
rs
molecules. Consequently, to avoid confusion, we must Mole and Gram Molecular Mass
be careful to specify the kind of particles when the The mass of 1 mole i.e., 6.022 × 1023 molecules of a
term mole is used. For example, one mole of oxygen substance is equal to its gram molecular mass or gram
atoms contains 6.022 × 1023 atoms but one mole of molecule. For example, the mass of 1 mole molecules
oxygen molecules (O 2 ) contains 6.022 × 10 23 (6.022 × 1023 molecules) of water is equal to 18 g,
.
molecules. Therefore, a mole of oxygen molecules is
ed
1 mole molecules of oxygen is 32 g and that of 1 mole
equal to two moles of oxygen atoms. However, if it is
molecules of carbon dioxide is 44 g. Thus,
not mentioned, then we should consider that it is
rv
the natural form of that substance. These days, the mass of one mole (or 6.022 × 1023 molecules) of
molecular forms of hydrogen, nitrogen and oxygen any substance in grams is equal to its gram
se
are called dinitrogen (N2), dihydrogen (H 2) and molecular mass or one gram molecule.
dioxygen (O2) respectively to avoid confusion.
One mole of molecules ≡ 6.022 × 1023 molecules
re
≡ Gram molecular mass
Example 32. Mole Concept as applied to Ionic Compounds
ts
How many molecules and atoms of sulphur are The term mole is also applied for ionic compounds
present in 0.1 mole of S8 molecules ? and one mole of formula units means 6.022 × 1023
gh
Solution: formula units. The mass of one mole formula units in
1 mol of S8 molecule = 6.022 × 1023 molecules grams is equal to formula mass expressed in grams or
∴ 0.1 mol of S8 molecules = 6.022 × 1023 × 0.1
ri
gram formula mass of the compound. Thus,
= 6.022 × 1022 molecules
One molecule of S8 contains 8 atoms of sulphur ll mass of one mole formula units (or
∴ 6.022 × 1022 molecules of S8 contain = 6.022 × 1022 × 8 6.022 × 10 23 formula units) of any ionic
= 4.816 × 1023 atoms
.A
substance in grams is equal to its gram
Example 33. formula mass.
Calculate the number of moles of iodine in a sample For example, a mole of NaCl weighs equal to 58.5 g
rs
containing 1.0 × 1022 molecules. (one gram formula mass) and contains 6.022 × 1023
Solution: 6.022 × 1023 molecules of iodine = 1 mol formula units of NaCl or 6.022 × 1023 Na+ ions and
he
The mass of 1 mol of a substance is also called its temperature and pressure. However, the molar
molar mass (M). The units of molar mass are volumes of gases change considerably with
g mol–1 or kg mol–1. Therefore, the molar mass is temperature and pressure. It has been observed that
equal to atomic mass or molecular mass expressed one mole of an ideal gas (i.e., 6.022 × 1023 molecules)
in grams depending upon whether the substance occupies 22.4 litres at N.T.P. (0 °C and 1 atm pressure).
.
contains atoms or molecules. For example,
ed
1 mole of hydrogen gas at N.T.P. = 22.4 litres
Mole in Terms of Volume 1 mole of CO2 gas at N.T.P. = 22.4 litres
Mole is also related to the volume of the gaseous This provides us with a method to determine
rv
substance. Volume of one mole of any substance is molecular masses (weights) of gases from
called its molar volume. The molar volume of solids experimentally determined density as
se
and liquids can be easily calculated if we know the Molecular mass = Molar volume × Density
molar mass and density at any given temperature and The summary of various relationships of mole may
pressure because these do not change much with be illustrated as shown below :
re
ts
gh
ri
ll 1 gram formula
mass of substance
.A
One mole of atoms = 6.022 × 1023 atoms = Gram atomic mass of the element
he
Mass of element
Moles of an element =
Atomic mass
Atomic mass
lis
Mass of compound
Moles of a compound =
Molecular mass
P
Molecular mass
Mass of one molecule =
6.022 × 1023
n
The scheme for calculation of mole, mass and number of particles can be summarised below :
od
M
©
12
Mass of one carbon atom =
6.022 × 1023
Type I. Calculation of mass of atom(s) or
molecule(s) = 1.99 × 10–23 g
1
Example 34. ∴ 1u= × 1.99 × 10–23 g = 1.66 × 10–24 g
.
12
ed
(i) Calculate the mass of an atom of silver
(atomic mass = 108). Type II. Calculation of number of molecules
present in a given volume of a gas
(ii) 1 molecule of naphthalene (C10H8)
rv
Solution: Example 37.
Calculate the number of molecules and number of
se
(i) Mass of 6.022 × 1023 atoms of silver = 108 g
atoms present in 11.2 litres of oxygen (O2) at N.T.P.
108 Solution: We know that one mole of O 2 at N.T.P.
Mass of 1 atom of silver = = 1.793 × 10–22 g
re
6.022 × 1023 occupies 22.4 litres.
(ii) Molecular mass of naphthalene (C10H8)
1
= 10 × 12 + 8 × 1 = 128 11.2 litres of O2 at N.T.P. = × 11.2 = 0.5 mole
22.4
ts
Mass of 6.022 × 1023 molecules of naphthalene = 128 g
Now, 1 mole of O2 contains = 6.022 × 1023 molecules
128
gh
Mass of 1 molecule of naphthalene = 0.5 mole of O2 contains = 6.022 × 1023 × 0.5
6.022 × 1023
= 3.01 × 1023 molecules
= 2.12 × 10–22 g
1 molecule of oxygen = 2 × 3.01 × 1023
ri
Example 35.
Calculate the mass of = 6.02 × 1023 atoms.
(i) 1 atom of 14C
ll
Example 38.
(ii) 1 molecule of N2 The mass of 94.5 mL of a gas at S.T.P. is found to
.A
(iii) 1 molecule of water be 0.2231 g. Calculate its molecular mass.
(iv) 100 molecules of sucrose (C12H22O11). Solution:
rs
6.022 × 1023
22.4 × 1000 × 0.2231
(ii) 6.022 × 1023 molecules of N2 = Gram molecular mass ∴ Molecular mass = = 52.9
94.5
ub
= 28 g
Type III. Converting mass to moles or to atoms/
Mass of 1 molecule of N2 =
28
= 4.65 × 10–23 g
molecules
6.022 × 1023
P
Example 39.
(iii) 6.022 × 1023 molecules of H2O = Gram molecular mass
Calculate the number of moles in the following :
n
= 18 g
(a) 7.85 g of iron (b) 4.68 mg of silicon (c) 65.6 μg
er
18
Mass of 1 molecule of H2O = = 2.99 × 10–23 g of carbon
6.022 × 1023
(iv) 6.022 × 1023 molecules of sucrose = Gram molecular Solution: (a) 7.85 g of iron
od
mass = 342 g
342 Atomic mass of Fe = 55.8 u
Mass of 100 molecules of sucrose = 23
× 100
6.022 × 10
M
Mass of iron
Mole of iron =
= 5.68 × 10–20 g Atomic mass
Example 36. 7.85
= = 0.141 mol
©
.
ed
Mass of carbon (ii) 7.9 mg of Ca = 7.9 × 10–3 g of Ca
Molecules of carbon =
Atomic mass Atomic mass of Ca = 40.1
65.6 × 10−6
rv
= Mass of Ca
12 Moles of Ca =
Atomic mass
= 5.47 × 10–6 mol
se
Example 40. 7.9 × 10−3
=
40.1
Calculate the number of molecules in a drop of
re
= 1.97 × 10–4 mol
water weighing 0.05 g (H = 1, O = 16).
Example 43.
Solution: Gram molecular mass of H2O = 18.0 g
Suppose the chemists had chosen 10 20 as the
ts
18.0 g of H2O contain = 6.022 × 1023 molecules number of particles in a mole. What would be the
molecular mass of oxygen gas ?
gh
6.022 × 1023 × 0.05
∴ 0.05 g of H2O contain = Solution: According to present concept of mole,
18.0
= 1.672 × 1021 molecules 6.022 × 1023 molecules of oxygen weigh = 32 g
ri
Example 41. ∴ Mass of 1 molecule of oxygen =
32
Calculate the number of atoms in each of the ll 6.022 × 1023
following : = 5.32 × 10–23 g
.A
(i) 52 mol of Ar (ii) 52 u of He (iii) 52 g of He. If the mole contained 1020 particles
The mass of one mole i.e., 1020 particles
rs
1
52 u of He = 4 × 52 = 13 atoms Gram molecular mass of Na2CO3 = 2 × 32 + 12 + 3 × 16
= 106.0 g
(iii) 4 g of He contain = 6.022 × 1023 atoms 106.0 g of Na2CO3 = 1 mol
P
4 106.0
sodium atoms
Example 42.
∴ 0.05 mol of Na2CO3 contains = 0.05 × 2 × 6.022 × 1023
od
.
Solution: (a) 1 mole of oxygen or 32 g oxygen contains = 2.99 × 10–23 mL
ed
6.022 × 1023 molecules of oxygen. Thus,
(b) Since water molecule is assumed to be spherical, its
32 g of oxygen contain molecules = 6.022 × 1023 4 3
volume is equal to πr , where r is the radius of water
rv
3
6.022 × 1023 molecule.
1000 g of oxygen contain molecules = × 1000 4 3
32 πr = 2.99 × 10–23
se
∴
3
= 1.88 × 1025 2.99 × 10−23 × 3
or r3 = (π = 3.143)
4 × 3.143
(b) We know that 1 mole of H2 at S.T.P. occupies 22.4 dm3
re
1/3
and contains 6.022 × 1023 molecules. ⎛ 2.99 × 10−23 × 3 ⎞
or r = ⎜⎜ ⎟
⎟
22.4 dm3 of hydrogen contain molecules = 6.022 × 1023 ⎝ 4 × 3.143 ⎠
ts
= (7.13 × 10 ) = 1.925 × 10–8 cm
–24 1/3
6.022 × 1023
1 dm3 of hydrogen contains molecules = = 1.925 Å
gh
22.4
ri
Chlorophyll, the green colouring matter of plants
Example 48.
contains 2.68% of magnesium by weight. Calculate ll
the number of magnesium atoms in 2.00 g of KBr (potassium bromide) contains 32.9% by weight
.A
chlorophyll (at. mass of Mg = 24). of potassium. If 6.40 g of bromine reacts with
3.60 g of potassium, calculate the number of moles
Solution: Mass of chlorophyll = 2.0 g
of potassium which combine with bromine to form
rs
32.9
6.022 × 1023 × 6.4 = 3.14 g
=
0.054 g of Mg contains = × 0.054 67.1
P
24
Atomic mass of potassium = 39 u
= 1.3 × 1021 atoms ∴ Moles of potassium which combine with bromine to form
n
3.14
Type IV. Calculation of sizes of individual atoms/ KBr = = 0.0805 mol
er
39
molecules.
Example 49.
Example 47.
od
(a) the actual volume of one molecule of water. Solution: (a) Cost of table salt per mole :
(b) the radius of a water molecule assuming to be Molecular mass of table salt (NaCl) = 23 + 35.5 = 58.5
spherical (density of water = 1 g mL–1).
©
.
36 Molecules of CHCl3 present in the drop
∴ 342 g of sugar will cost = × 342 = ` 12.312
ed
1000
5.0 × 10 −12
= ` 12 (approx) = × 6.022 × 1023 = 2.5 × 1010.
119.5
Example 50.
rv
Silver is a very precious metal and is used in Example 53.
Jewellery. One million atoms of silver weigh 1.79 Oxygen and nitrogen are present in a mixture in
se
× 10–16 g. Calculate the atomic mass of silver. the ratio of 1 : 4 by weight. Calculate the ratio
between their molecules.
Solution: Atomic mass of an element is the mass of
re
6.022 × 1023 atoms. Solution: Let wt. of given mixture = Wg
Now, 106 atoms of silver weigh = 1.79 × 10–16 g W ×1 W
Amount of O2 in mixture = = g
ts
6.022 × 1023 atoms of silver weigh 5 5
gh
= = 107.8 g Amount of N2 in mixture = = g
106 5 5
Atomic mass of silver = 107.8 W
ri
Moles of O2 =
Example 51. 5 × 32
Calculate the weight of carbon monoxide having ll W
same number of oxygen atoms as are present in 88 Molecules of O2 = × 6.022 × 1023
5 × 32
.A
g of carbon dioxide.
Solution: Molecular mass of CO2 = 12 + 2 × 16 = 44 Moles of N2 =
4W
1 mole of CO2 = 44 g 5 × 28
rs
6.022 × 10 × 88 23 5 × 28
88 g of CO2 contain =
44 Ratio of molecules of O2 and N2
= 12.044 × 1023
lis
W
Each molecule of CO2 contains 2 oxygen atoms × 6.022 × 1023 : 4W × 6.022 × 1023
5 × 32 5 × 28
No. of oxygen atoms in 12.046 × 1023 molecules of CO2
ub
= 28 g
er
24
= × 2.4088 × 10 = 112 g
6.022 × 1023
(ii) one molecule of sulphur dioxide (SO2).
Example 52. 36. Calculate the number of atoms in
M
A certain public water supply contained 0.10 parts (i) 0.5 mole atoms of carbon (C12)
per billion of chloroform (CHCl3). How many
(ii) 3.2 g of sulphur
©
.
number of rupees at the rate of 10 lakh rupees per 49. Calculate the mass of oxygen in grams present in
ed
second ? 0.1 mole of Na2CO3.10H2O.
40. One atom of an element ‘X’ weighs 6.644 × 10–23 g.
rv
Calculate the number of gram atoms in 80 kg of it.
41. Calculate the number of molecules and number of 35. (i) 6.64 × 10–23 g (ii) 1.06 × 10–22 g
se
atoms present in 5.60 L of ozone (O3) at N.T.P.
36. (i) 3.011 × 1023 atoms (ii) 6.022 × 1022 atoms
42. Calculate the number of gold atoms in 300 mg of a
gold ring of 20 carat gold (atomic mass of gold = 197, (iii) 1.445 × 1024 atoms (iv) 2.409 × 1023 atoms
re
pure gold is 24 carat). 37. 8.624 g
43. What mass in kilogram of K2O contains the same 38. (i) 1.12 L (ii) 0.224 L (iii) 4.48 L
number of moles of K atoms as are present in
ts
1.0 kg of KCl ? 39. 1.91 × 1010 years 40. 1999.5 gram atoms
44. How many molecules of water of hydration are 41. 1.506 × 1023 molecules, 4.518 × 1023 atoms.
gh
present in 630 mg of oxalic acid (H2C2O4 . 2H2O) ? 42. 7.64 × 1020 43. 0.631 kg
45. How many molecules of CO2 are present in one litre 44. 6.022 × 1021 molecules 45. 8.07 × 1018 molecules
of air containing 0.03% by volume of CO 2
ri
46. 5.019 × 1016 atoms 47. 0.426 litres
at STP ?
46. A dot ‘.’ containing carbon has 1 microgram weight. ll 48. 7.2 × 1021 49. 20.8 g.
Calculate number of carbon atoms used to make the Hints & Solutions on page 79
.A
dot.
rs
2
he
16
= 6.022 × 1022 molecules
One molecule of CH4 contains = 6 + 4 = 10 electrons
P
Q.2. How many molecules of aspirin (molar mass = 180 amu) are present in 50 mg tablet ?
er
50 × 10−3
Ans. Molecules of aspirin = × 6.022 × 1023 = 1.673 × 1020.
180
od
Q.3. Lithium exists in nature in the form of two isotopes, Li-6 and Li-7 with atomic masses 6.015 μ
and 7.016 μ and the percentages 8.24 and 91.76 respectively. Calculate the average atomic
mass of Li.
M
100
Q. 4. What is the ratio of molecules between 1 mole of H2O and 1 mole of sucrose (C12H22O11) ?
Ans. 1:1
Q. 5. If atomic weight of C and S are 12 and 32 respectively, then an atom of S is ............... time heavier
than an atom of C.
Ans. 8/3.
Q. 6. What is the mass of a mole of water containing 50% of heavy water (D2O) ?
Ans. 19 g
Q. 7. What is the mass of a molecule of carbon-14 dioxide (14CO2) ?
Ans. 7.64 × 10–23 g.
.
Q. 8. Why do atomic masses of most of the elements in atomic mass units involve fraction ?
ed
Ans. This is because atomic mass of an element is the average relative mass of its various isotopes. While taking
average, the result comes out to be fraction.
rv
Q.9. What is the approximate molecular mass of dry air containing 78% N2 and 22% O2 ?
Ans. Molecular mass = 28 × 0.78 + 32 × 0.22 = 28.88.
se
Q.10. 35Cland 37Cl are the naturally occurring isotopes of chlorine. What percentage distribution
accounts for the atomic mass 35.45 ?
re
Ans. Let fraction of 35C = x, ∴ Fraction of 37C = 1 – x
∴ Atomic mass = x (35) + (1.0 – x) 37 = 35.45
∴ x = 0.775
ts
∴ 35Cl = 77.5% and 37Cl = 22.5%
gh
Q. 11. Two bulbs A and B of equal capacity contain 10 g of oxygen (O2) and ozone (O3) respectively.
Which bulb will have
(i) larger number of molecules?
ri
(ii) larger number of oxygen atoms?
Ans. 10 g of O2 =
10
mol =
10
ll
× 6.022 × 1023 molecules = 1.88 × 1023 molecules
.A
32 32
or = 2 × 1.88 × 1023 O-atoms = 3.76 × 1023 O-atoms
10 10
rs
1 6.022
(i) 1 g Au = × 6.022 × 1023 = × 1023
197 197
6.022
(ii) 1 g Na = × 1023
23
.
ed
Q.14. Calculate the average atomic mass of chlorine using the following data.
% Natural abundance Molar mass
35Cl 75.77 34.9689
rv
37Cl 24.23 36.9659
se
34.9689 × 75.77 + 36.9659 × 24.23
Ans. Average atomic mass =
75.77 + 24.23
re
= 35.45
ts
gh
Problem 1. An alloy of metals X and Y weighs 12 g Now, 1 mol of C14 contains = 6.022 × 1023 atoms
and contains atoms X and Y in the ratio of 2 : 5. The 0.5 × 10–3 mol of C14 contains
ri
percentage by mass of X in the sample is 20. If
atomic mass of X is 40, what is the atomic mass of = 6.022 × 1023 × 0.5 × 10–3
metal Y ? ll = 3.011 × 1020 atoms
12 × 20 No. of electrons in C14 = 6
.A
Solution Mass of metal X in alloy = = 2.40 g No. of protons in 1 atom of C14 = 6
100
∴ No. of neutrons in 1 atom of C14 = 14 – 6 = 8
Mass of metal Y in alloy = 12 – 2.4 = 9.6 g (i) 3.011 × 1020 atoms of C14 contain
rs
= 8 × 3.011 × 1020
6.022 × 1023 × 2.4 = 2.409 × 1021 neutrons
Number of atoms of X =
40 (ii) Mass of one neutron = Mass of H atom
he
= 3.61 × 1022 1
=
Ratio of atoms of X and Y = 2 : 5 6.022 × 1023
lis
21
Mass of 2.409 × 10 neutrons
3.61 × 1022 × 5
No. of atoms of Y = 1
2 = × 2.409 × 1021
6.022 × 1023
ub
× 6.022 × 1023
9.025 × 1022
Solution First of all, we have to calculate the volume of
er
= 64.0 g
gas at STP by applying gas equation.
∴ Atomic mass of Y = 64 a.m.u.
Given conditions At STP
od
T1 T2
or 14 g of C14 = 1 mol
1 2.5 × 300 1 × V2
7 × 10–3 g of C14 = × 7 × 10–3 = 0.5 × 10–3 mol =
14 273 273
∴ V2 = 2.5 × 300 = 750 mL
.
22400 3
ed
= 2.01 × 1022 molecules 4
= × 3.143 × 0.001 = 0.00419 cm3
Problem 4. If atomic mass of carbon was taken as 3
100 u, then what would be the value of Avogadro’s Mass of drop of water = Volume × Density
rv
number? = 0.00419 cm3 × 1 g cm–3 = 0.0419 g
Solution Avogadro number would be the number of Now, 1 mole of water = 18 g of water contain 6.022 × 1023
se
atoms in 100 g of carbon. molecules.
12 g of carbon contain 6.022 × 1023 atoms ∴ 0.00419 g of water contain molecules
re
then 100 g of carbon would contain atoms 6.022 × 1023
= × 0.00419
18
6.022 × 1023 × 100
= = 1.40 × 1020
ts
12
= 5.01 × 1024 atoms Problem 7. 1 × 1021 molecules are removed from 280
mg of carbon monoxide. Calculate the number of
gh
Problem 5. A 0.005 cm thick coating of copper is
deposited on a plate of 0.5 m2 total area. Calculate moles of carbon monoxide left.
the number of copper atoms deposited on the plate Solution Step I. To calculate the number of moles of
ri
(density of copper = 7.2 g cm–3, atomic mass = 63.5). carbon monoxide in 280 mg.
Solution Area of plate = 0.5 m2 = 0.5 × 104 cm2 Wt. of CO in grams
ll No. of moles of CO =
Thickness of coating = 0.005 cm Gram molecular mass of CO
.A
Volume of copper deposited = 0.5 × 104 × 0.005 280 1
×
= = 0.01 mol.
= 25 cm3 1000 28
Mass of copper deposited = 25 × 7.2 = 180 g Step II. To calculate the number of moles of carbon
rs
The composition is generally expressed as the mass The formula of glucose = C6H12O6
percentage composition. Molar mass of glucose = 6 × 12 + 12 × 1 + 6 × 16 = 180
M
.
60
Molar mass of NH3 = 14 + 3 = 17
ed
Mass of oxygen = 16
The formula of NH3 shows that there are 1 N atom
16
and 3 H atoms Mass percentage of oxygen = × 100 = 26.67%
60
rv
Mass of N in 1 mol of NH3 Mass of nitrogen = 2 × 14 = 28
Mass percentage of N = × 100
Molar mass of NH3 28
se
14 Mass percentage of nitrogen = × 100 =46.66%
= × 100 = 82.35% 60
17 Mass of H = 4 × 1 = 4
Mass of H in 1 mol of NH3
re
Mass percentage of H = × 100 4
Molar mass of NH3 Mass percentage of hydrogen = × 100 = 6.67%
60
3 (ii) Copper sulphate, CuSO4.5H2O
= × 100 = 17.65%
ts
17 Molar mass of CuSO4. 5H2O = 63.5 + 32 + 4 × 16 + 5 × 18
Let us consider one more example of ethanol = 249.5
gh
having the molecular formula C2H5OH. Molar mass Mass of Cu = 63.5
of ethanol = 2 × 12 + 6 × 1 × 16 = 46 g 63.5
Mass of C = 2 × 12 = 24 Mass percentage of Cu = × 100 = 25.45
ri
249.5
24 Mass of S = 32
Mass % of C = × 100 = 52.17%
46 ll 32
Mass percentage of S = × 100 = 12.82%
Mass of H = 6 × 1 249.5
.A
6 Mass of O = 9 × 16 = 144
Mass % of H = × 100 = 13.04%
46
Mass of O atom = 16.0 144
rs
10
Mass percentage of H = × 100 = 4.01%
249.5
lis
Example 54.
Example 56.
Calculate the mass percentage composition of
Ferric sulphate is used in water and sewage
ub
Mass of Cu = 63.5
and oxygen in this compound.
63.5
× 100 = 34.64%
n
58.5 = 400
Mass percentage of iron = × 100 = 30.44%
Mass of iron = 2 × 56 = 112
od
183.3
Mass of sulphur = 2 × 32 = 64
2 × 56
Mass percentage of iron = × 100 = 28%
64 400
M
.
(vi) N2O4 (vii) H3PO4 (viii) Fe2O3 (ix) C2H2 (x) N2O5
ed
= 56 + (32 + 64) + 2 (14 + 4)
Solution: Empirical formula gives the simple whole
+ (32 + 64) + 6 (2 × 1 + 16)
number ratio of atoms in one molecule of the
= 56 + 96 + 36 + 96 + 108 = 392 compound.
rv
Mass of water = 6 × 18 = 108
(i) C6H6 = CH (ii) C6H12 = CH2
se
108
Percentage of water = × 100 = 27.55% (iii) H2O2 = HO (iv) Na2CO3 = Na2CO3
392
∴ Water of crystallisation = 27.55%. (v) B2H6 = BH3 (vi) N2O4 = NO2
re
Empirical and Molecular Formula (vii) H3PO4 = H3PO4 (viii) Fe2O3 = Fe2O3
The chemical formula may be of two types : (ix) C2H2 = CH (x) N2O5 = N2O5
(i) Empirical formula and
ts
Relation between Empirical and Molecular
(ii) Molecular formula.
Formulae
Empirical formula
gh
Molecular formula and empirical formula are
The formula which gives the simplest
related as
whole number ratio of the atoms of various
ri
elements present in one molecule of the Molecular formula = n (Empirical formula)
compound is called empirical formula. where n is a simple whole number and may have
For example, empirical formula of hydrogen
ll
values 1, 2, 3... It is equal to
.A
peroxide is HO. It represents that hydrogen and Molecular mass
n=
oxygen (i.e., H : O) are present in the ratio of 1 : 1 in Empirical formula mass
hydrogen peroxide. Similarly, empirical formula of
For example, the molecular mass of benzene is 78.
rs
Molecular formula
The formula which gives the actual Molecular mass
Thus, n =
number of atoms of various elements
lis
Step I. Divide the percentage of each element by Steps for Determination of the Molecular
its atomic mass. This gives the moles of atoms of Formula of a Compound
various elements in the molecule of the compound.
Step I. Determine the empirical formula as
Percentage of an element described above.
Moles of atoms =
.
Atomic mass of the element Step II. Calculate the empirical formula mass
ed
by adding the atomic masses of the atoms in the
Step II. Divide the result obtained in the above
empirical formula.
step by the smallest value among them to get the
rv
simplest ratio of various atoms. Step III. Determine the molecular mass by a
suitable method.
se
Step III. Make the values obtained above to the
nearest whole number and multiply, if necessary, Step IV. Determine the value of n as
by a suitable integer to make the values whole
re
Molecular mass
numbers. This gives the simplest whole n =
Empirical formula mass
number ratio.
ts
Step IV. Write the symbols of the various elements Change n to the nearest whole number.
side by side and insert the numerical value at the Step V. Multiply empirical formula by n to get
gh
right hand lower corner of each symbol. The the molecular formula.
formula thus obtained represents the empirical
Molecular formula = n × Empirical formula.
formula of the compound.
ri
ll
.A
Example 59.
The molecular mass of an organic compound is 78 and its percentage composition is 92.4% C and 7.6% H.
rs
92.4 7.7
C 92.4 12.0 = 7.7 = 1.01 1
12.0 7.6
7.6 7.6
P
H 7.6 1 = 7.6 =1 1
1 7.6
n
Molecular mass = 78
Molecular mass 78
M
∴ n = = =6
Empirical formula mass 13
Thus, molecular formula of the compound = 6 × (CH) = C6H6.
©
Example 60.
An organic compound on analysis gave the following percentage composition : C = 57.8%,
H = 3.6% and the rest is oxygen. The vapour density of the compound was found to be 83. Find out the
molecular formula of the compound.
.
ed
57.8 4.82
C 57.8 12 = 4.82 =2 4
12 2.41
3.6 3.60
rv
H 3.6 1 = 3.60 = 1.49 3
1 2.41
se
38.6 2.41
O 100 – (57.8 + 3.6) 16 = 2.41 =1 2
16 2.41
= 38.6
re
∴ Empirical formula = C4H3O2
Calculation of molecular formula
ts
Empirical formula mass = 4 × 12 + 3 × 1 + 2 × 16 = 83
Molecular mass = 2 × V.D. = 2 × 83 = 166
gh
Molecular mass
n =
ri
Empirical formula mass
166
= =2
ll
83
.A
Molecular formula = n (Empirical formula)
= 2 (C4H3O2) = C8H6O4.
Example 61.
rs
Four gram of copper chloride on analysis was found to contain 1.890 g of copper (Cu) and 2.110 g of chlorine
(Cl). What is the empirical formula of copper chloride ?
he
Solution:
Step I. Calculation of percentage of different elements
lis
Wt .of copper
Percentage of Cu in copper chloride = × 100
Wt.of copper chloride
ub
1.890
= × 100 = 47.3
4.000
P
Weight of chlorine
Percentage of Cl in copper chloride = × 100
Weight of copper chloride
n
2.110
= × 100 = 52.7
er
4.000
Step II. Calculation of empirical formula
od
52.7 1.84
Cl 52.7 35.5 = 1.48 =2 2
35.5 0.74
Example 62.
A compound contains 4.07 % hydrogen, 24.27% carbon and 71.65% chlorine. Its molecular mass is 98.96. What
are its empirical and molecular formulae ?
Solution: Step I. Calculation of empirical formula
Elements Percentage At. mass Moles of atoms Mole ratio or
.
ed
composition atomic ratio
24.27 2.02
C 24.27 12 = 2.02 =1
rv
12 2.02
4.07 4.07
H 4.07 1 = 4.07 =2
se
1 2.02
71.65 2.02
Cl 71.65 35.5 = 2.02 =1
35.5 2.02
re
∴ Empirical formula of the compound = CH2Cl
Step II. Calculation of molecular formula
ts
Empirical formula mass = At. mass of C + 2 × At. mass of H + At. mass of Cl
= 12 + 2 × 1 + 35.5 = 49.5
gh
Molecular mass = 98.96
Molecular mass 98.96
n = = = 2.
Empirical formula mass 49.5
ri
∴ Molecular formula = (CH2Cl)2 = C2H4Cl2.
Example 63.
ll
Determine the empirical formula of an oxide of iron which has 69.9% iron and 30.1% oxygen by mass.
.A
Elements At. mass Percentage Moles of atoms Mole ratio or Simplest whole
composition atomic ratio number ratio
he
69.9 1.25
Fe 56 69.9 = 1.25 =1 2
lis
56 1.25
30.1 1.88
O 16 30.1 = 1.88 = 1.50 3
ub
16 1.25
Example 64.
(a) Butyric acid contains only C, H and O. A 4.24 mg sample of butyric acid is completely burnt. It gives
n
8.45 mg of carbon dioxide and 3.46 mg of water. What is the mass percentage of each element in butyric
er
acid ?
(b) The molecular mass of butyric acid was determined by experiment to be 88u. What is the molecular
od
formula ?
Solution: (a) Calculation of mass percentage of different elements
Percentage of carbon can be calculated as :
M
CO2 ≡ C
44 mg 12 mg
44 mg of CO2 contain C = 12 mg
©
12
8.45 mg of CO2 contain C = × 8.45 mg
44
Weight of carbon
∴ Percentage of C = × 100
Weight of compound
12 8.45
= × × 100 = 54.3%
44 4.24
Percentage of hydrogen can be calculated as :
H2O ≡ 2H
.
18 mg 2 mg
ed
18 mg of H2O contain H = 2 mg
2
3.46 mg of H2O contain H = × 3.46 mg
rv
18
Weight of hydrogen
se
∴ Percentage of H = × 100
Weight of compound
2 3.46
re
= × × 100 = 9.0%
18 4.24
The sum of the percentages of C and H = 54.3 + 9.0 = 63.3%
ts
∴ Percentage of O = 100 – 63.3 = 36.7%
(b) Calculation of molecular formula
gh
Element Percentage Atomic Moles Mole Simplest
mass of atoms ratio or whole
ri
atomic ratio no. ratio
C 54.3 12.0
54.3
12.0
ll
= 4.52
4.52
2.29
= 1.97 2
.A
9.0 8.93
H 9.0 1.008 = 8.93 = 3.90 4
1.008 2.29
36.7 2.29
rs
= 44 a.m.u.
Molecular mass = 88 a.m.u.
P
Molecular mass
n =
Empirical formula mass
n
er
88
n = =2
44
od
Example 65.
M
are present in combination with oxygen as water of crystallisation. The molecular mass of the compound is
322. [At. mass : Na = 23, S = 32, H = 1, O = 16].
Solution:
Step I. Calculation of empirical formula
Element Atomic Percentage Moles Mole ratio or
mass composition of atoms atomic ratio
.
14.31 0.62
Na 23 14.31 = 0.62 =2
ed
23 0.31
9.97 0.31
S 32 9.97 = 0.31 =1
32 0.31
rv
6.22 6.22
H 1 6.22 = 6.22 = 20
1 0.31
se
69.50 4.34
O 16 69.50 = 4.34 = 14
16 0.31
re
Empirical formula of the compound is Na2SH20O14.
Step II. Calculation of molecular formula
Empirical formula mass
ts
= 2 × At. mass of Na + At. mass of S + 20 × At. mass of H + 14 × At. mass of O
= 2 × 23 + 1 × 32 + 20 × 1 + 14 × 16 = 322
gh
Molecular mass = 322
Molecular mass 322
Now, n = = =1
ri
Empirical formula mass 322
Molecular formula = (Na2SH20O14) = Na2SH20O14
ll
Since all hydrogen atoms are present in combination with oxygen as water therefore, it means that all the 20 hydrogen
atoms must be combined with 10 atoms of oxygen to form 10 molecules of water of crystallisation. The remaining
.A
4 oxygen atoms are present with the compound. Thus,
Molecular formula of the compound = Na2SO4.10H2O
rs
Example 66.
A welding fuel gas contains carbon and hydrogen only. Burning a small sample of it in oxygen gives 3.38 g
he
of carbon dioxide, 0.690 g of water and no other products. A volume of 10.0 L (measured at S.T.P.) of this
welding gas is found to weigh 11.6 g. Calculate (i) empirical formula (ii) molar mass of the gas, and
(iii) molecular formula.
lis
Solution: CO2 ≡≡ C
44 g 12 g
ub
12
Mass of carbon = × 3.38 = 0.92 g
44
H2O ≡ H2
P
18 g 2 g
2
Mass of hydrogen = × 0.690 = 0.077
n
18
er
0.922
Percentage of C = × 100 = 92.2%
(0.922 + 0.077)
od
0.077
Percentage of H = × 100 = 7.7%
(0.922 + 0.077)
M
92.2 7.7
C 92.2 12 = 7.7 =1
12 7.7
7.77 7.7
H 7.77 1 = 7.7 =1
1 7.7
Empirical formula = CH
(ii) Calculation of molar mass
10.0 L of gas at S.T.P. weigh = 11.6 g
11.6
22.4 L of gas at S.T.P. weigh = × 22.4
10.0
= 26 g mol–1.
.
(iii) Calculation of molecular formula
ed
Empirical formula mass = 12 + 1 = 13
Molecular mass = 26
rv
Molecular mass
n =
Empirical formula mass
se
26
= =2
13
∴ Molecular formula = 2 (CH)
re
= C2H2
Example 67.
A crystalline salt on being rendered anhydrous loses 45.6% of its weight. The composition of anhydrous salt
ts
is : Al = 10.5%, K = 15.1%, S = 24.8% and O = 49.6%. Calculate the formula of anhydrous salt and crystalline
salt (At. mass of Al = 27, K = 39, S = 32, O = 16).
gh
Solution:
Step I. Calculation of empirical formula
ri
Element Percentage Atomic Moles Mole Simplest
of element mass of atoms ratio whole no. ratio
ll
15.1 0.38
.A
K 15.1 39 = 0.38 =1 1
39 0.38
10.5 0.38
Al 10.5 27 = 0.38 =1 1
rs
27 0.38
24.8 0.775
he
49.6 3.1
lis
The simplest atomic ratio of K : Al : S : O is 1 : 1: 2 : 8. The empirical formula of the anhydrous salt is : KAlS2O8.
ub
Step III. To calculate the amount of water associated with 258 parts by weight of compound
Let the weight of hydrated salt be 100 g
n
If the weight of anhydrous salt is 54.4 gm, the weight of water in the crystalline salt = 45.6 gm.
If the weight of anhydrous salt in 258 gm, the weight of water in the crystalline salt
45.6
M
= × 258 = 216.2 g
54.4
Step IV. To calculate the number of molecules of water of crystallisation
It is clear from the above calculations that 216.2 parts by weight of water are present in combination with 258 parts be
©
.
meaning measure. It is based on the fact that the
ed
dichromate.
51. An organic compound containing carbon, hydrogen stoichiometric coefficients in a chemical equation can
and oxygen gave the following percentage be interpreted as the number of moles of each substance
composition : (reactants or products). For example, consider the reaction
rv
C = 40.68%, H = 5.08% between nitrogen and hydrogen to form ammonia as :
The vapour density of the compound is 59. Calculate N2(g) + 3H2(g) ⎯⎯→ 2NH3 (g)
se
the molecular formula of the compound. The stoichiometric coefficients in the equation
52. (i) The elemental composition of butyric acid was indicate that 1 molecule of nitrogen combines with 3
found to be 54.2% C, 9.2% H and 36.6% O. Determine molecules of hydrogen to form 2 molecules of ammonia.
re
its empirical formula.
(ii) The molecular mass of butyric acid was N2(g) + 3H2(g) ⎯⎯→ 2NH3 (g)
determined by an experiment to be 88 u. What is its 1 molecule 3 molecules 2 molecules
ts
molecular formula ? Multiplying by 6.022 × 1023
the entire equation,
53. An oxide of nitrogen contains 30.43% of nitrogen. The we get
molecular weight of the compound is equal to
gh
1 × 6.022 × 1023 3 × 6.022 × 1023 2 × 6.022 × 1023
92 a.m.u. Calculate the molecular formula of the
compound. molecules molecules molecules
54. Calculate the empirical and molecular formula of the This means
ri
compound having the following percentage 1 mol 3 mol 2 mol
composition :
Na = 36.5%, H = 0.8%, P = 24.6%, O = 38.1% ll Taking molar masses into consideration
The molecular mass of the compound is 126 a.m.u. 28.0 g 3×2 2 × 17
A
Also name the compound. = 6.0 g = 34 g
55. A crystalline salt when heated becomes anhydrous
The interpretation of the coefficients as the
and loses 51.2% of its weight. The anhydrous salt on
.
and the crystalline salt. Molecular weight of the first step is writing the chemical equation for the
anhydrous salt is 120. reaction and then balancing it. The balanced chemical
56. Calculate the empirical formula of a mineral which equation gives the stoichiometric coefficients which
lis
57. Determine the empirical formula of a compound A chemical equation is a statement of a chemical
having percentage composition as : reaction in terms of the symbols and formulae of the
Iron = 20% ; sulphur = 11.5% ; oxygen = 23.1% and species involved in the reaction. The chemical
water molecules = 45.4%
P
Reactants Products
53. N2O4 The substances which react among themselves to
54. The empirical and molecular formula is, Na2HPO3, bring about the chemical changes are known as
©
(i) It must be consistent with the experimental facts. Removal of Drawbacks of Chemical Equations
i.e., a chemical equation must represent a true The chemical equation can be made more
chemical change. If a chemical reaction is not possible informative by the following modifications;
between certain substances it cannot be represented (a) The physical states of the reactants and
by a chemical equation. products can be specified. For this, we use s (for solids),
l (for liquids) and g (for gases). Sometimes, the letter
.
(ii) It should be balanced i.e., the total number of
ed
atoms on both sides of the equation must be equal. aq is given to represent that the given substance has
been dissolved in excess of water. For example,
(iii) It should be molecular i.e., the elementary
gases like hydrogen, oxygen, nitrogen, etc., must be Zn(s) + 2HCl(aq) ⎯⎯→ ZnCl2 (aq) + H2(g)
rv
represented in molecular form as H2, O2, N2, etc. (b) In order to express the strength of acid or base
Information Conveyed by a Chemical Equation used in the reaction, the words conc. for concentrated
se
and dil. for dilute are used before the formula of the
A chemical equation conveys both qualitative and acid or base. For example,
quantitative informations.
Zn (s) + dil. 2HCl (aq) ⎯⎯→ ZnCl2(aq) + H2(g)
re
Qualitatively, a chemical equation tells the names
(c) The conditions of the reaction such as
of the various reactants and the products.
temperature, pressure, catalyst, etc., may be written
Quantitatively, a chemical equation represents : on the arrow between the reactants and products. For
ts
(i) the relative number of reactant and product example,
species (atoms or molecules) taking part in the Fe /Mo, 723 K
N2(g) + 3H2(g) ⎯⎯⎯⎯⎯⎯ → 2NH (g)
gh
reaction. 600 atm 3
(ii) the relative number of moles of the reactants The above equation indicates that the reaction
and products. between hydrogen and nitrogen has been carried out
ri
in the presence of Fe/Mo (catalyst) at 723 K and 600
(iii) the relative masses of the reactants and
atm pressure.
products.
ll (d) Heat changes taking place during the reaction
(iv) the relative volumes of gaseous reactants and
may be expressed in the equation. For example,
products.
.A
C(s) + O2 (g) ⎯→ CO2(g) + 394 kJ (Heat is evolved)
Thus, the chemical equation for the reaction :
N2(g) + O2(g) ⎯→ 2NO (g) – 180.5 kJ (Heat is absorbed)
CH4(g) + 2O2(g) ⎯⎯→ CO2(g) + 2H2O (g)
(e) The formation of a precipitate, if any, in a
rs
gives the following information : reaction can be expressed by writing the word ppt. or
(i) One molecule of CH 4 (g) reacts with by an arrow pointing downward. For example,
he
2 molecules of O2(g) to give one molecule of CO2(g) AgNO3(aq) + NaCl(aq) ⎯→ AgCl(s)↓ + NaNO3(aq)
and 2 molecules of H2O(g). (ppt)
(ii) One mole of CH4(g) reacts with 2 moles of Similarly, the evolution of a gas in a chemical
lis
O2(g) to give one mole of CO2(g) and 2 moles of reaction can be indicated by an arrow pointing upward.
H2O(g). For example,
(iii) 16 g of CH4(g) reacts with 2 × 32 g of O2(g) to Zn(s) + 2HCl (aq) ⎯→ ZnCl2(aq) + H2(g)↑
ub
give 44 g of CO2 and 2 × 18 g of H2O(g). (f) The distinction between slow and fast reactions
(iv) 22.4 L of CH4(g) reacts with 44.8 L of O2(g) to can be made by writing word slow and fast on the
arrow head.
P
The chemical equations, as such do not give us the reaction is occurring in the forward as well as in
er
Backward
(ii) The concentration of the reactants and products. Balancing of Chemical Equation
(iii) The conditions such as temperature, pressure or A correct chemical equation must be in accordance
catalyst, etc., which affect the reaction. with the law of conservation of mass according to which
M
(iv) The heat changes accompanying the reaction i.e., the total mass of the reactants must be equal to the
whether heat is evolved or absorbed during the total mass of the products. In other words, the number
©
H2 + O2 ⎯→ H2O
But this equation has two atoms of O on the
reactants side but only one atom of O on the products Example 68.
side. Therefore, it does not obey law of conservation of Balance the equation
mass. By choosing suitable coefficients, the equation C3H8 + O2 ⎯⎯→ CO2 + H2O
may be balanced as :
.
Solution: Converting the oxygen to elementary
ed
2H2 + O2 ⎯→ 2H2O state
The balancing of a chemical equation means to C3H8 + O ⎯⎯→ CO2 + H2O
equalise the number of atoms of each element on both unbalanced equation
rv
sides of the equation. There are many methods to
Balancing carbon atoms
balance a chemical equation. Some of these
C3H8 + O ⎯⎯→ 3CO2 + H2O
se
methods are :
(i) Hit and Trial method or Trial and Error method. Balancing hydrogen atoms
(ii) Partial equation method C3H8 + O ⎯⎯→ 3CO2 + 4H2O
re
(iii) Oxidation number method Balancing oxygen atoms
(iv) Ion-electron method. C3H8 + 10 O ⎯⎯→ 3CO2 + 4H2O
The first two methods are discussed below while
ts
Changing to molecular form
the other two methods will be taken up in Unit 8
C3H8 + 5O2 ⎯⎯→ 3CO2 + 4H2O
(Redox reactions).
gh
Hit and Trial Method or Trial and Error Example 69.
method Balance the following skeleton equation :
This method involves the following steps :
ri
As2O3 + SnCl2 + HCl ⎯⎯→ SnCl4 + As + H2O
(i) Write the symbols and formulae of the reactants
Solution: The skeleton equation is :
and the products in the form of a skeleton ll
equation. As2O3 + SnCl2 + HCl ⎯⎯→ SnCl4 + As + H2O
.A
(ii) If any elementary gas appears, change it to its Balancing arsenic and oxygen atoms by multiplying As by
atomic state. 2 and H2O by 3
(iii) Select the formula containing maximum number As2O3 + SnCl2 + HCl ⎯⎯→ SnCl4 + 2As + 3H2O
rs
is balanced in the final equation. This is a better method than the hit and trial
For example, let us consider the equation : method. It involves the following steps :
Fe + H2O ⎯⎯→ Fe3O4 + H2
P
(iii) Converting to molecular form, original reactants and the final products.
3Fe + 4H2O ⎯⎯→ Fe3O4 + 4H2 (iv) Finally, the partial equations are added to get
is a balanced equation. the final equation.
©
Similarly, some other balanced equations are : Let us illustrate this method with an example of
4Fe (s) + 3O2 (g) ⎯⎯→ 2Fe2 O3 (s) reaction between copper and nitric acid. The reaction
P4 (s) + 5O2 (g) ⎯⎯→ P4O10 (s) is written as :
2Mg (s) + O2 (g) ⎯⎯→ 2MgO (s) Zn + HNO3 ⎯⎯→ Zn (NO3)2 + N2O + H2O
(i) The probable partial equations for the above (iii) Multiply the first partial equation by 4 in
reaction are : order to cancel out H atoms in the two equations since
Zn + HNO3 ⎯⎯→ Zn(NO3)2 + 2H they do not appear in the final equation. Finally add
HNO3 + H ⎯⎯→ N2O + H2O the two equations
(ii) Balance the partial chemical equations [Zn + 2HNO3 ⎯⎯→ Zn(NO3)2 + 2H] × 4
.
2HNO3 + 8H ⎯⎯→ N2O + 5H2O
ed
separately by hit and trial method as
Zn + 2HNO3 ⎯⎯→ Zn(NO3)2 + 2H ———————————————————————
2HNO3 + 8H ⎯⎯→ N2O + 5H2O 4Zn + 10HNO3 ⎯⎯→ 4Zn(NO3)2 + N2O + 5H2O
rv
se
re
58. Balance the following equations by hit and trial method :
(a) KMnO4 + HCl ⎯⎯→ KCl + MnCl2 + H2O + Cl2
ts
(b) H2S + SO2 ⎯⎯→ S + H2O
(c) K2Cr2O7 + H2SO4 ⎯⎯→ K2SO4 + Cr2(SO4)3 + H2O + O2
gh
(d) KMnO4 + KOH ⎯⎯→ K2MnO4 + O2 + H2O
(e) Mg3N2 + H2O ⎯⎯→ Mg(OH)2 + NH3
ri
(f ) Al4C3 + H2O ⎯⎯→ Al(OH)3 + CH4
( g) FeS2 + O2 ⎯⎯→ Fe2O3 + SO2
(h) KMnO4 + H2S + H2SO4 ⎯⎯→
ll
KHSO4 + MnSO4 + S + H2O
.A
(i) C3H8 (g) + O2 (g) ⎯⎯→ CO2(g) + H2O (l)
59. Balance the following equations by partial equation method :
rs
.
Example 71.
ed
concept, balancing of chemical equations and care in
How many moles of iron can be made from Fe2O3
the conversion of units.
by the use of 16 mol of carbon monoxide in the
The problems based upon chemical equations may
rv
following reaction :
be classified as : Fe2O3 + 3CO ⎯⎯→ 2Fe + 3CO2
(i) Mole to mole relationships. In these
se
Solution: The balanced equation is :
problems, the moles of one of the reactants/ Fe2O3 + 3CO ⎯⎯→ 2Fe + 3CO2
products is to be calculated if that of other 16 mol ?
re
reactants/products are given. 3 mol of CO are used to make = 2 mol Fe
(ii) Mass-mass relationships. In these problems, 2
the mass of one of the reactants/products is to 16 mol of CO are used to make = × 16
ts
3
be calculated if that of the other reactants/
= 10.67 mol
products are given.
gh
(iii) Mass-volume relationship. In these Type II. Mass to mass or Mole to mass
problems, mass or volume of one of the relationship
ri
reactants or products is calculated from the Example 72.
mass or volume of other substances. Calculate the amount of water in grams produced
(iv) Volume-volume relationship. In these llby combustion of 16 g methane (CH4).
problems, the volume of one of the reactants/ Solution: The balanced chemical equation is
.A
products is given and that of the other is to be CH4(g) + 2O2 (g) ⎯⎯→ CO2(g) + 2H2O(g)
calculated. 1 mol 2 mol
The main steps for solving such problems are : 16 g 36 g
rs
16 g ?
(i) Write down the balanced chemical equation.
From the equation, it is clear that 16 g of methane
he
(ii) Write down the moles or gram atomic or gram produces 36 g of water.
molecular masses of the substances whose quantitites Example 73.
are given or have to be calculated. In case, there are
How many moles of methane are required to
lis
two or more atoms or molecules of a substance, multiply produce 22g of CO2(g) after combustions ?
the mole or gram atomic mass or molecular mass by
the number of atoms or molecules.
ub
44
= 0.5 mol.
Type I. Mole to mole relationship Example 74.
M
.
Solution: The balanced chemical equation is :
ed
5g ?
H2 + Cl2 ⎯⎯→ 2HCl
87 g of MnO2 react with 146.0 g of HCl
1 mol 1 mol
146.0 2g 2 × 35.5 = 71 g
rv
5 g of MnO2 will react with = ×5
87 2 g of H2 react with = 71 g Cl2
= 8.39 g
se
71
0.40 g of H2 would react with = × 0.4 = 14.2 g Cl2
Example 75. 2
What mass of calcium oxide will be obtained by Now,
re
heating 3 mol of CaCO3 ? H2 + Cl2 ⎯⎯→ 2HCl
Solution: The balanced chemical equation is : 1 mol 2 mol
CaCO3 ⎯⎯→ CaO + CO2 2g 73 g
ts
1 mol 1 mol 2 g of hydrogen give HCl = 73 g
40 + 16
73
= 56 × 0.40 = 14.6 g
gh
0.40 g of hydrogen will give HCl =
2
3 mol ?
1 mol of CaCO3 on heating gives CaO = 56 g Example 79.
ri
3 mol of CaCO3 on heating gives CaO = 56 × 3 What weight of zinc would be required to produce
= 168 g enough hydrogen to reduce completely 8.5 g of
Example 76. llcopper oxide to copper ?
Oxygen is prepared by the catalytic decomposition Solution: (i) Calculation of H2 required to reduce
.A
of potassium chlorate (KClO3). Decomposition of CuO to Cu.
potassium chlorate gives potassium chloride (KCl) CuO + H2 ⎯⎯→ Cu + H2O
and oxygen (O2). If 2.4 mol of oxygen is needed for 1 mol 1 mol
rs
2
2 × (30 + 35.5 + 3 × 16) 8.5 g of CuO require H2 for reduction to copper = × 8.5
79.5
= 245 g
= 0.214 g
ub
? 2.4 mol
3 mol of O2 is produced by decomposition of KClO3 (ii) Calculation of weight of zinc required to
produce H2
= 245 g
P
3
? 0.214 g
er
into its oxide (Fe3O4) by the action of 14.4 g of steam 0.214 g of H2 would be produced from zinc = × 0.214
2
on it.
= 6.955 g
Solution: The balanced chemical equation is:
M
3 × 56 4 × 18
= 168 g = 72 g containing 16.2 g of calcium bicarbonate per
Now 72 g of steam react with = 168 g of Fe hundred litre. (Atomic masses Ca = 40, C = 12,
168
O = 16, H = 1)
14.4 g of steam will react with = × 14.4 Solution: (i) Calculation of the weight of calcium
72
bicarbonate present.
= 33.6 g
.
Type III. Mass – Volume relationship
ed
(ii) Calculation of the quantity of lime required.
The equation involved is : Example 82.
Ca(HCO3)2 + Ca(OH)2 ⎯⎯→ 2CaCO3 + 2H2O A 1.20 g of an impure sample of sodium chloride,
rv
40 + 2 + 24 + 96 40 + 32 + 2
on treatment with excess of AgNO3 solution gave
= 162 = 74
2.40 g of silver chloride as a precipitate. Calculate
se
162 ?
the percentage purity of the sample.
∴ One mole of calcium bicarbonate requires one mole
of calcium hydroxide Solution: NaCl + AgNO3 ⎯⎯→ AgCl + NaNO3
re
or 162 g of Ca(HCO3)2 require = 74 g of Ca(OH)2 23 + 35.5 108 + 35.5
= 58.5 = 143.5
74 ? 2.40 g
∴ 9720 g of Ca(HCO3)2 will require = × 9720 g of
ts
162 143.5 g of AgCl is obtained from 58.5 g of pure NaCl
Ca(OH)2 2.40 g of AgCl is obtained from pure NaCl
= 4440 g of Ca(OH)2
gh
58.5
= × 2.40 = 0.978 g
Hence the quantity of lime required = 4440 g = 4.44 kg 143.5
Now, 1.20 g of NaCl contain pure NaCl = 0.978 g
Example 81.
ri
A 1.84 g mixture of calcium carbonate and 100 g of NaCl contain pure NaCl =
0.978
× 100 = 81.5 g
magnesium carbonate upon heating gave 0.96 g ll 1.20
residue. Calculate the percentage composition of the Percentage purity of sample = 81.5%
.A
mixture (At. mass of Ca = 40, Mg = 24, C = 12 and
Example 83.
O = 16).
A 2.0 g of sample containing Na2CO3 and NaHCO3
Solution: Calcium carbonate and magnesium carbonate
loses 0.248 g when heated to 300°C, the temperature
rs
100 g 56 g mixture ?
Solution: The balanced chemical equation is :
MgCO3 ⎯⎯⎯
heat
→ MgO + CO2
lis
Wt. of MgCO3 in the mixture = (1.84 – x) g CO2 and H2O will escape as gases at 300°C. Therefore,
Now, 100 g of CaCO3 give CaO = 56 g loss in weight would correspond to the weight of H2O
and CO2.
P
56
x g of CaCO3 give CaO = ×xg Total mass of CO2 + water lost by heating 168 g of
100 NaHCO3 = (18 + 44) = 62 g
n
100 84
1.344
4704 x + 7360 – 4000 x = 8064 ∴ Percentage of Na2CO3 = × 100 = 67.2%
2
©
704 x = 704
704 The equations involving volume are based upon the fact
x = =1 g
704 that 1 mol of all gases at N.T.P. occupy 22.4 L. The
Weight of CaCO3 in the mixture = 1 g problems involving different conditions of temperature
and pressure are described in next unit.
Weight of MgCO3 in the mixture = 1.84 – 1 = 0.84 g
.
of CO. = 2.6 L
ed
1 Example 87.
CO + O ⎯⎯→ CO2
2 2 What volume of oxygen at S.T.P. is required to effect
complete combustion of 200 cm3 of acetylene and
rv
1 mol 0.5 mol
22.4 L at N.T.P. what would be the volume of carbon dioxide
22.4 L of CO at N.T.P. require O2 = 0.5 mol formed ?
se
0.5 Solution: The balanced chemical equation is :
112 L of CO at N.T.P. require O2 = × 112 2C2H2 + 5O2 ⎯⎯→ 4CO2 + 2H2O
22.4
re
= 2.5 mol 2 mol 5 mol 4 mol
This O2 is to be obtained by heating KClO3. 2 × 22400 cm3 5 × 22400 cm3 4 × 22400 cm3
2KClO3 ⎯⎯→ 2 KCl + 3O2 (i) Calculation of volume of O2 at S.T.P. required to
ts
effect complete combustion of 200 cm 3 of
2 mol 3 mol
acetylene.
2 (39 + 35.5 + 3 × 16) = 245 g
2 × 22400 cm3 of acetylene require O2 for complete
gh
3 mol of O2 is produced from KClO3 = 245 g
combustion = 5 × 22400 cm3
245 × 25 200 cm3 of acetylene require O2 for complete
2.5 mol of O2 is produced from KClO3 =
ri
3 5 × 22400
= 204.167 g combustion = × 200
2 × 22400
Example 85.
ll = 500 cm3 at S.T.P
(ii) Calculation of volume of carbon dioxide formed
What volume of air at N.T.P. containing 21% of
.A
oxygen by volume is required to completely burn 2 × 22400 cm3 of acetylene produce CO2
1000 g of sulphur containing 4% incombustible = 4 × 22400
matter ? ∴ 200 cm3 of acetylene produce CO2
rs
1000 × 96
= = 960 g
100 60. How much iron can be obtained by the reduction of
ub
.
(iii) Both will react completely because it is
ed
60. 0.7 kg stoichiometric mixture. No limiting reagent.
(iv) 2.5 mol of B will react with 2.5 mol of A and hence
61. 916.2 g B is limiting reagent.
rv
62. 462.6 g (v) 2.5 mol of A will react with 2.5 mol of B. Hence A
63. 0.25 mole is limiting reagent.
se
64. 2.27% Example 89.
65. 8.47 g 50.0 kg of N2(g) and 10.0 kg of H2(g) are mixed to
66. 90.5 ml produce NH3(g)
re
Hints & Solutions on page 79 (a) Calculate the NH3(g) formed.
(b) Identify the limiting reagent in this reaction if
ts
LIMITING REACTANT any.
Solution:
In many cases, the substances in a mixture are
gh
not present in exactly the same amount as required Moles of N2 =
Mass in g
by the balanced chemical equation. In such situations, Molar mass
one reactant is in excess over the other. The reactant 50.0 × 103
ri
= = 1.786 × 103 mol
which is present in lesser amount gets consumed after 28
sometime and after that no further reaction takes place ll Mass in g
Mole of H2 =
even though the other reactant is present. Thus, Molar mass
.A
the reactant which gets completely 10 × 10 3
= = 5.0 × 103 mol
consumed in a reaction is called limiting 2
reactant. N2 (g) + 3H2(g)
2NH3 (g)
rs
1 mol 3 mol
The concentration of the limiting reactant limits According to the equation 1 mol of N2(g) requires
the amount of products formed. The other reactants 3 mol of H2(g) for the reaction.
he
present in quantities greater than those needed to 1.786 × 103 mol of N2(g) will require H2(g)
react with the quantity of the limiting reagent present = 3 × 1.786 × 103
would be left unreacted. It is also called excess = 5.36 × 103 mol
lis
reagent. For example, in the reaction But moles of H2 present = 5.0 × 103 mol
Therefore, N2 is in excess and dihydrogen is limiting
2H2 + O2 ⎯⎯→ 2H2O
reagent
if the reaction mixture contains 2 mol of H2 and 2 mol
ub
= 3.3 × 103 × 17
er
.
O2 = 1 mol mass ?
ed
1.50 Solution: The balanced chemical equation is
∴ 1.50 mol of H2 require O2 = = 0.75 mol
2
But moles of O2 actually present = 0.906 mol N2 (g) + 3H2 (g)
2NH3 (g)
rv
Therefore, O2 is in excess and H2 is the limiting
reactant. 2.00 × 103
Moles of N2 = = 71.43 mol
(ii) Calculation of maximum amount of H2O formed. 28
se
Now, 2 mol of H2 form H2O = 2 mol
∴ 1.50 mol of H2 form H2O = 1.50 mol 1.00 × 103
Moles of H2 = = 500 mol
Thus, the maximum amount of H2O formed = 1.50 mol 2
re
(iii) Calculation of the amount of one of the According to above equation, 1 mol of N2 require 3 mol
reactants which remains unreacted i.e., O2 of H2.
Number of moles of O2 added = 0.906 mol ∴ 71.43 mol of N2 will require H2 = 3 × 71.43
ts
Number of moles of O2 used up = 0.75 mol
= 214.29 mol
∴ Number of moles of O2 unreacted = 0.906 – 0.75
But moles of H2 actually present = 500 mol
gh
= 0.156 mol.
∴ H2 is in excess and will remain unreacted and N2 is
Example 91. limiting reagent.
If 20.0 g of CaCO3 is treated with 20.0 g of HCl,
ri
(i) 1 mol of N2 react with H2 to form NH3 = 2 mol
how many grams of CO2 can be produced according
71.43 mol of N2 will react with H2 to form NH3
to the reaction :
CaCO3 (s) + 2HCl (aq) ⎯⎯→ CaCl2 (aq) + H2O(l)
ll =
2
× 71.43
+ CO2(g)
.A
1
= 142.86 mol
Solution: In this problem, CO2 can be calculated by Mass of NH3 produced = 142.86 × 17 = 2428.6 g
rs
using either of the reactants CaCO3 or HCl. So, first of (ii) Hydrogen will remain unreacted.
all we are to calculate the limiting reactant. (iii) Moles of H2 initially taken = 500 mol
he
CaCO3 (s) + 2HCl (aq) ⎯⎯→ CaCl2 (aq) + H2O(l) Moles of H2 reacting = 214.29 mol
+ CO2(g) Moles of H2 remaining unreacted = 500 – 214.29
1 mol 2 mol = 285.71 mol
lis
But amount of HCl actually present = 20.0 g present in lesser amount and is called solute while
Therefore, CaCO3 is limiting reactant and HCl is excess the other present in excess is called the solvent. The
reactant.
n
44
20 g of CaCO3 will produce CO2 = × 20 the solution. For example, if WA is the mass of
100 component A and WB is the mass of component B in a
= 8.80 g
solution, then
©
Example 92.
Dinitrogen and dihydrogen react with each other Mass percentage of A =
WA
× 100
to produce ammonia according to the following WA + WB
chemical equation : This can be expressed as w/w. For example, a 10%
N2 (g) + 3H2 (g) ⎯⎯→ 2NH3 (g) (w/w) solution of sodium chloride means that 10 g of
(i) Calculate the mass of ammonia produced if sodium chloride is present in 90 g of water so that the
.
percentage is defined as the volume of the
as ‘M’. Thus, a solution which contains one gram mole
ed
component per 100 parts by volume of the
of the solute dissolved per litre* of the solution, is
solution. For example, if VA and VB are the volume of
regarded as one molar solution. For example, 1M
two components A and B respectively in a solution, then
Na2CO3 (molar mass = 106) solution has 106 g of the
rv
Volume percentage of A solute present per litre of the solution.
Volume of A
=
se
× 100 Moles of solute
Volume of A + Volume of B Molarity =
Volume of solution in litres
This may be expressed as v/v. Sometimes, we It is convenient to express volume in cm 3 or mL
re
express the concentrations as weight/volume. For so that
example, a 10% solution of sodium chloride (w/v)
Moles of solute
means that 10 g of sodium chloride are dissolved in Molarity = × 1000
100 mL of solution. Volume of solution (in mL or cm 3 )
ts
Parts per million. When a solute is present in (∵ 1 litre = 1000 mL)
very minute amounts (trace quantities), the
gh
Thus, the units of molarity are moles per
concentration is expressed in parts per million
litre (mol L–1) or moles per cubic decimetre
abbreviated as ppm. It is the parts of a component
(mol dm–3) . The symbol M is used for mol L–1 or mol
per million parts of the solution. It is expressed as:
ri
dm–3 and it represents molarity.
Mass of component A
ppm A = × 106 If nB moles of solute are present in V mL of
Total mass of solution
For example, suppose a litre of public supply water
ll
solution, then
.A
nB
contains about 3 × 10 –3 g of chlorine. The mass Molarity = × 1000
percentage of chlorine is : V
Moles of solute can be calculated as :
rs
3.0 × 10−3
Mass percentage of chlorine × 100 Mass of solute
1000 Moles of solute =
Molar mass of solute
he
= 3 × 10–4
Molarity is one of the common measures of
The parts per million parts of chlorine is :
expressing concentration which is frequently used in
3 × 10−3 × 106
lis
Moles of NaOH =
Example 93. Molar mass
If 11 g of oxalic acid are dissolved in 500 mL of 2.46
solution (density = 1.1 g mL–1), what is the mass %
M
= = 0.0615 = 0.0615
40
of oxalic acid in solution ?
Solution : 11 g of oxalic acid are present in 500 mL of Moles of NaOH
©
.
nA + nB
ed
Weight of solvent in kg Thus, if the mole fraction of one component of a binary
solution is known, that of the other can be calculated. For
Moles of solute
or = × 1000 example, the mole fraction xA is related to xB as :
rv
Mass of solvent in gram xA = 1 – xB or xB = 1 – xA
Thus, the units of molality are moles per It may be noted that the mole fraction is
kilogram i.e., mol kg–1. It is represented by the independent of temperature.
se
symbol m. Example 96.
If nB moles of solute are dissolved in W grams of A solution is prepared by adding 60 g of methyl
re
solvent, then alcohol to 120 g of water. Calculate the mole
nB fraction of methanol and water.
Molality = × 1000 T Solution : Mass of methanol = 60 g
W
ts
60
Moles of methanol = = 1.875
KEY POINT 32
gh
(Molar mass = 32)
From the discussion of molarity and molality, it is
Mass of water = 120 g
evident that in molarity we consider the volume of
the solution while in molality, we take the mass of 120
Moles of water = = 6.667
ri
the solvent. Therefore, the two are never equal. 18
Molality is considered better for expressing (Molar mass = 18)
the concentration as compared to molarity ll Total number of moles = 1.875 + 6.667
because the molarity changes with = 8.542
.A
temperature because of expansion or Mole fraction of methanol =
1.875
= 0.220
contraction of the liquid with temperature. 8.542
However, molality does not change with 6.667
Mole fraction of water = = 0.780.
rs
Equivalent mass
Moles of solute
Molality = × 1000 Like molarity, normality of a solution also
Mass of solvent in gram changes with temperature.
od
0.15
= × 1000 = 0.313 m. Example 97.
479.3
Calculate the normality of solution containing
M
.
Molar mass
Normality = Molarity × Moles of solute
ed
solute (xB) =
Equivalent mass Moles of + Moles of
For acids,
solute solvent
Normality = Molarity × Basicity
rv
where basicity is the number of H+ ions that a Mole fraction of solute + Mole fraction of solvent = 1
molecule of an acid can give in solution. or Mole fraction of solvent = 1 – Mole fraction of solute
se
For bases,
Normality = Molarity × Acidity
where acidity is the number of OH – ions that a
re
molecule of a base can give in solution. Example 98.
For example, A solution is prepared by dissolving 18.25 g of
1 M H2SO4 solution = 2 N H2SO4 solution NaOH in distilled water to give 200 ml of solution.
ts
(basicity = 2) Calculate the molarity of the solution.
1 M H3PO4 solution = 3 N H3PO4 solution Solution: Moles of NaOH =
18.25
gh
(basicity = 3) 40
(Molecular mass of NaOH = 40)
and 1 M Ca(OH)2 solution = 2 N Ca(OH)2 solution
Vol. of solution = 200 mL
(acidity = 2)
ri
18.25 × 1000
Alternatively, 1 N H2SO4 solution Molarity = = 2.28 M
40 × 200
= 0.5 M H2SO4 solution
Example 99.
ll
(basicity = 2) and so on. How many grams of Na2CO3 should be dissolved to
.A
It may be noted that these days the terms normality or make 100 cm3 of 0.15 M Na2CO3 solution ?
equivalent weight are not commonly used. Solution: 1000 cm3 of 0.15 M Na2CO3 contain Na2CO3
Sometimes, the term formality is also used. It gives = 0.15 mole
rs
as discrete molecules. In such cases, we do not use the A solution is prepared by dissolving 2 g of substance
term mole for expressing the concentration. The sum A in 18 g of water. Calculate the mass percentage of
of the atomic masses of various atoms constituting the
solute.
P
Mass of water = 18 g
er
Mass of solution = 2 + 18 = 20 g
• Molarity of a solution changes with temperature
due to accompanied changes in volume of the Mass of A
Mass percent of A = × 100
od
Example 101.
REMEMBER Calculate the concentration of nitric acid in moles
©
.
Moles of HNO3 Moles of solute
∴ Molarity = × 1000 Molarity =
b g × 1000
ed
Volume of solution (in mL) Vol. of solution in mL
69/63 Moles of NaCl
= × 1000 = 15.44 M 0.50 = × 1000
rv
70.92 250
Example 102. 0.50 × 250
∴ Moles of NaCl = = 0.125 mol
A sample of NaNO3 weighing 0.38 g is placed in a 1000
se
250 ml volumetric flask. The flask is then filled with Gram molecular mass of NaCl = 23 + 35.5 = 58.5 g
water to the mark on the neck. What is the molarity ∴ Mass of NaCl solution in grams
of the solution ?
re
= Moles of NaCl × Molecular mass
Solution: Mass of NaNO3 dissolved = 0.38 g = 0.125 × 58.5 = 7.3125 g
Molecular mass of NaNO3 = 23 + 14 + 48 = 85 Example 106.
ts
0.38 Calculate the number of Cl– ions in 100 ml of 0.001
Moles of NaNO3 dissolved =
85 M HCl solution.
gh
Volume of solution = 250 mL Solution : Since HCl is a strong acid, it ionises
0.38 / 85 completely so that the concentration of HCl is equal to
Molarity = × 1000 that of Cl– ions.
250
ri
–
= 0.018 M 1000 ml of 0.001 M HCl solution contains Cl = 0.001
Example 103. mole
ll –
100 ml of 0.001 M HCl solution contains Cl
What is the concentration of sugar (C12H22O11) in
mol L–1 if its 20 g are dissolved in enough water to 0.001 × 100
.A
= 1000 = 1 × 10–4 mole
make a volume upto 2 L ?
Solution:Mass of sugar = 20 g No. of Cl– ions = 6.022 × 1023 × 1.0 × 10–4
rs
126
solution ? Volume of solution = 250 mL
er
2.5 Basicity
∴ Moles of CH3OH = 0.25 × 2.5 = 0.625 moles 126
= 63
=
Mass of CH3OH = 0.625 × 32 = 20 g 2
©
.
Mass of glucose = 2.82 g
A solution of glucose in water is labelled as 10%
ed
Moles of glucose =
2.82
(Molar mass = 180) (w/w). The density of the solution is 1.20 g mL–1.
180 Calculate
Mass of water = 30 g
rv
(i) molality
Moles of glucose
Molality = × 1000 (ii) molarity, and
se
Mass of water
(iii) mole fraction of each component in solution.
2.82 × 1000
= = 0.522 m. Solution : 10% (w/w) solution of glucose means that 10 g
180 × 30 of glucose is present in 100 g of solution or in 90 g of water.
re
(b) Calculation of mole fraction
(i) Calculation of molality
2.82
Moles of glucose = = 0.0157 Mass of glucose = 10 g
180
10
ts
30 Moles of glucose = = 0.0556
Moles of water = = 1.67 180
18
(Molar mass of glucose = 180)
gh
0.0157
Mole fraction of glucose = = 0.009. Mass of water = 90 g
0.0157 + 1.67
Moles of glucose
1.67 Molality = × 1000
ri
Mole fraction of water = = 0.991. ∴
Mass of water
0.0157 + 1.67
Example 109. ll 0.0556
= × 1000 = 0.618 m.
Calculate the molarity of pure water (density of 90
water = 1 g mL–1)
.A
(ii) Calculation of molarity
Solution : Density of water = 1 g mL–1 Moles of glucose = 0.0556
Mass of 1000 mL of water = Volume × Density Mass
rs
18 1.20
Now, 55.55 moles of H2O are present in 1000 ml or 1 L
Moles of glucose
of water Molarity = × 1000
Vol. of solution
lis
∴ Molarity = 55.55 M.
0.0556
Example 110. = × 1000 = 0.667 M.
83.3
A solution is 25% water, 25% ethanol and 50%
ub
5.0556
25
Moles of water = = 1.388 5.0
18 Mole fraction of water = = 0.989.
(·.· Molar mass of H2O = 18) 5.0556
od
Example 112.
25
Moles of ethanol = = 0.543 A sugar syrup of weight 214.2 g contains 34.2 g of
46
sugar (C12H22O11). Calculate :
M
· ·
( . Molar mass of C2H5OH = 46)
(i) molal concentration, and
50 (ii) mole fraction of sugar in the syrup.
Moles of acetic acid = = 0.833
©
60
Solution : (i) Weight of sugar syrup = 214.2 g
(·.· Molar mass of CH3COOH = 60)
Weight of sugar in syrup = 34.2 g
Total number of moles = 1.388 + 0.543 + 0.833 Weight of water in syrup = 214.2 – 34.2 = 180.0 g
= 2.764
34.2
1.388 Moles of sugar = = 0.1
Mole fraction of water = = 0.502. 342
2.764
(Molar mass = 342)
.
18
ed
0.1 M1 V1 M2 V2
Mole fraction of sugar =
10 + 0.1
= 0.0099. =
Initial Final
Molarity Equation 0.015 M × V1 = 5.25 × 10–3 M × 250 mL
rv
To calculate the volume of a definite molarity
of a solution required to prepare solution of other 5.25 × 10−3 × 250
∴ V1 =
se
molarity, we can use the relation 0.015
M1V1 = M2V2 = 87.5 mL
where M1 = Initial molarity,
Example 114.
re
M2 = Molarity of new solution
V1 = Initial volume, 250 mL of 1.5 M solution of sulphuric acid is
V2 = Volume of new solution diluted by adding 5L of water. What is the
ts
This is known as molarity equation and is molarity of the diluted solution ?
commonly used to calculate the molarity of solution Solution: Volume of diluted solution = 5000 + 250
after dilution. = 5250 mL
gh
Applying molarity equation
Normality Equation
Like molarity equation, to calculate the volume M1 V1 M2 V2
=
ri
of a definite normality of a solution required to Initial Final
prepare solution of other normality, we can use the
1.5 M × 250 mL = M2 × 5250 mL
relation : ll
N1V1 = N2V2 1.5 × 250 mL
.A
∴ M2 =
where N1 is the initial normality of a solution 5250 mL
having volume V1 and N2 is the normality of new = 0.0714 M
solution having volume V2. This is known as Example 115.
rs
normality equation.
What volume of 10 M HCl and 3 M HCl should be
Molarity equation and normality equation are
mixed to get 1L of 6 M HCl solution?
he
commonly used to
Solution : Suppose volume of 10 M HCl required to
(i) calculate the molarity or normality of a
prepare 1 L of 6 M HCl = x litre
solution after mixing two or more solutions.
lis
1 1 =
3 3
(a) If V1 mL of a solution of molarity M1 are mixed 10 M HCl 3 M HCl 6 M HCl
10x + 3 – 3x = 6
M1V1 + M2V2 = M3(V1 + V2)
3
(b) If V2 mL of a solution of molarity M2 is to be 7x = 3 or x = = 0.428 L
n
7
prepared from a solution of molarity M1 (concentrated)
er
then volume of solution (say V1) of molarity M1 ∴Vol. of 10 M HCl required = 0.428 L = 428 mL
required can be calculated from the molarity equation:
Vol. of 3 M HCl required = 1 – 0.428 = 0.572 L
od
M1 × V1 = M2 × V2
= 572 mL
M 2 × V2
or V1 = Example 116.
M1
M
Mass
=
100 Example 118.
Volume of solution = = 84.03 mL
Density 1.19 The density of 3 M solution of NaCl is 1.25 g mL–1.
Moles of HCl =
38
= 1.04 Calculate the molality of the solution.
36.5
Solution : 3 M solution means that 3 moles of NaCl are
1.04 × 1000 present in 1 L solution.
.
Molarity = = 12.38 M
ed
84.03 Mass of NaCl in 1 L solution = 3 × 58.5
(ii) The volume of this solution required to make = 175.5 g
1.0 L of 0.10 M HCl can be calculated by applying ..
( . Molecular mass of NaCl = 58.5)
rv
molarity equation as
Density of solution = 1.25 g mL–1
M 1 V1 M V
= acid
2 2
Mass of 1 L solution= 1000 × 1.25 = 1250 g
se
acid1 2
Mass of water in solution = 1250 – 175.5
12.38 M × V1 = 0.10 M × 1.0 L
= 1074.5 g
0.10 × 10
.
re
∴ V1 = = 0.00808 L or = 8.08 cm3
12.38 Moles of solute
Molality = × 1000
Example 117. Mass of solvent (in g)
Commercially available sulphuric acid
ts
contains 93% acid by mass and has a density of
3
= × 1000
1.84 g mL–1. Calculate (i) the molarity of the 1074.5
gh
solution (ii) volume of concentrated acid required = 2.79 m
to prepare 2.5 L of 0.50 M H2SO4 Calculating molality from mole fraction
Solution: (i) 93% H2SO4 by mass means that 93 g of Molality is the moles of solute dissolved per 1000 g
ri
H2SO4 is present in 100 g of solution. of solvent whereas mole fraction is the ratio of moles
Mass of solute present to the total moles of solute and
100 ll
Vol. of 100 g of solution= Density = = 54.3 mL solvent present in the solution. Mole fraction also
1.84
tells us that out of total of 1 mol of solution, xsolute
.A
93 mol of solute and (1 – x) mol of solvent are present.
Moles of H2SO4 = = 0.95 mol
98 Molality can be calculated from the mole
rs
2
solvent calculate molality.
17.5 M × V1 = 0.5 M × 2.5 L Example 119.
0.5 × 2.5 What is the molality of a solution of methanol in
ub
∴ V1 =
17.5 water in which the mole fraction of methanol is
= 0.071 L = 71 mL 0.25?
P
Calculate the total mass of 1 L (1000 mL) of Calculating mole fraction from molality
solution by using density of the solution. Mole fraction is the ratio of moles of solute to the
Calculate mass of solute from the molarity (no.
©
.
Example 120. the reactants and their concentrations. These are
ed
illustrated below:
What is the mole fraction of the solute in 2.5 m
aqueous solution?
rv
Solution: 2.5 m aqueous solution means that 2.5 moles
of solute are present in 1000 g of water. Thus, Example 121.
se
250 ml of 0.5 M sodium sulphate (Na2SO4 ) solution
Moles of solute = 2.5 mol are added to an aqueous solution containing 10.0 g
1000 of BaCl2 resulting in the formation of white
re
Moles of water = = 55.6 precipitate of BaSO4 . How many moles and how
18
many grams of barium sulphate will be obtained ?
2.5
Mole fraction of solute = Solution: The balanced chemical equation is :
2.5 + 55.6
ts
BaCl2 (aq) + Na2SO4 (aq) ⎯⎯→ BaSO4 (s) + 2NaCl (aq)
= 0.043
Let us first calculate moles of Na2SO4 and BaCl2
gh
0.5 M solution of Na2SO4 means that 0.5 mol of Na2SO4
are present in 1000 mL of solution.
67. A sample of NaOH weighing 0.38 g is dissolved in
ri
1000 mL of solution contain Na2SO4 = 0.5 mol
water and the solution is made to 50.0 mL in a
volumetric flask. What is the molarity of the 0.5
ll 250 mL of solution contain Na2SO4 = × 250
resulting solution ? 1000
= 0.125 mol
.A
68. The density of 3 molal solution of NaOH is
1.110 g mL–1. Calculate molarity of the solution. 10
Moles of BaCl2 in solution =
(NCERT Exemplar Problem) 208
rs
71. 0.63 g of oxalic acid, (COOH)2 .2H2O are dissolved Now, according to the equation,
ub
in 500 ml of solution. Calculate the molarity of the 1 mol of BaCl2 produces BaSO4 = 1 mol
solution. 0.048 mol of BaCl2 produces BaSO4 = 1 × 0.048
72. How many moles of NaOH are contained in 27 mL = 0.048 mol
P
Example 122.
What volume of 0.6 M HCl has enough
67. 0.19 M
od
70. 15.8 M NaOH (aq) + HCl (aq) ⎯⎯→ NaCl (aq) + H2O
Let us first calculate the moles of HCl and NaOH
71. 0.01 M
©
.
0.6 mol of HCl of 0.6 M concentration are present in Now, according to the equation, 1 mol of CaCO3(s)
ed
= 1000 mL requires 2 mol of HCl(aq). Hence, for 10 mol of
0.0125 mol of HCl of 0.6 M concentration are present in CaCO3(s), moles of HCl(aq) required would be
1000 × 0.0125
rv
= 2
0.6 = × 10 = 20 mol
1
= 20.83 mL But we have only 0.19 mole of HCl (aq)
se
Example 123. ∴ HCl (aq) is limiting reagent.
What volume of 0.250 M HCl (aq) is required to 2 mol of HCl (aq) form CaCl2 = 1 mol
re
react completely with 22.6 g of sodium carbonate
according to the reaction : 1
0.19 mol of HCl (aq) would form CaCl2 = × 0.19
2
Na2CO3 (s) + 2HCl (aq) ⎯⎯→ 2NaCl (aq)
s
= 0.095 mol
+ H2O + CO2
Mass of CaCl2 formed = 0.095 × 111 = 10.54 g
t
Solution: The balanced chemical equation is :
gh
Na2CO3 (s) + 2HCl (aq) ⎯⎯→ 2NaCl (aq) + H2O
1 mol 2 mol + CO2
ri
74. What mass of solid AgCl is obtained when 25 ml of
22.6
Moles of Na2CO3 present = = 0.213 0.068 M AgNO3 reacts with excess of aqueous HCl?
106
ll75. What volume of 0.34 M KOH is sufficient to react
(Molar mass = 2 × 23 + 12 + 3 × 16 = 106) with 20 ml of 0.15 M H2SO4 solution ?
.A
Now, according to the equation, 76. Calculate the volume of 1.00 mol L –1 aqueous
1 mol of Na2CO3 requires moles of HCl = 2 sodium hydroxide that is neutralised by 200 mL of
2.00 mol L–1 aqueous hydrochloric acid and the
0.213 mol of Na2CO3 requires moles of HCl
rs
0.426 mol of 0.250 M HCl would be present in be added for completing the reaction. How many
1000 millilitre of 0.150 M NaOH should be added for this
= × 0.426 = 1704 mL. requirement ?
ub
0.250
78. 500 mL of 0.250 M Na2SO4 solution is added to an
Example 124.
aqueous solution of 15.00 g of BaCl2 resulting in the
Calcium carbonate reacts with aqueous HCl to formation of the white precipitate of BaSO4. How
P
+ H2O (l) CaCO3 (s) + 2HCl (aq) ⎯⎯→ CaCl2 (aq) + CO2 (g)
What mass of CaCl 2 will be formed when + H2O (l)
od
250 mL of 0.76 M HCl react with 1000 g of What mass of CaCO3 is required to react completely
CaCO3 ? Name the limiting reagent. Calculate with 25 mL of 0.75 M HCl ?
the number of moles of CaCl 2 formed in the
M
1000
Moles of CaCO3 = = 10 mol Hints & Solutions on page 79
100
.
Q. 1. Give two examples of molecules having molecular formula same as empirical formula.
ed
Ans. CO2, CH4.
Q. 2. Give an example of molecule in which
rv
(i) Ratio of molecular formula and empirical formula is 6 : 1.
(ii) Molecular weight is two times of the empirical formula weight.
se
(iii) The empirical formula is CH 2O and ratio of molecular formula weight and
empirical formula weight is 6.
re
Ans. (i) C6H6 (ii) H2O2 (iii) C6H12O6
Q.3. Write the empirical formula of (i) glucose (ii) sucrose.
ts
Ans. (i) CH2O (ii) C12H22O11
Q. 4. What are the the SI units of molarity ?
gh
Ans. mol dm–3
Q. 5. 1.615 g of anhydrous ZnSO4 was left in moist air. After a few days its weight was found to be
ri
2.875 g. What is the molecular formula of hydrated salt ?
(At. masses : Zn = 65.5, S = 32, O = 16, H = 1)
ll
Ans. Molecular mass of anhydrous ZnSO4
.A
= 65.5 + 32 + 4 × 16 = 161.5
1.615 g of anhydrous ZnSO4 combines with water
rs
1.260
161.5 g of anhydrous ZnSO4 combine with = × 161.5 = 126 g
1.615
lis
126
No. of moles of water = =7
18
Formula : ZnSO4.7H2O.
ub
Q. 6. Density of water 1000 kg m–3 corresponds to ........ g cm–3. Complete the statement.
Ans. 1.
P
Ans. 0.50 m Na2CO3 means that 0.50 moles of Na2CO3 are dissolved in 1000 g of water.
er
0.50 M Na2CO3 solution means that 0.50 moles of Na2CO3 are dissolved in 1000 mL of solution.
Q. 8. Calculate the amount of carbon dioxide that could be produced when
od
.
(i) Which is the limiting reagent ? (i) B is the limiting reagent.
ed
(ii) Calculate the amount of C formed. (ii) 4 moles of B give 3 mole of C.
Ans. According to the equation, 2 moles of A require 4 3
rv
moles of B for the reaction. ∴ 6 moles of B will give = × 6 = 4.5 mol of C .
4
se
re
Problem 8. 1.0 g of a mixture of carbonates of
p1 V1 p V
calcium and magnesium gave 240 mL of CO2 at N.T.P. Applying = 2 2
ts
T1 T2
Calculate the percentage composition of the mixture.
p1 = 0.92 atm p2 = 1 atm
gh
Solution CaCO3 ⎯⎯→ CaO + CO2 V1 = 1.20 L V2 = ?
100 g 22400 mL T1 = 273 K T2 = 273 K
MgCO3 ⎯⎯→ MgO + CO2
ri
p1 V1T2 0.92 × 1.20 × 273
84 g 22400 mL ∴ V2 = =
p2 T1 1 × 273
Let CaCO3 present in the mixture = x g ll = 1.104 L
Now 100 g of CaCO3 give CO2 = 22400 mL Let mass of Al in alloy = xg
.A
22400 × x Mass of Mg in alloy = (1 – x) g
x g of CaCO3 will give CO2 =
100 ⎯⎯→
= 224 x g 2Al + 6HCl 2AlCl3 + 3H2
2 × 27 3 × 22.4
rs
84 24 g 22.4 L
= 266.7 (1 – x) 54 g of Al give H2 at N.T.P. = 67.2 L
Total CO2 evolved at N.T.P. = 240 mL
lis
67.2 × x
Now, 224 x + 266.7 (1 – x) = 240 x g of Al give H2 at N.T.P. =
54
224x – 266.7x = 240 – 266.7
Similarly, 24 g of Mg give H2 at N.T.P.= 22.4 L
ub
– 42.7x = – 26.7
22.4
26.7 (1 – x) g of Mg will give H2 at N.T.P.= × (1– x)
x = = 0.625 24
42.7
P
Total H2 liberated
Wt. of CaCO3 = 0.625 g
Wt. of MgCO3 = 0.375 g
67.2 x
+
22.4 1 − x b g
= 1.104
n
54 24
Solving for x, we get
er
0.625
% CaCO3 = × 100 x = 0.5486 g
1
∴ Mass of Al in the alloy = 0.5486 g
= 62.5
od
0.5486
0.375 × 100 % of Al = × 100 = 54.86
% MgCO3 = 1
1
% of Mg in alloy = 100 – 54.86 = 45.14.
M
= 37.5
Problem 10. A mixture of sodium iodide and
Problem 9. 1.0 g of an alloy of aluminium and sodium chloride when treated with sulphuric acid
©
magnesium when treated with excess of dil. HCl form gave sodium sulphate equal to the weight of the
magnesium chloride and aluminium chloride and
original mixture. Find the percentage composition
hydrogen collected over mercury at 0°C has a volume
of the mixture.
of 1.20 L at 0.92 atmospheric pressure. Calculate the
composition of the alloy. Solution 2NaI + H2SO4 ⎯⎯→ Na2SO4 + 2HI
2(23 + 127) 46 + 32 + 64
Solution Let us first calculate the volume of H2 at N.T.P.
= 300 = 142
2NaCl + H2SO4 ⎯⎯→ Na2SO4 + 2HCl Solution Since no water is added, the volume of 0.25 M
2(23 + 35.5) 46 + 32 + 64 HCl cannot be more than 2 litres. Let x litre of 0.40 M HCl
= 117 = 142 (more concentrated) be added to 1 litre of 0.15 M HCl.
Let the wt. of mixture = 1g Applying molarity equation :
Wt. of NaI in the mixture = xg M1V1 + M2V2 = M3V3
.
Wt. of NaCl in the mixture = (1 – x) g 0.15 × 1 + 0.40 × x = 0.25 × (1 + x)
ed
(Total volume V3 becomes 1 + x litres)
Now, 300 g of NaI give Na2SO4 = 142 g 0.15 + 0.40 x = 0.25 + 0.25 x
142 0.40x – 0.25 x = 0.25 –0.15
x g of NaI give Na2SO4 = ×x 0.15 x = 0.10
rv
300
Similarly, 0.10
x = = 0.667 litre
se
117 g of NaCl give Na2SO4 = 142 g 0.15
Total volume of 0.25 M solution = 1 + x
142 = 1 + 0.667 = 1.667 L
(1 – x) g of NaCl give Na2SO4 = × (1 – x)
re
117
Problem 13. A mixture of FeO and Fe 3O4 when
Weight of Na2SO4 formed = Wt. of original mixture
heated in air to a constant mass gains 5% by mass.
142 142 Calculate the composition of the mixture (atomic
ts
x+ × (1 – x) = 1
300 117 mass of Fe = 55.8).
Solving for x, we get Solution The chemical equations are:
gh
x = 0.2886 g 1
∴ Wt. of NaI = 0.2886 2FeO + O ⎯⎯→ Fe2O3 ... (i)
2 2
ri
0.2886 2 (55.8 + 16) 2 × 55.6 + 3 × 16
% NaI in mixture = × 100 = 28.86% = 143.6 = 159.6
1
% NaCl in mixture = 100 – 28.86 = 71.14% ll 1
2Fe3O4 + O ⎯⎯→ 3Fe2O3 ... (ii)
2 2
.A
Problem 11. How many mL of H2SO4 of density 1.8 2 (55.6 × 3 + 4 × 16) 3(2 × 55.6 + 3 × 16)
g/mL containing 92.5% by volume of H2SO4 should be = 462.8 = 478.8
added to 1 litre of 40% solution of H2SO4 (density 1.30
Let the weight of mixture = 100 g
rs
98 × 100
Molarity of solution containing 40% H2SO4, 462.8 g of Fe3O4 produce Fe2O3 = 478.8 g
40 × 1.3 × 1000 478.8
M2 = = 5.31 M
P
50 × 1.4 × 1000
M = = 7.14 M 159.6 478.8
98 × 100
er
x+ (100 – x)
Let V litre of solution with molarity (M1) is added to 143.6 462.8
1 litre of solution with molarity (M2) to prepare (1 + V) litre 100 × 5
Gain in weight of mixture = =5g
od
.
Solution HCl and Al(OH)3 react as Problem 16. (a) Calculate the number of chloride
ed
ions in 100 mL of 0.01 M AlCl3 solution.
3HCl + Al(OH)3 ⎯⎯→ AlCl3 + 3H2O
36.5 × 3 78 (b) What will be the change in number of chloride
rv
= 109.5 ions if the solution is diluted by 100 mL water?
Volume of gastric juice produced per day = 225 mL Moles of AlCl 3
Mass of HCl produced per day = 225 × 3 × 10–3 Solution (a) Molarity = × 1000
se
Vol. of solution
= 0.675 g
Now 109.5 g of HCl require Al(OH)3 = 78 g Molarity × Vol. of Solution
∴ Moles of AlCl3 =
1000
re
78
0.675 g of HCl will require Al(OH)3 = × 0.675 0.01 × 100
109.5 = = 0.001 mol
= 0.481 g 1000
Now, 1 molecule of AlCl3 contains 3 Cl– ions
ts
0.48 ∴ 1 mol of AlCl3 contains 3 mol of Cl– ions
Number of tablets required = = 1.92
0.250 0.001 mol of AlCl3 contains Cl– ions = 0.001 × 3
gh
= 2 tablets. = 0.003 mol
Problem 15. A mixture of formic acid and oxalic 1 mol of Cl– ions = 6.022 × 1023 Cl– ions
ri
acid is heated with concentrated H2SO4. The gas 0.003 mol of Cl– ions = 6.022 × 1023 × 0.003
produced is collected and on its treatment with KOH = 1.81 × 1021 Cl– ions
solution, the volume of the solution decreases by (b) No change in number of Cl– ions.
ll
1/6th. Calculate the molar ratio of the two acids in
.A
the original mixture. Problem 17. The mole fraction of urea in an aqueous
solution of urea containing 900 g of water is 0.05. If
Solution Let x moles of formic acid and y moles of oxalic the density of the solution is 1.2 g cm–3, calculate the
acid are heated. molarity of the solution. (JEE Advance 2019)
rs
Conc. H 2SO 4
HCOOH ⎯ ⎯⎯⎯⎯⎯ → H2O + CO Solution Let number of moles of urea = n
x mol x mol 900
he
Moles of water = = 50
18
COOH
n
Conc. H SO
⎯ ⎯⎯⎯⎯⎯
2 4
→ H2O + CO + CO2 x(urea) =
n + 50
lis
1057.8
Moles of CO2 y 1 Volume of solution = = 881.5 mL
er
= = 1.2
Moles of both gases x + 2y 6
2.63
6y = x + 2y or 4y = x Molarity = × 1000 = 2.98
881.5
od
Practice Problems
= (0.921 g cm–3) × (439 cm3)
©
.
or = 1 × 10–7 (upto one significant figure)
ed
4.86 kg 4.86 kg 1000 g litre
∴ = × × 3 = 4.86 g/mL.
3.25 × 0.08621 L L kg 10 mL
(d) = 0.0700 (upto 3 significant figures)
4.002 (ii) 1.86 km to cm
rv
(e) (1.0042 – 0.0034) (1.23) 1 km = 105 cm
(1.0008) (1.23) = 1.23 (upto 3 significant figures) 105 cm
Unit factor : 1 =
se
6. (i) Area of square = (side)2 ( km )
= (1.2)2 = 1.44 m2 105 cm
Correct answer = 1.4 m2 (upto 2 significant places). 1.86 km = 1.86 km × = 1.86 × 105 cm.
( km )
re
4 3 (iii) 1 m = 106 μm
(ii) Volume of sphere = πr
3
4 22 106 μ m
= × (1.6)3 Unit factor : 1 =
ts
× m
3 7 6
= 17.164 10 μm
∴ 6.92 ×10–7 m × = 0.692 μm
gh
= 17 cm3 (upto two significant figures). m
–7
6.92 × 10 m to Angstroms
Area 10.25
(iii) Length of rectangle = = 1Å = 10–10 m
Breadth 2.5
ri
= 4.1 m (upto 2 significant figures). 1Å
Unit factor : 1 =
–6 –5 10 –10 m
7. (i) (1.20 × 10 ) + (6.00 × 10 ) ll
0.120 × 10–5 + 6.00 × 10–5 = 6.12 × 10–5. 1Å
∴ 6.92 × 10–7 m × = 6920Å.
(upto second place of decimal as in 6.00 × 10–5) 10 –10 m
.A
(ii) (2.164 × 105)½ = (21.64 × 104)½ = 4.6519 × 102 (iv) 1 litre =103 cm3
= 4.652 × 102 (upto 4 significant figures) Unit factor : 1 =
1 litre
(iii) (9.13 × 10–2) (7.006 × 10–3) = 63.9648 × 10–5 103cm 3
rs
= 64.0 × 10–5 (upto 3 significant figures) 9.2 × 10–3 cm3 = 9.2 × 10–3 cm3 × 1 litre
or = 6.40 × 10–4 103cm 3
he
= 0.004 or 4 × 10–3 m3
0.0001 × 10−2 1 litre = 10–3 m3
= 4.33 × 10–2 1 litre
4.3261 × 10−2 Unit factor : 1 =
ub
2.130
3 3
figures because 2.36 contains 3 significant figures. 15. 100 L = 100 × 10 cm (1L = 103 cm3)
= 105 cm3.
( 28.2–21.2 ) (1.79 × 106)
n
1kg
10. (3.0 × 108 m s–1) × (2.00 × 10–9s) = 6.0 × 10–1 = 0.60 m Unit factor : 1 =
answer should be in two significant figures as in 103 g
3.0 × 108. 1kg
500 × 106 g = 500 × 106 × = 500 × 103 kg
M
0.1(nm) 103 g
11. Unit factor =
1(Å) = 5.0 × 105 kg.
3
(ii) 1 kg = 10 g
©
0.1(nm)
∴ 5896 Å = 5896Å ×
1(Å) Unit factor : 1 =
1 kg
= 589.6 nm. 103 g
12. 1 m = 102 cm 1 kg
∴ 3.34 × 10–24 g = 3.34 × 10–24 g ×
1 m3 = (102 cm)3 = 106 cm3. 103 g
13. (i) 1kg = 1000 g; 1 litre = 103 mL
= 3.34 × 10–27 kg.
.
1 nm
ed
1 pm
Unit factor : 1 =
10 –9 mm ⎛ 1 0 –9 m ⎞
∴ 7 nm = 7nm × ⎜ ⎟ = 7 × 10–9 m.
1 μs = 10–6 s ⎝ 1 nm ⎠
rv
1 μs 10 –6s (ii) 1 pm = 10–12 m
Unit factor : 1 = –6 or =
10 s 1 μs 10 –12 m
1 =
se
1 pm 10 –6s 1 pm
1.54 mms–1 = 1.54 mms–1 × –9
×
10 mm 1 μ s ⎛ 10 –12 m ⎞
= 1.54 × 103 pm μs–1. 41 pm = 41 pm × ⎜ 1 pm ⎟ = 41 × 10–12 m.
re
∴
(ii) 1g = 1000 mg ⎝ ⎠
1000 mg 24 h
Unit factor : 1 = 19. Unit factor : 1 =
ts
1g 1 day
10 dL = 1L 1 hr = 60 min
1dL–1
gh
10 dL 60 min
Unit factor : 1 = = Unit factor : 1 =
1L 10 L–1 1 hr
1000 mg 1dL–1 1 min = 60s
∴ 25 g L–1 = 25 g L–1 × ×
ri
1g 10 L–1 60s
3 –1
= 2.5 × 10 mg dL . Unit factor : 1 =
1 min
(iii) 1L = 1dm3 ll ⎛ 24 h ⎞ ⎛ 60 min ⎞ ⎛ 60 s ⎞
1dm 3 ∴ 2 days = 2day × ⎜ × ⎜ ⎟×⎜ ⎟
.A
Unit factor : 1 = ⎝ 1 day ⎟⎠ ⎝ 1 h ⎠ ⎝ 1 min ⎠
1L
= 172800 s
103dm3 = 1m3 20. (i) 1 pm = 10–12 m
rs
1m 3
Unit factor : 1 = 10 −12
103dm 3 Unit factor : 1 =
1 pm
he
1dm 3 1m 3 ⎛ 10−12 m ⎞
∴ 25 L = 25 L × × 3 = 2.5 × 10–2 m3. 28.7 pm = 28.7 pm × ⎜ –11
⎟ = 2.87 × 10 m
1L 10 dm 3 1 pm
(iv) 1μg = 10–6 g ⎝ ⎠
(ii) 1 μs = 10–6 s
lis
1μ g
Unit factor : 1 = 1 0 −6 s
10 –6g Unit factor : 1 =
1 μs
ub
1 m3 = 106 cm3
⎛ 1 0 −6 s ⎞ –5
106cm 3 ∴ 15.15 μs = 15.15μs × ⎜ ⎟ = 1.515 × 10 s
Unit factor : 1 = ⎝ 1 μs ⎠
1m 3
P
(iii) 1 mg = 10–6 kg
1 m3 = (106)3μm
10−6 kg
n
⎛ 10 −6 kg ⎞
2.66 g 1 μ g 106cm3 1m3 ∴ 25365 mg = 25365 mg × ⎜ ⎟
∴ × –6 × × 18 ⎝ 1 mg ⎠
od
3 3
cm 10 g 1m 10 μ m
= 2.66 × 10–6 μg μm–3. = 2.5365 × 10–2 kg
(v) 1L = 1000 mL
21. Mass of reactants = 4.2 g + 10.0 g = 14.2 g
M
1000 mL
Unit factor : 1 = Mass of products = 2.2 g + 12.0 g = 14.2 g
1L
∴ Mass of reactants = Mass of products
1h = 3600 s
©
22. In H2O2,
1h
Unit factor : 1 = Mass of hydrogen = 5.93 g
3600 s
Mass of oxygen = 100 – 5.93 = 94.07 g
2
4.2 L 4.2 L ⎛ 1000 mL ⎞ ⎛ 1 h ⎞ In water,
∴
h2
=
h2
× ⎜ ⎟×⎜ ⎟ Hydrogen = 11.2 g
⎝ 1L ⎠ ⎝ 3600 s ⎠
–4 –2 Oxygen = 100 – 11.2 = 88.8 g
= 3.2 ×10 mL s
.
ed
Mass of oxygen combining with 5.93 g of hydrogen with fixed mass of carbon (i.e. 1 g)
= 94.07 g 0.333 : 0.167 : 0.083
Mass of oxygen combining with 1 g of hydrogen 4 : 2 : 1
rv
94.07 This is simple whole number ratio and therefore,
= = 15.86 g it illustrates the law of multiple proportions.
5.93
se
In H2O, 25. First experiment
Mass of oxygen combining with 11.2 g of hydrogen Mass of copper oxide = 1.375 g
= 88.8 g Copper left = 1.098 g
re
Mass of oxygen combining with 1 g of hydrogen Mass of oxygen present= 1.375 – 1.098 = 0.277 g
88.8 0.277 × 100
= = 7.93 g % of oxygen in CuO =
11.2 1.375
ts
The ratio of masses of oxygen combining with 1g of = 20.14
hydrogen Second experiment
gh
15.86 : 7.93 Mass of copper taken = 1.178 g
2 : 1 Mass of CuO formed = 1.476 g
This is simple ratio and therefore, it illustrates Mass of oxygen present = 1.476 – 1.178
ri
the law of multiple proportions. = 0.298
23. Let us fix the mass of A to be 1.0 g 0.298 × 100
ll % of oxygen in CuO = = 20.19
In compound X, 1.476
.A
Percentage of oxygen in the two compounds is the
0.3 g of A combines with B = 0.4 g
same, therefore, these results illustrate the law of
0.4 constant composition.
1.0 g of A will combine with B = × 1.0
0.3 26. (i) 22400 mL of SO2 at N.T.P = 64 g
rs
= 1.33 g 64
224 mL of SO2 at N.T.P = × 224
In compound Y, 22400
he
× 100
1.33 : 2.66 0.64
1 : 2 = 50%
This is a simple ratio and hence it illustrates the (ii) Percentage of sulphur in second sample = 50%
P
25 copper = 25.45
= = 0.333 g
75 40 g of crystalline copper sulphate sample will
In compound B, 25.45
contain copper = × 40 = 10.18 g
M
79.9 g of Cu combine with 20.1 g of oxygen in Mass of oxygen which combines with 1 g of metal
copper oxide. 0.2011
Let us fix the mass of copper as 1 g = = 0.2517 g
.
0.7989
ed
In copper sulphide, The ratio of masses of oxygen which combine with
66.5 g of copper combine with sulphur = 33.5 g fixed mass of metal (1 gram)
1 g of copper will combine with sulphur 0.1255 : 0.2517
rv
33.5 1 : 2
= = 0.503 g Since this is a simple ratio, therefore the above
66.5
data illustrates the law of multiple proportions.
se
In copper oxide,
79.9 g of Cu combine with oxygen = 20.1 g 30. Average atomic mass of Si
1 g of Cu will combine with oxygen 28 × 92.25 + 29 × 4.65 + 30 × 3.10
re
=
20.1 100
= = 0.251 g = 28.11 u.
79.9
31. (a) 1.6 gram atoms of oxygen = 16 × 1.6 = 25.6 g.
ts
The ratio of weights of sulphur and oxygen which
combine with the fixed mass of copper (1 g) 32
(b) 5.6 gram atoms of sulphur = × 5.6
0.503 : 0.251 1
gh
2 : 1 ...(i) = 179.2 g.
In sulphur trioxide,
ri
Percentage of sulphur = 40 (c) 2.4 gram atoms of iodine = 127 × 2.4 = 304.8 g.
Percentage of oxygen = 60 32. (i) 2.5 gram molecules of H2S = 34 × 2.5 = 85 g.
The ratio of sulphur and oxygen in sulphur trioxide ll (ii) 3.6 gram molecules of glucose = 180 × 3.6 = 648 g
40 : 60 Molecular mass of glucose = 12 × 6 + 1 × 12 + 16 × 6
.A
2 : 3 ...(ii) = 180
The ratios (i) and (ii) are 33. (i) Atomic mass of iron = 55.8
2 2 55.8 g of iron ≡ 1 gram atom
rs
: or 3:1
1 3 1
This is a simple ratio. Hence law of reciprocal 669.6 g of iron = × 669.6 = 12 gram atom.
55.8
he
proportions is illustrated.
(ii) Molecular mass of C2H5OH
29. Calculation of weights of oxygen in each oxide.
= 2 × 12 + 5 × 1 + 1 × 16 + 1 × 1 = 46
Mass of each oxide taken = 1 g
lis
16 46
= × 0.1254 = 0.1115 g
18 = 1.6 gram molecule.
0.2263 g of water in second oxide contains oxygen 34. (a) 2.6 gram atoms of sulphur = 2.6 × 32 = 83.2 g
P
Mass of metal =1 – 0.1115 = 0.8885 g 35. (i) 6.022 × 1023 atoms of calcium have mass = 40 g
In second oxide
od
40
Mass of oxygen = 0.2011 g 1 atom of calcium has mass =
6.022 × 1023
Mass of metal = 1 – 0.2011 = 0.7989 g
Let us fix the weight of metal as 1 g = 6.64 × 10–23 g.
M
23
First oxide (ii) Mass of 6.022 × 10 molecules of SO2 = 64 g
Mass of oxygen which combines with 0.8885 g of 64
Mass of 1 molecule of SO2 =
©
.
1
ed
= 6.022 × 1022 atoms. 80 × 103 g of X = × 80 × 103
40.01
(iii) 180 g of glucose = 6.022 × 1023 molecules
= 1999.5 gram atoms.
6.022 × 1023 × 18 41. 22.4 L of ozone (O3) at N.T.P = 6.022 × 1023 molecules
rv
18 g of glucose =
180
6.022 × 1023
= 6.022 × 1022 molecules 5.60 L of ozone at N.T.P = × 5.6
se
22.4
1 molecule of glucose = 6 + 12 + 6 = 24 atoms
= 1.506 × 1023 molecules.
6.022 × 1022 molecules of glucose
Now,1 molecule of ozone = 3 atoms
= 6.022 × 1022 × 24 =1.445 × 1024 atoms.
re
1.506 × 1023 molecules of ozone = 1.506 × 1023 × 3
(iv) 1 mole molecules of oxygen = 6.022 × 1023 molecules
= 4.518 × 1023 atoms.
0.20 mol molecules of oxygen = 6.022 × 1023 × 0.2
= 1.2046 × 1023 molecules 300 × 20
ts
42. Gold present in 300 mg gold ring =
1 molecule of oxygen = 2 atoms 24
1.2046 × 1023 molecules = 1.2046 × 1023 × 2 = 250 mg
gh
= 2.409 × 1023 atoms. Now, number of atoms present in 197 g
37. 40 g of calcium = 6.022 × 1023 atoms = 6.022 × 1023
15 g of calcium contain atoms No. of atoms present in 250 × 10–3 g
ri
6.022 × 10 23 6.022 × 1023 × 250 × 10 –3
= × 15 = 2.258 × 1023 atoms. =
40 ll 197
Now, number of sodium atoms = 2.258 × 1023 = 7.64 × 1020.
.A
6.022 × 1023 atoms of sodium has mass = 23 g 43. Molecular mass of KCl = 39 + 35.5 = 74.5 g
2.258 × 1023 atoms of sodium has mass 74.5 g of KCl = 1 mole atoms of K
23 1 × 1000
rs
1.4 g of nitrogen at N.T.P will occupy volume 2 mol atoms of K in K2O weigh = 94 g
22.4 94
= × 1.4 = 1.12 L. 13.42 mol atoms of K in K2O weigh = × 13.42
lis
28 2
(ii) 6.022 × 1023 molecules of O2 at N.T.P occupy = 631 g = 0.631 kg.
volume = 22.4 L 44. Gram molecular mass of H2C2O4.2H2O = 126 g
ub
6.022 × 1021 molecules of O2 at N.T.P will occupy No. of water molecules in 1 mol of oxalic acid
22.4 × 6.022 × 1021 = 2 mol
volume = No. of water molecules in 126 g of oxalic acid
6.022 × 1023
P
= 2 × 6.022 × 1023
= 0.224 L ∴ No. of water molecules in 630 × 10–3 g of oxalic acid
(iii) 1 mol of NH3 occupy volume = 22.4 L
n
Rupees spent per second = 106 45. Volume of CO2 present in air = 1000 ×
100
Rupees spent per year = 106 × 60 × 60 × 24 × 365 = 0.03 mL
∴ 106 × 60 × 60 × 24 × 365 rupees are spent in = 1year 22400 ml of CO2 at S.T.P. contain = 6.022 × 1023
M
6
10 × 60 × 60 × 24 × 365 22400
= 8.07 × 1018 molecules.
=1.91 × 1010 years. 46. Mass of dot = 10–6
40. 1 gram atom corresponds to mass of 6.022 × 1023 12 g of C contain atoms = 6.022 × 1023
atoms 6.022 × 1023
Mass of 6.022 × 1023 atoms = 6.644 × 10–23 × 6.022 × 1023 10–6 g of C contain atoms = × 10–6
12
= 40.01 g = 5.019 × 1016 atoms.
47. 1 mol of CCl4 = 154 g 49. Molecular weight of Na2CO3. 10H2O = 286
154 g of CCl4 contains 6.022 × 1023 × 4 Cl atoms Mass of oxygen present in Na2CO3.10H2O = 208
or 6.022 × 1023 × 4 Cl atoms are present in 154 g of
.
Now 1 mol of Na2CO3.10H2O = 286 g
ed
CCl4
0.1 mol of Na2CO3.10H2O = 28.6 g
154 × 1 × 1025
1 × 1025 Cl atoms are present in = Wt. of oxygen present in 286 g of Na2CO3 = 208
6.022 × 1023 × 4
rv
= 639.2 g Wt. of oxygen present in 28.6 g of Na2CO3.10H2O
208
Mass = × 28.6
se
Volume of CCl4 = 286
Density
639.2 = 20.8 g.
= = 426 cc
re
1.5 50. (i) Molecular mass of MgSO4
or = 0.426 litre. = 24 + 32 + 4 × 16 = 120
48. Number of C atoms in 1.0 g of C-14 isotope 14 g of
Mass of Mg = 24
ts
C-14 isotope contains C atoms = 6.022 × 1023
1 g C-14 isotope contain C atoms 24
Mass percentage of Mg = × 100 = 20%
120
gh
23
6.022 × 10
= ×1
14 Mass of S = 32
= 4.30 × 1022 32
ri
Mass percentage of S = × 100 = 26.67%
Number of C atoms in 1.0 g of C-12 isotope 120
12 g of C-12 isotope contains C atoms = 6.022 × 1023 Mass of O = 4 × 16 = 64
1 g of C-12 isotope contain C atoms
ll Mass percentage of O =
64
× 100 = 53.33%.
.A
6.022 × 1023 120
= ×1
12
(ii) Molar mass of (NH4)2 Cr2O7 = 2 × (14 + 4) + 2 × 52 +
= 5.02 × 1022
7 × 16 = 252
rs
ratio
40.68 3.39
C 40.68 12 = 3.39 =1 2
P
12 3.39
5.08 5.08
n
54.24 3.39
O 100 – (40.68 + 5.08) 16 = 3.39 =1 2
16 3.39
od
= 54.24
.
mass ratio ratio
ed
54.2 4.50
C 54.2 12 = 4.50 =2
12 2.29
rv
9.2 9.2
H 9.2 1 = 9.2 =4
1 2.29
se
36.6 2.29
O 36.6 16 = 2.29 =1
16 2.29
re
Empirical formula = C2H4O
(ii) Calculation of molecular formula
Empirical formula mass = 2 × 12 + 4 × 1 + 1 × 16 = 44
ts
Molecular mass = 88
Molecular mass 88
gh
n = = =2
Empirical formula mass 44
∴ Molecular formula = 2(C2H4O) = C4H8O2.
ri
53. (i) Calculation of empirical formula
Molecular mass 92
n = = =2
Empirical formula mass 46
ub
36.5 1.59
er
Na 36.5 23 = 1.59 =2
23 0.79
0.8 0.8
od
H 0.8 1 = 0.8 =1
1 0.79
24.6 0.79
P 24.6 31 = 0.79 =1
31 0.79
M
38.1 2.38
O 38.1 16 = 2.38 =3
16 0.79
©
126
n = =1
126
.
Molecular formula = Na2HPO3 Name : Sodium hydrogen phosphite.
ed
55. (i) Calculation of empirical formula
rv
Element Percentage Atomic mass Atomic ratio Simple atomic ratio
0.83
se
20.0
Mg 20.0 24 = 0.83 =1
24 0.83
0.83
re
26.66
S 26.66 32 = 0.83 =1
32 0.83
53.33 3.33
O 53.33 16 = 3.33 =4
ts
16 0.83
gh
(ii) Calculation of molecular formula
Empirical formula mass = 24 + 32 + 4 x 16 = 120
ri
Molecular mass = 120
n =
120
120
=1
ll
.A
Molecular formula of anhydrous salt = MgSO4
Let molecular formula mass = 100
rs
Now, if molecular mass of anhydrous salt is 48.8, then that of hydrated salt is = 100
100
lis
If molecular mass of anhydrous salt is 120, then that of hydrated salt is = × 120 = 245.9
48.8
Loss in weight due to dehydration = 245.9 – 120 = 125.9
ub
125.9
No. of water molecules in hydrated sample = =7
18
P
56.
n
44.82 0.564
od
34.83 0.580
SiO2 34.83 60 = 0.580 = 1.03 1
60 0.564
©
20.35 1.12
H2O 20.35 18 = 1.12 = 2.0 2
18 0.564
57.
Element / Percentage At. mass/ Relative Simple Simplest whole
.
Compound Molecular mass no. ratio no. ratio
ed
20 0.36
Fe 20 56 = 0.36 =1 1
56 0.36
rv
11.5 0.36
S 11.5 32 = 0.36 =1 1
32 0.36
se
23.1 1.44
O 23.1 16 = 1.44 =4 4
16 0.36
re
45.4 2.52
H2O 45.4 18 = 2.52 =7 7
18 0.36
ts
60. Fe2O3 + 3H2⎯⎯⎯→ 2Fe + 3H2O 37.8 g of NaBH4 give BI3 = 391.8 g
gh
2 × 56 + 3 × 16 2 × 56 391.8
= 160 = 112 64 g of NaBH4 will give BI3 = × 64 = 663.36 g
1 kg ? 37.8
160 g of Fe2O3 on reduction gives iron = 112 g Experimental yield = 15.0 g
ri
1000 g of Fe2O3 on reduction will give iron 15.0 × 100
112 Percentage yield =
= × 1000 = 700 g ll 663.36
160
= 2.27%.
.A
= 0.7 kg.
65. 5KNO2 + 2KMnO4 + 3H2SO4 ⎯⎯⎯→
61. C12H22O11 + 12O2⎯⎯⎯→ 12CO2 + 11H2O
5(39 + 14 + 2 × 16) 2(39 + 54.9 × 4 × 16)
342g 12 × 32g
= 425 g = 315.8 g
?
rs
34 × 24 g
5KNO3 + 2MnSO4 + K2SO4 + 3H2O
Energy requirement in a day = 34 × 24 g of sucrose
For oxidising 425 g of KNO2, KMnO4 needed
Amount of oxygen needed for burning 342 g of sucrose
he
= 315.8 g
= 12 × 32 g
Amount of oxygen needed for burning 34 × 24 g of For oxidising 11.4 g of KNO2, KMnO4 will be needed
12 × 32 315.8
lis
sucrose = × 34 × 24 = × 11.4
342 425
= 916.2 g. = 8.47 g.
ub
62. CaCO3 + 2HCl ⎯⎯⎯→ CaCl2 + H2O + CO2 66. 2KMnO4 + 8H2SO4 + 10KI ⎯⎯⎯→
100 g 22.4 L at
2(39 + 54.9 +4 × 16) 10(39.5 + 127)
N.T.P. = 315.8 g = 1665 g
P
22.4 L of CO2 at N.T.P. is obtained from CaCO3 6K2SO4 + 2MnSO4 + 5I2 + 8H2O
= 100 g 1665 g of KI require for conversion to I2 = 315.8 g
100 L of CO2 at N.T.P. will be obtained from CaCO3 KMnO4
n
= × 100 = 446.4 g
22.4 315.8 × 75
446.4 × 100 = = 14.22 g
1665
od
5.6 L ? 1000
= × 14.22 = 90.05 mL.
22.4 L of methane on ignition give = 1 mol of CO2 158
67. Mass of NaOH = 0.38 g
5.6 L of methane on ignition will give CO2
©
1 0.38
= × 5.6 = 0.25 mol. Moles of NaOH =
22.4 40
64. Let us first calculate the theoretical yield of BI3 Volume of solution = 50.0 mL
NaBH4 + 4I2 ⎯⎯⎯→ BI3 + NaI + 4HI 0.38 / 40
Molarity = × 1000
23+10.8 + 4 × 1 10.8 + 3 × 127 50.0
= 37.8 g = 391.8 g = 0.19 M.
68. 3 molal solution of NaOH means that 3 mol of NaOH 74. AgNO3 + HCl ⎯⎯→ AgCl + HNO3
are present in 1000 g of solvent. 1000 mL of 0.068 M AgNO3 contain AgNO3
.
Mass of 3 mol of NaOH = 3 × 40 = 120 g = 0.068 mol
ed
Mass of solution = 1000 + 120 = 1120 g 25 mL of 0.068 AgNO3 contain AgNO3
1120 g 0.068
Volume of solution = = × 25
1.110 g mL−1 1000
rv
= 0.0017 mol
= 1009.0 mL
1 mol of AgNO3 give 1 mol of AgCl and therefore,
se
3 0.0017 mol of AgNO3 will form 0.0017 mol of AgCl,
Molarity = × 1000
1009.0
Amount of AgCl obtained = 0.0017 × 143.5
re
= 2.97 M = 0.244 g .
69. M1V1 = M2V2 75. 2KOH + H2SO4 ⎯⎯→ K2SO4 + 2H2O
2.4 × 500 = 1.6 × V2 1000 mL of 0.15 M H2SO4 contain = 0.15 mol
ts
2.4 × 500 20 mL of 0.15M H2SO4 contain
or V2 = = 750 mL
1.6 0.15
gh
= × 20
Volume of water to be added = 750 – 500 = 250 mL. 1000
70. 86% by weight of H2SO4 means that 86 g of H2SO4 is = 0.003 mol
present in 100 g of solution According to equation, 2 mol of KOH react with 1 mol
ri
100 of H2SO4 so that 0.003 mol of H2SO4 will react with
Volume of 100 g solution = = 55.6 mL 2 × 0.003 = 0.006 mol of KOH
1.80 ll 0.34 mol of 0.34 M KOH are present in = 1000 mL
86 0.006 mol of 0.34 M KOH are present in
.A
Moles of H2SO4 =
98
1000
86 / 98 = × 0.006
Molarity = × 1000 0.34
55.6
rs
= 17.65 mL.
= 15.8 M. 76. NaOH (aq) + HCl (aq) ⎯⎯→ NaCl (aq) + H2O (l)
he
71. Molecular mass of (COOH)2. 2H2O = 126 1000 mL of 2.0 mol L–1 contain NaOH = 2.0 mol
0.63 200 mL of 2.0 mol L–1 contain NaOH
Moles of oxalic acid = = 0.005 mol 2.0
126
lis
= × 200
0.005 1000
Molarity = × 1000 = 0.4 mol
500
ub
Molarity × Volume in mL
73. Moles of oxalic acid = 1 M solution of NaOH contains 40 g L–1
1000
0.150 M solution of NaOH contains 4.0 × 0.150
©
78. BaCl2 (aq) + Na2SO4 (aq) ⎯⎯→ BaSO4 (s) + 2NaCl 79. The balanced chemical equation is :
1000 mL of 0.250 M Na2SO4 contain Na2SO4 CaCO3 (s) + 2 HCl (aq) ⎯⎯→ CaCl2 (aq) + CO2 (g) + H2O (l)
= 0.250 mol Let us first calculate the moles of HCl in 25 mL of
0.75 M HCl
500 mL of 0.250 M Na2SO4 cantain Na2SO4
1000 mL of 0.75 M HCl contain HCl = 0.75 mol
0.250
× 500 = 0.125 mol 0.75
.
=
1000 25 mL of 0.75 M HCl contain HCl = × 25
ed
1000
15 = 0.01875 mol
Moles of BaCl2= = 0.0721 mol
208 2 mol of HCl completely react with 1 mol of CaCO3
rv
∴ BaCl2 is limiting reactant. 0.01875 mol of HCl will completely react with
Molecular mass of BaSO4 = 233 1
× 0.01875
se
1 mol of BaCl2 react with Na2SO4 to give =
2
= 233 g BaSO4 = 0.009375 mol
0.072 mol of BaCl2 will react with Na2SO4 to give Mass of 0.009375 mol of CaCO3 = 0.009375 × 100
re
= 233 × 0.072 = 16.78 g. = 0.9375 g
ts
gh
ri
ll
Precision refers for the closeness of the set of values obtained from identical measurements of a quantity.
Accuracy refers to the closeness of a single measurement to its true value.
.A
All numbers, small or large are expressed as a number between 1.000 ................ and 9.999 multiplied or divided by
10, an appropriate number of times.
⇒ 1426.2 = 1.4262 × 103
rs
In addition and subtraction, the final result should be reported to the same number of decimal places as the
number with the minimum number of decimal places.
In multiplication and division, the final result should be reported as having the same number of significant digits
lis
Law of constant composition (Proust in 1798). A chemical compound always contains same elements combined
together in same proportion by mass.
Law of multiple proportions (John Dalton in 1803). When two elements combine together to form two or more
P
than two compounds then the masses of one of the elements that combine with fixed mass of the other bear a simple
whole number ratio to one another.
Law of reciprocal proportions (Richter in 1792). When two different elements combine separately with a fixed
n
mass of a third element, then the ratio of their masses in which they do so is either the same or some whole number
er
The smallest particle of an element that takes part in chemical reactions is atom.
The smallest particle of a substance that has independent existence is molecule.
One twelveth (1/12) of the mass of an atom of carbon (C-12) is atomic mass unit. It is equal to 1.66 × 10–27 kg.
©
Atomic mass is the average relative mass of an atom of element as compared with mass of a carbon atom (C-12)
taken as 12 a.m.u.
Molecular mass is the average relative mass of a molecule of the substance as compared with mass of a carbon atom
(C-12) taken as 12 a.m.u.
Atomic mass expressed in grams is gram atomic mass and molecular mass expressed in grams is gram molecular
mass.
One mole is 6.022 × 1023 specified particles.
Volume occupied by one mole of a gaseous substance is called gram molecular volume (G.M.V.). Its value is 22.4 L at
N.T.P.
Mass percentage composition of a compound gives the mass of each element expressed as percentage of the total
mass.
Empirical formula is the formula which gives the simplest whole number ratio of atoms of different elements
present in the molecule of a compound.
.
ed
Molecular formula is the formula which gives the actual number of atoms present in the molecule of the compound.
It is whole number multiple of empirical formula.
Molarity is the number of moles of solute dissolved per litre of solution. It is expressed as mol dm–3 or M.
rv
Molarity changes with temperature because volume of the solution changes with temperature.
Limiting reactant is the reactant which is completely consumed in the reaction.
se
QUICK CHAPTER ROUND UP
Matter
re
anything which occupies space and has mass.
ts
Physical classification Chemical classification
gh
solids. definite shape and definite volume
liquids. definite volume but no definite Pure substances Mixtures
shape
ri
gases. neither definite shape nor definite Element: simplest form Homogeneous: Uniform
volume of matter composition throughout
ll
Compound: contain
more than one kind of
Heterogeneous: compo-
sition is not uniform
.A
heat heat
liquid
solid gas elements combined in a throughout
cool cool
definite proportion
be reported to the same number of decimal places as the which has independent existence ⇒ molecule
number with the minimum number of decimal places.
• In multiplication and division, the final result should
lis
th mass of 12C
→
12
1 gram atom of an 1 gram molecule of a
• One twelveth (1/12) of the mass of an atom of carbon
element substance
n
.
of atoms present in the molecule of the compound. Normality (N) = × 1000
Vol. of solution (in mL)
ed
It is whole number multiple of empirical formula.
• Molecular formula = n (Empirical formula) Mol. mass
Normality = Molarity ×
Molecular mass Eq. mass
rv
n = Empirical formula mass Moles of solute
Mole fraction of solute(xB) =
Moles of solute + Moles of solvent
se
Mole fraction of solute + Mole fraction of solvent = 1
Molarity equation :
or Mole fraction of solvent=1 – Mole fraction of solute.
M1V1 = M2V2
re
Limiting reactant the reactant which reacts completely in a
reaction.
ts
gh
Textbook Exercises
ri
NCERT
Q. 1. Calculate the molecular mass of the following: Ans.
ll The balanced chemical equation of combustion of
(i) H2O (ii) CO2 (iii) CH4 carbon in dioxygen (or air) is
.A
C(s) + O2 (g) ⎯⎯→ CO2(g)
Ans. (i) Molecular mass of H2O 1 mol 1 mol 1 mol
= 2 (1.008 u) + (16.00 u) = 18.016 u (i) In air, carbon will be completely burnt. 1 mol of
rs
(ii) Molecular mass of CO2 carbon will give 1 mol of CO2 or = 44g
(ii) Since only 16 g of dioxygen is available (0.5
= 1 (12.01 u) + 2(16.00 u) = 44.01 u
he
produce 22 g of CO2.
= 2 × At. mass of Na + At. mass of S + 4 × At. mass of O Q. 5. Calculate the mass of sodium acetate
= 2 × 23.0 + 32 + 4 × 16 = 142 (CH3COONa) required to make 500 mL of 0.375
P
32
Mass % of sulphur = × 100 = 22.53% sodium acetate are present in 1000 mL of solution.
er
142
500 mL of the solution should contain sodium acetate
4 × 16
Mass % of oxygen = × 100 = 45.07% =
0.375
mol
142
od
2
Q. 3. Determine the empirical formula of an oxide of Molar mass of sodium acetate = 82.0245 g mol–1
iron which has 69.9% iron and 30.1% dioxygen
0.375
by mass. (Atomic masses : Fe = 55.85 amu, Mass of sodium acetate required = × 82.0245
M
O = 16.00 amu). 2
= 15.38 g
Ans. Refer Solved Example 63 (Page 53). Alternatively, it may be solved as :
©
0.375 × 82.0245 × 500 Q. 11. What is the concentration of sugar (C12H22 O11)
∴ Mass of sodium acetate = in mol L–1 if its 20g are dissolved in enough
1000
= 15.38 g water to make a final volume upto 2 L ?
Q. 6. Calculate the concentration of nitric acid in Ans. Refer Solved Example 103 (Page 70).
moles per litre in a sample which has a density, Q. 12. If the density of methanol is 0.793 kg L–1, what
1.41 g mL–1 and the mass per cent of nitric acid is its volume needed for making 2.5 L of its
.
in it being 69%. 0.25 M solution ?
ed
Ans. Refer Solved Example 101 (Page 69). Ans. Refer Solved Example 104 (Page 70).
Q. 7. How much copper can be obtained from 100 g Q. 13. Pressure is defined as force per unit area of the
of copper sulphate (CuSO4) ? (Atomic mass of surface. The SI unit of pressure, pascal is shown
rv
Cu = 63.5 amu). below :
Ans. 1 mol of CuSO4 contains 1 mol (1 gram atom) of Cu 1 Pa = 1N m–2
se
Molar mass of CuSO4 = 63.5 + 32 + 4 × 16 If mass of air at sea level is 1034 g cm–2, calculate
= 159.5 g the pressure in pascal.
∴159.5 g of CuSO4 will give Cu = 63.5 g Ans. Pressure is force (i.e., weight) acting per unit area
re
But weight = mg
63.5
100 g of CuSO4 will give Cu = × 100 ∴ Pressure = Weight per unit area
159.5
= 39.81 g 1034g
ts
= × 9.8 ms–2
Q. 8. Determine the molecular formula of an oxide cm 2
of iron in which the mass per cent of iron and
gh
oxygen are 69.9 and 30.1 respectively. Given 1034 g 1 kg
= × 9.8 ms–2 ×
that the molecular mass of iron oxide is 159.8 cm 2 1000 g
and atomic masses : Fe = 55.85 amu and O =
ri
16.00 amu. 100 cm × 100 cm 1N 1Pa
× × × −2
Ans. Calculation of empirical formula = Fe2O3 (See Solved 1m ×1m kg m s−2 1Nm
Example 65) ll = 1.01332 × 105 Pa
Empirical formula mass of Fe2O3
Q. 14. What is the SI unit of mass ? How is it defined?
.A
= 2 × 55.85 + 3 × 16.00
Ans. The SI unit of mass is kilogram (kg). Kilogram is
= 111.7 + 48.00 = 159.7 g mol–1
defined as the mass of platinum-iridium (Pt-Ir)
Molecular formula mass = 159.8 g mol–1
block, stored at the International Bureau of Weights
rs
Molecular formula mass and Measures in France. Thus it is the mass of the
n=
Empirical formula mass international prototype of the kilogram.
he
35
Cl 75.77 34.9689 (iv) giga 10–15
37
Cl 24.23 36.9659 (v) femto 10
Ans. Average atomic mass of Cl Ans. (i) micro – 10–6 (ii) deca – 10 (iii) mega –106
P
75.77 × 34.9689 + 24.23 × 36.9659 (iv) giga – 109 (v) femto – 10–15
=
100 Q. 16. What do you mean by significant figures ?
n
(ii) Number of moles of hydrogen atoms is reported to be 18.25 mL, the digits 1, 8 and 2 are
(iii) Number of molecules of ethane certain while 5 is doubtful. So, it has four significant
Ans. (i) 1 mole of C2H6 contains 2 moles of carbon figures (three certain plus one doubtful).
M
∴ Number of moles of carbon in 3 moles of Q. 17. A sample of drinking water was found to be
C2H6 = 6 severely contaminated with chloroform, CHCl3,
(ii) 1 mole of C2H6 contain 6 mole atoms of hydrogen supposed to be carcinogenic in nature. The
©
∴ Number of moles of hydrogen atoms in 3 level of contamination was 15 ppm (by mass).
moles of C2H6 = 3 × 6 = 18 (i) Express this in per cent by mass.
(iii) 1 mole of C2H6 = 6.022 × 1023 molecules (ii) Determine the molality of chloroform in
∴ Number of molecules in 3 moles of the water sample.
C2H6 = 3 × 6.022 × 1023 Ans. (i) 15 ppm means that 15 parts of chloroform is
= 1.807 × 1024 molecules present in 106 parts.
15
∴ % by mass of CHCl3 = × 100 = 1.5 × 10–3% 1000 m 1pm
106 1 km = 1 km × × = 1015 pm
1 km 10−12 m
(ii) Molar mass of CHCl3 = 12 + 1 + 3 × 35.5 = 119.5
Correct answer : 106, 1015
1.5 × 10 −3 g
Moles of CHCl3 =
119.5 g mol −1 1g 1 kg
.
(ii) 1mg = 1mg × × = 10–6 kg
= 1.255 × 10–5
ed
1000 mg 1000 g
Mass of water = 100 g 1g 1 ng
1 mg = 1 mg × × = 106 ng
1.225 × 10−5 1000 mg 10 −9 g
rv
∴ Molality = × 1000
100 Correct answer : 10–6, 106
–4
= 1.255 × 10 m 1L
se
Q. 18. Express the following in the scientific notation. (iii) 1 mL = 1mL × 1000 mL = 10–3 L
(i) 0.0048 (ii) 234,000 (iii) 8008
3
(iv) 500.0 (v) 6.0012 ⎛ 1dm ⎞
re
1 mL = 1 cm3 × ⎜ = 10–3 dm3
Ans. Refer Solved Example 2 (Page 15). ⎝ 10 cm ⎟⎠
Q.19. How many significant figures are present in Correct answer : 10–3, 10–3
the following ?
ts
Q. 22. If the speed of light is 3.0 × 108 ms–1, calculate
(i) 0.0025 (ii) 208 (iii) 5005 the distance covered by light in 2.00 ns.
(iv) 126,000 (v) 500.0 (vi) 2.0034 Ans. Distance covered by light = Speed × Time
gh
Ans. (i) 2 (ii) 3 (iii) 4
⎛ 10 −9 ⎞
(iv) 3 (v) 4 (vi) 5 = 3.0 × 108 ms–1 × 2.00 ns × ⎜
⎝ 1ns ⎟⎠
ri
Q. 20. Round up the following upto three significant
figures : = 6.00 × 10–1 m = 0.600 m
(i) 34.216 (ii) 10.4107 (iii) 0.04597 (iv) 2808 Q. 23. In a reaction
ll
Ans. (i) 34.2 (ii) 10.4 (iii) 0.0460 (iv) 2810 A + B2 ⎯⎯→ AB2
.A
Q. 21. The following data were obtained when identify the limiting reagent if any in the
dinitrogen and dioxygen react together to form following reaction mixtures :
different compounds : (i) 300 atoms of A + 200 molecules of B
rs
(a) Which law of chemical combination is Q. 24. Dinitrogen and dihydrogen react with each
obeyed by the above experimental data ? other to produce ammonia according to the
Give its statement.
ub
unreacted ?
of dioxygen combined will be
(iii) If yes, which one and what would be its
16 : 32 : 16 : 40
mass ?
od
or 2 : 4 : 2 : 5
Ans. Refer Solved Example 92 (Page 66).
This is a simple whole number ratio and hence, the
Q. 25. How are 0.50 m Na2CO3 and 0.50 M Na2CO3
data illustrate the law of multiple proportions.
M
different ?
The law of multiple proportions states that when
two elements combine to form two or more than two Ans. Molar mass of Na2CO3 = 2 × 23 + 12 + 3 × 16
compounds, then the mass of one of the elements = 106 g mol–1
©
which combine with a fixed mass of the other bear a Mass of 0.5 mol of Na2CO3 = 106 × 0.5 = 53 g
simple whole number ratio. 0.50 m Na2CO3 solution means that 0.5 mol or 53g of
(b) (i) Na2CO3 are present in 1000 g of solvent.
1000 m 100 cm 10 mm 0.50 M Na2CO3 solution means that 0.5 mol or 53 g
1 km = 1 km × × × =106mm of Na2CO3 are present in 1 L of the solution.
1 km 1m 1 cm
.
2 volumes of dihydrogen react with 1 volume of O2 to Ans. 1 mole of 12C atoms = 6.022 × 1023 atoms = 12g
ed
produce 2 volumes of water vapour. Thus, ∴ 6.022 × 1023 atoms of 12C have mass = 12g
10 volumes of dihydrogen will react completely with 12 12
1 atom of C will have mass =
6.022 × 10 23
rv
5 volumes of dioxygen to produce 10 volumes of water.
10 volumes of water would be produced. = 1.9927 × 10–23 g
Q. 31. How many significant figures should be present
se
Q. 27. Convert the following into basic units :
(i) 28.7 pm (ii) 15.15 μs (iii) 25365 mg in the answer of the following calculations ?
0.02856 × 298.15 × 0.112
10−12 m (i)
re
Ans. (i) 28.7 pm = 28.7 pm × = 2.87 × 10 –11 m 0.5785
1 pm
(ii) 5 × 5.364
10−6 s (iii) 0.0125 + 0.7864 + 0.0215
ts
(ii) 15.15 μs = 15.15 μs × = 1.515 × 10–5 s Ans. Refer Solved Example 8 (Page 17).
1 μs
Q. 32. Use the data given in the following table to
gh
1g 1 kg calculate the molar mass of naturally occurring
(iii) 25365 mg = 25365 mg × 1000 mg ×
1000 g argon isotopes :
= 2.5365 × 10–2 kg Isotope Isotopic molar mass Abundance
ri
36
Q. 28. Which one of the following will have largest Ar 35.96755 g mol–1 0.337%
38
number of atoms ? ll Ar 37.96272 g mol–1 0.063 %
40
(i) 1 g Au (s) (ii) 1g Na(s) (iii) 1g Li (s) Ar 39.9624 g mol–1 99.600 %
.A
(iv) 1g of Cl2 (g) Ans. Refer Solved Example 26 (Page 36).
Ans. No. of atoms can be calculated as : Q. 33. Calculate the number of atoms in each of the
1 6.022 following :
(i) 1g Au = × 6.022 × 1023 = × 1023
rs
To calculate molarity, we need to calculate moles of 1000 mL of 0.75 M HCl contain HCl = 0.75 × 36.5 g
ethanol in 1 L of solution or nearly 1L of water ∴ 25 mL of 0.75 M HCl will contain HCl
because the solution is dilute.
M
0.75 × 36.5
1000 = × 25 = 0.684 g
No. of moles of water in 1 L of water = 1000
18
CaCO3 + 2HCl ⎯→ CaCl2 + CO2 + H2O
©
= 55.55 mole
100g 2 × 36.5 g
n ( C2H 5OH )
∴ = 0.040 2 × 36.5 g of HCl react completely with CaCO3 = 100 g
n ( C2H 5OH ) +55.55
0.684 g of HCl will react completely with CaCO3
n (C2H5OH) = 0.04 [n (C2H5OH)] + 2.222 100
= × 0.684 = 0.937 g
or 0.96 n (C2H5OH) = 2.222 2 × 36.5
.
ed
NCERT Exemplar Problems
rv
8. One mole of any substance contains 6.022 × 1023
se
atoms/molecules. Number of molecules of H 2SO 4
1. Two students performed the same experiment present in 100 mL of 0.02M H 2 SO 4 solution
separately and each one of them recorded two is _______.
re
readings of mass which are given below. Correct (a) 12.044 × 1020 molecules
reading of mass is 3.0 g. On the basis of given data (b) 6.022 × 1023 molecules
mark the correct option out of the following (c) 1 × 1023 molecules
statements. (d) 12.044 × 1023 molecules
ts
Student Readings 9. What is the mass percent of carbon in carbon
(i) (ii) dioxide ?
gh
(a) 0.034 % (b) 27.27 %
A 3.01 2.99
(c) 3.4 % (d) 28.7%
B 3.05 2.95
10. The empirical formula and molecular mass of a
ri
(a) Results of both the students are neither accurate compound are CH2O and 180 g respectively. What
nor precise. will be the molecular formula of the compound ?
(b) Results of student A are both precise and
accurate.
ll (a) C9H18O9 (b) CH2O
.A
(c) C6H12O6 (d) C2H4O2
(c) Results of student B are neither precise nor
11. If the density of a solution is 3.12 g mL–1, the
accurate.
mass of 1.5 mL solution in significant figures
(d) Results of student B are both precise and
rs
is _________ .
accurate.
(a) 4.7 g (b) 4680 × 10–3g
2. A measured temperature on Fahrenheit scale is
he
(a) 4 mol L–1 (b) 20 mol L–1 constituent elements by physical methods of
(c) 0.2 mol L–1 (d) 2 mol L–1 separation.
P
5. The number of atoms present in one mole of an 13. Which of the following statements is correct about
element is equal to Avogadro number. Which of the the reaction given below?
od
blood ? followed.
(a) 5 M (b) 50 M (c) Amount of Fe2O3 can be increased by taking any
(c) 0.005 M (d) 0.5 M one of the reactants (iron or oxygen) in excess.
7. What will be the molality of the solution containing
(d) Amount of Fe2O3 produced will decrease if the
18.25 g of HCl gas in 500 g of water ?
amount of any one of the reactants (iron or
(a) 0.1 m (b) 1 M
oxygen) is taken in excess.
(c) 0.5 m (d) 1 m
14. Which of the following reactions is not correct (b) Carbon forms two oxides namely CO2 and CO,
according to the law of conservation of mass? where masses of oxygen which combine with
(a) 2Mg (s) + O2 (g) ⎯→ 2MgO (s) fixed mass of carbon are in the simple ratio
2 : 1.
(b) C3H8 (g) + O2 (g) ⎯→ CO2(g) + H2O(g)
.
(c) When magnesium burns in oxygen, the amount
(c) P4(s) + 5O2(g) ⎯→ P4O10 (s)
ed
of magnesium taken for the reaction is equal to
(d) CH4(g) + 2O2(g) ⎯→ CO2(g) + 2H2O(g) the amount of magnesium in magnesium oxide
rv
15. Which of the following statements indicates that law formed.
of multiple proportions is being followed? (d) At constant temperature and pressure 200 mL
se
(a) Sample of carbon dioxide taken from any source of hydrogen will combine with 100 mL oxygen
will always have carbon and oxygen in the to produce 200 mL of water vapour.
ratio 1 : 2.
re
ANSWERS / HINTS
ts
MCQs Type-I
1. (b) : Average reading of student Thus, 12 g of He contain maximum number of
gh
3.01 + 2.99 atoms.
A= = 3.00 6. (c) Molar mass of glucose (C6H12O6)
2
ri
Average reading of student = 6 × 12 + 1 × 12 + 6 × 16 = 180
0.9
3.05 + 2.95 Molarity = = 0.005 M.
B= = 3.00 ll 180 × 1
2
Moles of HCl
.A
Correct reading = 3.0 7. (d) Molality = × 1000
Mass of water (in g)
For both the students average value is same as
18.25/36.5
the correct value. Hence readings of both the = × 1000 = 1 m.
500
rs
1000
0.02) and hence readings of student A are
precise. Thus, results of student A are both Molecules = 0.002 × 6.022 × 1023
= 12.044 × 1020
lis
44
5
= × (200 − 32) 10. (c) : Empirical formula mass
9
= 12 + 2 + 16 = 30
P
= 93.3°C
Molecular formula mass
5.85 / 58.5 n =
× 1000 Empirical formula mass
3. (c) : Molarity =
n
500 180
= =6
= 0.2 mol L–1
er
30
4. (b) M1 V 1 = M2 V 2 Molecular formula = (CH2O)6
5 M × 500 mL = M2 × 1500 mL = C6H12O6
od
4 = 4.7
5. (d) : 4 g He = = 1 mol = NA atoms. (upto 2 significant figures)
4
12. (c) A compound does not retain the physical
©
46
46 g Na = = 2 mol = 2NA atoms. properties of its constituent elements.
23
0.40 13. (a)
0.40 g Ca = = 0.01 mol = 10–2 NA atoms. 14. (b) Equation is not balanced.
40
12 15. (b) Statement (b) is correct according to law of
12 g He = = 3 mol = 3 NA atoms. conservation of mass.
4
.
(b) 6.022 × 1023 atoms of oxygen
ed
(c) 16 g of oxygen (c) 40 g of NaOH in 100 mL of solution
rv
17. Sulphuric acid reacts with sodium hydroxide as 20. 16 g of oxygen has same number of molecules as in
se
follows : (a) 16 g of CO (b) 28 g of N2
H2SO4 + 2NaOH ⎯→ Na2SO4 + 2H2O (c) 14 g of N2 (d) 1.0 g of H2
re
When 1L of 0.1M sulphuric acid solution is allowed 21. Which of the following terms are unitless ?
to react with 1L of 0.1M sodium hydroxide solution,
(a) Molality (b) Molarity
the amount of sodium sulphate formed and its
ts
molarity in the solution obtained is (c) Mole fraction (d) Mass percentage
22. One of the statements of Dalton’s atomic theory is
(a) 0.1 mol L–1
gh
(b) 7.10 g
given below :
(c) 0.025 mol L–1 (d) 3.55 g
“Compounds are formed when atoms of different
ri
18. Which of the following pairs have the same number elements combine in a fixed ratio.”
of atoms ? Which of the following laws is not related to this
(a) 16 g of O2(g) and 4 g of H2(g)
ll statement ?
.A
(b) 16 g of O2 and 44 g of CO2 (a) Law of conservation of mass
(b) Law of definite proportions
(c) 28 g of N2 and 32 g of O2
(c) Law of multiple proportions
rs
ANSWERS / HINTS
MCQs Type-II
lis
32
= 142
= 1 × 2 × NA atoms = 2 NA atoms
Mass of Na2SO4 formed = 0.05 × 142 = 7.10 g
©
12
Volume of solution after mixing = 2 L 12 g C = = 1 mol = NA atoms
12
H2SO4 left unreacted = 0.05 mol 23
23 g Na = = 1 mol = NA atoms
0.05 23
Molarity of solution = = 0.025 mol L–1
2 ∴ (c) and (d) are correct
20/40 = 1.78 M
19. (a, b) Molar conc. of NaOH solution = × 1000
200 Thus, (a) and (b) have the same concentration.
= 2.5 M. 16
20. (c, d) 16 g CO = = 0.57 mol = 0.57 NA molecules
0.5 28
.
Molar conc. of KCl solution = × 1000 = 2.5 M 28
ed
200 28 g N2 = = 1.0 mol = NA molecules
28
40/40
Molar conc. of NaOH solution = × 1000 14
14 g N2 = = 0.5 mol = 0.5 NA molecules
rv
100
28
= 10 M 1.0
1.0 g H2 = = 0.5 mol = 0.5 NA molecules.
se
20/56 2
Molar conc. of KOH solution = × 1000 21. (c, d) 22. (a, d)
200
re
Mass of P = 2 × 31 = 62
62
ts
Mass percentage of P = × 100 = 20%
Q.23. What will be the mass of one atom of C-12 in 310
grams ? Mass of O = 8 × 16 = 128
gh
Ans. 1 mole of carbon atoms = 12 g = 6.022 × 1023 atoms. 128
6.022 × 1023 atoms of carbon-12 have mass = 12 g Mass percentage of O = × 100 = 41.29%
310
ri
12 Q.28. 45.4L of dinitrogen reacted with 22.7L of
∴1 atom of carbon has mass =
6.022 × 1023 dioxygen and 45.4L of nitrous oxide was
= 1.99 × 10–23 g. ll formed. The reaction is given below :
Q.24. How many significant figures should be 2N2(g) + O2(g) ⎯→ 2N2O(g)
.A
present in the answer of the following
Which law is being obeyed in this experiment ?
calculations ?
Write the statement of the law.
2.5 × 1.25 × 3.5
Ans. Gases are reacting together to form gaseous products.
rs
2.01
Ratio of volumes of gases :
Ans. Least precise number in the calculations is 2.5 or
N2 : O2 : N2O = 45.4 : 22.7 : 45.4
he
the mole defined ? volumes. This law states that gases combine or are
Ans. Symbol for SI unit of mole is mol. produced in a chemical reaction in a simple whole
number ratio by volume provided that all gases are
ub
one kilogram of solvent whereas molarity is the (a) Is this statement true ?
er
number of moles of solute dissolved in one litre of (b) If yes, according to which law ?
solution. (c) Give one example related to this law.
Molality is independent of temperature whereas
od
phosphorus and oxygen in calcium phosphate (H2O) and hydrogen peroxide (H2O2) as :
Ca3(PO4)2. 1
H2 + O ⎯→ H2O
Ans. Molecular mass of Ca3(PO4)2 2 2
©
Water
= 3 × 40 + 2(31 + 4 × 16)
2g 16 g 18 g
= 310
Mass of Ca = 3 × 40 = 120 H2 + O2 ⎯→ H2O2
Hydrogen peroxide
120 2g 32 g 34 g
Mass percentage of Ca = × 100 = 38.71%
310
.
using the following data : 18 g
ed
Isotope % Natural abundance Molar mass 0.1 0.1
1 Mole fraction of NaOH = = = 0.047
H 99.985 1 2 + 0.1 2.1
2
rv
H 0.015 2 2 2
Mole fraction of water = = = 0.95
2 + 0.1 2.1
1 × 99.985 + 2 × 0.015 Mass of solution = Mass of water + Mass of NaOH
Ans. Average atomic mass =
se
99.985 + 0.015 = 36 g + 4 g = 40 g
= 1.00015 u. Volume of solution = 40 × 1 = 40 mL
re
Q.31. Hydrogen gas is prepared in the laboratory by (Since specific gravity of solution is 1 g mL–1)
reacting dilute HCl with granulated zinc. Moles of solute
Following reaction takes place. Molarity = × 1000
Volume of solution (in mL)
ts
Zn + 2HCl ⎯→ ZnCl2 + H2
0.1
Calculate the volume of hydrogen gas liberated = × 1000 = 2.5 M.
40
gh
at STP when 32.65 g of zinc reacts with HCl. Q.35. The reactant which is entirely consumed in
1 mol of a gas occupies 22.7 L volume at STP; reaction is known as limiting reagent. In the
Atomic mass of Zn = 65.3 u. reaction 2A + 4B → 3C + 4D, when 5 moles of A
ri
Ans. 1 mol of Zn i.e., 65.3 g of Zn produces 22.7 L of H2 react with 6 moles of B, then
22.7 (i) which is the limiting reagent ?
∴ 32.65 g of Zn will liberate H2 = × 32.65 ll
65.3 (ii) calculate the amount of C formed.
.A
= 11.35 L. Ans. Refer Conceptual Questions 3 , Q.9 (Page 77).
Q.32. The density of 3 molal solution of NaOH is 1.110
g mL–1. Calculate the molarity of the solution.
rs
1.110
= 1009.00 mL 37. Match the following physical quantities with units :
n
Moles of solute
Molarity = × 1000 Physical quantity Unit
Vol. of solution (in mL)
er
ANSWERS / HINTS
Matching Type
36. (i) – (b); (ii) – (c); (iii) – (a); (iv) – (e); (v) – (d).
37. (i) – (e); (ii) – (d); (iii) – (b); (iv) – (g); (v) – (c); (vi) – (f); (vii) – (a); (viii) – (i).
.
ed
(c) A is true but R is false.
rv
(d) Both A and R are false.
In the following questions a statement
of Assertion (A) followed by a statement of 40. Assertion (A) : Significant figures for 0.200 is 3
se
Reason (R) is given. Choose the correct option where as for 200 it is 1.
out of the choices given below each question. Reason (R) : Zero at the end or right of a number
re
38. Assertion (A) : The empirical mass of ethene is half are significant provided they are not on the right side
of its molecular mass. of the decimal point.
Reason (R) : The empirical formula represents the (a) Both A and R are true and R is the correct
ts
simplest whole number ratio of various atoms present explanation of A.
in a compound.
gh
(b) Both A and R are true but R is not a correct
(a) Both A and R are true and R is the correct
explanation of A.
explanation of A.
ri
(c) A is true but R is false.
(b) A is true but R is false.
(d) Both A and R are false.
(c) A is false but R is true. ll
(d) Both A and R are false. 41. Assertion (A) : Combustion of 16 g of methane gives
.A
18 g of water.
39. Assertion (A) : One atomic mass unit is defined as
one twelfth of the mass of one carbon-12 atom. Reason (R) : In the combustion of methane, water is
rs
explanation of A.
(a) Both A and R are true and R is the correct
explanation of A. (b) A is true but R is false.
lis
(b) Both A and R are true but R is not the correct (c) A is false but R is true.
explanation of A. (d) Both A and R are false.
ub
ANSWERS / HINTS
Assertion Reason Type
P
(ii) No. of molecules in 1.6 g or 0.05 mol Q. 45. A box contains some identical red coloured
= 6.022 × 10 23 22
× 0.05 = 3.011 × 10 . balls, labelled as A, each weighing 2 grams.
Another box contains identical blue coloured
Q.43. Calcium carbonate reacts wth aqueous HCl to balls, labelled as B, each weighing 5 grams.
give CaCl2 and CO2 according to the reaction Consider the combinations AB, AB2, A2B and
given below : A2B3 and show that law of multiple proportions
.
ed
CaCO3(s) + 2HCl(aq) → CaCl2(aq) + CO2(g) + H2O(l) is applicable.
What mass of CaCl2 will be formed when 250 Ans. Combination of A and B AB AB2 A2B A2B3
mL of 0.76 M HCl reacts with 1000 g of CaCO3? Mass of A (in g) 2 2 4 4
rv
Name the limiting reagent. Calculate the
Mass of B (in g) 5 10 5 15
number of moles of CaCl 2 formed in the
Masses of B which combine with fixed mass of A
se
reaction.
(say 4 g) are :
Ans. Refer Solved Example 124 (Page 75).
10 g , 20 g , 5g , 15 g
Q.44. Define the law of multiple proportions. Explain
re
it with two examples. How does this law point Ratio 2 : 4 : 1 : 3
to the existence of atoms ? This is a simple whole number ratio. Hence the law
Ans. Refer Text (Page 24). of multiple proportions is applicable.
ts
gh
ri
ll
.A
9. How many moles of Li2O are needed to completely
remove 50 kg of water?
rs
questions 1–5 that follow: Predict which of the following statements are true
Glucose (dextrose) solutions are given intravenously or false.
lis
3. What is the mole fraction of glucose in the ozone are 6.022 × 1023.
solution? 4. There is no difference between mass and weight of a
n
13. 1 gram atom of C and 1 gram atom of sulphur have 3. Assertion : The standard unit for expressing the
same mass. mass of atoms is a.m.u.
14. When 3.0 g of H2 react with 29.0g of O2 to form water, Reason : a.m.u. stands for mass of 1 atom of
O2 is the limiting reactant. carbon.
15. Nitrogen and oxygen combine to form N2O, NO and 4. Assertion : The sum of 154.2 + 6.1 + 23 is 183.
.
NO2. This is in accordance with law of reciprocal Reason : The result of addition is reported to the
ed
proportions. same number of decimal places as that
of the term with least number of decimal
Fill in the Blanks Questions places.
rv
5. Assertion : 1 mol of O and 1 mol of O2 contain equal
1. AZT is used for the victims of ……….
number of particles.
se
2. The prefix pico stands for ……….
Reason : 1 mol of molecules is always double than
3. Pascal are the units for the physical quantity ………. 1 mol of atoms in all diatomic molecules.
4. The number of significant figures in 0.00030 is ……….
re
6. Assertion : Graphite is an element.
5. Decimal equivalent of 2/3 is ………. upto three Reason : Element is the pure form of a substance
significant figures. containing same kind of atoms.
ts
6. The empirical formula of hydrogen peroxide is ………. 7. Assertion : Steam is a mixture.
7. The law which does not follow from Dalton’s atomic Reason : In a compound, the composition of the
theory is ……….
gh
elements must be fixed.
8. The mass of a molecule of carbon-14 dioxide (14CO2) 8. Assertion : Empirical and molecular formula of
is ………. g. Na2CO3 is same.
ri
9. An atom of sulphur is ……… times heavier than an Reason : Na2CO3 does not form hydrate.
atom of carbon.
9. Assertion : The empirical mass of ethene is half of
10. The ratio of atoms of hydrogen in 1 mole of methane
and 1 mole of sucrose (C12H22O11) is ……… .
ll its molecular mass.
.A
Reason : The empirical formula represents the
11. ………. mol of N2 are needed to produce 3.8 mol of simplest whole number ratio of various
NH3 by reaction with hydrogen. atoms present in a compound.
12. If mole fraction of sodium chloride in sodium chloride
rs
followed by a statement of reason is given. Choose One Word/Very Short Sentence Answer
the correct answer out of the following choices. 1. Define Avogadro’s law.
n
(a) Assertion and reason both are correct statements 2. What is meant by a.m.u. ?
and reason is correct explanation for assertion.
3. Define significant figure.
er
.
(i) same and (ii) different. (ii) sulphur from a mixture of carbon and sulphur ?
ed
18. Define mole in terms of number. 11. State Avogadro’s hypothesis. In what way, has it
given support to Dalton atomic theory ?
19. Balance the equation :
12. How can you deduce the atomicity of hydrogen with
rv
CaF2 + H2SO4 + H3BO3 ⎯⎯→ CaSO4 + BF3 + H2O the help of Avogadro’s hypothesis ?
20. How many atoms of carbon are present in 13. State the following laws of chemical combination and
se
0.1 mole of C12H22O11 ? give one example in each case
21. How many hydrogen atoms are present in 60 a.m.u. (i) Law of constant composition.
of ethane ?
(ii) Law of multiple proportions.
re
22. What is meant by one gram of atom of iron ? 14. What do you understand by a balanced chemical
23. What is the S.I. unit of density ? equation ? What quantitative information does a
24. Name the law which deals with the ratios of the balanced chemical equation convey ?
ts
volumes of the gaseous reactants and products. 15. Explain (i) molarity (ii) limiting reagent.
25. Which isotope of C is used for expressing relative 16. Write the balanced chemical equations for the
gh
atomic masses of elements ? following reactions :
26. An atom of an element is 13 times heavier than the (i) Manganese dioxide and concentrated hydrochloric
mass of a carbon atom. What is its mass in a.m.u. ? acid.
ri
27. What is the standard for the molecular weights of (ii) Sodium thiosulphate and iodine.
molecules ? (iii) Copper and dilute nitric acid.
28. What is the ratio of molar volumes of SO2 and SO3 ?
ll (iv) Sulphur dioxide and hydrogen sulphide.
.A
29. State law of reciprocal proportions.
17. Write the empirical formulae of the compounds
30. What volume will 250 g of mercury occupy ? having the following molecular formulae :
(Density of mercury = 13.6 g cm–3).
(i) C6H6 (ii) C6H12 (iii) H2O2 (iv) H2O
rs
1. What do you understand by the terms element, (i) H3PO3 ⎯⎯→ H3PO4 + PH3
compound and mixture ? Give two examples in each
(ii) Ca + H2O ⎯⎯→ Ca(OH)2 + H2
case.
lis
2. Explain the term mole. What does one mole of (iii) Fe2(SO4)3 + NH3 + H2O ⎯→ Fe(OH)3 + (NH4)2SO4
ammonia represent ? 19. What do you understand by the term formula mass ?
3. Give the SI units for (i) volume (ii) speed and How does it differ from molecular mass ?
ub
8. When two substances A and B are mixed together in Outline briefly the various steps for balancing a
a pestel and mortar, a large amount of heat is evolved chemical equation by hit and trial method.
and a new substance C is formed. C has the properties 4. Write short notes on
different from A and B. Is C an element, compound (i) Limiting reagent (ii) Avogadro hypothesis
or a mixture? (iii) Dalton’s atomic theory.
.
(a) 2 moles .......... .............. ............
7. One atom of nickel weighs 9.75 × 10–23 g. Calculate
ed
(b) ........ g ........g .........g ........2g. the atomic mass of nickel.
(c) ..... H ..... H 6 × 1023 H ..... H
8. How many molecules are present in 1 kg of
rv
atoms atoms atoms atoms hydrogen ?
(d) 6 × 1020 .....total .....total .....total
9. Calculate the total charge of a mole of electrons if
se
total atoms atoms atoms atoms the electrical charge on a single electron is
7. What are the main postulates of Dalton's 1.60 × 10–19 C.
atomic theory ? What were its limitations ? How has
10. The volume of a drop of rain was found to be 0.448
re
the theory been modified ?
ml at N.T.P. How many molecules of water and number
8. Define Avogadro number and mole. What is their
of atoms of hydrogen are present in this drop ?
importance ?
ts
9. What are the essentials of a chemical 11. Assuming the atomic mass of a metal M to be 56,
equation ? What is the information conveyed by a calculate the empirical formula of its oxide containing
70.0% M.
gh
chemical equation ?
10. Explain the following : 12. Calculate the number of molecules of oxygen in
(a) Gay Lussac law 150 ml of it at 20°C and 750 mm pressure.
ri
(b) Law of definite composition 13. How many moles of hydrogen, phosphorus and
(c) Empirical and molecular formula oxygen are there in 0.4 moles of phosphoric acid
(d) Relation between mole and volume of gases
ll (H3PO4) ?
14. A solution has been prepared by dissolving 5.6 g of
.A
(e) Limiting reagent.
KOH in 250 mL of it. Calculate the molarity of the
solution.
15. A chemist wishes to prepare 6.022 × 1024 molecules
rs
2. Express the following numbers to three significant 16. A sample of iron has a mass of 1.68 g. Calculate
figures : (a) the number of moles of iron present, (b) the number
of atoms of iron in the sample.
ub
3. Express the result of the following calculations to measured at 27°C and 760 mm pressure. Calculate
appropriate number of significant figures :
the percentage purity of the sample.
n
with the law of multiple proportions. 20. What volume of concentrated aqueous sulphuric acid
5. Calculate the amount in grams of : which is 98.0% H2SO4 by mass and has a density of
(a) 2.5 gram atoms of nitrogen 1.84 g mL–1 is required to prepare 10.0 L of 0.200 M
H2SO4 solution ?
(b) 3.6 gram mole of carbon dioxide
.
Passage Based Questions Moles of Li2O required to completely remove water =
ed
2.78 103 mol
5 10. The reactant which gets completely consumed in a
1. Moles of glucose = = 0.0278
180
rv
reaction is called limiting reactant.
Mass of water = 95 g
True or False Questions
0.0278
se
Molality = × 1000 = 0.293 m
95 1. False : Properties of a compound are entirely different
100 from those of the constituent atoms.
2. Volume of solution = = 98.04 mL
re
1.02 2. False : because 7.00 g is more precise than 7.0 g.
3. True
0.0278 4. False : Mass of a substance is a constant quantity but
Molarity = × 1000 = 0.284 M
98.04
ts
its weight varies from place to place.
95 5. True
3. Moles of water = = 5.278
18 6. False : Homogeneous as well as heterogeneous mixtures
gh
Total moles = 5.278 + 0.0278 = 5.3058 do not have sharp melting and boiling points.
7. True
0.0278
8. False : 0.006 g contains one significant figure while
ri
Mole fraction of glucose = = 0.0052
5.3058 5.00 × 10–3 contains 3 significant figures.
4. 98.04 mL of solution contain glucose = 5g 9. True
ll 10. True 11. True
5 12. True 13. False 14. False
250 mL of solution contain glucose = × 250
.A
98.04 15. False.
= 12.75 g Fill in the Blanks Question
12.75
rs
180 × volume
Assertion Reason Questions
Mass of glucose
0.05 = × 1000
ub
= 4.5 g
10. 0.286
90 × 103
n
6. Moles of water = = 5 × 103 mol 11. Both have same molar volume
18
er
30
Li2O is limiting reactant because 1 mol of Li2O reacts 19. 3CaF2 + 3H2SO4 + 2H3BO3 ⎯→ 3CaSO4 + 2BF3
with 1 mol of water + 6H2O
M
7. Moles of water which will react = 1.5 × 103 mol 20. 7.224 × 1023
Mass of water which will be removed 21. 12
= 1.5 × 103 × 18 = 27 × 103 g 22. One gram atom of iron means atomic mass of iron
©
.
Short Answer Questions
ed
is in agreement with law of multiple proportions.
3. (i) m3 (ii) m s–1 (iii) kg m s–2 or N 5. (a) 1 gram atom of nitrogen = 14 g
6. Compounds : (i), (v) ; 2.5 gram atoms of nitrogen = 14 × 2.5 = 35 g.
rv
Mixtures : (ii), (iii), (vi), (viii), (x); (b)1 gram mole of carbon dioxide = 44 g
Elements (iv), (vii). 3.6 gram mole of carbon dioxide = 44 × 3.6 = 158.4 g.
se
7. Tap water and clouds are homogeneous. 6. (i) 22.4 dm3 of CO2 at N.T.P. has molecules
18. (i) 4H3PO3 → 3H3PO4 + PH3 = 6.02 × 1023
3
(ii) Ca + 2H2O → Ca(OH)2 + H2 2.24 dm of CO2 at N.T.P. has molecules
re
(iii) Fe2(SO4)3 + 3NH3 + 3H2O ⎯→ 2Fe(OH)3 6.02 × 1023
+ 3(NH4)2 SO4 = × 2.24
22.4
20. (i) (c) (ii) (b). = 6.02 × 1022.
ts
Long Answer Questions (ii) Mass of 6.02 × 1023 atoms of oxygen = 16 g
Mass of 1 atom of oxygen
gh
6. (a) 4, 2, 4 moles (b) 21, 18, 37 g
16
(c) 6 × 103, 12 × 103, 12 × 103 H atoms = = 2.66 × 10–23 g.
6.02 × 10 23
(d) 12 × 1020, 10 × 1020, 8 × 1020
ri
(iii) No. of ozone molecules in 2 mol = 6.02 × 1023 × 2
Numerical Problems One ozone molecule = 3 atoms of oxygen
ll No. of oxygen atoms in 2 mol of ozone
1. (a) 2, (b) 3, (c) 5, (d) 3, (e) 3
= 6.02 × 1023 × 2 × 3
.A
2. (a) 6.03, (b) 2.36, (c) 6.00 × 104, (d) 2.86 = 3.61 × 1024.
7.5 × 206.8 (iv) Molecular mass of SO2 = 64
3. (a) = 30.232 = 30.
0.0512 × 1002 64 g of SO2 at N.T.P. occupy volume = 22.4 L
rs
= × 4.4
1.6523 64
0.015 = 1.54 L.
5.8673= 5.87 [upto two decimal places (as in 4.20] 7. Atomic mass is the mass of 6.022 × 1023 atoms
lis
figures as in 8.5)
4. Ist oxide 6.022 × 1023 × 1000
Mass of carbon = 42.9 g = = 3.01 × 1026.
n
2
Mass of oxygen = 100 – 42.9 = 57.1 g 9. 1 mole of electrons = 6.022 × 1023
er
=
57.1 18
= = 1.33 g
42.9 = 1.5 × 1022.
In second oxide,
1 molecule of water = 2 H atoms
Mass of oxygen combining with 27.3 g of carbon
= 72.7 g No. of hydrogen atoms = 1.5 × 1022 × 2 = 3.0 × 1022
Mass of oxygen combining with 1 g of carbon atoms.
11.
70 1.25
M 70 56 = 1.25 =1 2
.
56 1.25
ed
50 1.875
O 30 16 = 1.875 = 1.5 3
16 1.25
rv
Empirical formula = M2O3.
se
12. Volume of oxygen at N.T.P. p1 = 760 mm, p2 = 760 mm
p1 = 750 mm p2 =760 mm V1 = 300 mL, V2 = ?
re
V1 = 150 mL V2 = ? T1 =273 + 27 = 300 K, T2 = 273
T1 =273 + 20 = 293 K T2 = 273 K
p1 V1 p V p1 V1T2
p1 V1 p2 V2 pVT Applying = 2 2 or V2 =
V2 = 1 1 2 T1 T2 p2T1
ts
Applying = or
T1 T2 T1 p2
760 × 300 × 273
750 × 150 × 273 ∴ V2 = = 273 mL
∴ V2 = = 137.9 mL 760 × 300
gh
293 × 760
22400 mL of oxygen at N.T.P. has molecules Calculation of pure NaHCO3
= 6.022 × 1023 22400 mL of CO2 at N.T.P. are produced by heating
ri
137.9 mL of oxygen at N.T.P. has molecules 168 g of NaHCO3
ll 273 mL of CO2 at N.T.P. are produced by heating
6.022 × 1023 × 137.9
= 168
22400 NaHCO3 = × 273 = 2.0475
.A
22400
= 3.71 × 1021. Wt. of sample taken = 2.5 g
13. 0.4 moles of H3PO4 ≡ 0.4 × 3 = 1.2 mol of H 2.0475
rs
5.6 56 36.5
14. Moles of KOH = = 0.1
56 56 g of KOH require HCl for neutralisation = 36.5 g
lis
S + O2 ⎯⎯⎯→ SO2 40
1 mole of SO2 is prepared from = 1 gram atom of S Cost = 11.4 × 6 = ` 68.4
n
10 mole of SO2 are prepared from = 10 gram atoms of S 2 KOH + H2SO4 ⎯⎯⎯→ K2SO4 + 2H2O
er
56
98
(b) 1 mole of iron = 6.022 × 1023 atoms = × 7 × 103 g = 6.125 kg
112
0.03 mole of iron = 6.022 × 1023 × 0.03 = 1.8 × 1022.
©
6.125 × 100
17. 2NaHCO3 ⎯⎯⎯⎯→ Na2CO3 + CO2 + H2O Wt. of 80% H2SO4 = = 7.66 kg
80
2 (23 + 1 + 12 + 48) 22.4 L
Cost = 7.66 × 3.50 = ` 26.81
= 168 g at N.T.P.
∴ It is cheaper to neutralise KOH by H2SO4 than
Volume of CO2 at N.T.P. HCl.
.
H 6.50 1 = 6.50 = 11 11
ed
1 0.59
28.4 1.77
O 28.4 16 = 1.77 =3 3
16 0.59
rv
8.28 0.59
N 8.28 14 = 0.59 =1 1
14 0.59
se
Empirical formula = C8H11O3N.
re
20. 98.0% H2SO4 means that 98 g of H2SO4 is present in Now applying molarity equation,
100 g of solution.
M1V1 = M2V2
98
Moles of H2SO4 = = 1 mol 18.4 × V1 = 0.20 × 10.0L
ts
98
100 0.20 × 10
Volume of solution = = 54.35 mL ∴ V1 = = 0.1087L
gh
1.84 18.4
1
Molarity = × 1000 = 18.4 M = 108.7 mL.
54.35
ri
ll
.A
Q.1. In the combustion of methane in air, what solution (1000 mL solution = amount of solute + amount of
rs
is the limiting reactant and why ? solvent). Thus, 1 molar solution is more concentrated.
Ans. Methane is the limiting reactant because the Q.4. Will the molarity of a solution at 50°C be
he
other reactant is oxygen of the air which is always present same, less or more than molarity at 25°C?
in excess. Thus, the amounts of carbon dioxide and water Ans. Molarity at 50°C of a solution will be less than
formed will depend upon the amount of CH4 burnt.
lis
1kg-mole of CH4 contains 6.022 × 1026 molecules. Ans. No, law of constant composition is not true for
n
Since one molecule of methane contains 10 electrons all types of compounds. It is true for only those compounds
and therefore, 1 kg mole of CH4 contains 10 × 6.022 × 1026 which are obtained from one isotope. For example, carbon
er
= 6.022 × 1027 electrons. exists in two common isotopes : 12C and 14C. When it forms
12
CO2, the ratio of masses is 12 : 32 or 3 : 8. However, when
Q.3. Which aqueous solution has higher
od
Q.7. What is the difference in expressing a From the data find out
weight of a solid as 36.5 × 103 g and 36.50 × 103 g ? (i) atomic masses of the elements A, B, C and D,
Ans. 36.5 × 103 g has three significant figures while (ii) simple ratio,
36.50 × 10 3 has four significant figures. Hence 36.50
represents greater accuracy than 36.5. (iii) molecular formula of the compound.
.
Ans. Step I. To calculate the atomic masses :
ed
Q.8. How many significant figures are there
in ‘π
π’ ? The relative number of atoms
Ans. Infinite number. Percentage of element
rv
=
Q.9. In calculations involving more than one Atomic mass
arithmetic operation, rounding off to the proper Percentage of element
se
number of significant figures may be done once at Atomic mass =
Relative number of atoms
the end if all the operations are multiplication and/
re
or division or if they are all additions and/or 9.76
∴ Atomic mass of A = = 24,
subtractions but not if they are combinations of 0.406
additions or subtractions with multiplications or
13.01
divisions. Explain. Atomic mass of B = = 32
ts
0.406
Ans. There are different rules for the number of
significant digits in the answer to an addition or subtraction 26.01
gh
∴ Atomic mass of C = = 16,
and to a multiplication or division. Therefore, they must be 1.625
applied separately when a mixed calculation is performed. 51.22
Atomic mass of D = = 18
ri
Q.10. Calculate the molarity of water if its density 2.846
is 1000 kg/m3. Step II. To calculate the simple ratio of atoms.
Ans. Molarity of water means the number of moles of ll
water in 1 litre of water. Element Relative no. of atoms Simple atomic ratio
.A
1 L of water = 1000 cm3 = 1000 g 0.406
A 0.406 =1
3 3
(∵ 1000 kg/m = 1g/cm ) 0.406
1000 0.406
rs
C 1.625 =4
∴ Molarity = 55.56 M. 0.406
Q.11. Sulphuric acid is generally available in 2.846
=7
D 2.846
lis
0.50 × 250
∴ V2 = = 6.94 mL
er
B 13.01 0.406 on the assumption that all the hydrogen atoms in the
compound are present in combination with oxygen
C 26.01 1.625 as water of crystallisation. The molecular weight of
D 51.22 2.846 the compound is 248 a.m.u.
.
Na 18.59 23 Amount of Pb(NO3)2 in the mixture = (5 – x) g
ed
23 0.80
= 0.80 =1 2NaNO3 ⎯⎯→ 2NaNO2 + O2 ↑
25.80 0.80 2(23 + 14 + 3 × 16) 2 × 16
rv
S 25.80 32
32 0.80 = 170 g = 32
= 0.80 =1 2Pb(NO3)2 ⎯⎯→ 2PbO + O2↑ + 4NO2 ↑
se
4.03 4.03
H 4.03 1 2(207 + 28 + 96) 2 × 16 4 × (14 +32)
1 0.80
= 4.03 =5 = 662 g = 32 = 184 g
re
51.58 3.22 170 g of NaNO3 give O2 = 32 g
O 51.58 16
16 0.80 32
= 3.22 =4 x g of NaNO3 give O2 = ×x g
ts
170
Thus, the simple ratio of Na : S : H : O is 1 : 1 : 5 : 4. Similarly,
gh
Therefore, the empirical formula of the compound is 662 g of Pb(NO3)2 give O2 and NO = 216 g
NaSH5O4.
Step II. Calculation of molecular formula 216
(5 –x) g of Pb(NO3)2 give gases = × (5 − x) g
ri
662
Empirical formula mass = 23 + 32 + 5
× 1 + 4 × 16 = 124 32 x 216
ll Total loss on heating = + (5 − x)
Molecular mass = 248 170 662
.A
Molecular mass 248 28 × 5
n= = =2 Actual loss on heating = 28% of 5 g = = 1.4 g
Empirical formula mass 124 100
32 x 216
∴ + (5 − x) = 1.4
rs
Since all the 10 hydrogen atoms are present as water ∴ Wt. of NaNO3 = 1.676 g
molecules (H2O), the total number of water molecules is
Wt. of Pb(NO3)2 = 5 –1.676
5H2O.
lis
= 3.324 g.
Molecular formula = Na2S2O3 . 5H2O
Q.16. A sample of hard water contains 20 mg of
Q.14. The vapour density of a mixture of NO2 and Ca2+ ions per litre. How many milliequivalents of
ub
N2O4 is 38.3 at 26.7°C. Calculate the number of moles Na2CO3 would be required to soften 1 litre of sample ?
of NO2 in 100 g of the mixture.
Ans. Ca2+ + Na2CO3 ⎯⎯→ CaCO3 + 2Na+
Ans. V.D. of mixture of NO2 and N2O4 = 38.3
P
40 g 106 g
Mol. wt. of mixture of NO2 and N2O4 = 38.3 × 2 = 76.6
40 g of Ca2+ react with Na2CO3 = 106 g
(Mol. wt. = 2 × V.D.)
n
46 1 × 10 −3
= 0.437 Milliequivalents of Na2CO3 =
10 −3
Q.15. A solid mixture (5.0 g) consisting of lead = 1 milliequivalent.
nitrate and sodium nitrate is heated below 600°C until
.
ed
What is the percentage of Mn in the ignited sample ?
(At. wt. of Mn = 55). P4 + 3O2 ⎯⎯→ P4O6
rv
3 (55 + 2 × 16) 3 × 55 + 4 × 16 What are the masses of P4O6 and P4O10 that will
= 261 g = 229 g be produced by the combustion of 2.0 g of P4 in 2.0 g
se
Let the amount of pyrolusite ignited = 100 g of oxygen leaving no P4 and O2 ?
Wt. of MnO2 = 80 g
Ans. P4O6 and P4O10 are formed as :
re
Wt. of SiO2 and other inert contents = 15 g
P4 + 3O2 ⎯⎯→ P4O6
Wt. of water = 100 – (80 + 15) = 5 g
P4 + 5O2 ⎯⎯→ P4O10
Now, 261 g of MnO2 gives Mn3O4 = 229 g
ts
Let x be the mass of P4 that is converted into P4O6 so
229
80 g of MnO2 give Mn3O4 = × 80 that
gh
261
= 70.19 g. Mass of P4 which is converted to P4O10 = 2 – x
ri
During ignition, water present in pyrolusite is Mass of oxygen required for forming P4O6
removed while SiO2 and other inert contents remain as such.
x
ll = × 96
Total wt. of residue = 70.19 + 15 = 85.19 g 4 × 31
.A
Mass of oxygen required for forming P4O10
70.19
∴Percentage of Mn3O4 in the residue = × 100
85.19 2− x
= × 160
rs
= 82.39% 4 × 31
3 × 55 3 × 55 + 4 × 16
⎛ x ⎞ ⎛ 2− x ⎞
= 165 = 229 ⎜⎝ × 96⎟ + ⎜ × 160⎟ = 2.0
4 × 31 ⎠ ⎝ 4 × 31 ⎠
229 g of Mn3O4 contain Mn = 165
lis
∴
96 x 320 160 x
165 or + − = 2.0
82.39 g of Mn3O4 contain Mn = × 82.39 124 124 124
ub
229
64 x 320
= 59.36 g or − = 2.0 –
124 124
∴ Percentage of Mn in ignited sample = 59.36%
P
64 x 72
Q.18. The density of gold is 19.3 g cm–3. Calculate or − = −
124 124
the diameter of a solid gold sphere having a mass of
n
422 g. 72
x = = 1.125 g
er
Mass 64
Ans. Volume of gold sphere = Mass of P4O6 formed
Density
od
Mass of P4
=
422 g = × Molar mass of P4O6
19.3 g cm −3 Molar mass of P4
M
1.125
= 21.865 cm3 = × 220 = 1.996 g
124
4 3
Volume of sphere = πr Mass of P4O10 formed
©
3
1/ 3 Mass of P4
⎛ 3⎞ = × Molar mass of P4O10
or radius, r = ⎜ Volume ×
4 π ⎟⎠
⎝ Molar mass of P4
1/3
⎛ 21.865 × 3 × 7 ⎞ 0.875
= ⎜ = × 284 = 2.004 g
⎝ 4 × 22 ⎠⎟ 124
.
ed
rv
CONCEPT OF EQUIVALENT MASS 1 part by mass of hydrogen ≡
se
The equivalent mass of a substance is defined 49 parts by mass of H2SO4
as the number of parts by mass of a substance which
combines with or displaces directly or indirectly 1.008 Hence, equivalent mass of H2SO4 = 49
re
parts by mass of hydrogen or 8 parts by mass of oxygen Equivalent mass can also be calculated from
or 35.5 parts by mass of chlorine. the formulae of the compounds.
ts
The following examples clearly illustrate the definition For example :
of equivalent mass :
• AlCl3 : 3 × 35.5 parts by mass of chlorine combine
• Chlorine combines with hydrogen to form hydro-
gh
with 27 parts by mass of Al.
gen chloride; HCl as :
35.5 parts by mass of chlorine will combine with
H2 + Cl2 ⎯⎯→ 2 HCl
ri
9 parts by mass of Al
2 71
ll ∴ Equivalent mass of Al = 9
Hence, 2 parts by mass of hydrogen combine with
71 parts by mass of chlorine. • FeO : 16 parts by mass of oxygen ≡ 56 parts by
.A
mass of Fe
or 1 part by mass of hydrogen will combine with
35.5 parts by mass of chlorine. 8 parts by mass of oxygen ≡ 28 parts by mass of Fe
rs
taken, instead of 1.008 parts] The mass of a substance in grams which is nu-
• Magnesium wire burns in oxygen gas to form merically equal to its equivalent mass is called
gram equivalent mass or one gram equiva-
lis
magnesium oxide :
lent. It is simply the equivalent mass of a sub-
2Mg + O2 ⎯⎯→ 2MgO
stance expressed in grams. For example, 1 gram
2 × 24 32
ub
Hence, equivalent mass of magnesium is 12. It may be noted that the elements with variable
•
er
Zinc metal reacts with dil. H2SO4 to liberate valency generally exhibit variable equivalent masses.
hydrogen gas : The equivalent mass of an element is related to its
Zn + H2SO4 ⎯⎯→ ZnSO4 + H2 valency as,
od
65 98 2 Atomic mass
2 parts by mass of hydrogen is liberated by 65 Equivalent mass =
Valency
M
mass of H+ ions in its aqueous solutions or one The number of OH− ions that one molecule of a
part by mass of replaceable hydrogen. base can give in its aqueous solution is called its
.
acidity. Therefore,
ed
The number of H+ ions that one molecule of an
acid can give in its aqueous solution is known as its Molecular mass
basicity. Equivalent mass of a base =
Acidity
rv
Therefore, For example,
Molecular mass
se
Equivalent mass of an acid = NaOH Na+ + OH− Acidity = 1
Basicity
For example, Mol. mass
Eq. mass =
re
1
HCl H+ + Cl− Basicity = 1 Ca(OH)2 Ca2+ + 2OH− Acidity = 2
Mol. mass
Eq. mass = Mol. mass
ts
1 Eq. mass =
2
Al(OH)3 Al3+ + 3OH− Acidity = 3
gh
H2SO4 2H+ + SO42 − Basicity = 2
Mol. mass
Eq. mass =
Mol. mass 3
ri
Eq. mass = Equivalent mass of a salt
2
and H3PO4 3H+ + PO43− Basicity = 3 ll The equivalent mass of a salt is that mass of it
which contains one equivalent mass of metal.
.A
Mol. mass Equivalent mass of salt
Eq. mass =
3
Mol. mass of salt
Equivalent mass of a base is the number of =
Total valency of the metal in the salt
rs
17 parts by mass of OH− ions in an aqueous atoms present in the formula of the salt. For example,
solution.
lis
(i) Equivalent mass of KMnO4 in the reaction : This relation can be used to calculate atomic
mass.
2KMnO4 + 3H2SO4 → K2SO4 + 2MnSO4 + 3H2O + 5(O)
.
First approximate atomic mass may be
ed
2(39 + 55 + 64) 5 × 16
calculated as:
= 316
6.4
rv
5 × 16 parts of oxygen are liberated from KMnO4 Approximate atomic mass =
Specific heat
= 316 parts
se
From this, valency can be calculated as
8 parts of oxygen are liberated from
316 Approx. atomic mass
re
KMnO4 = × 8 = 31.6 Valency =
5 × 16 Equivalent mass
∴ Eq. mass of KMnO4 = 31.6 Take the valency to nearest whole number, then
ts
(ii) Equivalent mass of K2Cr2O7 in the reaction : Exact atomic mass = Equivalent mass × Valency
gh
K2Cr2O7 + 4H2SO4 → K2SO4 + Cr2(SO4)3 + 4H2O + 3(O) EUDIOMETRY
2 × 39 + 2 × 52 + 7 × 16 3 × 16
This method
= 294 = 48
has been used for
ri
48 parts of oxygen are liberated from K2Cr2O7 = 294 determining the
8 parts of oxygen are liberated from K2Cr2O7 molecular formula
ll
or percentage
294
.A
= × 8 = 49 composition of a
48
gaseous mixture of
∴ Eq. mass of K2Cr2O7 = 49
hydrocarbons. The
rs
graduated tube
2(56 + 32 + 64) 16
open at one end.
= 304
The closed end is provided with platinum electrodes
lis
16 parts of oxygen react with FeSO4 = 304 parts for the passage of electricity through the gas. Such
304 tube is called eudiometer tube. It is filled with
8 parts of oxygen react with FeSO4 = × 8 = 152
ub
respectively.
316
8 parts of oxygen react with Na2S2O3 = × 8 = 158 (iii) The mixture is then cooled so that water vapour
16
M
DULONG AND PETIT’S LAW (iv) KOH is introduced which absorbs CO2 and only
According to Dulong and Petit’s law, the product unused O2 is left. Thus,
of atomic mass and specific heat for a solid element is
decrease in volume on adding KOH = volume of
approximately equal to 6.4.
CO2 produced.
Atomic mass × Specific heat = 6.4
The volume of unused O2 can be measured by Volume of CO2 produced for 8 mL of hydrocarbon
absorbing in alkaline solution of pyrogallol or = 8 x mL
.
calculated by difference method.
ed
∴ 8x = 16 or x=2
Eudiometry is based on Avogadro’s law which
states that equal volumes of all gases under similar Contraction in volume on cooling for 8 mL of
rv
conditions of temperature and pressure contain equal ⎛ y⎞
number of molecules or moles. This means that hydrocarbon = 8 ⎜⎝ 1 + ⎟⎠
4
volume ratio among gases is same as mole ratio at
se
same conditions of temperature and pressure. ⎛ y⎞
∴ 8 ⎜⎝ 1 + ⎟⎠ = 16 or 8 + 2y = 16 or y = 4
The equation for combustion of hydrocarbon is 4
re
⎛ y⎞ y Hence, formula of compound is C2H4.
CxHy(g) + ⎜⎝ x + ⎟⎠ O2(g) ⎯→ x CO2(g) + H O(l)
4 2 2
Illustration 2. 10 mL of a gaseous hydrocarbon was
ts
⎛ y⎞ y burnt completely in 100 mL of O2 at NTP. On cooling
1 mol ⎜⎝ x + ⎟⎠ mol ⎯→ x mol + mol
4
gh
2 the gas occupied 80 mL at N.T.P. This volume became
⎛ y⎞ 50 mL on treatment with KOH solution. What is the
1 cc ⎜⎝ x + ⎟⎠ cc ⎯→ x cc – formula of the hydrocarbon?
ri
4
Values of x and y can be calculated from the Solution. Volume of CO 2 produced + Volume of
ll
following relations: unreacted O2 = 80 mL
.A
Volume of O2 used per cc of hydrocarbon Volume of unreacted O2 (CO2 absorbed by KOH)
= 50 mL
⎛ y⎞
= ⎜⎝ x +
rs
⎡ ⎛ y⎞ ⎤ ⎛ y⎞
= ⎢1 + ⎜ x + ⎟ ⎥ − x = ⎜ 1 + ⎟ cc ⎛ y⎞ y
⎣ ⎝ ⎠
4 ⎦ ⎝ 4⎠ Cx Hy(g) + ⎜ x + ⎟ O2(g) ⎯→ x CO2(g) + H O (l)
⎝ 4⎠ 2 2
ub
of the hydrocarbon. ⎛ y⎞
10 ⎜ x + ⎟ = 50 mL
⎝ 4⎠
Solution.
od
.
ed
A8. Which of the following has maximum number of atoms?
cwise MULTIPLE CHOICE QUESTIONS (a) 18 g of water (b) 16 g of O2
i
Top (c) 4.4 g of CO2 (d) 16 g of CH4.
rv
A9. Which of the following contains more molecules ?
Select the Correct Answers:
(a) 1 g CO2 (b) 1 g N2.
se
Matter, Physical Measurements, Laws of (c) 1 g H2 (d) 1 g CH4.
Combination A10. Which of the following weighs least :
A1. The number of significant figures in 0.0101 is : (a) 2.24 litres of CO2 at N.T.P.
re
(a) 3 (b) 2 (b) 6.02 × 1021 molecules of CO2
(c) 4 (d) 5. (c) 1 g of CO2
A2. Two elements A and B combine to form two compounds (d) 6.02 × 1022 atoms of carbon.
ts
in which ‘a’ gm of A combines with b1 and b2 gm of B A11. The number of atoms present in 0.1 mole of P4 are :
respectively. According to law of multiple proportions, (a) 2.4 × 1023 atoms
(a) b1 = b2
gh
(b) 6.02 × 1022 atoms
(b) b1 and b2 bear a simple whole number ratio
(c) same as in 0.2 mole of S8
(c) a1 and b1 bear whole number ratio
(d) no relation exists between b1 and b2. (d) same as in 3.1 g of phosphorus
ri
A3. Two different oxides of a metal contain 20% and (atomic mass of P = 31).
27% oxygen by weight. This is in accordance with A12. The total number of atoms in 8.5 g of NH3 is :
the law of : (a) 9.03 × 1023 (b) 3.01 × 1023
ll
(c) 1.204 × 1023 (d) 6.02 × 1023
(a) conservation of mass (b) constant composition
.A
A13. The number of atoms of oxygen present in
(c) multiple proportion (d) reciprocal proportion.
11.2 L of ozone at N.T.P. are :
A4. Two elements X and Y have atomic masses 14 and 16
(a) 3.01 × 1022 (b) 6.02 × 1023
respectively. They can form a series of five compounds (c) 9.03 × 1024 (d) 1.20 × 1024.
rs
A, B, C, D and E in which for the same amount of X, Y A14. The number of atoms of He in 104 a.m.u. is :
is present in the ratio of 1: 2: 3: 4: 5. If the compound (a) 3.1 × 1025 (b) 6.2 × 1025
he
(d) 4.1 parts by weight of X water in a glass. The number of oxygen atoms in the
A5. Two oxides of an element contain 57.1% and 72.7% of solution are :
oxygen. If the first oxide is MO, the second oxide is : (a) 3.66 × 1026 (b) 6.6 × 1023
P
A6. Calculate to the correct number of significant (a) 6.02 × 1023 atoms of H
figures : 4.26 − (15.635/5.0)
(b) 4 gram atoms of hydrogen
er
(d) 1.5 mole of water. (a) N/2 (b) 3N/2 (c) 2N (d) 6 N.
A1. (a) A2. (b) A3. (c) A4. (b) A5. (d) A6. (c) A7. (d) A8.
(d) A9. (c) A10. (b) A11. (a)
A12. (a) A13. (c) A14. (c) A15. (c) A16. (c) A17. (b) A18. (c)
A19. Which of the following contain highest number of (a) twice that in 60 g carbon
atoms ? (b) 6.023 × 1022
.
(a) 1.0 g of water (b) 1.0 g of silver
ed
(c) half that of 8 g He
(c) 1.0 g of nitrogen (d) 1.0 g of propane (C3H8). (d) 558.6 × 6.023 × 1023
A20. Which of the following has maximum mass ?
(a) 1.0 mole of H2 gas Percentage Composition, Molecular formula
rv
(b) 0.5 mole of sucrose (C12H22O11) and Stoichiometry
(c) 1.2 mole of silver A31. The empirical formula of sucrose is :
se
(d) 22.4 L of N2 at N.T.P. (a) CH2O (b) CHO
A21. Number of molecules present in 1 ml of water is : (c) C12H22O11 (d) C(H2O)2.
(a) 1 (b) 1000 A32. One mole of calcium phosphide on reaction with excess
re
(c) 2.69 × 1019 (d) 6.02 × 1020. of water gives
A22. 2 g of oxygen contain number of atoms equal to that (a) One mole of phosphine
contained by : (b) Two moles of phosphoric acid
ts
(a) 0.5 g hydrogen (b) 4 g sulphur (c) Two moles of phosphine
(c) 7 g nitrogen (d) 2.3 g sodium. (d) One mole of phosphorus pentoxide
A23. 40 g of caustic soda contain :
gh
A33. A sample of water contains x % of D2O. Its molecular
(a) 6. 02 × 1023 atoms of H weight is 19. The value of ‘x’ is
(b) 22.4 litres of N2
(a) 25 (b) 50 (c) 33.33 (d) 75
(c) 6.02 × 1024 molecules of O2
ri
(d) 4 g of Na. A34. A compound contains 8% sulphur. The minimum
A24. 0.6 g of carbon was burnt in air to form CO2. The molecular weight of the compound is
number of molecules of CO2 introduced into air will (a) 100 (b) 200
ll (c) 350 (d) 400
be : A35. An aqueous solution of 6.3 g of oxalic acid dihydrate
.A
(a) 6.02 × 1022 (b) 3.01 × 1022 is made upto 250 mL. The volume of 0.1 N NaOH
required to completely neutralize 10 mL of this
(c) 6.023 × 10 (d) 3.01 × 1023.
23
solution is :
A25. The total number of electrons present in 3.2 g of
(a) 40 mL (b) 20 mL
rs
methane are :
(a) 2 × 10 × 6.022 × 1022 (b) 10 × 6.022 × 1023 (c) 10 mL (d) 5 mL
(c) 10 × 6.022 × 1022 (d) 6.022 × 1023. A36. Mixture X = 0.02 mol of [Co(NH 3 ) 5 SO 4 ]Br and
he
A27. Haemoglobin contains 0.33% of iron by weight. The 1 L of mixture X + excess of BaCl2 → Z
molecular mass of haemoglobin is about 67200. The Number of moles of Y and Z are :
number of iron atoms (at mass of Fe = 56) present (a) 0.01, 0.01 (b) 0.02, 0.01
ub
6.022 1
(c) × 1054 (d) × 108 aforesaid conditions in the end ?
er
(c) three (d) two A38. A gaseous mixture contains 50% He and 50% CH4 by
A30. Number of atoms in 558.6 g Fe (atomic mass of volume. What is the percent by weight of CH4 in the
Fe = 55.86 g mol–1) is mixture ?
©
A19. (d) A20. (b) A21. (c) A22. (b) A23. (a) A24. (b) A25. (a) A26. (d) A27. (b) A28. (d) A29. (d)
A30. (a) A31. (c) A32. (c) A33. (b) A34. (d) A35. (a) A36. (a) A37. (b) A38. (d)
A39. The mass of carbon anode consumed (giving only (a) 10⋅40 M (b) 5⋅70 M
carbon dioxide) in the production of 270 kg of (c) 12⋅38 M (d) 13⋅46 M
.
aluminium metal from bauxite by Hall process is : A44. The molarity of a solution obtained by mixing 800
ed
(a) 180 kg (b) 270 kg mL of 0.5 M HCl with 200 mL of 1 M HCl will be
(c) 540 kg (d) 90 kg (a) 0.8 M (b) 0.6 M
(c) 0.4 M (d) 0.2 M
rv
A40. The crystalline salt Na2SO4.xH2O on heating loses A45. 4L of water is added to 2L of 6M HCl. The molarity
55.9 % of its weight. The formula of crystalline salt is of the final solution is
se
(a) Na2SO4.5H2O (b) Na2SO4.7H2O (a) 4 M (b) 2 M
(c) Na2SO4.2H2O (d) Na2SO4 . 10H2O (c) 1 M (d) 0⋅5 M
A46. The volume of 10.50 M solution required to prepare
A41. 20 kg of N2(g) and 3.0 kg of H2(g) are mixed to produce
re
1.0 L of 0.25 M solution of HNO3 is :
NH3(g). The amount of NH3(g) formed is
(a) 250 mL (b) 500 mL
(a) 17 kg (b) 51 kg (c) 230 mL (d) 23.8 mL.
(c) 60 kg (d) 34 kg A47. The moles of sodium chloride in 250 cm3 of 0.50 M
ts
A42. A phosphorus oxide has 43.6% phosphorus (at. mass NaCl are :
= 31). The empirical formula of the compound is (a) 0.250 mol (b) 2 mol
gh
(a) P2O5 (b) P4O6 (c) 0.125 mol (d) 1.0 mol.
A48. 6 mL of a gaseous hydrocarbon was exploded with
(c) P2O3 (d) P4O8
excess of oxygen and the product cooled. A contraction
A43. Commercially available concentrated HCl contains
ri
of 9 mL was observed. A further contraction of 12 mL
38⋅0% HCl by mass (density = 1⋅19 g mL–1). The was observed on treatment with aqueous KOH. The
molarity of the solution is ll formula of hydrocarbon is
(a) CH4 (b) C2H4
(c) C2H6 (d) C2H2
.A
A39. (d) A40. (d) A41. (a) A42. (a) A43. (c) A44. (b) A45. (b) A46. (d) A47. (c) A48. (d)
rs
gas (C3H8) measured under the same conditions? (a) 6.02 × 1022 (b) 6.02 × 1021
(a) 6 L (b) 5 L (c) 6.02 × 1020 (d) 6.02 × 1018
(c) 10 L (d) 7 L (AMU Med. 2010)
(C.B.S.E. PMT 2008)
P
(C.B.S.E. PMT 2008) (c) 16 moles (d) 4 moles
B4. Which of the following concentration terms is/are (e) 1 mole (Kerala PMT 2010)
independent of temperature ? B9. One kilogram of a sea water sample contains 6 mg of
©
B1. (b) B2. (c) B3. (b) B4. (b) B5. (b) B6. (c) B7. (b) B8. (d) B9. (b)
B10. 25.3 g of sodium carbonate, Na2 CO3 is dissolved in B18. The mass of CaCO3 required to react completely with
enough water to make 250 mL of solution. If sodium 20 mL of 1.0 M HCl as per the reaction:
.
carbonate dissociates completely, molar concentration
ed
CaCO3 + 2HCl → CaCl2 + CO2 + H2O is
of Na+ and CO32– ions are respectively (molar mass of
Na2CO3 = 106 g mol–1) (At. mass: Ca = 40, C = 12, O = 16)
(a) 0.477 M and 0.477 M (a) 1 g (b) 2 g
rv
(b) 0.955 M and 1.910 M (c) 10 g (d) 20 g
(c) 1.910 M and 0.955 M (e) 200 g (Kerala PMT 2015)
se
(d) 1.90 M and 1.910 M (C.B.S.E PMT 2010) B19. Which one of the following has maximum number of
B11. The number of atoms in 0.1 mol of triatomic gas is molecules?
(NA = 6.02 × 1023) (a) 16 g of O2 (b) 16 g of NO2
re
(a) 1.800 × 1022 (b) 6.026 × 1022 (c) 4 g of N2 (d) 2 g of H2
(c) 1.806 × 1023 (d) 3.600 × 1022 (e) 32 g of N2 (Kerala PMT 2015)
(C.B.S.E. PMT 2010)
B20. A mixture of gases contains H2 and O2 gases in the
ts
B12. Which one of the following sets of compounds correctly
ratio of 1 : 4 (w/w). What is the molar ratio of the two
illustrate the law of reciprocal proportions ? gases in the mixture?
gh
(a) P2O3, PH3, H2O (b) P2O5, PH3, H2O
(a) 16 : 1 (b) 2 : 1
(c) N2O5, NH3, H2O (d) N2O, NH3, H2O
(c) 1 : 4 (d) 4 : 1
(e) NO2, NH3, H2O (Kerala P.M.T. 2011) (A.I.P.M.T. 2015)
ri
B13. 20.0 kg of N2(g) and 3.0 kg of H2(g) are mixed to B21. If Avogadro number NA is changed from 6.022 × 1023
produce NH3(g). The amount of NH3(g) formed is mol–1 to 6.022 × 1020 mol–1, this would change:
(a) 17 kg
(c) 20 kg
(b) 34 kg
(d) 3 kg
ll
(a) the ratio of chemical species to each other in a
balanced equation.
.A
(e) 23 kg (Kerala P.M.T. 2011) (b) the ratio of elements to each other in a compound.
(c) the definition of mass in units of grams.
B14. What is the volume of CO2 liberated (in litres) at 1
atmosphere and 0°C when 10 g of 100% pure calcium (d) the mass of one mole of carbon.
rs
B15. Which one of the following is the lightest ? (d) 1.8 gram of water. (A.I.P.M.T. 2015)
(a) 0.2 mole of hydrogen gas. B23. 20.0 g of a magnesium carbonate sample decomposes
on heating to give carbon dioxide and 8.0 g magnesium
ub
(a) 60 (b) 84
(e) 1 g of water. (Kerala P.M.T. 2012) (c) 75 (d) 96
B16. When 22.4 litres of H2(g) is mixed with 11.2 litres of (A.I.P.M.T. 2015)
n
Cl2(g), each at S.T.P, the moles of HCl(g) formed is B24. What is the mole fraction of the solute in a 1.00 m
er
B10. (c) B11. (c) B12. (a) B13. (a) B14. (b) B15. (c) B16. (a) B17. (a) B18. (a) B19. (e)
B20. (d) B21. (d) B22. (b) B23. (b) B24. (b) B25. (a)
B26. Suppose the elements X and Y combine to form two B33. The volume of 10 N and 4 N HCl required to make
compounds XY2 and X3Y2. When 0.1 mole of XY2 1 L of 7 N HCl are
.
weighs 10 g and 0.05 mole of X3Y2 weighs 9 g, the (a) 0.50 L of 10 N HCl and 0.50 L of 4 N HCl
ed
atomic weights of X and Y are
(b) 0.60 L of 10 N HCl and 0.40 L of 4 N HCl
(a) 40, 30 (b) 60, 40
(c) 20, 30 (d) 30, 20 (NEET 2016) (c) 0.80 L of 10 N HCl and 0.20 L of 4 N HCl
rv
B27. Which of the following is dependent on temperature? (d) 0.75 L of 10 N HCl and 0.25 L of 4 N HCl
(a) Molality (b) Molarity (Orissa J.E.E. 2009)
se
(c) Mole fraction (d) Weight percentage B34. Excess of carbon dioxide is passed through
(NEET 2017) 50 mL of 0.5 M calcium hydroxide solution. After
B28. In which case is number of molecules of water the completion of the reaction, the solution was
re
maximum? evaporated to dryness. The solid calcium carbonate
was completely neutralised with 0.1 N hydrochloric
(a) 18 mL of water
acid. The volume of hydrochloric acid required is
(b) 0.18 g of water
ts
(a) 200 cm3 (b) 500 cm3
(c) 0.00224 L of water vapours at 1 atm and 273 K (c) 400 cm3 (d) 300 cm3
(d) 10–3 mol of water
gh
(NEET 2018) (Karnataka C.E.T. 2009)
B29. The number of moles of hydrogen molecules required B35. How much time (in hours) would it take to distribute
to produce 20 moles of ammonia through Haber’s
one Avogadro number of wheat grains if 1020 grains
ri
process is
are distributed each second ?
(a) 40 (b) 10
(c) 20 (d) 30 (NEET 2019) (a) 0.1673
ll (b) 1.673
(c) 16.73 (d) 167.3
JEE (Main) & Other State Boards’
.A
(e) 1673 (Kerala PET 2010)
Engineering Entrance B36. Two oxides of a metal contain 36.4% and 53.4% of
oxygen by mass respectively. If the formula of the
B30. If we consider that 1/6 in place of 1/12 mass of carbon
rs
and pressure for every mole of Al that reacts. (a) 0.50, 0.50 (b) 0.75, 0.25
(b) 67.2 L H2(g) at STP is produced for every mole (c) 0.67, 0.33 (d) 0.25, 0.75
n
B26. (a) B27. (b) B28. (a) B29. (d) B30. (a) B31. (c) B32. (b) B33. (a) B34. (b) B35. (b)
B36. (b) B37. (c) B38. (b)
B39. 50 cm3 of 0.2 N HCl is titrated against 0.1 N NaOH calcium oxide in the mixture is approximately (Given
solution. The titration was discontinued after adding molar mass of BaO = 153, CaO = 56).
.
50 cm3 of NaOH. The remaining titration is completed (a) 52.6 (b) 55.1
ed
by adding 0.5 N KOH. The volume of KOH required (c) 44.9 (d) 47.4
for completing the titration is (Karnataka CET 2014)
(a) 10 cm3 (b) 12 cm3 B48. 25 cm3 of oxalic acid completely neutralised 0.064 g
rv
(c) 16.2 cm3 (d) 21.0 cm3 of sodium hydroxide. Molarity of oxalic acid solution is
(Karnataka C.E.T. 2011) (a) 0.064 (b) 0.045
se
B40. A 100% pure sample of a divalent metal carbonate
(c) 0.015 (d) 0.032
weighing 2 g on complete thermal decomposition
(Karnataka CET 2014)
releases 448 cc of carbon dioxide at STP. The
B49. A 5.82 g silver coin is dissolved in nitric acid. When
re
equivalent mass of the metal is
sodium chloride is added to the solution, all the silver
(a) 40 (b) 20 is precipitated as AgCl. The AgCl precipitate weighs
(c) 28 (d) 12 7.20 g. The percentage of silver in the coin is
(e) 56 (Kerala P.E.T. 2012)
ts
(a) 60.3% (b) 80%
B41. The total number of electrons in 18 mL of water (c) 93.1% (d) 70%
(density = 1 g mL–1) is (A.M.U. Egg. 2014)
gh
(a) 6.02 × 1023 (b) 6.02 × 1025 B50. The ratio of masses of oxygen and nitrogen in a
(c) 6.02 × 1024 (d) 6.02 × 18 × 1023 particular gaseous mixture is 1 : 4. The ratio of
(Karnataka C.E.T. 2012) number of their molecules is
ri
B42. Two solutions of HCl, A and B, have concentrations of (a) 3 : 16 (b) 1 : 4
0.5 M and 0.1 M respectively. The volume of solutions
(c) 7 : 32 (d) 1 : 8 (JEEMain2014)
A and B required to make 2 litres of 0.2 M HCl are ll
(a) 0.5 L of A + 1.5 L of B B51. The number of Cl– ions in 100 mL of 0.001 M HCl
.A
(b) 1.5 L of A + 0.5 L of B solution is
(c) 1.0 L of A + 1.0 L of B (a) 6.022 × 1023 (b) 6.022 × 1020
(d) 0.75 L of A + 1.25 L of B (A.M.U. Engg. 2012) (c) 6.022 × 1019 (d) 6.022 × 1024
rs
B43. The number of water molecules present in a drop of (A.M.U. Engg 2015)
water weighing 0.018 g is B52. 0.30 g of an organic compound containing C, H and
O on combustion yields 0.44 g CO2 and 0.18 g H2O.
he
(c) C3H4 (d) C6H6 (JEEMain2013) 51.2g of naphthalene, (C10H8) in 500 mL of carbon
B47. 10 g of a mixture of BaO and CaO requires 100 cm3 tetrachloride. The density of CCl4 is 1.60 g/mL.
(a) 0.250 m (b) 0.500 m
©
B39. (a) B40. (b) B41. (c) B42. (a) B43. (d) B44. (c) B45. (b) B46. (a) B47. (a) B48. (d)
B49. (c) B50. (c) B51. (c) B52. (b) B53. (b) B54. (e) B55. (b)
B56. An organic compound contains C = 40%, H = 13.33% B64. 1 gram of a carbonate (M2CO3) on treatment with
and N = 46.67%. Its empirical formula is excess HCl produces 0.01186 mole of CO2. The molar
.
(a) C2H2N (b) C3H7N mass of M2CO3 in g mol–1 is
ed
(c) CH4N (d) CHN (a) 1186 (b) 84.3
(Karnataka CET 2016) (c) 118.6 (d) 11.86
rv
B57. At 300 K and 1 atm, 15 mL of a gaseous hydrocarbon (Kerala PET 2017)
requires 375 mL air containing 20% O2 by volume B65. Calculate the molarity of a solution of 30 g of
for complete combustion. After combustion the gases Co(NO3)2.6H2O in 4.3 L of solution? Consider atomic
se
occupy 330 mL. Assuming that the water formed is mass of Co = 59u, N = 14u, O = 16u, H = 1u.
in liquid form and the volumes were measured at the
same temperature and pressure the formula of the (a) 0.023 M (b) 0.23 M
re
hydrocarbon is (c) 0.046 M (d) 0.46 M
(a) C3H6 (b) C3H8 (J.K. CET 2018)
(c) C4H8 (d) C4H10 B66. How many moles of electrons will weigh one kilogram?
ts
(JEE Main 2016) 1
(a) 6.023 × 1023 (b) × 1021
B58. You are supplied with 500 mL each of 2 N HCl and 9.108
5N HCl. What is the maximum volume of 3 M HCl 6.023 1
gh
(c) × 1054 (d) × 108
that you can prepare using only these two solutions? 9.108 9.108 × 6.023
(a) 250 mL (b) 500 mL (WB JEE 2018)
ri
(c) 750 mL (d) 1000 mL B67. A metal M (specific heat 0.16) forms a metal chloride
(WB-JEE 2017) with a 65% chlorine present in it. The formula of the
B59. In a flask, the weight ratio of CH4(g) and SO2(g) at ll metal chloride will be
298 K and 1 bar is 1 : 2. The ratio of the number of (a) MCl (b) MCl2
.A
molecules of SO2(g) and CH4(g) is (c) MCl3 (d) MCl4
(a) 1 : 4 (b) 4 : 1 (WB JEE 2018)
(c) 1 : 2 (d) 2 : 1 (WB-JEE 2017) B68. 1.0 g of Mg is burnt with 0.28 g of O2 in a closed vessel.
rs
B60. What will be the normality of the salt solution obtained Which reactant is left in excess and how much?
by neutralizing x mL of y (N) HCl with y mL of x (N) (a) Mg, 5.8 g (b) Mg, 0.58 g
NaOH and finally adding (x + y) mL distilled water ?
he
(a) 0.5 × 10–3 mol (b) 0.1 × 10–3 mol compound (CxHyOz) is 6 : 1. If one molecule of the
(c) 9.95 × 10–2 mol (d) 1.66 × 10–3 mol above compound (CxHyOz) contains half as much
n
The value of 'x' is of NaOH in solution and molality (in mol kg–1) of the
(a) 4 (b) 5 solution respectively are
(a) 0.167, 11.11 (b) 0.2, 22.20
©
(c) 6 (d) 7
(c) 8 (Kerala PET 2017) (c) 0.2, 11.11 (d) 0.167, 22.20
(JEE Main 2019)
B56. (c) B57. (b) B58. (c) B59. (c) B60. (b) B61. (a) B62. (e) B63. (c) B64. (b) B65. (a)
B66. (d) B67. (b) B68. (b) B69. (a) B70. (d) B71. (a)
.
(a) 68.4 g (b) 17.1 g
ed
B80. At 300 K and 1 atmospheric pressure, 10 mL of
(c) 34.2 g (d) 136.8 g a hydrocarbon required 55 mL of O2 for complete
(JEE Main 2019) combustion and 40 mL of CO2 is formed. The formula
B73. A solution of sodium sulphate contains 92 g of Na+ of the hydrocarbon is :
rv
ions per kilogram of water. The molality of Na+ ions (a) C4H8 (b) C4H7Cl
in that solution in mol kg–1 is
(c) C4H10 (d) C4H6
se
(a) 16 (b) 8
(J.E.E. Main 2019)
(c) 4 (d) 12
B81. The amount of sugar (C12H22O11) required to prepare
(JEE Main 2019)
re
22 L of its 0.1 M aqueous solution is
B74. Total number of atoms in 44 g of CO2 is
(a) 6.02 × 1023 (b) 6.02 × 1024 (a) 68.4 g (b) 17.1 g
(c) 1.806 × 1024 (d) 18.06 × 1022 (c) 34.2 g (d) 136.8 g
ts
(J & K CET 2019) (J.E.E. Main 2019)
B75. The combining ratios of hydrogen and oxygen in water B82. A mixture of 100 mmol of Ca(OH)2 and 2 g of sodium
gh
and hydrogen peroxide are 1:8 and 1:16. Which law sulphate was dissolved in water and the volume was
is illustrated in this example? made up to 100 mL. The mass of calcium sulphate
(a) Law of definite proportions formed and the concentration of OH- in resulting
ri
(b) Law of multiple proportions solution respectively, are : (Molar mass of Ca(OH)2,
(c) Law of conservation of mass Na 2SO 4 and CaSO 4 are 74, 143 and 136 g mol -1
(d) Gay Lussac’s law of combining volumes of gases. ll respectively)
(MH–CET 2019) (a) 1.9 g, 0.14 mol L-1 (b) 13.6 g, 0.14 mol L-1
.A
B76. The mass of AgCl precipitated when a solution (c) 1.9 g, 0.28 mol L-1 (d) 13.6 g, 0.28 mol L-1
containing 11.70 g of NaCl is added to a solution (J.E.E. Main 2019)
containing 3.4 g of AgNO 3 is [Atomic mass of B83. The percentage composition of carbon by mole in
rs
If the solution and pure methanol have densities of solute is 0.8. The molality (in mol kg–1) of the aqueous
0.964 kg L–1 and 0.793 kg L–1 respectively, find the solution is
per cent of methanol by weight?
ub
identify dihydrogen (H2) as a limiting reagent in the B86. Given that the abundances of isotopes 54Fe, 56Fe and
57Fe are 5%, 90% and 5% respectively, the atomic
following reaction mixtures.
(a) 14 g of N2 + 4 g of H2 mass of Fe is
©
B72. (a) B73. (c) B74. (c) B75. (b) B76. (a) B77. (b) B78. (a) B79. (c) B80. (d) B81. (a)
B82. (c) B83. (d) B84. (c) B85. (c) B86. (b)
.
with more than one correct answers (a) Mg = 80% (b) Mg = 20%
ed
(c) S = 26.7% (d) S = 53.3%
C1. Which of the following method/methods of expressing C6. The following substances are present in different
concentrations is/are independent of temperature ? containers
rv
(a) Mole fraction (b) Molarity (i) One gram atom of nitrogen
(c) Normality (d) Molality (ii) One mole of calcium
se
C2. A solution contains 25% water, 25% ethanol (C2H5OH) (iii) One atom of silver
and 50% acetic acid (CH3COOH) by mass. The mole (iv) One mole of oxygen molecules
fraction of
(v) 1023 atoms of carbon and
re
(a) Water = 0.502 (vi) One gram of iron.
(b) Ethanol = 0.302
(c) Acetic acid = 0.196 The correct order of increasing masses (in grams)
is/are :
ts
(d) Ethanol + acetic acid = 0.498
C3. 8 g of O2 has the same number of oxygen atoms as (a) (iii) < (vi) < (i) < (v)
(a) 11 g CO2 (b) 14 g CO (b) (iii) < (vi) < (iv) < (ii)
gh
(c) 32 g SO2 (d) 8 g O3 (c) (vi) < (v) < (i) < (iv)
1 (d) (iii) < (ii) < (v) < (iv)
C4. The mass of th of 12C is same as that of
ri
12 C7. Which of the following methods of expressing
1 concentration varies with temperature?
(a) th of N2 (b) 1u (a) Molality
ll (b) Weight per cent
28
1 1 (c) Normality (d) Molarity
.A
(c) th of O (d) th of He C8. Which of the following units are not correct for the
8 12
physical quantity ?
(a) Acceleration : ms–2 (b) Pressure : kg m–2s–2
rs
A mole is a collection of 6.022 × 1023 particles and (a) 1.79 × 10–16 g (b) 3.58 × 10–16 g
the number 6.022 × 1023 is called Avogadro number. The (c) 3.58 × 10–6 g (d) 1.79 × 10–16 g
mass of this number of atoms in an element is equal to its
Passage-II
P
N.T.P is 22.4 L. When 6.022 × 1023 molecules of a substance composition of proteins were found :
er
(c) 2.12 × 10–23 g (d) 1280 g D4. If haemoglobin contains 4 atoms of iron, then
approximate molecular mass of haemoglobin is
D2. Suppose the chemists would have choosen 1020 as the
©
Passage I. D1. (b) D2. (b) D3. (a) Passage II. D4. (c)
.
Oleum or fuming sulphuric acid contains SO3 dissolved in
ed
(a) 2.16 × 10–3 (b) 2.7 × 105 sulphuric acid and has the molecular formula H2S2O7. It is
formed by passing SO3 in H2SO4. When water is added to
(c) 3.67 × 10–3 (d) 1.83 × 10–3 oleum, SO3 reacts with water to form H2SO4.
rv
D6. If the cytochrome protein contains one atom per SO3 (g) + H2O (l) → H2SO4 (aq)
molecule then the molecular mass of protein is As a result, mass of H2SO4 increases. When 100 g sample
se
(a) 14850 u (b) 29600 u of oleum is diluted with desired amount of water (in gram)
then the total mass of pure H2SO4 obtained after dilution
(c) 32960 u (d) 12840 u
is known as percentage labelling of oleum.
D7. How many atoms of Se are present in 1 µg of peroxidase
re
% Labelling of oleum = Total mass of H2SO4 present in
enzyme assuming one molecule of enzyme contains oleum after dilution
1 atom of Se (at. mass of Se = 78.96) ?
or = Mass of H2SO4 initially present
(a) 2.23 × 1019 (b) 4.52 × 1014 + Mass of H2SO4 produced after
ts
(c) 3.82 × 1021 (d) 2.23 × 106 dilution
From this, the percentage composition of H2SO4 and SO3
D8. How many moles of iron are present in 1 mg of
gh
haemoglobin (assuming a molecule of haemoglobin (free) and SO3 (combined) can be calculated.
contains 4 Fe atoms)? Answer the following questions :
D9. The percentage of SO3 in 109% H2SO4 is
ri
(a) 1.50 × 10–8 (b) 6.0 × 10–8
(a) 9% (b) 36% (c) 40% (d) 60%
(c) 3.0 × 10–8 (d) 1.875 × 10–9
D10. The percentage of free SO3 and H2SO4 in 112% H2SO4 is
ll
(a) 53.6, 46.4 (b) 12.0, 88.0
.A
(c) 88.0, 12.0 (d) 26.8, 73.2
Passage II. D5. (c) D6. (a) D7.
(b) D8. (b) Passage III. D9. (c) D10. (a)
rs
Column I Column II
columns, which have to be matched. Statements in Column I
are labelled as A, B, C and D whereas statements in Column (A) Molality (p) independent of
II are labelled as p, q, r and s. Match the entries of Column I temperature
lis
with appropriate entries of Column II. Each entry in Column (B) Molarity (q) mol L–1
I may have one or more than one correct option from Column (C) Mole fraction (r) g equiv L–1
II. The answers to these questions have to be appropriately
ub
p q r S
A p q r S Column I Column II
n
C p q r S
molecules of H2O
D p q r S
(C) 5.6 litres of O2 (r) 1 mol
od
(1) : (A) – (s) (B) – (p), (r) (C) – (q) (D) – (p), (s) (2) : (A) – (p), (s) (B) – (q) (C) – (p) (D) – (r)
(3) : (A) – (q) (B) – (r) (C) – (p) (D) – (t) (E) – (s)
.
ed
(I.I.T. J.E.E. 2012)
Integer Type: The answer to each of the following
8. If the value of Avogadro number is 6.023 × 1023
question is a single digit integer ranging from 0 to 9.
and the value of Boltzmann constant is
1. 1.420 g of anhydrous ZnSO4 was left in moist air. 1.380 × 10–23 JK–1, then number of significant digits
rv
After a few days its weight was found to be 2.528 g. in the calculated value of the universal constant is
How many water molecules are present in its (JEE Advance 2014)
se
hydrated salt formula (molar mass of ZnSO4 = 161.5)?
Numerical Value Type: Give the correct numerical
2. Moles of iron which can be made from Fe2O3 by the value (in decimal notation truncated/rounded off to the
use of 294 g of carbon monoxide in the reaction :
re
second decimal place.
Fe2O3 + 3CO → 2Fe + CO2 are :
9. To measure the quantity of MnCl2 dissolved in an
3. 428 mL of 10 M HCl and 572 mL of 3 M HCl are aqueous solution, it was completely converted to
mixed. The molarity of the resulting solution is
ts
KMnO4 using the reaction,
4. Silver (atomic mass = 108 g mol–1) has a density of
MnCl2 + K2S2O8 + H2O → KMnO4 + H2SO4 + HCl
10.5 cm–3. The number of silver atoms on a surface of
gh
(equation not balanced)
area 10–12m2 can be expressed in scientific notation
Few drops of concentrated HCl were added to this
as y × 10x. The value of x is
solution and gently warmed. Further, oxalic acid
(I.I.T. JEE 2010) (225 mg) was added in portions till the colour of
ri
5. The value of n in the formula Ben Al2 Si6 O18 is the permanganate ion disappeared. The quantity
(I.I.T. 2010) ll of MnCl2 (in mg) present in the initial solution
6. Reaction of Br2 with Na2CO3 in aqueous solution is _________. (Atomic weights in g mol–1: Mn = 55,
gives sodium bromide and sodium bromate with Cl = 35.5) (JEE Advanced 2018)
.A
evolution of CO 2 gas. The number of sodium 10. The mole fraction of urea in an aqueous urea solution
bromide molecules involved in the balanced chemical containing 900 g of water is 0.05. If the density of the
equation is solution is 1.2 g cm–3, the molarity of urea solution
rs
g mL–1. The molecular weight of HCl is 36.5 g mol–1. 60 g mol–1 and 18 g mol–1, respectively.)
(JEE Advanced 2019)
lis
1. (7) 2. (7) 3. (6) 4. (7) 5. (3) 6. (5) 7. (8) 8. (4) 9. 126.00 10. 2.98
ub
x p la n ations for
s & E
P
A5. (d) : In first oxide : Metal = 42.9 parts and Ratio of M : O = 0.636 : 1.267
oxygen = 57.1 parts or = 1 : 2
Since the formula is MO. Formula = MO2.
∴ 42.9 parts of metal = 1 atom A6. (c) : 4.26 – (15.635/5.0)
57.1 parts of oxygen = 1 atom 4.26 – 3.1 (upto two significant figures)
In second oxide, metal = 27.3 parts and = 1.2 (upto first decimal place after rounding off).
A7. (d) : Different weights are (a) 88g (b) 34.2g. (c) 40g A23. (a) : Caustic soda is NaOH
(d) 27g. 27g (1.5 mol) of water weigh least.
.
40 g of NaOH contain 6.02 × 1023 molecules
ed
A8. (d) : (a) 6.022 × 1023 × 3 (b) 0.5 × 6.022 × 1023
× 2
= 6.02 × 1023 atoms of H.
(c) 0.1 × 6.022 × 1023 × 3 (d) 1 × 6.022 × 1023 × 5.
A11. (a): No. of atoms of P4 = 0.1 × 6.02 × 1023 × 4 A24. (b) : C + O2 CO2
rv
= 2.4 × 1023 atoms. 12 g of C gives CO2 = 44 g
8.5 44
se
A12. (a) : Moles of NH3 = = 0.5 0.6 g of C will give CO2 = × 0.6 = 2.2 g
17 12
No. of H atoms = 0.5 × 6.022 × 1023 × 3 2 .2
Moles of CO2 = = 0.05
re
44
= 9.03 × 1023 atoms.
No. of molecules = 0.05 × 6.02 × 1023
A13. (c) : No. of molecules of O3 in 22.4 L = 6.022 × 1023
= 3.01 × 1022
ts
No. of molecules of O3 in 11.2 L
1 A25. (a) : 1 molecule of CH4 contains electrons = 6 + 4 = 10
= × 6.022 × 1023
gh
2 16 g of CH4 contain 6.022 × 1023 molecules
1
No. of O atoms = × 3 × 6.022 × 1023 = 6.022 × 10 23 × 10 electrons
2
ri
= 9.03 × 1023. 6.022 × 1023 × 10 × 3.2
∴ 3.2 g of CH4 contain =
A14. (c) : Mass of 1 He atom = 4 a.m.u. 16
No. of He atoms =
104
= 26
ll
= 2 × 10 × 6.022 × 1022 electrons.
.A
4
4.25
0.018 A26. (d) : Moles of NH3 =
A15. (c) : Moles of water = = 1 × 10–3 17
18
4.25
rs
342
= 221.8 u
= 6.62 × 1023
221.8
No. of iron atoms present in 67200 u = =4
ub
= 3.01 × 1024 1
1 kg of electrons = × 108 mol.
Total number of O atoms 9.108 × 6.02
n
of N
= 3.67 × 1024. 14 × x
∴ % of N = × 100 = 17.28
162
od
= 10 × 6.02 × 1023
2 1
A22. (b) : 2 g O =
= mol 60
16 8 × 6.02 × 1023
No. of atoms in 60 g of C =
4 1 12
4 g S = = mol = 5 × 6.02 × 1023
32 8
These will contain same number of atoms. ∴ No. of atoms in 558.6 g of Fe = twice the no.
of atoms in 60 g of C.
A
31. (c) : Molecular formula of sucrose = C12H22O11 A38. (d): Equal volumes contain equal no. of moles
∴ Molar ratio = 1 : 1
.
Empirical formula = C12H22O11.
ed
A32. (c) : Ca3P2 + 6H2O 2PH3 + 3Ca(OH)2 Ratio by weight = 4 : 16 = 1 : 4
1 mol of Ca3P2 gives 2 moles of PH3. ∴ CH4 present by weight
A33. (b) : Out of 1 mole of water, number of moles of D2O 4
rv
= × 100 = 80%
x ×1 5
= = 0.01 x A39. (d): 2Al2O3 + 3C → 4Al + 3CO2
100
se
Number of moles of D2O × mass of D2O + Number 4 × 27 kg of Al consumes C of anode
of moles of H2O × mass of H2O = 19 = 3× 12 = 36 kg
0.01x × 20 + (1 – 0.01x) × 18 = 19
re
Production of 270 kg of Al consumes C of anode
0.01x × 20 + 18 – 0.01x × 18 = 19
36
0.2x – 0.18x = 1 = × 270 = 90 kg
4 × 27
0.02x = 1
ts
A40. (d): Molecular mass of Na2SO4 = 2 × 23 + 32 + 4 × 16
x = 1/0.02 = 50
= 142
A34. (d) : Let ‘a’ atoms of sulphur be present in x g of the
gh
compound. A crystalline salt on becoming anhydrous loses
55.9% by mass.
8% of weight of compound
44.1 g of anhydrous salt contain H2O = 55.9
ri
= weight of sulphur in compound
8 142 g of anhydrous salt contain H2O
x = 32 × a
100 ll 55.9
3200 = × 142 = 180 g
x = a = 400 a 44.1
8
.A
180
The minimum molecular weight is the one in No. of water molecules = = 10 molecules
18
which a = 1 (i.e. 1 atom of sulphur is present).
A41. (a) : N2 + 3H2 —→ 2NH3
Hence, minimum molecular weight = 400
rs
mol SO2− 4
31 1.41
With excess of AgNO3, 0.01 mol of AgBr i.e., Y
56.4 3.52
n
38
of NH3. × 1000
Molarity = 36 .5
Since actual yield is 50% of the expected value, 100
NH3 formed = 10 L
©
1.19
N2 reacted = 5 L
= 12.38 M
N2 Unreacted = 30 – 5 = 25 L
A44. (b): M1V1 + M2V2 = M(V1 + V2)
H2 reacted = 15 L
0.5 × 800 + 1 × 200 = M (800 + 200)
H2 Unreacted = 30 – 15 = 15 L
400 + 200
∴ Mixture will contain 10 L NH3, 25 L N2, 15 L H2. M= = 0 .6 M
1000
A45. (b): Final volume = 2 L + 4 L = 6 L B2. (c) : PbO + 2HCl PbCl2 + H2O
.
6×2 Molar mass of PbO = 207 + 16 = 223 g mol–1
Molarity = =2M
ed
6 6 .5
Moles of PbO = = 0.029 mol
223
A46. (d): M1V1 = M2V2 3 .2
rv
Moles of HCl = = 0.088 mol
10.50 × V1 = 0.25 × 1.0 36.5
0.25 × 1.0 1 mol of PbO reacts with 2 mol of HCl
se
V1 = ∴ PbO is a limiting reagent
10.50
= 0.0238 L = 23.8 mL 0.029 mol of PbO will produce PbCl2 = 0.029 mol.
Moles 18
re
A47. (c) : Molarity = × 1000 B3. (b) : Mass of 1 molecule of water = g
Vol (in mL) 6.022 × 1023
18 g
Moles Volume = = 3.0 × 10–23 cm3
0.50 = × 1000 6.022 × 1023 × 1 g/cm 3
ts
250
B4. (b) : Molality and mole fraction are independent of
0.50 × 250
Moles of NaCl = temperature.
gh
1000
= 0.125 mol. B5. (b) : 2H2 + O2 → 2H2O
ri
Limiting reagent is O2
y y 1 mole of O2 gives 2 moles of H2O
C x H y ( g) + x + O2
→ xCO2 ( g) + H O(l)
4 2 2 ll
∴ 2 moles of O2 will give 4 moles of H2O.
y
.A
1 mL x + mL x mL negligible B6. (c) Normality = n × Molarity
4
y For acids, n = Basicity
6 mL 6 x + mL 6 x mL
4 ∴ For H2SO4, n = 2
rs
Equating theoretical and experimental value of CO2 0.02 N = 2 × Molarity
6x = 12 or x = 2
he
9 = 0.001 moles
cooling= = 1.5 mL
6 Number of molecules in 0.001 moles
n
y y
1 + = 1.5 or = 0.5 or y = 2
4 4 B 7. (b) 4Al + 3O2 → 2Al2O3
\ Formula of hydrocarbon = C2H2 4 × 27 = 54 g 3 mol
od
B1. (b) : C3H8(g) + 5O2(g) → 3CO2(g) + 4H2O(l) ∴ Amount of Al that combines with 1.5 moles of O2 = 27 g
22.4 L 5 × 22.4 L B 8. (d) A + 2B → C
1 mol 2 mol 1 mol
©
at N.T.P. at N.T.P.
22.4 L of C3H8 at N.T.P. require O2 = 5 × 22.4 L 8 moles of B require 4 moles of A. Therefore,
limiting reagent is B.
1 L of C3H8 at N.T.P. will require O2
2 moles of B give 1 mole of C
5 × 22.4
= L at N.T.P. 1
22.4 ∴ 8 moles of B give = × 8 = 4 moles of C.
2
= 5 L
B 9. (b) 1 kg of sea water = 106 mg sea water B16. (a) 22.4 L of H2 at S.T.P. = 1 mol
106 mg sea water has 6 mg dissolved O2 11.2 L of Cl2 at S.T.P. = 0.5 mol
.
ed
6 So, Cl2 is limiting reagent.
The concentration of O2 = × 106 ppm
106 H2 + Cl2 → 2HCl
= 6 ppm 1 mol of Cl2 reacts with 1 mol of H2 to give 2 mol
rv
25.3 / 106 of HCl
B10. (c) Molarity = × 1000
250 ∴ 0.5 mol of Cl2 will react with 0.5 mol of Cl2 to give
se
= 0.955 M 1 mol of HCl.
Na2CO3 → 2Na+ + CO32– 1
B17. (a) Moles of Mg =
= 0.0417 mol
∴ [Na+] = 2 × 0.955 = 1.910 M
re
24
[CO32–] = 0.955 M 0.56
Moles of O2 = = 0.0175 mol
B11. (c) No. of atoms in 0.1 mol 32
ts
= 6.022 × 1023 × 0.1 × 3 1
Mg + O → MgO
= 1.806 × 1023 2 2
1 mol 0.5 mol 1 mol
gh
B12. (a) P O2 is limiting reagent
0.5 mol of O2 react with Mg to form 1 mol of MgO
ri
1 × 0.0175
PH3 0.0175 mol of O2 react with Mg =
P2O3 0 .5
ll = 0.035
Moles of Mg left unreacted = 0.0417 – 0.035
.A
H O = 6.7 × 10–3 mol
H 2O
Amount of Mg left unreacted = 6.7 × 10–3 × 24
Ratio of number of hydrogen and oxygen combining = 0.1608 g
rs
with one P is
B18. (a) CaCO3 + 2HCl → CaCl2 + H2O + CO2
3 : 1.5 i.e., 2 : 1
According to equation, 1 mol of CaCO3 reacts with
he
28 2 × 3 34 1000
1 mole of N2 (28 g) combine with 3 moles of H2 (6 g) 0.02
Moles of CaCO3 required = = 0.01 mol
28 2
ub
× 3 = 17 kg.
6
16
B14. (b) CaCO3 + H2SO4 → CaSO4 + H2O + CO2 (b) 16 g NO2 = = 0.35 mol = 0.35 × NA molecules
46
n
(b) 6.023 × 1023 molecules of N2 = 28g ∴ (e) 32 of N2 has maximum number of molecules
6.023 × 1022 molecules of N2 B20. (d) Let masses of H2 and O2 be x and 4x g
28
©
B21. (d) Mass of 1 mol (6.022 × 1023 atoms of carbon = 12 g) 0.049 mol of AgNO3 combines with 0.049 mol of
.
If Avogadro number is changed to 6.022 × 1020 NaCl to give AgCl
ed
atoms, then mass of 1 mol of carbon = 0.049 mol
12 × 6.022 × 10 20 = 0.049 × 143.5
=
=7g
rv
6.022 × 1023
= 12 × 10–3 g B26. (a) : Let atomic weight of element X is x and that of
element Y is y.
se
18
B22. (b) (a) 18 g of water = = 1 mol For XY2
18
0.1 mol of XY2 = 10 g
= NA molecules
re
10
(b) 18 mol of water = 18 × NA molecules 1.0 mol of XY2 = × 1.0 = 100 g
0 .1
(c) 18 molecules of water or x + 2y = 100 g mol–1 .....(i)
ts
For X3Y2
1.8
(d) 1.8 g of water = = 0.1 × NA molecules 0.05 mol of X3Y2 = 9 g
18
gh
∴ 18 mol of water has maximum number of water 9
1 mol X3Y2 = = 180 g
molecules. 0.05
B23. (b) MgCO3 (s) → MgO (s) + CO2 (g) or 3x + 2y = 180 g mol–1 .....(ii)
ri
1 mol 1 mol Subtracting eq. (i) from eq. (ii)
84 g 40 g 2x = 80 g mol–1 or x = 40 g mol–1
40 g of MgO are produced from MgCO3 = 84 g
ll
and 40 + 2y = 100
100 − 40
.A
84 or y= = 30 g mol–1
8 g of MgO are produced from MgCO3 = ×8 2
40
B27. (b) : Molarity depends upon temperature.
= 16.8 g
B28. (a) : 18 mL of water has maximum number of molecules.
rs
= 84 % Molecules of water = × NA = NA
18
B24. (b) 1 m aqueous solution means 1 mol of solute is (b) 0.18 g of water
present in 1000 g of water
lis
0.18
∴ Moles of solute = 1 mole Molecules of water = × N A = 0.01 × N A
18
1000
Moles of water = = 55.55 mol 0.00224 L
ub
B25. (a) 16.9 % AgNO3 solution means that 16.9 g of AgNO3 (d) 10–3 mol of water
is present in 100 mL of solution. Molecules of water = 10–3 × NA = 0.001 NA
B29. (d) : N2 + 3H2 2NH3
n
16.9
50 mL of solution contains AgNO3 = × 50 2 mol of NH3 are formed from 3 mol of H2
er
100
∴ 20 mol of NH3 will be formed from
= 8.45 g
8.45 3
= × 20 = 30 mol
od
or = 0.049 mol 2
170 1
5.8 % NaCl solution means that 5.8 g of NaCl is B30. (a) : If atomic mass unit on the scale of of C – 12 is
1 6
present in 100 mL of solution. 2 a.m.u. on the scale of of C–12, the mass
M
12
5 .8 of one mole of substance will become half of the
50 mL of solution contains NaCl = × 50 = 2.9 g
100 normal value.
©
.
Moles of hydrogen in 5g = = 5 mol
ed
1 C2H4 + 3O2 → 2CO2 + 2H2O
No. of atoms of hydrogen = 5 × 6.022 × 1023 pV
Moles of mixture, n =
B33. (a) : N1V1 + N2V2 = NV RT
rv
1 atm × 41 L
where V1 + V2 = 1L =
(0.082 L atm K −1mol −1 ) × 500K
or V2 = (1 – V1)L
se
= 1 mol
10 × V1 + 4 (1–V1) = 1 × 7
Let moles of C2H6 = x, moles of C2H4 = 1 – x
10V1 + 4 – 4V1 = 7
Oxygen required
6V1 = 3
re
7 10
3 x + (1 – x)3 =
V1 = = 0.5 L 2 3
6 21x + 18 – 18x = 20
V2 = 1 – 0.5 = 0.5 L
ts
3x = 2 or x = 0.67 mol
B34. (b) : Ca(OH)2 + CO2 → CaCO3 + H2O
Moles of ethane = 0.67,
Moles of Ca(OH)2 present in 50 mL of 0.5 M solution
gh
Moles of ethene = 1 – 0.67 = 0.33
0 .5
= × 50 = 2.5 × 10–2 mol 0.67
1000 Mole fraction of ethane = = 0.67,
1
ri
Moles of CaCO3 formed = 2.5 × 10–2 mol 0.33
CaCO3 + 2HCl → CaCl2 + H2O + CO2 Mole fraction of ethene = = 0.33
1
Moles of HCl required
= 2 × 2.5 × 10–2 mol
ll
B38. (b) NaCl + CaCl2
2 3 Na CO
→ CaCO3
.A
= 5.0 × 10–2 mol Let wt. of CaCl2 in mixture = x
Volume of 0.1 N HCl required and wt. of NaCl in mixture = 4.44 – x
CaCO3 → CaO + CO2
rs
0 .1
= × x = 5.0 × 10−2
1000 Moles of Ca in CaCl2 = Moles of Ca in CaCO3
5.0 × 10−2 × 1000 Moles of Ca in CaCO3 = Moles of Ca in CaO
he
x = = 500 mL 0.56
0 .1 x
=
35. (b) Time taken to distribute 1020 grains
B 111 56
lis
0.56
= 1 second ∴ x = × 111 = 1.11 g
56
Time taken to distribute 6.022 × 1023 grains
Wt. of NaCl in mixture = 4.44 – 1.11 = 3.33 g
ub
1 23
= 20 × 6.022 × 10 seconds 3.33
10 % of NaCl = × 100 = 75%
4.44
6.022 × 1023
= 20 hours 0.2 × 50
P
B36. (b) Let atomic mass of metal = x, then Equivalents of NaOH = = 5.0 × 10–3
1000
er
Now, in second oxide, oxygen and metal are 53.4% Normality = × 1000 = 0.05 N
100
and 46.6% respectively.
100 cm3 of 0.05 N HCl is titrated against 0.5 N
©
∴ Atomic ratio = M : O
KOH.
46.6 53.4 N1 × V1 ×V
:
13.978 16 = N
22
HCl KOH
3.3 : 3.3 0.05 × 100 = 0.5 × V2
or 1:1
0.05 × 100
∴ Formula of metal oxide = MO ∴ V2 = = 10 cm3.
0 .5
.
1 mol 22400 cc 0.5 × 750 + 2 × 250 = Mmix (1000)
ed
2g 448 cc 0.5 × 750 + 2 × 250
Mmix =
448 cc of CO2 is evolved from MCO3 = 2g 1000
rv
22400 cc of CO2 will be evolved from 375 + 500
2 = = 0.875 M
MCO3 = × 22400 1000
se
448
3.08
= 100 g B46. (a) Moles of CO2 produced = = 0.07
44
∴ 1 mol of MCO3 = 100 g
re
0.72
or Molecular mass of MCO3 = 100 g mol–1 Moles of H2O produced = = 0.04
18
If M is the atomic mass of metal,
Combustion of hydrocarbon C x H y may be
ts
M + 12 + 3 × 16 = 100 represented as :
∴ M = 100 – 12 – 48 = 40 y y
Cx Hy + x + O2 → x CO2 + H2O
gh
Atomic mass 40 4 2
Equivalent wt. = = = 20
Valency 2 y
x = 0.07, = 0.04
41. (c) Mass of water = 18 mL × 1g mL–1 = 18 g
B 2
ri
or = 1 mol ∴ y = 0.08
No. of molecules in 1 mol of water
ll x 0.07 7
= 6.02 × 1023 = =
y 0.08 8
.A
No. of electrons in 1 molecule of water
∴ Empirical formula of hydrocarbon C7H8.
= 2 + 8 = 10
∴ No. of electrons in 1 mol of water B47. (a) BaO + 2HCl → BaCl2 + H2O
rs
B42. (a) Let volume of solution A = x L BaO and CaO require 4 moles of HCl to react
Volume of solution B = (2 – x) L completely.
2.5 × 100
M1 V1 + M2V2 = M(V1 + V2) Moles of HCl in 100 cm3 = = 0.25 mol
lis
1000
0.5 × x + 0.1 (2 – x) = 0.2 (2)
0.5x + 0.2 – 0.1x = 0.4 4 moles of HCl react with 2 moles of mixture
ub
2 × 0.25
0.4x = 0.2 0.25 mol of HCl reacts with mixture =
4
0.2
x = = 0.5L = 0.125 mol
P
0.4
Let mass of CaO be x g so that mass of BaO
Vol. of A = 0.5 L, Vol. of B = 2.0 – 0.5 = 1.5 L
= (10 – x) g
n
0.018 x 10 − x
B43. (d) Moles of water in a drop of water = = 1 × 10–3 + = 0.125
er
18 56 153
1 mole of water = 6.022 × 1023 molecules 153x + 560 – 56x = 0.125 × 56 × 153 = 1071
od
1 × 10–3 mol of water = 6.022 × 1023 × 1 × 10–3 or 97x = 1071 – 560 = 511
= 6.022 × 1020 molecules. 511
∴ x = = 5.268
Moles of HNO3 97
B44. (c) Molarity = × 1000
M
Volume 5.268
∴ % CaO = × 100
Molarity × Volume 10
or Moles of HNO3 =
= 52.68%
©
1000
2.0 × 250 B48. (d) COOH COONa
= = 0 .5
1000 + 2NaOH → + 2H2O
Mass of HNO3 = 0.5 × 63 = 31.5 g. COOH COONa
.
2
ed
Propene
0.0016
Moles of oxalic acid = = 8 × 10–4
2 3 mol (or 54g) of water are obtained from 1 mol
(42g) of propene
Volume of solution = 25 mL
rv
or 54g of water is obtained from propene = 42 g
810 4
Molarity of oxalic acid solution = × 1000 27g of water is obtained from propene
25
se
42
= 0.032 M = × 27 = 21 g
54
B49. (c) 143.5 g of AgCl contain Ag = 108 g B54. (e) : 2.46g of hydrated salt (MSO4.xH2O) gives
re
108 anhydrous salt = 1.20 g
Amount of Ag in 7.20 g of AgCl = × 7.20
143.5 ∴ Mass of water = 2.46 – 1.20 = 1.26 g
ts
= 5.42 g 1.26
5.42 Percentage of water = × 100 = 51.2%
% of Ag = × 100 = 93.1% 2.46
gh
5.82
Percentage of anhydrous salt = 100 – 51.2 = 48.8%
B50. (c) Ratio of masses of O2 and N2 = 1 : 4
1 4 48.8 g of anhydrous salt contain water = 51.2 g
∴ :
Ratio of moles of O2 and N2 =
ri
32 28 120 g (1 mol) of anhydrous salt contain water
= 7 : 32 51.2
ll = × 120 = 126 g
∴ Ratio of molecules of O2 and N2 = 7 : 32 48.8
.A
B51. (c) 1000 mL of 0.001 M HCl contain Cl– ions 126
= 0.001 mol No. of water molecules = = 7 molecules
18
100 mL of 0.001 M HCl will contain Cl– ions
rs
51.2
12 0.44 Moles of naphthalene =
52. (b) Percentage of C =
B × × 100 = 40% 128
44 0.30 51.2 × 1000
ub
Percentage of O = 100 – (40 + 6.67) = 53.33% Element Percentage Atomic Moles Simplest
mass ratio
Calculation of empirical formula
n
40
Element Percentage At. Moles of Mole ratio or C 40.0 12 = 3.33 3.33/3.33=1
er
12
composition mass atoms atomic ratio
13.3
C 40 12 40/12 = 3.33 1 H 13.33 1 = 13.33 13.33/3.33=4
od
1
H 6.67 1 6.67/1 = 6.67 2
46.67
O 53.33 16 53.33/16 = 3.33 1 N 46.67 14 = 3.33 3.33/3.33=1
14
M
Molecular mass = 60
B57. (b) : CxHy + x + O2 → xCO2 + H2O
60 4 2
∴ n= =2
30 y
15mL 15 x +
∴ Molecular formula = (CH2O)2 = C2H4O2 4
0 0 15x mL
.
Vo2 = = 75mL 98
ed
100 No. of moles of 3.01 × 1020 molecules of
y 3.01 × 1020
15 x + H2SO4 =
= 75
4 6.022 × 1023
rv
= 0.5 × 10–3
y Moles of H2SO4 left = 1 × 10–3 – 0.5 × 10–3 = 0.5 × 10–3
x + 4 = 5
se
Heat
B62. (e) : MgCO3 → MgO + CO2
This corresponds to formula C3H8. 10g 4g 0.1 mol
It may be noted that in this case the further
re
Molar mass of MgCO3 = 24 + 12 + 3 × 16
information i.e, 330 mL volume is neglected. If
= 84 g mol–1
we use that information then none of the answer
is correct. Molar mass of MgO = 24 + 16 = 40 g mol–1
ts
B58. (c) : Maximum volume of 3M HCl solution can be 40 g of MgO is obtained from 84 g of MgCO3
prepared by taking 500 mL of 2N HCl and x mL of 84
4 g of MgO will be obtained from = ×4
gh
5 N HCl so that total volume becomes (500 + x) mL 40
= 8.4 g of MgCO3
For HCl, molarity = normality
8.4
Applying N1V1 + N2V2 º N3V3 \ Percentage purity = × 100 = 84%
ri
10
500 × 2 + x × 5 º (500 + x) × 3 B63. (c) : Molar mass of Na2CO3 = 2 × 23 + 12 + 3 × 16
1000 + 5x = 1500 + 3x = 106 g mol–1
ll
Na2CO3. x H2O has 50% H2O by mass
2x = 500 or x = 250 mL 50
.A
Thus, maximum volume of 3 M solution formed (106 + 18x) × = 18x
100
= 500 + 250 = 750 mL 53 + 9 x = 18x
9x = 53 or x = 5.88 » 6
B59. (c) : Let mass of CH4(g) = 1 g
rs
M
2 M =
1
= 84.3
Moles of SO2 (g) =
64 0.01186
2 Molar mass of M2CO3 = 84.3 g mol–1
ub
2 1 30
× 6.022 × 1023 : × 6.022 × 1023 Moles of Co(NO3)2⋅6H2O =
64 16 291
1 1 30
n
: or 1 : 2 Molarity = = 0.024 M
32 16 291 × 4.3
er
xy × 10–3 xy × 10–3 0 0 1
added = × 108
9.108 × 6.023 × 1023
Equivalents left 0 0 xy × 10–3 xy × 10–3
B67. (b) : According to Dulong-Petit’s law
Equivalents of NaCl present in (x + y) mL solution = xy × 10–3
©
35.5 x × 100 8
or = 65 B71. (a) : Moles of NaOH =
= 0.2 mol
.
40 + 35.5 x 40
ed
35.5 x 65 18
= Moles of water = = 1.0 mol
40 + 35.5 x 100 18
40 100 0 .2
+ 1 =
rv
or Mole fraction of NaOH = = 0.167
35.5 x 65 0 .2 + 1 .0
40 100 35 0.2 × 1000
= −1 =
se
Molality = = 11.11 m
35.5 x 65 65 18
40 7 Moles of sugar
= B72. (a) : Molarity =
35.5 x 13 Vol. of solution in L
re
40 × 13 Moles of sugar
35.5 x = 0 .1 =
7 2L
40 × 13
x= = 2.09 2 Moles of sugar = 0.1 × 2 = 0.2 mole
ts
7 × 35.5
Mass of sugar = 0.2 × 342
Formula = MCl2
= 68.4 g
gh
B68. (b) : 2Mg + O2 —→ 2 MgO
1 .0 92
Moles of Mg = B73. (c) : Moles of Na =
+
=4
= 0.042 23
24
4
ri
0.28 Molality = = 4 m
Moles of O2 = = 0.00875 1
32
1 mol of O2 requires Mg = 2 mol ll 44
B74. (c) : Moles of CO2 = = 1 mol
0.00875 mol of O2 requires Mg = 2 × 0.00875 44
.A
= 0.0175 Molecules of CO2 = 6.022 × 1023
\ Mg is in excess. Atoms in 44 g of CO2 = 3 × 6.022 × 1023
Moles of Mg left in excess = 0.042 – 0.0175 = 0.0245 = 1.806 × 1024
rs
Mass of Mg left in excess = 0.0245 × 24 = 0.58 g B76. (c) : NaCl + AgNO3 —→ AgCl + NaNO3
+2 +3
B69. (a) : 2FeSO4
→ 2Fe2 (SO4 )3 11.70
Moles of NaCl = = 0.2
he
Change in oxidation state per Fe atom is 1. 58.5
\ Equivalent wt. = Atomic wt. = 55.84 g mol–1 3 .4
Moles of AgNO3 = = 0.02
B70. (d) : 170
lis
2 15.86
Mass % of methanol = × 100 = 16.45%
Oxygen atoms required for complete combustion 96.4
od
of CxHy 8
B78. (a) Moles of NaOH = = 0.2 mol
y 40
= 2 x +
4 18
M
Moles of water =
= 1.0 mol
1 y y 18
z = 2 x + = x +
2
4 4 0 .2
Mole fraction of NaOH = = 0.167
2 3
©
z=1+ = 0 .2 + 1 .0
4 2
0.2 × 1000
3 Molality = = 11.11 m
Ratio of x : y : z = 1 : 2 : 18
2
B79. (c) N2 (g) + 3 H2(g) → 2NH3(g)
=2:4:3
(a) 0.5 mol 2 mol
Formula : C2H4O3 (Limiting reagent)
.
1. (7) : 1.420 g of anhydrous ZnSO4 combine with water
ed
l
D1. (b) : Molar mass of C10H8 = 10 × 12 + 8 × 1 = 128
128 = 2.528 – 1.420
Mass of 10 molecules of C10H8 = × 10
6.02 × 1023 = 1.108 g
rv
= 2.12 × 10–21 g ∴ 161.5 g of anhydrous ZnSO4 combines with
32 × 1020 1.108
water = × 161.5
se
D2. (b) : = 5.32 × 10–3 g 1.420
6.02 × 1023
107.81 = 126
D3. (a) : × 106 = 1.79× 10–16 g 126
re
6.02 × 1023 No. of moles of water =
=7
4 × 55.85 18
D4. (c) : % Fe = No. of water molecules in the salt = 7
Mol. mass of haemoglobin
l 2. (7) : Fe2O3 + 3CO → 2Fe + 3CO2
ts
4 × 55.85 × 100
∴ Mol. mass = = 66687 294
0.335 Moles of CO used = = 10.5
gh
or = 66800 28
0.29 3 mol of CO are used to make = 2 mol of Fe
D5. (c) : = 3.672 × 10–3
78.96 2
ri
10.5 mol of CO are used to make = × 10.5 = 7
1 × 55.85 × 100 3
D6. (a) : Mol. mass =
0.376 l
ll 3. (6) : M1V1 + M2V2 ≡ M3V3
= 14853 14850 10 × 428 + 3 × 572 = M3 × 1000
.A
1 × 78.96 × 100 ∴ M3 = 6
D7. (b) : Mol. mass of peroxidase =
0.29 Mass
l 4. (7) : Density =
= 27228 Volume
rs
−3
1 × 10 No. of atoms in 1 cm = (58.5 × 1021)1/3
Moles of haemoglobin = = 3.882 × 107
66800
= 1.497 × 10–8 No. of atoms in 1 cm2 = (3.882 × 107)2
ub
Moles of iron = 1.497 × 10–8 × 4
No. of atoms in 10–12m2 or 10–8 cm2
= (3.882 × 107)2 × 10–8
= 5.99 × 10–8 ≈ 6.0 × 10–8
= 1.506 × 107
P
9 2n = 6
Moles of water added = = Moles of free SO3
present 18 or n = 3
od
.
k = 225 × 10−3
ed
Avogadro number, N A Moles of oxalic acid =
90
or R = k × NA = 2.5 × 10–3
= 1.380 × 10–23 × 6.023 × 1023 From equation (ii)
rv
Since both the terms have 4 significant figures.
Moles of KMnO4 react with 5 mol of H2C2O4
The number of significant figures in the term = 2 mol
se
having least number of significant figures is 4. No. Moles of KMnO4 react with 2.5 × 10–3 mol of
of significant figures in calculated value will be 4. 2
H2C2O4 = × 2.5 × 10−3 = 1 × 10–3 mol
5
re
l 9. (126.00) :
Moles of MnCl2 required initially = 1 × 10–3 mol
2MnCl2 + 5K2S2O8 + 8H2O → 2KMnO4
Mass of MnCl2 required initially = 1 × 10–3 × 126
+ 4K2SO4 + 6H2SO4 + 4HCl ...(i)
ts
= 126 mg
2KMnO4 + 5H2C2O4 + 3H2SO4 → K2SO4 l 10. (2.98) Refer Advanced Level Problems, Problem 17,
+ 2MnSO4 + 8H2O + 10CO2 ...(ii) Page 79.
gh
ri
ll
.A
rs
he
lis
ub
P
n
er
od
M
©
.
ed
Time Allowed : 1½ Hrs. Maximum Marks : 25
1.
The ratio of masses of oxygen and nitrogen in a gaseous mixture is 1 : 4 (w/w). The ratio of the number
rv
of their molecules is
se
(a) 3 : 16 (b) 1 : 4
(c) 1 : 8 (d) 7 : 32 (1)
2.
A mixture of CO and CO2 is passed over red hot carbon when 1 mole of mixture changes to 33.6 L at
re
NTP. The mole fraction of CO2 in the mixture is
(a) 0.62 (b) 0.50
ts
(c) 0.74 (d) 0.32 (1)
3.
An alkaloid contains 17.28% of nitrogen and its molecular mass is 162. The number of nitrogen atoms
gh
present in one molecule of alkaloid is
(a) 2 (b) 3
ri
(c) 4 (d) 6 (1)
Q. 4 and 5 are assertion reason type questions
ll
In the following questions a statement of assertion followed by a statement of reason is given.
.A
Choose the correct answer out of the following choices.
(a) Assertion and reason both are correct statements and reason is correct explanation for assertion.
(b) Assertion and reason both are correct statements but reason is not correct explanation for assertion.
rs
4. Assertion: Both 100 g of CaCO3 and 12 g of carbon have same number of carbon atoms.
Reason: Both contain 1 gram atom of carbon which contains 6.022 × 1023 carbon atoms.
(1)
lis
5. Assertion: When two elements combine to form more than one compound, then the masses of one
element that combine with a fixed mass of the other element, are in the whole number ratio.
ub
Reason: A chemical compound is always made up of the same elements combined together in the same
fixed proportion by mass. (1)
P
12
6. What is the mass of one C atom in grams? (1)
–23
7. One atom of an element weighs 9.75 × 10 g. Calculate its atomic mass. (1)
n
8. Two oxides of a metal contain 27.6% and 30.0% of oxygen respectively. If the formula of first oxide
er
12. Commercially available sulphuric acid contains 93% acid by mass and has density of
1.84 g mL–1. Calculate
(i) the molarity of the solution
.
(ii) volume of concentrated acid required to prepare 2.5 L of 0.50 M H2SO4. (3)
ed
13. (a) What is meant by empirical formula and molecular formula? How are they related to each
other? Explain with an example.
rv
(b) A compound on analysis gave the following percentage composition: Na = 14.31%, S = 9.97%,
H = 6.22% and O = 69.50%. Calculate the molecular formula of the compound assuming that
se
all the hydrogens in the compound is present in combination with oxygen as water of
crystallisation. The molecular mass of the compound is 322. (5)
re
To check your performance, see HINTS and SOLUTIONS to some questions at the end
ts
of Part I of the book.
gh
ri
ll
.A
rs
he
lis
ub
P
n
er
od
M
©
.
2
ed
rv
se
re
ts
gh
ri
STRUCTURE OF ll
ATOM
.A
The existence of atoms has been proposed since the times of early Indian and Greek philosophers around 400 B.C., who
rs
were of the view that the atoms are the fundamental building blocks of matter. The word ‘atom’ has been derived from
he
theory. According to his theory, all matter are composed of extremely small, structureless, hard spherical particles called
atoms. However, the discoveries towards the end of 19th and early 20th centuries showed that atom has a complex
structure and is not indivisible. These studies further revealed that atom consists of still smaller particles such as electron,
ub
proton and neutron, into which it may be divided. These particles are regarded as fundamental particles because these
are the main constituents of all atoms.
P
OBJECTIVES
Building on..... Assessing..... Preparing for Competition.....
n
SOLUTION FILE HOTS & Advanced Level AIPMT, NEET & Other State Boards’
Hints & Solutions for Practice Questions with Answers 95 Medical Entrance 102
Problems 68 JEE (Main) & Other State Boards’
M
CHAPTER SUMMARY & QUICK UNIT PRACTICE TEST 121 Engineering Entrance 104
CHAPTER ROUND UP 73 JEE (Advance) for IIT Entrance 107
©
2/1
.
These experiments were known as discharge tube
ed
Whenever an object is placed inside the tube, it casts
experiments. The experiment in its simplest form a shadow on the wall opposite to the cathode.
consists of a cylindrical hard glass tube (about 50 cm 2. These rays themselves are not visible but their
long) closed at both ends [Fig.1]. It is known as discharge
rv
behaviour can be observed with the help of certain
tube or Crookes tube. It is fitted with two metallic kinds of materials (fluorescent or phosphorescent)
electrodes. The tube is connected to a side tube, through
se
which glow when hit by them. It may be remembered
which it can be evacuated to any desired pressure with that television tubes are also cathode tubes and
the help of a vacuum pump. The discharge tube is filled television pictures result due to fluorescence on the
re
with the gas under study and the two electrodes are television screen coated with certain fluorescent or
connected to a source of high voltage. The pressure of phosphorescent materials.
different gases could be adjusted by evacuation. 3. Cathode rays produce mechanical effects. For
Under ordinary conditions, gases are poor
ts
example, when a small paddle wheel is placed between
conductors of electricity. However, when a sufficiently the electrodes, it starts rotating.
high voltage is applied across the electrodes, at very 4. In the presence of electrical or magnetic field,
gh
low pressures, the gases become conductors and the behaviour of cathode rays are similar to that
electricity begins to flow in the form of rays. These rays expected from negatively charged particles, suggesting
are called cathode rays. The existence of these rays that the cathode rays consist of negatively charged
ri
was shown by scientists like Plucker, Crookes, etc., but particles called electrons.
the main credit goes to J.J. Thomson. He studied the ll 5. When the cathode rays are allowed to strike a
properties of cathode rays in detail which led to the thin metal foil, it gets heated up. Thus, the cathode
.A
discovery of an electron. rays possess heating effect.
6. The characteristics of cathode rays do not depend
upon the nature of electrodes and the nature of gas
rs
(e/m) of electrons
In 1897, J.J. Thomson determined the ratio of the
charge (e) of the electron to its mass (m) by measuring
n
Fig. 1. Discharge tube experiments (a) Discharge tube the deflection under the simultaneous influence of
er
containing a gas. (b) Emission of cathode rays at high electric and magnetic fields, applied perpendicular to
voltage and low pressure. each other.
od
Charge/mass (e/m) = 1.76 × 1011 C kg–1 However, this mass is very small and for all practical
Charge (e) = 1.60 × 10–19 C purposes, it may be taken as negligible. The charge of
−19 the electron is the smallest known electrical charge
e 1.60 × 10 C and is usually referred to as unit negative charge.
Mass of electron (m) = =
e /m 1.76 × 1011C kg −1 Thus, an electron may be defined as
.
= 9.10 × 10–31kg a sub-atomic particle which carries one unit
ed
The mass of the electron is much smaller than negative charge (1.6022 × 10–19 C) and has a
the mass of an atom of hydrogen. It has been found mass (9.10 × 10–31 kg) equal to 1/1837th of that
that the mass of an electron is approximately 1/1837th of hydrogen atom.
rv
(or 5.45 × 10 –4 times) the mass of an atom of hydrogen.
se
LEARNING PLUS
J Thomson’s Experiment for Determination of Charge/Mass (e/m) of the electron
The apparatus is shown in Fig. 2. A high potential is maintained between the cathode and the anode.
re
Electrons emitted from the cathode are accelerated by the high voltage. The circular disc after the anode
selects the beam moving in a straight line. The beam then passes through electric and magnetic fields which
are perpendicular to each other and also to the direction of the motion. Thomson suggested that the amount
ts
gh
ri
ll
.A
rs
he
Fig. 2. Apparatus for determining the ratio of electric charge (e) to mass (m) of electrons.
lis
of deviation of the particles from their path in the presence of electrical and magnetic fields depends upon the
charge on the electrons, mass of the electrons and strengths of the electric and magnetic fields.
When only electric field is applied, the electrons deviate from their path and hit the cathode ray tube at
ub
point A. Similarly, when only magnetic field is applied, electrons deviate from their path and hit the cathode
ray tube at point C. By carefully balancing the electrical and magnetic field strength, it is possible to bring
P
back the electron to the path followed as in the absence of electric or magnetic field and they hit the screen at
point B. By carrying out accurate measurements on the deflections observed by electron on the electric field
strength or magnetic field strength, Thomson was able to calculate the value of charge / mass ratio i.e, e/m.
n
where m is the mass of electron in kg and e is the magnitude of the charge on the electron in Coulombs (C).
Since electrons are negatively charged, the charge on electron is negative, – e.
M
The relative strengths of electric and magnetic fields and the ratio e/m control the deflections. Hence, by
measuring the deflection and the field strength, e/m can be calculated.
J Determination of charge on the electron.
©
The charge on the electron was measured by R.A. Millikan in 1909 by a method known as oil drop method.
The apparatus used is shown in Fig 3. Small drops of oil in the form of mist are formed by a sprayer and these
are allowed to fall in between two metal plates, which could be electrically charged. A single drop between the
plates is observed by means of a telescope equipped with a micrometer eye piece. The oil in the form of mist
drop falls through the air under the influence of gravitational force. He then irradiated the space between the
plates with X-rays. These knocked electrons out of some of the molecules of the air and some of these electrons,
were caught by oil droplets which acquired electrical charge. By charging the upper plate positive and the lower
plate negative, the oil drop experiences electric field in the upper direction. By adjusting the electrical field
strength, the upward electrical field on the oil droplet was balanced against the downward gravitational force.
Under these conditions, the drop remains stationary. From the amount of charge on the plates and the mass
.
of the droplet, the charge on the droplet was determined. The mass of the droplet was determined from the rate
ed
of fall of droplet through the air when the plates were uncharged.
rv
se
re
ts
gh
ri
Fig. 3. Millikan’s experiment for the determination of charge of electron.
ll
.A
From his experiments, Millikan found the charge of the electron to be 1.6022 × 10–19 coulombs.
The presence of negatively charged electrons in Some of the characteristics of anode rays are :
an atom suggests that there must be some positively 1. The anode rays travel in straight lines and cast
shadow of the object placed in their path.
he
are also found streaming behind the cathode in of anode rays is in the opposite direction to that of the
discharge tube experiments (Fig. 4). These rays cathode rays. For example, these rays are attracted
travelled in opposite direction to the cathode rays. towards the negative plate in the electric field.
ub
Fig. 4 Generation of anode rays (protons). of anode rays was found to depend upon the nature of
the gas taken in the discharge tube. In case of
©
These rays are also deflected by the magnetic and hydrogen, the charge to mass (e/m) ratio was
electric fields like cathode rays. But the deflection of maximum. The value was found to be 9.58 × 107
anode rays is in the opposite direction to that of cathode coulombs per kg. Its charge has been found to be
rays. This means that these rays consist of positively 1.6022 × 10–19 coulombs which is equal in magnitude
charged particles and were also named positive rays but opposite in sign to that of an electron i.e., it has one
or canal rays or anode rays. unit of positive charge.
Since e/m is maximum for hydrogen, the mass have the same e/m value as the cathode rays.
(m) of the positive particle obtained from hydrogen is Therefore, β-rays were considered to be the streams
the smallest. Its mass can be calculated from the values of electrons.
of e and e/m as : (iii) Gamma (γγ) rays. The rays which are not
.
−19
e 1.6022 × 10 C deflected at all and are therefore, neutral are called
ed
m= = = 1.67 × 10–27 kg
e/ m 7
9.58 × 10 C kg
−1 gamma (γγ) rays. These rays were regarded as high
energy electromagnetic radiations having no charge
This mass is about 1837 times the mass of an
rv
and negligible mass.
electron. This lightest positively charged particle was
named proton and is also regarded as a fundamental
se
particle. Thus, a proton is
a sub-atomic particle which carries one unit
re
positive charge (1.6022 × 10–19 coulombs) and
has mass (1.67 × 10 –27 kg) equal to that of
an atom of hydrogen.
ts
LEARNING PLUS
gh
J Discovery of Radioactivity
Electrically charged
The discovery of cathode rays and anode rays plates
ri
showed that atoms are divisible into subatomic Fig. 5. The effect of the electric field on the radiations
particles. This was further supported by the discovery ll from a radioactive substance.
of radioactivity by a French scientist Henri Becquerel
.A
in 1896. Discovery of Neutron
Radioactivity is the phenomenon of spontaneous After the discovery of electron and proton, a need
emission of active radiations by certain elements. was felt for the presence of electrically neutral particle
rs
The substances which emit such radiations are as one of the constituent of atom. The neutral particle,
called neutron, was discovered by Chadwick and it
he
The effect of magnetic and electric fields on these beryllium element with α-particles and observed highly
radiations was studied by placing a small sample of penetrating rays which consist of streams of neutral
ub
radioactive substance, uranium, in a cavity of a block particles. The neutral particles were found to have
of lead. The radioactive radiations coming out from a mass of 1.675 × 10–27 kg which is nearly the same as
P
narrow slit were allowed to pass through a strong that of hydrogen atom and have no charge. These were
electric or magnetic field. The deflections of these named neutrons.
n
α-particle Neutron
types of rays. These were : Thus, a neutron is a sub-atomic particle having
(i) Alpha (α
α) rays. The rays which are deflected mass (1.675 × 10–27 kg) equal to that of hydrogen
od
towards the negative electrode are called alpha (αα) atom and carrying no electrical charge.
rays. These rays were found to consist of positively Thus, an atom consists of three fundamental
M
charged He2+ particles. Each particle has a mass of 4 particles; electron, proton and neutron. Their mass
a.m.u. (m = 6.6 × 10–24g) and carries two units of and charge are summarized below in Table 1.
©
Note : Not in the syllabus of CBSE
.
Neutron (n) 1.67493 × 10–27 1.00867 1 0 0
ed
REMEMBER
rv
One unit charge = 4.80298 × 10–10 esu or = 1.60210 × 10–19 Coulombs
se
1
One u = th the mass of C–12 or = 1.66056 × 10–27 kg
12
re
the distribution of subatomic particles in an atom. The performed a series of experiments known as
first simple model was proposed by J.J. Thomson Rutherford’s scattering experiments. In their
known as Thomson’s atomic model. experiments, they bombarded a target of atoms by
ts
THOMSON’S MODEL OF ATOM subatomic projectiles. These projectiles called alpha
(α ) particles, were obtained from a radioactive
gh
J.J. Thomson proposed that an atom consists of a
uniform sphere in which positive charge is uniformly substance. Alpha particles are high energy positively
distributed. The electrons are embeded into it in such charged helium ions having charge +2 and mass 4 u.
ri
a way as to give the most stable electrostatic They bombarded alpha (α) particles emitted from a
arrangement (Fig. 6). The radius of the sphere is of radioactive substance on a piece of thin foil of gold or
ll
the order of 10–10 m, which is equal to the size of the some other heavy metals.
.A
atom. This model was much like pudding or cake (of
positive charge) with raisins (electrons) embedded into In this experiment, a piece of radioactive
it. Therefore, this model is also known as raisin substance (radium) is placed in a lead block (Fig. 7).
rs
pudding model. This model was also compared with The block is constructed in such a way with slits that
water melon model of positive charge in which seeds only a narrow beam of α-particles could escape. The
he
(electrons) are embedded. Therefore, this model is beam of high energy α-particles was directed at a thin
given different names such as raisin pudding, plum gold foil (thickness about 100 nm). In order to detect
pudding or watermelon model. the α-particles after scattering, a movable circular
lis
electron
n
Gold
er
Foil
α-particle
This model was soon discarded, when Rutherford source Lead Zinc sulphide
shield Very few few
and his co-workers observed unusual scattering of α- α-particle
M
detector
particles by the thin metal foils. Photographic plate
of the α-particles which got deflected through various (ii) Alpha particles are positively charged and have
angles. The following observations were made from considerable mass. They can be deflected only if they
these experiments : come close to some heavy positively charged mass due
to enormous force of repulsion. Since some of the
(i) Most of the α-particles (nearly 99%) passed
α-particles are deflected to certain angles, it means
through the gold foil undeflected.
.
that there is a heavy positively charged mass present
ed
(ii) A small fraction of α - particles got deflected in the atom. Moreover, this mass must be occupying a
through small angles. very small space within the atom because only a few
(iii) Very few (about one in 20,000) did not pass α-particles suffered large deflections.
rv
through the foil at all but suffered large (iii) The strong deflections or even bouncing back
deflections (more than 90°) or even came back of α-particles from the foil, were explained to be the
se
suffering a deflection of 180°. direct collision with the heavy positively charged mass.
Now, according to Thomson’s model, if the positive The positively charged heavy mass which occupies
re
charge of the atom was uniformly distributed, then only a small volume in an atom is called nucleus. It is
positively charged α-particles with a considerable mass supposed to be present in the centre of the atom.
(4 a.m.u.) would pass through weak electric field largely All these types of deflections of α-particles from
ts
undeflected or slightly deflected. However, he noticed atoms are shown in Fig. 8. It is clear from the figure
that some of the α-particles experienced strong that the α-particles which pass at large distances from
deflections. Even some particles returned back from
gh
the nucleus do not suffer any deflections (marked a),
the foil. Thus, Thomson’s model could not provide those which pass close to the nucleus suffer small
answers for these observations and therefore, was deflections (marked b), while very few which hit the
ri
discarded. nucleus are either deflected to large angles or they
Rutherford explained these observations as retraced their paths (marked c).
follows :
ll On the basis of the above experiments and
(i) Since most of the α-particles passed through observations, Rutherford proposed a model for the
.A
the gold foil undeflected, it means that there must be structure of the atom called Rutherford’s nuclear model
very large empty space within the atom. of atom.
rs
he
Beam of α-particles
lis
ub
P
Rutherford’s Nuclear Model of Atom (iii) The electrons and the nucleus are held together
by electrostatic forces of attraction.
er
the centre known as nucleus. It was observed that CONCEPT OF ATOMIC NUMBER AND MASS
the volume occupied by the nucleus is negligibly small NUMBER
as compared to the total volume of the atom.
M
Atomic Number
(ii) The nucleus is surrounded by negatively The number of unit positive charges carried by the
charged electrons which are revolving around the nucleus of an atom is termed as the atomic number.
©
nucleus at very high speeds in circular paths called Since the positive charge on the nucleus is due to the
orbits. presence of protons in it and each proton carries one
Thus, the Rutherford’s model of atom resembles unit positive charge, therefore, the atomic number is
the solar system in which the nucleus plays the role of numerically equal to the number of protons present in
the sun and revolving electrons play the role of planets. the nucleus of an atom. For example, the number of
protons in the hydrogen nucleus is 1 and therefore,
Note : Not in the syllabus of CBSE atomic number of hydrogen is 1.
.
Z. Thus, Since the atomic number of different isotopes of
ed
the same element is same, it means that they have
Atomic number (Z) = Number of protons (p)
same number of electrons and protons. The
= Number of electrons (e) difference in their mass numbers is due to different
rv
Mass Number number of neutrons present in their nuclei.
Since the electrons are of negligible mass, the entire Hydrogen is the common example which has three
se
mass of the atom is due to protons and neutrons only. isotopes. These are commonly known as hydrogen,
These particles are present in the nucleus and are deuterium and tritium. These three isotopes have the
collectively known as nucleons. The sum of the same atomic number, one, but different mass numbers
re
neutrons and protons is known as mass number. 1, 2 and 3 respectively as given below.
Mass number =
No. of protons + No. of neutrons Isotope Atomic Mass No. of No. of No. of
ts
Mass number is generally represented by the number number electrons protons neutrons
letter A. Hydrogen 1 1 1 1 0
1H
Therefore, from the knowledge of atomic number
gh
1
(or protium)
and mass number of an element, the number of
Deuterium 1 2 1 1 1
electrons, protons and neutrons can be easily predicted. 2 H(or D)
We know 1
ri
Tritium 1 3 1 1 2
Atomic number, Z = No. of protons (p) 3 H(or T)
1
= No. of electrons (e) ll
Mass number, A = No. of protons (p) + Similarly, chlorine has two isotopes having same
No. of neutrons (n)
.A
atomic number, Z = 17 whereas, their mass numbers
Therefore, for an atom with mass number A and are 35 and 37.
atomic number Z :
Isotope Atomic Mass No. of No. of No. of
Number of electrons = Z, Number of protons = Z
rs
17
= 3 and mass number (A) = 7. Therefore,
Number of electrons = Atomic number = 3 37
Cl 17 37 17 17 20
17
Number of protons = Atomic number = 3
lis
7
N , 7
N
Mass number 32 33 34 36
Sulphur
er
S , S , S , S
A 16 16 16 16
Z X Symbol of Uranium
235
92
U ,
238
92
U ,
239
92
U
od
While using the notation AZ X , it is essential to Therefore, all the isotopes of a given element will
know whether the species is neutral atom, a cation show almost same chemical properties.
or an anion. If it is neutral, then number of Isobars
©
protons = number of electrons = atomic number. Atoms of different elements having the same
If the species is an ion, determine whether the mass number but different atomic numbers are
number of protons are larger (for cations) or called isobars.
smaller (for anions) than the number of electrons. Since isobars have same mass number, therefore,
Number of neutrons is always given by A – Z the sum of protons and neutrons in the nucleus of
whether the species is neutral or ion. each atom is the same. These atoms differ in their
atomic number and therefore, they have different number No. of protons = No. of electrons = Z = 35
of protons (or electrons) and also different number of No. of neutrons = A – No. of protons
neutrons. = 80 – 35 = 45.
For example, 40 40 40 Example 2.
18 Ar, 19K and 20Ca are isobars.
The number of electrons, protons and neutrons
The characteristics of these isobars are: in a species are equal to 18, 16 and 16 respectively.
.
ed
Isobar Atomic Mass No. of No. of No. of Assign proper symbol to the species.
number number electrons protons neutrons
40 Solution: Atomic number is equal to number of protons
Ar 18 40 18 18 22
18 = 16
rv
40
19
K 19 40 19 19 21 So, the element is sulphur (S)
40 Mass number = No. of protons + No of neutrons
se
Ca 20 40 20 20 20
20
= 16 + 16 = 32
Isobars are atoms of different elements and Species is not neutral because the number of protons
hence they have different properties. is not equal to number of electrons. It is anion with
re
charge equal to excess electrons = 18 – 16 = 2
Isotones
32 2 −
Atoms having same number of neutrons but ∴ Symbol is 16 S
ts
different mass numbers are called isotones. Example 3.
Thus, isotones have same number of Complete the following table :
neutrons. These atoms differ in mass number as well
gh
as atomic number. For example, 30 Particle Mass Atomic Protons Neutrons Electrons
14Si (14 protons, 16
neutrons), 31 P (15 protons, 16 neutrons) and 32 Number Number
15 16S
ri
(16 protons, 16 neutrons) are isotones because all have O 8 8
16 neutrons. Similarly, 146C (6 protons, 8 neutrons), Al 27 13
15N (7 protons, 8 neutrons) and 16 O (8 protons,
7 8
ll Cl– 35 17
8 neutrons) are isotones because all have 8 neutrons. Mg2+ 12 10
.A
n = 8
different number of neutrons. ∴ Mass number = 8 + 8 = 16
• Isobars have different number of protons as well Now e = p=8
he
as neutrons but same sum of protons and neutrons. Second row : Atomic number
• Isotones have same number of neutrons but = p = e = 13
different number of protons. Protons = 13
lis
∴ n = 35 – 17 = 18
How many protons, electrons and neutrons are Now electrons in Cl atom = p = 17
there in the following nuclei ? ∴ Electrons in Cl– ion = 17 + 1 = 18
n
∴ p = 12
No. of neutrons + No. of protons = A Mass number = 12 + 12 = 24
No. of neutrons + 8 = 17
Atomic No. = p = 12
or No. of neutrons = 17 – 8 = 9
M
.
Mass number = No. of protons + No. of neutrons = 81 negative charge. If the ion contains 11.1% more
ed
i.e., p + n = 81 neutrons than the electrons, find the symbol of
Let number of protons = x the ion.
x × 31.7
rv
Number of neutrons = x +
100
= 1.317 x
se
∴ x + 1.317x = 81
1. (i) 6, 6 (ii) 26, 30 (iii) 38, 50 (iv) 92, 146
81
x = = 34.96 = 35 2. A = 13, A – Z = 7 ∴ Z = 6
2.317
3. No, in all atoms except hydrogen.
re
81
Symbol = 35 Br 4. These are isotopes.
Example 5. 5. (i) 54, 56, 81 (ii) 10, 9, 10 (iii) 88, 88, 138
An ion with mass number 56 contains 3 units of positive 238 234 234 234 234
ts
6. isotopes : 92 U , 92 U; isobars : 90Th , 92 U, 91 Pa
charge and 30.4% more neutrons than electrons. Assign 7. 118
the symbol for the ion.
gh
37 −
8. 17 Cl
Solution: Since the ion carries 3 units of positive charge,
it will have 3 electrons less than the number of protons. Hints & Solutions on page 68
ri
Let number of electrons = x
No. of protons = x + 3
x × 30.4 DRAWBACKS OF RUTHERFORD MODEL
ll
No. of neutrons = x +
100 According to Maxwell theory of electromagnetic
.A
= x + 0.304 x = 1.304 x radiation, a charged body moving under the influence
Now, No. of protons + No. of neutrons = 56 of attractive forces loses energy continuously in the
x + 3 + 1.304 x = 56 form of electromagnetic radiation. Thus, the electron
rs
and smaller and finally the electron would fall into the
nucleus. In other words, the atom should collapse. The
56 3+
Symbol = 26 Fe
calculations have shown that it should take only 10–8 s
ub
12 56
(i) 6C (ii) 26 Fe
er
88
(iii) 38
Sr (iv) 238 .
92 U
2. An atom having atomic mass number 13 has
od
38 Sr 38 Sr
5. How many electrons, protons and neutrons are
Therefore, Rutherford model cannot explain
present in each of the following ?
the stability of an atom.
137 2+ 19 − 226
(i) 56 Ba (ii) 9F (iii) 88 Ra
There is another serious drawback of the
6. From the following nuclei select the isotopes and Rutherford model. This model does not explain the
isobars :
.
During the period of development of new models
ed
to improve Rutherford model of an atom, two new
concepts played a major role. These are :
rv
1. Dual behaviour of the electromagnetic Fig. 10. Wave motion.
radiation. This means that light has both
Characteristics of Wave Motion
se
particle like and wave like properties.
2. Atomic spectra. The experimental results All waves are characterised in terms of their
regarding atomic spectra of atoms can only be wavelength, frequency, velocity and amplitude. These
re
explained by assuming quantized (fixed) are discussed below :
electronic energy levels in atoms. 1. Wavelength (λ λ, lambda)
Let us briefly learn about these concepts before The distance between two adjacent crests or
ts
studying a new model proposed by Niels Bohr known troughs is called wavelength.
as Bohr Model of Atom. It is denoted by the Greek letter lambda (λ) and is
gh
generally expressed in terms of Angstrom units,
NATURE OF LIGHT AND ELECTROMAGNETIC denoted as Å [1Å = 10–8 cm or 10–10 m]. It can also be
RADIATION expressed as micron meter (μ), millimicron meter
ri
The earliest view of light, due to Newton, regarded (mμ), nanometer (nm), pico meter (pm), etc. These
light as made up of particles (commonly termed as units are related to SI unit metre (m) as :
ll
corpuscles of light). The particle nature of light explained 1 Å = 10–10 m ; 1 μ = 10–6 m, 1 mμ = 10–9 m,
.A
some of the experimental facts such as reflection and 1 nm = 10–9 m, 1 pm = 10–12 m
refraction of light. However, it failed to explain the 2. Frequency (ν ν, nu)
phenomena of interference and diffraction. The The number of waves which pass through a
rs
corpuscular theory was, therefore, discarded and given point in one second is known as the
Huygens proposed wave like character of light. With frequency.
he
the help of wave theory of light, Huygens explained the It is denoted by the Greek letter (ν) nu. The units
phenomena of interference and diffraction. of frequency are cycles per second or simply reciprocal
of seconds (s–1). The SI unit of frequency is hertz
lis
Electromagnetic Radiation
(Hz, s–1) after the name of Heinrich Hertz.
In 1870, James Clark Maxwell proposed that light
and other forms of radiant energy propagate through 1 Hz = 1 cycle per second.
ub
space in the form of waves. These waves have electric A cycle is said to be completed when a wave
and magnetic fields associated with them and are, consisting of crest and trough passes through a point.
therefore, called electromagnetic radiations or 3. Velocity (c)
P
quiet pond. The waves originate from the centre of It is the height of crest or depth of trough of
disturbance and propagate in the form of up and down wave.
M
movements. The point of maximum upward It is generally expressed by the letter ‘a’. The
displacement is called crest while the point of amplitude of the wave determines the intensity or
maximum downward displacement is called trough. brightness of radiation.
©
It is denoted by nu bar ( ν ) and is expressed in two field components have the same wavelength and
frequency.
cm–1. It can also be expressed as m–1 (S.I. units).
Thus, (ii) All electromagnetic waves travel with the same
1 speed. In vacuum, the speed of all types of
Wave number = electromagnetic radiation is 3.00 × 108 m s–1 (2.997925
Wavelength
.
× 108 ms1 to be more precise). This speed is called the
ed
1
ν= speed of light.*
λ
Relationship between wavelength, wave (iii) These electromagnetic radiations do not
require any medium for propagation. For example,
rv
number, frequency and velocity. These three
characteristics are related as : light reaches us from the sun through empty space.
se
c=λ×ν Electromagnetic Spectrum
c 1 As already discussed, all electromagnetic waves
or ν= and = ν
λ λ have the same speed (3·0 × 108 m s–1). However, the
re
∴ ν = cν
different types of radiations differ from one another in
Characteristics of Electromagnetic Radiations their wavelengths and therefore, in frequency
ts
The important characteristics of electromagnetic c
(ν = and c is constant). Thus, the electromagnetic
radiations are : λ
radiation with a long wavelength has low frequency
gh
(i) These consist of electric and magnetic fields
that oscillate in the directions perpendicular to each while the radiation with short wavelength has a large
other and both are perpendicular to the direction in frequency. The complete range of electromagnetic
ri
which the wave is travelling as shown in Fig. 11. The waves is called electromagnetic spectrum. Thus,
* The speed of light in air is slightly less than its speed in vacuum. However, the difference is so small that it can be ignored in most cases.
It is interesting to note that visible light, which the wavelength of the electromagnetic radiation
human eye can detect constitutes only small portion emitted by the transmitter. Which part of the
of the total electromagnetic spectrum. The visible electromagnetic spectrum does it belong ?
spectrum is also shown separately. Different colours
in the spectrum correspond to waves of different
.
Solution: The frequency of the radiation emitted,
wavelengths and frequencies. Out of the various
ed
ν = 1368 kHz = 1368 × 103 Hz
colours in the visible range, violet colour corresponds
= 1368 × 103 s–1
to radiation of maximum frequency (7.5 × 1014 Hz) and
rv
minimum wavelength (380 nm) while the red colour c
We know that λ × ν = c or λ=
corresponds to the radiation of minimum frequency ν
se
(4.0 × 1014 Hz) and maximum wavelength (760 nm). where c = 3.0 × 108 m s–1
The frequencies and wavelengths of the radiations
corresponding to colours in between lie between these 3.0 × 108 ms−1
re
λ = = 219.3 m
two extreme values. 1368 × 103 s−1
It is clear from Fig. 12 that radiowaves have the Thus, wavelength of radiowave broadcast by the station
ts
lowest frequencies or highest wavelengths and the = 219.3 m.
gamma rays have the highest frequencies or lowest It belongs to radiowave region of the electromagnetic
wavelengths. The different types of radiations are
gh
spectrum.
arranged in the decreasing order of frequencies or
increasing order of wavelengths as : Example 8.
ri
The wavelength range of the visible spectrum
Gamma rays, X-rays, ultra-violet radiations, extends from violet (400 nm) to red (750 nm).
visible radiations, infra-red radiations, ll Express these wavelengths in frequencies (Hz).
microwaves and radiowaves.
.A
Example 6. c
Frequency, ν =
Yellow light emitted from a sodium lamp has a λ
he
wavelength (λ) of 580 nm. Calculate the frequency where c is speed of light = 3.0 × 108 m s–1
and wave number of this light.
3.0 × 108 m s –1
lis
We know that frequency (ν) is related to wavelength Wavelength of red light, λ = 750 nm
as: = 750 × 10–9 m
P
c c
λ × ν = c or ν = ∴ Frequency, ν =
λ λ
3.0 × 108 m s –1
n
8 −1
3.0 × 10 m s
∴ ν = Thus, the range of visible spectrum in terms of
580 × 10−9 m frequency is from 4.0 × 1014 Hz to 7.5 × 1014 Hz.
od
λ
1
∴ ν =
580 × 10−9 m
= 1.72 × 106 m–1 9. The wavelength of a spectral line of cesium is
©
11. Calculate the wave number of radiations having a radiation does not show this expected behaviour. An ideal
frequency of 4 × 1011 kHz. body which emits and absorbs radiations of all
12. The wavelength of blue light is 480 nm. Calculate wavelengths or frequencies is called black body and
the frequency and wave number of this light. the radiation emitted by this body is called black body
13. Calculate (i) wave number and (ii) frequency of yellow radiation. It has characteristic distribution at a given
.
ed
radiation having wavelength 5800 Å. temperature as shown in Fig. 13. At a given temperature,
intensity of radiation emitted from a black body increases
with decrease of wavelength. It reaches a maximum value
rv
14. The wavelength of a beam of light is 25.0 μm. What
is its frequency and wave number ?
at a given wavelength and then starts decreasing with
further decrease of wavelength.** The figure shows the
se
variation of intensity with wavelength at two
temperatures T1 and T2 (T2 >T1). These results could not
9. 6.52 × 1014 sec–1
re
be explained by the classical wave theory of light.
10. 306 m
According to this theory, energy is emitted or absorbed
11. 1.33 × 106 m–1
continuously. Therefore, the energy of any
12. 6.25 × 1014s–1, 2.08 × 106 m–1
ts
electromagnetic radiation is proportional to its intensity
13. (i) 1.724 × 106 m–1, (ii) 5.172 × 10 14 s–1
and independent of its frequency or wavelength. Thus, the
gh
14. 1.2 × 1013Hz, 4.0 × 104 m
radiation emitted by the body being heated should have
Hints & Solutions on page 68
the same colour (wavelength or frequency) throughout,
although, the intensity of the colour might change with
ri
PARTICLE NATURE OF ELECTROMAGNETIC variation in temperature.
RADIATION AND PLANCK’S QUANTUM ll
THEORY
.A
The electromagnetic wave theory of radiation
believed in the continuous generation of energy. This
theory explained the phenomenon of propagation of
rs
becomes brighter orange, then yellow, then white and different temperatures (T2, T1).
finally, it becomes blue at very high temperatures. As we
It is clear from the figure that at each
n
know, red light has higher wavelength and blue light has
temperature, there is a wavelength at which the
lower wavelength so in terms of wavelength, the radiation
er
the intensity would keep on increasing indefinitely as the experiment. He observed that when light of certain
wavelength becomes shorter and shorter and may enter frequency strikes the surface of some metals, electrons
ultra violet region. However, the intensity of black body (or electric current) are ejected from the metals.
©
.
The apparatus showing photoelectric effect is
ed
intensity as shown in Fig. 15 (b).
shown in Fig. 14 below. The cell consists of an All these observations could not be explained on
evacuated chamber which contains two electrodes the basis of classical laws of physics. According to the
rv
connected to an external circuit. The metal that classical laws of physics, the energy content of beam of
exhibits the photoelectric effect is made negative light depends upon the brightness of the light. In other
se
electrode. When light of sufficiently high energy words, number of electrons ejected and the kinetic energy
strikes the metal, the electrons are ejected from its associated with them should depend on the brightness of
surface and move towards the positive electrode and light. However, as has been discussed above though the
re
form the current flowing through the circuit. number of electrons ejected depends upon the brightness
of light, the kinetic energy of the electrons does not. For
example, red light (ν = 4.3 – 4.6 ×1014 s–1) of any
ts
brightness may shine on a potassium surface for hours
but it does not eject photoelectrons. But yellow light
gh
(ν = 5.1 – 5.2 × 1014 s–1) of even a very weak brightness
ejects photoelectrons. This is because the threshold
frequency (ν0) for potassium metal is 5.0 × 1014 s–1 and
ri
light of frequency more than ν0 (i.e., yellow light and not
red light) can cause photoelectric effect.
ll
.A
no matter how long it falls on the surface or Fig. 15 Variation of kinetic energy of
how high is its intensity. photoelectrons with frequency and intensity.
M
.
ed
continuously but discontinuously in the form of Explanation of Photoelectric Effect using
small packets of energy called quanta. Each such Quantum Theory
quantum is associated with a definite amount of According to Einstein, when a photon strikes a
rv
energy. metal surface, some of its energy is used up to eject
In case of light, the quantum of energy is often the electron from the metal atom (equal to the energy
se
called photon. binding the electron with the nucleus) and the
(ii) The amount of energy associated with a quantum remaining energy is given to eject electron in the form
of radiation is proportional to the frequency of of kinetic energy. This may be expressed as :
re
light, Energy of striking photon = Binding energy +
Ε ∝ ν or E = hν ...(i) Kinetic energy of ejected electron
where the proportionality constant, h, is a This means that a certain minimum amount of
ts
universal constant known as Planck’s constant. energy corresponding to the binding energy (also called
It has the value of 6.626 × 10–34 J s or 3.99 × 10–13 threshold energy) is necessary to detach the electron
kJ sec mol–1. This relation was found to be valid from the metal. Thus, when a photon of energy hν,
gh
for all types of electromagnetic radiations. strikes a metal surface, (Fig. 16), some of its energy,
(iii) The total amount of energy emitted or absorbed called threshold energy x is used up to remove the
by a body will be some whole number multiple of electron from the surface and the remaining energy is
ri
quantum, i.e., imparted to the ejected electron as kinetic energy
nhc 1
llmv2.Therefore,
E = nhν or E= ... (ii)
λ 2
.A
where n is an integer such as 1, 2, 3, .... 1
hν = x + mv2
This means that a body can emit or absorb energy 2
equal to hν, 2 hν, 3 hν.... or any other integral If the threshold frequency is ν 0 , then the
rs
multiple of hν but cannot emit or absorb energy threshold energy, x = hν0 so that
equal to 1.6 hν, 3.2 hν or any other fractional 1
value of hν. hν = hν0 + mv2
he
2
The relation [equations (i) and (ii)] give the 1
or mv2 = hν – hν0
relation between energy of the radiation and its 2
lis
wavelength or frequency. It shows that the higher the Therefore, the kinetic energy of ejected electron,
frequency (or the lower the wavelength), the more FG 1 mv IJ
K.E.
H2 K = h (ν – ν0)
2
energetic are the corresponding photons. For example,
ub
R.U. Curious...
od
.
(hν – hν0) is imparted to the ejected electron as kinetic Ratio of energy of first and second radiations,
ed
energy. Thus, as the frequency of radiation increases, E1 2.48 × 10 –19 J 1
the kinetic energy of the electron increases. = =
−19 2
(iii) Each photon can eject the electron. On
E2 4.97 × 10 J
rv
increasing the intensity of light of a given frequency, ∴ E1 : E2 = 1 : 2 or E2 = 2E1
the number of photons striking the surface is increased Thus, energy of the radiation with wavelength 400 nm
se
but the kinetic energy remains unchanged. is twice that of the radiation of wavelength 800 nm.
Consequently, the greater intensity of light of given Example 11.
frequency (more than ν0) results into more electrons A 100 watt bulb emits electro-magnetic light of
re
being ejected but their kinetic energy does not change. wavelength 400 nm. Calculate the number of photons
REMEMBER emitted per second by the bulb.
The energy acquired by an electron when it is Solution: Energy emitted by the bulb = 100 watt
ts
= 100 J/s
accelerated through a potential difference of 1 volt is
Energy of one photon,
called one electron volt (1eV)
gh
hc
1eV = charge on electron × 1 volt E = hν =
λ
= 1.602 × 10–19C V = 1.602 × 10–19J λ = 400 nm = 400 × 10–9 m, h = 6.62 × 10–34 J s
c = 3.0 × 108 m s–1
ri
Dual nature of Electromagnetic radiation.
Light has been regarded as waves to explain the −34
× 108 m s −1 )
∴ E = (6.62 × 10 J s) × (3.0
phenomena of reflection, refraction, diffraction, etc. ll −9
400 × 10 m
However, in order to explain the photoelectric effect, = 4. 965 × 10–19 J
.A
Einstein regarded the light as tiny particles called No. of photons emitted per sec
photons. In other words, light behaves like waves
100
as well as like particles. Since light is a kind of =
radiation, it may be concluded that all radiations 4.965 × 10 −19
rs
known as dual nature of radiation. Calculate the energy of one mole of photons of
radiation whose frequency is 5 × 1014 Hz.
lis
other with wavelength of 400 nm. photons must possess to eject electrons from cesium
Solution: Energy of photon, metal. The threshold frequency of cesium metal is
4.6 × 1014 s–1 (h = 6.63 × 10–34 J s).
©
hc
E = hν =
λ Solution : [Threshold frequency (ν0) is the minimum
Here c = 3.0 × 108 m s–1 frequency that the photons must possess to eject
In first case, λ = 800 nm = 800 × 10–9 m electrons from metals. Therefore, the energy
corresponding to ν0 is the minimum energy required,
(6.626 × 10 −34 J s) × (3 × 108 m s−1) also called work function].
∴ E1 =
800 × 10−9 m Threshold frequency, ν0 = 4.6 × 1014 s–1
= 2.48 × 1019 J
Minimum energy required to eject the electrons from Solution : Energy of the striking photon,
cesium metal,
hc
E0 = hν0 E = hν =
λ
h = 6.63 × 10–34 J s h = 6.626 × 10–34 J s, c = 3.0 × 108 m s–1,
∴ E0 = (6.63 × 10–34 J s) × (4.6 × 1014 s–1) λ = 300 nm = 300 × 10–9 m
= 6.63 × 4.6 × 10–20 J = 3.05 × 10–19 J
d.
6.626 × 10 –34 J s × 3.0 × 108 m s –1
∴ E =
Example 14. 300 × 10 –9 m
ve
Calculate the kinetic energy of the ejected electron = 6.626 × 10–19 J
when ultra-violet radiation of frequency 1.6 × Kinetic energy of emitted electrons = 1.68 × 105 J mol–1
1015 s–1 strikes the surface of potassium metal.
r
1.68 × 105
Threshold frequency of potassium is 5 × 1014 s–1 Kinetic energy of emitted one electron =
se
6.022 × 1023
(h = 6.63 × 10–34 J s).
= 2.79 × 10–19 J
Solution : K.E. of the ejected electron is given by Now,
re
K.E. = hν – hν0 = h (ν – ν0) Energy of striking photon = Minimum energy required
ν = 1.6 × 1015 s–1, ν0 = 5 × 1014 s–1 to eject electron + Kinetic energy of electron or Minimum
∴ K.E. = (6.63 × 10–34 J s) × (1.6 × 1015 – 5 × 1014) s–1 energy required for ejection of an electron
ts
= (6.63 × 10–34 J s) × (11 × 1014 s–1) = 6.626 × 10–19 J – 2.79 × 10–19 J
= 7.29 × 10–19 J = 3.84 × 10–19 J
The wavelength which will cause photoelectron
gh
Example 15. emission,
When light of wavelength 470 nm falls on the surface λ =
hc
ri
of potassium metal, electrons are emitted with a E
velocity of 6.4 × 104 m s–1. What is the minimum 6.626 × 10–34 J s × 3.0 × 108 m s–1
=
energy required per mole to remove an electron ll 3.84 × 10 –19 J
from potassium metal ? = 5.17 × 10–7 m
.A
Solution : Velocity of emitted electrons or = 517 × 10–9 m = 517 nm.
= 6.4 × 104 m s–1
1
Kinetic energy of emitted electrons K.E. = mv2
rs
2
1
= × 9.1 × 10–31 × (6.4 × 104)2 15. Find energy of each of photons which
he
2
(i) have wavelength of 0.50 Å
∴ = 1.864 × 10–21 kg m2 s–2
(ii) correspond to light of frequency 3 × 1015 Hz.
= 1.864 × 10–21 J
16. Calculate the energy of one of the photons of a beam
Energy of photon,
lis
6.63 × 10 −34 × 3.0 × 108 18. A photochemical reaction requires 9.6 × 10–16 J
E =
470 × 10 −9 energy per molecule. Calculate the number of
photons per molecule of light with wavelength
= 4.23 × 10–19 J
P
or hν0 = hν – K.E.
= 4.23 × 10–19 – 1.864 × 10–21 electron emitted when radiation of frequency
er
Example 16.
When electromagnetic radiation of wavelength 300
nm strikes a metal surface of sodium, electrons 15. (i) 3.98 × 10–15 J (ii) 1.98 × 10–18 J
©
.
(i) Emission spectra.
ed
A similar spectrum is produced when a rainbow forms
Emission spectra are obtained when the radiations
in the sky. This means that sunlight is composed of
emitted from substances that have absorbed energy
collection of electromagnetic waves having different
rv
(either by passing electric discharge through a gas at
wavelengths. The prism bends the light of different
low pressure or by heating the substance to high
wavelengths to different extents. The red colour with
se
temperature) are analysed with the help of spectroscope.
the longest wavelength is deviated the least while the
Atoms, molecules or ions that have absorbed radiations
violet colour with the shortest wavelength is deviated are said to be excited. For example, when the gases or
re
the most. The splitting of light into series of colour vapour of chemical substances are heated by electric
bands is known as dispersion and the series of colour spark, light is emitted. The colour of the light depends
bands is called a spectrum. In this spectrum, there is upon the substance under investigation. For example,
ts
continuity of colours i.e., one colour merges into the sodium or salt of sodium gives off yellow light while
other without any gap or discontinuity and such a potassium or salt of potassium produces a violet colour.
gh
spectrum is known as continuous spectrum. When the radiations emitted by different substances
are analysed, the spectrum obtained consists of sharp
Atomic Spectra well-defined lines each corresponding to a definite
ri
Unlike the spectrum obtained by analysing the frequency (or wavelength).
sunlight, the spectra of atoms are not continuous. The ll The emission spectrum obtained by analysing the
spectra of atoms consist of sharp well-defined lines or radiation emitted by passing electric discharge through
hydrogen gas at low pressure is shown in Fig. 17.
.A
rs
he
lis
ub
Such a spectrum consisting of lines of definite of some salt and the transmitted light is analysed, we
frequencies is called line spectrum or discontinuous obtain a spectrum in which dark lines are observed in
an otherwise continuous spectrum. These dark lines
n
spectrum.
indicate that the radiations of corresponding
The line spectrum is also known as atomic
er
example, we always get two important lines at 589 nm characteristic of the substance. It may be noted that
and 589.6 nm in the spectrum of sodium whatever may these dark lines appear exactly at the same place where
the lines in the emission spectrum appear. For
©
.
ed
rv
Fig. 18. Absorption spectrum of sodium chloride.
se
Differences between Emission and n is an integer equal to or greater than 3
Absorption Spectra (i.e., n = 3, 4, 5 ...). It is known as Balmer formula.
The Balmer formula gives only the spectral lines
re
The essential differences between emission and
in the visible region. These series of lines which appear
absorption spectra are given below :
in visible region were named Balmer series.
Emission spectrum Absorption spectrum Soon afterwards, a series of spectral lines of
ts
1. Emission spectrum is 1. Absorption spectrum is
hydrogen atom in different regions were discovered.
obtained when the white These lines in different regions were grouped into five
gh
obtained when
radiations emitted by light is first passed different series of lines, each being named after the
the excited substance through the substance (in name of its discoverer. These are Lyman series, Balmer
are analysed with a gaseous state or in series, Paschen series, Brackett series and Pfund series.
ri
spectroscope. solution) and the Lyman series appears in the ultra-violet region. Balmer
transmitted light is
series appears in visible region while the other three
ll
analysed with a
spectroscope.
series lie in the infra-red region.
.A
2. Emission spectrum 2. Absorption spectrum
Rydberg Equation
consists of bright consists of dark lines in
coloured lines separated As the other series of hydrogen spectral lines
an otherwise continuous
by dark spaces. were discovered, a more general expression was found as :
rs
spectrum.
F1 I
Emission Spectrum of Hydrogen Atom 1
= ν (in cm–1) = R GH n −
1
JK
he
light is analysed with the help of spectroscope. The R is a constant, now called the Rydberg constant.
spectrum consists of a large number of lines appearing The value of R is 109677 cm–1. The expression is found
to be valid for all the lines in the hydrogen spectrum
ub
In 1885, J.J. Balmer developed a simple series in the hydrogen spectrum is the line when n2
relationship among the different wavelengths of the in the Rydberg equation is infinity i.e., n2 = ∞ . This
line will have the shortest wavelength and largest
n
2 n
M
©
.
ed
rv
Continuous emission spectrum
of white light
se
re
Orange
Yellow
Indigo
Green
Violet
Blue
ts
Red
gh
Visible spectrum
ri
ll
.A
rs
he
lis
Prism
Slit
ub
P
Source of
n
white light
er
od
M
©
re
ts
gh
ri
24 22 20 18 16 14 12 10 8 6 4 2 0
10 10 10 10 10 10 10 10 10 10 10 10 10 n(s-1 )
ll
g–rays X–rays UV IR Microwaves FM AM Long radio waves
.A
Radio waves
10 10 10 10 10 10 10 10 10 10 10 10
he
lis
V I S I B L E S P E C T R U M
ub
P
rv
se
re
ts
Film
gh
Excited
sample
ri
Prism
ll
.A
rs
he
lis
Absorbing sample
ub
Source of
white light
P
n
er
od
M
©
Emission spectra of some elements
.
ed
Hydrogen
rv
Helium
se
re
Neon
ts
gh
Sodium
ri
Mercury ll
.A
650 600 550 500 450 400 350
Wavelength(nm)
rs
It may be noted that the dark lines in absorption spectra appear exactly at
lis
the same place where the coloured lines appear in the emission spectrum.
ub
P
.
in which lines appear and the values of n1 and n2 are
(i) Calculate the wavelength and frequency of this
ed
given in Table 2.
spectral line.
Table 2. Different spectral lines in the (ii) To which spectral series does this line belong?
rv
spectrum of hydrogen atom.
(iii) In which region of the electromagnetic
Series Region n1 n2 spectrum, will this line fall ?
se
Lyman Ultra-violet 1 2, 3, 4, 5... Solution: (i) According to Rydberg equation,
Balmer Visible 2 3, 4, 5, 6 ... F1 I
GH n JK
re
1 1
=R −
Paschen Infra-red 3 4, 5, 6, 7 ... λ 1
2
n22
Brackett Infra-red 4 5, 6, 7, 8 ... where R = 109677 cm–1, n1 = 3 and n2 = 5
ts
Pfund Infra-red 5 6, 7, 8, 9 ...
Substituting the values,
It may be noted that the above equation is true FG IJ cm
gh
1 1 1
only for the spectral lines of hydrogen atom or λ
= 109677 −
H
32 52 K –1
ri
9 25 K
–1
The Rydberg equation for hydrogen like ions may
be expressed as : ll 16
or = 109677 × cm–1
225
.A
1 ⎛ 1 1⎞
= ν (cm −1 ) = R ⎜ 2 − 2 ⎟ Z2 225
λ ⎝ 1
n n2⎠ ∴ λ= cm
109677 × 16
rs
where Z is the nuclear charge, which is equal to = 12.82 × 10–5 cm = 1282 × 10–9 m
atomic number and R is Rydberg constant. For or λ = 1282 nm
example, for He+, Z = 2, for Li2+, Z = 3 and so on.
he
c
Now λ×ν = c or ν=
λ
where c = 3.0 × 108 m s–1, λ = 1282 nm = 1282 × 10–9 m
lis
Example 17.
3.0 × 108 m s−1 3
What is the wavelength of light emitted when the ν = = × 1017 s–1
1282 × 10−9 m
ub
1282
electron in hydrogen atom undergoes transition
from an energy level with n = 4 to an energy level = 2.34 × 1014 s–1
with n = 2 ? What is the colour of the radiation ? (ii) Since this line corresponds to n2 = 3, it
P
region.
F I
er
1
λ
=R GH
n
1
1
2
−
n
1
2
2 JK Example 19.
The wavelength of first spectral line in the Balmer
Here n1 = 2, n2 = 4 and R = 109677 cm–1 series is 6561 Å. Calculate the wavelength of the
od
1 FG
1 1 IJ cm second spectral line in Balmer series.
λ
= 109677
H−
22 42 K –1
Solution: According to Rydberg equation,
F I
M
F 1 1 IJ cm
= 109677 GH −
1
=R
1
−
1
GH JK
4 16 K
–1 λ n12 n22
For first line in Balmer series, n1 = 2, n2 = 3
©
12 1 1 1 FG 5 IJ FG IJ
= 109677 ×
64
cm–1 ∴
6561
= R 2 − 2 =R
2 3 H 36 K H K ...(i)
Dividing eq. (i) by (ii), 2. The energy of an electron in the orbit does
λ 5 16
×
not change with time. In other words, as long as the
=
6561 36 3 electron remains in a particular orbit, it does not lose or
6561 × 5 × 16 gain energy. This means that the energy of the electron
∴ = = 4860 Å
36 × 3 in a particular energy shell remains constant.
.
Therefore, these orbits are also called stationary
ed
states or allowed energy states. The term stationary
does not mean that electron is stationary but it means
21. The first line in Balmer series corresponds to
that the energy of the electron does not change with
rv
n1 = 2 and n2 = 3 and the limiting line corresponds
to n1 = 2 and n2 = ∞ . Calculate the wavelengths of time. This accounts for the stability of an atom.
the first and limiting lines in Balmer series. 3. Only those orbits are permitted in which the
se
22. Calculate the wavelength of spectral line in Lyman
angular momentum of the electron is a whole number
series corresponding to n2 = 3.
23. Calculate the wavelength and energy of radiation h
multiple of (where h is Planck’s constant). An
re
emitted for the electronic transition from 2π
infinity ( ∞ ) to the stationary state of the hydrogen electron, like any other moving body moving in a
atom. circular orbit has an angular momentum equal to mvr,
ts
24. Calculate the wave number for the longest where m is the mass of the electron moving with the
wavelength transition in the Balmer series of atomic velocity v and r is the radius of the orbit. Thus,
hydrogen.
gh
according to Bohr, the angular momentum mvr is a
h
whole number multiple of .
2π
ri
21. First line = 656 nm, limiting line = 364.7 nm.
22. For Lyman series n1 = 1; λ = 102.6 nm. h
23. λ = 9.11 × 10–8 m, E = 2.18 × 10–18 J ll i.e., mvr = n
2π
24. 1.523 × 106 m–1
where n = 1, 2, 3 .....
.A
Hints & Solutions on page 68
In other words, angular momentum of the
BOHR’S MODEL FOR HYDROGEN ATOM electron may be
rs
proposed a model of the atom which was based upon This postulate, therefore, introduces the concept
the Planck’s quantum theory of radiation. The new of quantization of angular momentum.
model is called Bohr’s model of atom. 4. The energy is emitted or absorbed only when
lis
Postulates of Bohr’s Model of the Atom the electrons jump from one energy level to another.
The basic postulates of Bohr’s theory are : When energy is supplied to an atom, its electrons
ub
1. An atom consists of a small heavy positively absorb one or more quantum of energy and jump to
charged nucleus in the centre surrounded by electrons. higher energy levels [Fig. 21 (a)].
The electrons in an atom revolve around the nucleus This higher energy state of electron is called its
P
only in certain selected circular paths which have a excited state. For example, as shown in Fig. 21 (a),
fixed value of radius and energy. These paths are called when the electron absorbs energy equal to E2 – E1, it
n
orbits. These orbits are associated with definite jumps to higher energy level. When the electron jumps
energies and are called energy shells or energy back to the lower energy level, it radiates the same
er
levels. These are numbered as 1, 2, 3, 4 ... etc., (from amount of energy [Fig. 21(b)]. This amount of energy
the nucleus) or alternatively these are designated as emitted or absorbed is given by the difference of the
od
K, L, M, N ... etc., shells (Fig. 20). energies of the two energy levels concerned. That is,
ΔE = E2 – E1
where E1 and E2 are the energies of two energy levels
M
.
H-atom, first energy level (n = 1) is called the ground
ed
state and the higher energy levels (n = 2 first excited
state, n = 3 second excited state and so on...) are called
excited states.
rv
This excited state is unstable and the electron
tends to come back to lower energy level. This
se
transition (change) from upper to lower level occurs
with a jump and energy is emitted in the form of a
quantum equal to difference in energies between the
re
two levels. When this quantum of energy strikes the
photographic plate it gives its impression in the form
of line. For example, if the electron comes back from
ts
energy level having energy E2 to energy level having
energy E 1, then the difference (E 2 – E 1) may be
gh
expressed in terms of energy of photon as :
E2 – E1 = hν
ri
Therefore, the frequency of the emitted radiation
Fig. 21. Energy changes in an electron jump. is given by
ll E − E1
Angular momentum ν= 2
h
.A
As we know a linear momentum is the product Since E2 and E1 can have only definite values for
of mass (m) and linear velocity (v) of the body. an atom therefore, ν will also have only fixed values
Similarly, angular momentum is the product of (depending upon energies of E2 and E1). Moreover, the
rs
moment of inertia (I) and angular velocity (ω). energies of different energy levels are characteristic
For an electron of mass m, moving in a circular of an atom, therefore, the emitted frequencies will also
he
velocity. ν= (∵ v × λ = c)
λ
∴ Angular momentum = mr2 ×
v
= mvr where c is the velocity of light.
ub
r
c E − E1 hc
Successes of Bohr’s Model Thus, = 2 or λ =
λ h E2 − E1
The main successes of Bohr’s model are :
P
1. Bohr ’s atomic model explained the Since h and c are constants and E 2 – E 1
stability of an atom. According to Bohr, an electron corresponds to definite energy, thus, each transition
n
revolving in a particular orbit cannot lose energy. from one level to another will produce a light of
definite wavelength. This is observed as a line in
er
as possible, the atom has the minimum possible energy (E4) to first (E1) or from fifth (E5) to first (E1) energy
and is said to be in the ground state. For example, in level, then the corresponding wavelengths of the
case of hydrogen atom there is only one electron and emitted light will be :
this should be present in the lowest energy shell i.e.,
in n = 1 shell. This represents the ground state for hc hc
λ’’ = and λ’’’ =
hydrogen atom. E4 − E1 E5 − E 1
.
ed
Fig. 22. Emission of radiation.
rv
These will give different lines in the spectrum of level (L), while in others may be raised to third (M),
se
the atom having definite wavelengths. fourth (N), fifth (O) energy levels and so on. Now, the
Thus, the different spectral lines in the spectra of excited electrons come back from the higher energy
re
atoms correspond to different transitions of electrons levels to the ground state in one or more jumps.
from higher energy levels to lower energy levels. For example, let us consider some electrons
Simultaneous appearance of a large number present in fourth energy level (n = 4). Some of these
ts
of lines in the spectrum of hydrogen electrons may directly jump to ground state (route a),
This behaviour is quite curious and a question others may first jump to second level and then to first
gh
arises as to why hydrogen atom gives so many spectral (route b) while some others may first drop to third
lines although, it contains only one electron. In fact, level and then finally to the first level either directly
or first to second level (routes c and d). These four
ri
any sample of hydrogen gas contains a large number of
atoms and when energy is supplied by passing electric routes are shown in Fig. 23. When these excited
discharge, the electrons in different hydrogen atoms electrons jump back to various lower energy levels,
ll
absorb different amounts of energies. Therefore, these they emit different amounts of energies. This results
.A
are raised to different energy states. For example, the into different lines depending upon the difference in
electrons in some atoms may jump to second energy energies of the levels concerned.
rs
he
lis
ub
These are also summarized below: charge e, revolving in a circular orbit. The energy of
Lyman series From n = 2 , 3, 4, 5... to n = 1 the electron in the nth orbit has been found to be
Balmer series From n = 3, 4, 5, 6... to n = 2 En = – RH ⎛⎜ 12 ⎞⎟ n = 1, 2, 3 ........
Paschen series From n = 4, 5, 6, 7... to n = 3 ⎝n ⎠
where RH is called Rydberg constant and it is equal to
Brackett series From n = 5, 6, 7,.......to n = 4
.
2 4
2π me
ed
Pfund series From n = 6, 7, 8, ......to n = 5 2 so that
h
2 π 2 me 4
KNOWLEDGE PLUS En = –
n2 h2
rv
The sixth series in the hydrogen spectrum was
Substituting the values of m, e and h, the above
reported by Curtis J. Humphrey in 1953. It was
se
named Humphrey series and are produced when expression becomes
the electron in the hydrogen atom jumps to 6th 2.18 × 10−18
En = – J per atom
energy level (n = 6) from higher energy levels (n = n2
re
7, 8, 9...). For these lines, 13.595 eV
or = – per atom
⎛ 1 1 ⎞ n2
ν = RH ⎜⎜ 2 − 2 ⎟⎟ where n2 = 7, 8, 9... (... 1 eV = 1.6022 × 10–19 J)
ts
⎝6 n2 ⎠
1312
1311.8
These lines lie in the far infrared region. or = – kJ mol–1 or ≈ – kJ mol–1
2
n2
gh
n
ri
In general, the number of emission lines when an of n as 1, 2, 3, 4... in the above relation, we get the
electron jumps from n2 level to n1 level are given energy of electron in various energy levels of hydrogen
by the relation :
ll
atom. These integral numbers (n = 1, 2, 3 .....)
( n2 − n1 )( n2 − n1 + 1) expressing stationary states for electron are known
.A
as principal quantum number. The values for
2
energies for first four orbits are given in Table 3.
For example, in the Lyman series when an
Table 3. The energies of electron in different
electron drops from fifth level (n2 = 5) to ground
rs
3 ⎯→ 2, 3 ⎯→ 1 (2 lines)
2 ⎯→ 1 (1 lines) It is clear from the table that as the value of n
Total (10 lines) increases, the energy difference between successive
Similarly, total spectral lines in Balmer series
P
4 ⎯→ 3, 4 ⎯→ 2 (2 lines) En = –
n2 h2
3 ⎯→ 2 (1 lines) ⎛ Z2 ⎞
Total (6 lines) or = –2.18 × ⎜⎜ 2 ⎟⎟ J per atom
10–18
M
⎝n ⎠
3. Bohr’s theory helped in calculating the ⎛ Z2 ⎞
energy of the electron in a particular orbit of En = – 1311.8 ⎜⎜ 2 ⎟⎟ kJ mol–1
©
.
The velocity of electron in any orbit is given by
ed
energy of an electron in the hydrogen atom is
the expression
negative. What does this negative sign convey ? The
nh
negative sign of energy means that the energy of v=
rv
the electron in the atom is lower than the energy 2π mr
of a free electron at rest. A free electron at rest is If we substitute the value of r, we get
se
an electron that is at sufficiently far away from the 2πe2 Z
v =
nucleus and its energy is assumed to be zero. nh
Mathematically, it corresponds to setting n equal
re
to infinity in the equation so that E∞ = 0. As the 2.19 × 106 Z
or v = m s–1
electron moves closer to the nucleus due to n
electrostatic attraction, work is done by the electron
ts
The velocities of the electrons in different orbits
itself and hence energy is released. Consequently,
may be given as :
its energy decreases and it takes energy values less
gh
Z
than zero, which means negative values. The vn = v0 ×
n
negative sign also indicates that the electron is
bound to the nucleus and a hydrogen atom is in a where v0 is the velocity of the electron in the first
ri
stable state in comparison to a state where electron orbit of hydrogen atom and v0 = 2.19 × 106 ms–1.
is sufficiently far away from the nucleus. ll No. of revolutions made by an electron
around nucleus
Bohr radius
.A
Velocity of electron v
= =
We can also calculate the radius of each circular Circumference of orbit 2πr
orbit. According to Bohr’s model, radius of nth orbit is Substituting the value of r, we get
rs
n2 h2
rn = 2πm v Ze2
4 π2 me2 Z No. of revolutions =
n2 h2
he
the Bohr radius is 52.9 pm (or 0.0529 nm or 0.529 Å). can be quantitatively explained using Bohr’s model.
Thus, for H atom, When an electron jumps from a lower orbit to a higher
orbit, it absorbs energy. When the electron jumps from
ub
ΔE = Ef – Ei
n = 3 , r3 = 52.9 × (3)2 = 52.9 × 9
er
or = 476.1 pm R R
Now, Ef = – H and Ei = – H2
2
Normally, the electron in hydrogen atom is found nf ni
in n = 1 orbit. As the value of n increases, the value of
od
⎛ RH ⎞
– ⎛⎜ − H2 ⎞⎟
R
r will increase. This means that the electron will be ∴ ΔE = ⎜ − 2 ⎟
⎜ ⎟ ⎜ ⎟
present away from the nucleus. ⎝ nf ⎠ ⎝ ni ⎠
M
rn = ⎛ 1 1 ⎞
Z = 2.18 × 10–18 J ⎜ n2 − n2 ⎟
rn ( H-atom ) ⎝ i f ⎠
or rn (H-like) =
Z The frequency (ν) associated with the absorption
52.9 n2 or emission of photon will be
or rn = pm ΔE
Z ΔE = hν or ν= so that
h
ΔE RH ⎛ 1 − 1 ⎞ = 0 – (–2.18 × 10–18)
ν = = h ⎜⎜ n2 n2 ⎟⎟
h ⎝ i f ⎠ = 2.18 × 10–18 J atom–1
2.18 × 10
−18
J ⎛ 1 − 1 ⎞
or = 0 – (–1311.8) kJ mol–1
= ⎜ 2 2⎟ = 1311.8 kJ mol–1
6.626 × 10 Js ⎜⎝ ni nf ⎟⎠
−34
For H-like ions,
.
ed
= 3.29 × 1015 ⎛ 12 − 12 ⎞ Hz 2.18 × 10−18 Z2
⎜ ⎟ En = − J atom −1
⎝ ni nf ⎠
n2
ν − 1311.8 Z2
In terms of wave number, ν =
rv
c
or = − kJ mol −1
n2
− ν RH ⎛ 1 − 1 ⎞ ∴ I.E. = E ∞ – E1
ν = = hc ⎜⎜ n2 n2 ⎟⎟
se
c ⎝ i f ⎠ = 0 – (–IE (H) Z2)
15 −1 ⎛ 1 1 ⎞
3.29 × 10 s = Z2 IE(H)
= ⎜ 2 − 2 ⎟⎟
−1 ⎜ n
⎝ i nf ⎠
re
8 +
3 × 10 ms ∴ For He , Z=2
⎛ 1 1 ⎞ I.E. = 4 × I.E.(H)
= 1.09677 × 107 ⎜ 2 − 2 ⎟ m–1 = 4 × 1311.8
⎜⎝ ni n f ⎟⎠
ts
= 5247.2 kJ mol–1
For 2+
Li = Z = 3
In case of absorption spectrum,
gh
nf > ni I.E. = 9 × I.E. (H)
= 9 × 1311.8
⎛ 1 1 ⎞
the term ⎜ − 2⎟
becomes positive so that ΔE = 11806.2 kJ mol–1
ri
⎜ 2
ni nf ⎟⎠
⎝ It may be noted that ionization of He+ is the
is positive and hence energy is absorbed.
In case of emission spectrum, second ionization energy of He and ionization
ll
nf < ni energy of Li2+ is the third ionization energy of
.A
lithium atom.
⎛ 1 1 ⎞
the term ⎜ − 2⎟
becomes negative so that ΔE is
⎜ 2
ni nf ⎟⎠
⎝
negative and energy is released.
rs
Example 20.
The above expression is similar to that used by
he
Rydberg which he derived empirically using the What are the frequency and wavelength of a photon
experimental data available at that time. Further, it emitted during a transition from n = 5 to n = 2 state in
is clear that each spectral line whether in absorption
lis
⎝ ⎠
absorbed or emitted.
⎛ 4 − 25 ⎞
er
= 2.18 × 10–18 ⎜⎝ 25 × 4 ⎟⎠
Ionization energy of hydrogen atom and
hydrogen like ions = –4.578 × 10–19 J
od
Ionization energy is the energy required to The negative sign indicates that the energy
remove the electron completely from the atom so is emitted. For calculating wavelength and
as to convert it to a positive ion. This means that it
M
.
2 −18 2
n
En = – 2.18 × 10 Z
ed
−1
Calculate the longest wavelength of light that will 2
J atom
n
be needed to remove an electron from the third For first orbit of He+,
orbit of He+ ion. Z = 2, n = 1
rv
Solution: For hydrogen like ion, i.e., He+, the relation 2.18 × 10
−18
−1
× ( 2) J atom
2
is : E1 = –
se
2
1
21 .76 × 10−19 Z 2 = – 8.72 × 10–18 J
En = – J per atom Radius of nth orbit is
n2
re
0.0529 n2
where Z is atomic number. For He+, Z = 2 rn = nm
Z
Energy of He+ ion in third orbit (n = 3) 0.0529 × 12
∴ r1 = = 0.02645 nm
ts
. × 10−19 × 4
2176 2
E3 = – J
32 Example 24.
gh
Energy required to remove an electron from third Bohr
orbit of He+ ion is : Radius of the fourth orbit in hydrogen atom is
ΔE = E∞ – E3 0.85 nm. Calculate the velocity of the electron in
ri
F . × 10 −19 × 4 IJ this orbit (mass of electron = 9.1 × 10–31 kg).
= 0– − GH 2176
9 JK ll Solution: From Bohr ’s postulate, the angular
= 9.67 × 10–19 J momentum (mvr) is given as :
.A
hc nh nh
Now, ΔE = mvr = or v =
λ 2π 2π mr
6.626 × 10−34 × 3.0 × 108 n = 4, m = 9.1 × 10–31 kg, r = 0.85 × 10–9 m
rs
hc
or λ= =
ΔE 9.67 × 10−19 J 4 × 6.626 × 10 −34
∴ v=
= 2.055 × 10–7 m = 205.5 nm
he
22
2× × 9.1 × 10 −31 × 0.85 × 10 −9
Example 22. 7
(i) The energy associated with the first orbit in = 5.45 × 105 m s–1
lis
°
2.17 × 10−18 J r1 = = 0.529 A
En = – atom–1 4π me2 2
n2
(i) For He+ ion, Z = 2, third orbit, n = 3
od
2
5
= – 8.68 × 10–20 J 9
= × 0.529 = 2.384 Å
(ii) Radius of Bohr’s nth orbit is given as : 2
©
∴ For n = 5 r2 (Li2+) =
22 h2
=
4 LM
h2 OP
r5 = 0.0529 × (5)2 nm = 1.3225 nm
2
4π m × 3 × e 2
MN
3 4 π me2
2
PQ
4
= × 0.529 = 0.7053 Å
3
Example 26. 26. The energy difference between two electronic states
is 399.1 kJ mol–1. Calculate the wavelength and
Calculate the ratio of the radius of 2nd orbit of H frequency of light emitted when an electron drops
atom and that of 3rd orbit. from a higher to a lower state. (Planck’s constant,
Solution: The radius of nth orbit (rn) of hydrogen atom h = 3.98 × 10–13 kJ s mol–1).
is given as : 27. How much energy is required to ionise a H-atom if
.
n 2 h2 the electron occupies n = 5 orbit ? Compare your
ed
rn = answer with the ionization energy of H atom.
4π 2me2
rn ∝ n2
28. Light of wavelength 1281.8 nm is emitted when an
rv
Radius of second orbit r2 ∝ 22 or 4 electron of H-atom drops from 5th to 3rd energy
Radius of third orbit, r3 ∝ 32 or 9 level. Calculate the wavelength of the photon emitted
r2 4 when electron falls from third to ground level.
se
∴ =
r3 9 29. What transition in the hydrogen spectrum would
have the same wavelength on the Balmer transition,
Example 27. n = 4 to n = 2 of He+ spectrum ?
The electronic energy of H atom is
re
30. Ionisation energy of hydrogen atom is 13.6 eV. What
1.312 ×106 will be the ionisation energy of He+ and Li2+ ions ?
En = – −1
J mol 31. What is the energy in joules required to shift the
n2
Calculate electron of the hydrogen from the first Bohr orbit to
ts
(i) First excitation energy of the electron in the the fifth Bohr orbit and what is the wavelength of
hydrogen atom. the light emitted when the electron returns to the
gh
(ii) Ionization energy of the hydrogen atom. ground state ? The ground state electron energy is
Solution: (i) First excitation energy is the amount of – 2.18 × 10–11ergs.
energy required to excite the electron from ground 32. Calculate the wave number for the longest
ri
state (n = 1) to first excited state (n = 2). wavelength transition in the Balmer series of atomic
ΔE = E2 – E1 hydrogen.
= −
1.312 × 106 ⎛ 1.312 × 106 ⎞
− ⎜− ⎟
ll 33. The electronic energy in hydrogen atom is given as
−18
.A
22 ⎝ 12 ⎠ 2.18 × 10 J
5 5 En = –
= – 3.28 × 10 + 13.12 × 10 n
2
= 0 – (–1.312 × 106)
= 1.312 × 106 J mol–1
Example 28.
lis
25. 15 lines
The ionization energy of He+ is 8.72 × 10–18 J
26. 300 nm, 1 × 1015 s–1
atom–1. Calculate the energy of first stationary
state of Li2+. 27. 8.72 × 10–20 J, 2.18 × 10–18 J
ub
I.E. of He+ = E∞ − E1 = 0 − ⎜ − 2 ⎟ = 4 k
⎝ 1 ⎠
Hints & Solutions on page 68
∴ 4k = 8.72 × 10–18 J atom–1
od
−18
8.72 × 10
or k= = 2.18 J atom −1
4 Bohr’s Theory and Concept of Quantisation
For Li2+, Z = 3 and first stationary state, n = 1.
M
In other words, n cannot have fractional values so 1. Bohr’s model of an atom could not account for
that the electron cannot have all possible values of the finer details (doublet, two closely spaced lines) of
energy but only those values for which n is a whole the hydrogen spectrum observed using sophisticated
number. Thus, electron can take only certain fixed spectroscopic techniques.
values of energy. It means that when an electron gains 2. Bohr’s model of an atom could not explain the
.
or loses energy, it does so in such a way that n has a line spectra of atoms containing more than one electron
ed
value which is a whole number. In other words, called multielectron atoms. According to Bohr’s theory,
electron does not gain or lose energy in a continuous one and only one spectral line can originate from an
manner but in jumps (or bursts). This led to the concept electron between any two given energy levels. But, if
rv
of quantisation of energy which means that radiant a powerful spectroscope is used, certain single lines
energy is emitted or absorbed in bursts (or jumps) are found to split into a number of very closely related
se
rather than as continuous flow. From the above facts, lines. The existence of such lines could not be explained
it can be concluded that the energy of the electron is on the basis of Bohr’s theory.
quantised. 3. Bohr’s theory failed to account for the effect of
re
magnetic field on the spectra of atoms or ions. It was
KEY NOTE observed that when the atom emitting radiations is
placed in a strong magnetic field, each spectral line is
ts
The quantisation concept may seem to be
mysterious but it is very easy to understand. The discrete further split into a number of lines. This phenomenon
energy levels in Bohr model may be compared with steps is known as Zeeman effect.
gh
of a ladder. Imagine a ladder having steps labelled as 1,
4. Bohr’s theory also could not explain the effect of electric
2, 3, 4... from the bottom.
field (known as Stark effect) on the spectra of atoms.
Your position on the ladder can only be on a step and
ri
never in between. These steps in a ladder are similar to 5. Bohr’s theory does not provide any clue to explain
energy levels (orbits) in an atom. the shapes of molecules arising out of the directional
bonding between atoms.
ll
6. The main objection to the Bohr’s theory came
.A
from the new principles namely dual nature of matter
and uncertainty principle. They introduced the idea of
wave character of electron in addition to its particle
rs
Definite steps (levels) in a ladder character and pointed out that the path of the motion
of the electron cannot be well-defined. Thus, these
he
Limitations of Bohr’s Atomic Theory overruled the Bohr’s idea of well defined circular paths.
Bohr’s theory of atomic structure was quite Thus, we find that Bohr’s model was only partially
successful in explaining the stability of atom and the
lis
the limitations of Bohr’s model are : ideas regarding the wave characteristics of matter.
P
1
n
er
λ
−
From (i) and (ii) ν = cν
Q. 3. Why are Bohr’s orbits called stationary states ?
Ans. Stationary orbits means that the energies of the orbits in which the electrons revolve are fixed.
Q. 4. Which series of hydrogen spectrum lies in the visible region ?
Ans. Balmer series.
Q. 5. To which Bohr’s orbit in hydrogen atom, the electric transition corresponds to third line in the
Balmer series ?
Ans. Fifth to second.
Q. 6. If the energy of an electron in the second Bohr orbit of H-atom is –E, what is the energy of the
electron in the Bohr’s first orbit ?
.
ed
1 1 1 E 4
Ans. En ∝ 2
∴ E1 ∝ and E2 ∝ ∴ 1 = or E1 = 4E2
n 1 4 E2 1
If E2 = – E, then E1 = – 4E.
rv
Q. 7. What do you mean by saying that energy of the electron is quantized ?
Ans. This means that the electrons in an atom have only definite values of energies.
se
Q. 8. The magnitude of charge on the electron is 4.8 × 10–10 e.s.u. What is the charge on the nucleus
of a helium atom ?
Ans. Helium nucleus contains 2 protons and charge of a proton is same as that of an electron. Therefore, the
re
charge on the nucleus of a helium atom is + 2 × 4.8 × 10–10 = + 9.6 × 10–10 e.s.u.
Q. 9. Which of the following relate to light as wave motion or stream of particles or both ?
(i) interference (ii) photoelectric effect (iii) E = mc2 (iv) E = hν
ts
Ans. (i) Wave motion (ii) Particles (iii) Particles (iv) both
Q. 10. Which transitions between Bohr orbits correspond to
(i) second line in the Balmer series and
gh
(ii) first line in Brackett series of the hydrogen spectrum?
Ans. (i) From 4th orbit to 2nd orbit (ii) From 5th orbit to 4th orbit.
Q. 11. Arrange the following types of radiations in increasing order of frequency ?
ri
(a) radiation from microwave oven
(b) amber light from traffic signal
(c) radiation from FM radio
(d) cosmic rays from outer space
ll
.A
(e) X-rays
Ans. Cosmic rays < X-rays < amber light < microwave < FM
Q. 12. Wavelengths of different radiations are given below :
rs
o
λ(A) = 300 nm, λ(B) = 300 μm, λ(C) = 3 nm, λ(D) = 30 A
he
o
λ (C) = 3 nm = 3 × 10–9 m, λ (D) = 30 A = 30 × 10–10 m or 3 × 10–9 m
1
ub
Ans. According to electromagnetic wave theory, energy is emitted or absorbed continuously whereas according
to Planck’s quantum theory, energy is emitted or absorbed discontinuously i.e., in certain definite packets
n
state?
Ans. Energy of an electron in nth level of H–atom is
od
13.6
En = – 2 eV
n
13.6
The energy in first excited state (n = 2), E2 = – eV = – 3.4 eV.
M
22
Q. 15. Although α-particle have a larger charge, but β-particles are deflected more than α-particles
in a given electric field. Why ?
©
Ans. β-particles have very-very small mass and therefore, their charge to mass ratio is large despite their
lower charge. Thus, these are deflected more.
Q. 16. What are nucleons ?
Ans. The protons and neutrons present in the nucleus are collectively called nucleons.
Q. 17. Which of the following has lowest frequency ?
X-rays, γ-rays, microwaves
Ans. Microwaves
.
emitted had twice the kinetic energy as did
ed
emitted when electron falls from this level to next
photoelectrons emitted when the same metal was
lower level.
irradiated with a light of frequency 2 × 1016 sec–1.
Calculate the threshold frequency of the metal. Solution Angular momentum of an electron in a Bohr’s
rv
orbit of H-atom.
Solution Kinetic energy of emitted photoelectrons is
nh
K.E. = hν – hνo = h (ν – νo) mvr =
se
2π
For the light of frequency 3.2 × 1016 sec–1
n × 6.626 ×10−34 kg m 2s−2
K.E1 = h (3.2 × 1016 – νo) 4.218 × 10–34 kg m2s–1 = 22
re
For the light of frequency 2 × 1016 sec–1 2×
7
KE2 = h (2.0 × 1016 – νo)
4.218 × 10 −34 × 2 × 22
It is given that KE1 = 2 KE2 or n = =4
6.626 × 10 −34 × 7
ts
∴ h (3.2 × 1016 – νo) = 2 h (2.0 × 1016 – νo)
3.2 × 1016 – νo = 2 (2.0 × 1016 – νo)
1 ⎛ 1 1 ⎞
Now ν = = 109678 ⎜ 2 − 2 ⎟ cm–1
λ ⎝ n1 n2 ⎠
gh
3.2 × 1016 – νo = 4.0 × 1016 – 2 νo
– νo + 2νo = (4.0 – 3.2) × 1016 The spectral line when electron falls from 4th level to
or ν o = 0.8 × 1016 3rd level so that
ri
or = 8.0 × 1015 sec–1 n2 = 4, n1 = 3
1 ⎛ 1 1⎞
Problem 2. Calculate the wavelength of the spectral
ll = 109678 ⎜⎝ 2 − 2 ⎟⎠ cm–1
λ 3 4
line obtained in the spectrum of Li 2+ ion when
.A
1 ⎛ 16 − 9 ⎞
transition takes place between two levels whose = 109678 ⎜ cm–1
λ ⎝ 9 × 16 ⎟⎠
difference is 2 and the sum is 4.
1 7
Solution Suppose the transition takes place between = 109678 × cm–1
rs
λ 9 × 16
two levels n1 and n2 and
9 × 16
∴ λ = = 1.876 × 10–4 cm
n2 – n1 = 2 ..(i)
he
109678 × 7
and n2 + n1 = 4 ...(ii)
Problem 4. Calculate the ratio of wavelength of first
In order to solve for the values of n1 and n2, add eqn (i)
spectral line of Lyman and Balmer series of hydrogen
lis
and (ii)
spectrum.
2n2 = 6 or n2 = 3
Solution According to Rydberg equation,
and 3 – n1 = 2 or n1 = 1 F I
ub
1 ⎛ 1 1⎞
= R ⎜ 2 − 2 ⎟ Z2
λ ⎝ n1 n2 ⎠ n1 = 1, n2 = 2
FG IJ
n
1 1 1
For Li2+, Z = 3, R = 109677 cm–1
λL
=R 2 − 2
1 2 H K
er
1 ⎛1 1⎞
= 109677 ⎜ 2 − 2 ⎟ × 32 1 3 4
λ ⎝1 3 ⎠ = R × or λ L =
λL 4 3R
od
1 ⎛ 1 1 ⎞ 5R
8 =R ⎜ − ⎟=
= 109677 × ×9 λB ⎝2 2
32 ⎠ 36
9
©
36
1 ∴ λB =
= 109677 × 8 cm–1 = 877416 cm–1 5R
λ
1 λL 4 5R 5
or λ= = 1.14 × 10−6 cm Now = × =
877416 cm −1 λB 3 R 36 27
or λ = 1.14 × 10–8 m or 11.4 nm Ratio of Lyman and Balmer series = 5 : 27.
.
of Li2+. (R = 1.09678 × 107 m–1, c = 3 × 108 m s–1, h = 6.625
ed
× 10–34 J s). 9
or λB =
5R
Solution According to Rydberg equation,
rv
For longest wavelength line in Lyman series,
F1
ν = RG
1 I n1 = 1, n2 = 2,
Hn −
n K
J
se
2 2
1 2 1 ⎛1 1⎞
= R(2)2 ⎜⎝ 2 − 2 ⎟⎠
For lowest frequency (or energy) in Lyman series : λL 1 2
re
n1 = 1, n2 = 2 ⎛ 1 1 ⎞ 4R × 3
= 4R ⎜⎝ − ⎟⎠ =
FG 1 − 1 IJ 1 4 4
∴ ν = 1.09678 × 107
H1 2 K
ts
2 2
1
or λL =
3R
gh
3
= 1.09678 × 107 ×
4 Now λ B – λ L = 133.8 × 10–9 m
ri
133.8 × 10–9 = −
5R 3R
1
Now, ν = ll 1 ⎛ 9 1 ⎞ 1 ⎛ 27 − 5 ⎞
λ = − =
R ⎜⎝ 5 3 ⎟⎠ R ⎜⎝ 15 ⎟⎠
.A
1 1
∴ λ = = or 133.8 × 10–9 =
1 22
×
ν 0.8226 × 107 R 15
rs
= 1.216 × 10–7 m 22
∴ R = = 1.0961 × 107 m −1
= 121.6 nm 15 × 133.8 × 10 −9
he
3.0 × 108 Solution Let nth level of Li2+has the same energy as
= the fourth energy level of H atom.
1.216 × 10 −7
ub
= 1.47 × 10–17 J. 42
Problem 6. Calculate the value of Rydberg constant,
ELi2+ (1) = EH(1) ×32 (∵ Z = 3) (iii)
od
2
4 n2
Solution For He+ ion spectrum
⎛ 1 1⎞ EH (1) E (1) × 32
©
1
= RZ2 ⎜ 2 − 2 ⎟ 2 = H 2
λ ⎝ n1 n2 ⎠ 4 n
.
(Z = 2 for He+ ion)
ed
(ii) Compare the shortest wavelength emitted by
hydrogen atom and He+ ion. E
He+ 8π2me 4 h2
= × =4
rv
Solution EH h 2
2π me 4
2
se
λ E
n1 = 1 to n2 = ∞
hc
λH =
1.312×106 EH
re
E1 = – J mol–1 , E∞ = 0
1
λ hc
∴ ΔE = Ionisation energy of H-atom He+
=
E +
= E∞ – E1
ts
He
gh
λ + EH He+
(ii) The shortest wavelength corresponds to electron He
jump from n2 = ∞ to n1 = 1 for H-atom and He+ Wavelength emitted by H-atom is four times that
ion.
ri
of He+ ion.
general model for an atom. Two important the mass-energy relationships proposed by Max Planck
developments which contributed significantly in the and Einstein.
formulation of a new model were:
he
wave as well as like a particle i.e., it has dual character. E = mc2 ...(ii)
In 1924, de-Broglie suggested that just as light exhibits where c is the velocity of light.
wave and particle properties, all microscopic material
P
hν = mc2
as well as a wave. This means that an electron which Now, since ν λ = c
er
h
or λ=
h h mc
λ= = This equation is valid for a photon. de-Broglie
mv p
suggested that on substituting the mass of the particle
where h is Planck’s constant,v is the velocity and p m and its velocity v in place of velocity of light c, the
(= mv) is momentum of the particles. The waves equation can also be applied to material particles.
.
ed
rv
se
Fig. 25. Electron diffraction experiment by Davisson and Germer.
re
Thus, the wavelength of material particles, influence of strong electrical field so as to accelerate
λ is : the speed of the electrons. When these electrons were
ts
h made to strike against nickel crystals, concentric dark
λ=
mv and bright rings were formed on screen (Fig. 25).
gh
This equation is known as de-Broglie’s equation. These rings were called the diffraction rings
h and this phenomenon was known as diffraction.
or λ= This diffraction pattern of electrons by crystals was
ri
p similar to that observed for X-rays. The diffraction
Planck' s constant patterns of X-rays and that of electrons are shown
= in Fig. 26 (a) and (b) respectively. The similarity of
ll
Momentum
these two patterns indicates that the electrons behave
where p stands for the momentum (mv) of the particle.
.A
like X-rays.
Since h is constant,
1
∴ λ∝
rs
Momentum
It means that the wavelength of a particle in
he
light known as scintillation. It has been observed Since X-rays have wave character, therefore,
that each striking electron produces only one the electrons must also have wave character associated
n
The particle character of matter (e.g. electron) had Let an electron of charge e be accelerated by a
been established from a number of experiments potential V. Then the kinetic energy acquired by the
and its wave character was confirmed by diffraction electron which is accelerated from rest by passing
and interference experiments. through a potential difference V is eV. The kinetic
.
ed
The wave like character of electron helped in energy of the electron moving with velocity v is also
making electron-microscope which is very 1
powerful tool in modern scientific research because
written as mv2. Thus,
2
rv
it achieves magnification of 15 million times. The
electron microscope utilises the wave like behaviour 1
1
mv2 = Ve or v = ⎛⎜ 2 Ve ⎞⎟
2
se
of electrons just as an ordinary microscope utilises
2 ⎝ m ⎠
the wave like nature of light.
Substituting the value of v in the de-Broglie
Protons, neutrons, hydrogen atoms or even fullerene
re
molecules (C60) have also been shown to have wave equation, we get
character. h h
λ = 1
or 1
ts
Significance of de-Broglie Relationship m(2Ve / m) 2 (2Vem) 2
Although the dual nature of matter is applicable Substituting the numerical values of various
gh
to all material objects but it is significant for quantities :
microscopic bodies only. For large bodies, the h = 6.626 × 10–34 kg m2 s–1, e = 1.602 × 10–19 C,
wavelengths of the associated waves are very small m = 9.11 × 10–31 kg
ri
and cannot be measured by any of the available methods.
6.626 × 10−34 kg m2s−1
Therefore, practically these bodies are said to have no ll λ= 1
wavelengths. Thus, any material body in motion can ( ) (
⎡2 × V × 1.602 × 10−19 C × 9.11 × 10−31 kg ⎤ 2 )
.A
⎣ ⎦
have wavelength but it is measurable or significant only
for microscopic bodies such as electron, proton, atom 1.226 × 10 −9
= m
or molecule. This may be illustrated as follows :
rs
V
The wavelength of an electron with mass where V is potential in volts.
If an electron is accelerated through a potential of
he
h
∴ λ =
magnetic radiation and cannot be measured by any 2 × K.E. × m
known method. Therefore, it is difficult to grasp the
©
idea of waves associated with the moving ball. In other Derivation of Bohr’s Postulate of Quantisation
words, we can say that the ball does not have waves of Angular Momentum from de-Broglie Relation
associated with it. Similarly, the other large bodies de-Broglie equation helped in explaining the
are said to be associated with waves but their Bohr’s postulate regarding the quantisation of angular
wavelengths are too small to be measured by any momentum of an electron. Consider an electron moving
conceivable method. around the nucleus in the form of wave in a circular
orbit of radius r. The wave train of electrons may be Distinction between Electromagnetic Waves and
continuously in phase or out of phase. If the two ends Matter Waves
of the wave meet to give a regular series of crests and
troughs, the wave motion is said to be in phase as The important differences between electro-
shown in Fig. 27 (a). If the two ends do not meet to magnetic waves and matter waves are given below :
give a regular series of crests and troughs, it is said to
.
Electromagnetic waves Matter waves
ed
be out of phase [Fig. 27 (b)]. As evident from Fig. 25 (a),
for wave motion to be continuously in phase, the 1. Electromagnetic waves are Matter waves may not be
associated with electric associated with electric
circumference of circular orbit must be an integral
and magnetic fields and magnetic fields.
rv
multiple of number of the wavelengths otherwise the perpendicular to each other
wave would interfere destructively and cancel each and to the direction of
other and, thus, destroy itself. Therefore, by
se
propagation of radiation.
considering the electron as wave, we automatically Matter waves are neither
2. Electromagnetic waves
impose a limit to the number of orbits. can be radiated into space radiated into space nor
re
or emitted. emitted by the particles.
These are simply associated
with the particles.
They require medium for
ts
3. They do not require any
medium for propagation propagation i.e., these
i.e., these waves can pass waves cannot pass
gh
through vacuum. through vacuum.
4. All electromagnetic waves Matter waves travel with
travel with the same velocity. different velocities.
ri
5. The velocity of all The velocity of matter
electromagnetic waves is waves is generally less
ll equal to that of light than that of light.
Fig. 27. Representation of electron waves and orbits (3 × 108 m/s).
.A
(a) in phase (b) out of phase. 6. The wavelengths of The matter waves have
electromagnetic radia- shorter wavelengths given
From the above discussion, it is clear that the tions are much large and by de-Broglie equation,
circumference of the orbit must be integral multiple
rs
ν
or 2πr = nλ where ν is the frequency.
h
But, according to de-Broglie equation, λ =
lis
mv
nh
∴ 2πr = R.U. Curious...
mv
ub
nh
or mvr = ...(iii)
2π J Is electron really a particle or wave ?
Thus, the angular momentum of the electron A question comes to our mind “Is the electron
P
should be an integral multiple of h/2π. In other words, really a particle or is it a wave ?” and “What does it
the angular momentum is quantised. This is the same look like ?”
as Bohr ’s condition for quantisation of angular ® Upto the year 1924, the electron was exclusively
n
.
associated with this moving tennis ball. Will the A beam of helium atoms moves with a velocity of
ed
movement of this ball exhibit a wave character ? 2.0 × 103 m s–1. Find the wavelength of the particle
Explain. (h = 6.63 × 10 –34 kg m2 s–1) constituting the beam (h = 6.626 × 10–34 Js).
Solution : According to de-Broglie equation : Solution :
rv
h 4
λ = Mass of helium atom =
se
mv 6.022 × 1023
m = 6.0 × 10–2 kg,
= 6.64 × 10– 24 g = 6.64 × 10–27 kg
v = 62 m s–1, h = 6.63 × 10–34 kg m2 s–1 According to de-Broglie equation,
re
6.63 × 10−34 kg m 2 s −1
∴ λ = λ =
h
6.0 × 10 −2 kg × 62 m s −1 mv
ts
= 1.8 × 10–34 m h = 6.626 × 10–34 J s,
This wavelength is too small to be measured and therefore, m = 6.64 × 10–27 k g
the wave nature of such a ball cannot be detected. Thus, the ball
gh
v = 2.0 × 103 m s–1
will not have wave character and will describe a fixed path.
6.626 × 10−34 kg m 2s−1
Example 30. λ =
(6.64 × 10−27 kg) × (2.0 × 103 ms−1 )
ri
Calculate de-Broglie wavelength of an electron
= 4.99 × 10–11 m
(mass = 9.1 × 10 –31 kg) moving at 1% speed of
ll = 49.9 pm
light. (h = 6.63 × 10 –34 kg m2 s–1)
Example 34.
.A
h Two particles A and B are in motion. If the
Solution : We know that λ =
mv wavelength associated with the particle A is
m = 9.1 × 10–31 kg, h = 6.63 × 10–34 kg m2 s–1 5 × 10–8 m, calculate the wavelength of particle B
rs
h
Calculate the wavelength of an electron moving For particle B : λB = ...(ii)
pB
with a velocity of 2.05 × 107 m s–1. pA
Solution : According to de-Broglie wavelength, But pB =
P
2
h
λ= h h 2h
mv ∴ λB = p = p = ... (iii)
n
A/2 pA
m = 9.11 × 10–31 kg, B
2
∴ λ= 6.63 × 10 kg m s
(9.11 × 10 kg) × (2.05 × 107 m s−1 )
−31 Now, λA = 5 × 10–8 m
= 3.55 × 10–11 m. ∴ λB = 2 × 5 × 10–8 = 10–7 m.
M
.
= 3.32 × 10–10 m = 332 pm 2
ed
Example 36. 2 × 1.6 × 10−19 × 108
∴ v2 =
An electron is moving with a kinetic energy of 9.1 × 10−31
rv
2.275 × 10–25 J. Calculate its de-Broglie wavelength. (Mass v2 = 0.352 × 1020 m2 s–2
of electron = 9.1 × 10–31 kg, h = 6.6 × 10–34 J s) or v = 5.93 ×109 m s–1
se
Solution : Kinetic energy of electron, h
1 Now, λ =
mv2 = 2.275 × 10–25 J or = 2.275 × 10–25 kg m2 s–2 mv
2
re
2 × 2.275 × 10−25 6.6 × 10−34 Js
or v2 = =
m
m = 9.1 × 10–31 kg
(9.1 × 10 −31
)(
kg 5.93 × 109 m s−1 )
ts
= 1.22 × 10–13 m.
2 × 2.275 × 10−25 kg m 2 s −2
v2 = −31 = 0.5 × 106 m2 s–2
9.1 × 10 kg Example 39.
gh
v = 0.707 × 103 m s–1
The mass of an electron is 9.1 × 10–31 kg. If its
h kinetic energy is 3.0 × 10 –25 J, calculate its
Now, λ =
mv wavelength.
ri
6.6 × 10 −34 kg m 2s −1 1
λ = −31 3 −1 Solution : Kinetic energy, K.E. = mv2
(9.1 × 10 kg) × (0.707 × 10 m s ) ll 2
= 1.026 × 10–6 m = 1026 nm 1
.A
mv2 = 3.0 × 10–25 J = 3.0 × 10–25 kg m2 s–2
Example 37. 2
Calculate the kinetic energy of moving electron 2 × 3.0 × 10–25 kg m 2 s –2
which has a wavelength of 4.8 pm. (mass of electron v2 = (J = kg m2 s–2 )
rs
m
= 9.11 × 10 –31 kg, h = 6.63 × 10 –34 J s).
Solution : According to de-Broglie equation, 2 × 3.0 × 10–25 kg m 2 s –2
=
he
h 9.1 × 10–31 kg
λ =
mv
F 2 × 3.0 × 10 I –25
½
h = 6.63 × 10–34 J s, or v= GH 9.1 × 10 JK m s–1 = 812 m s–1
lis
–31
λ = 4.8 pm = 4.8 × 10–12 m, m = 9.11 × 10–31 kg
6.63 × 10−34 kg m 2s −1 h
Now, λ =
ub
4.8 × 10–12m = mv
9.11 × 10 −31 kg × v
6.63 × 10 −34 kg m 2s −1
6.626 × 10–34 J s
=
or v =
e91. × 10 j
P
–31
(4.8 × 10 −12 m) × (9.11 × 10 −31 kg) kg × (812 m s –1 )
1 2
∴ Kinetic energy = mv
er
2
1
= × (9.11 × 10–31 kg)
od
2
× (1.516 × 108 m s–1)2
34. What will be the wavelength of a ball of mass 0.1
= 1.05 × 10 kg m2 s–2
–14
kg moving with a velocity of 10 ms–1.
or = 1.05 × 10–14 J.
M
a potential difference of 100 million volts. Broglie wavelength of 0.1 nm (h = 6.63 × 10–34 J s).
(1 eV = 1.6 × 10–19 C, me = 9.1 × 10–31 kg, h = 6.6 ×
10–34 Js) (D.S.B. 2003)
37. Calculate the kinetic energy of an α-particle which
Solution : The kinetic energy of the electron under the
potential difference of 100 million volts or 108 V is has a wavelength of 12 pm.
given by the relation :
38. The velocity associated with a proton moving in a be less than h/4π. The sign of equality refers to
potential difference of 1000 V is m s–1. If the hockey minimum uncertainty and is equal to h/4π.
ball of mass 0.1 kg is moving with the velocity, The constancy of the product of uncertainties
calculate the wavelength associated with this.
means that :
(i) If Δx is small i.e., the position of the particle
.
39. What accelerating potential is needed to produce is measured accurately, Δp would be large,
ed
an electron beam with wavelength of 9 pm ? i.e., there would be large uncertainty in its
momentum.
(ii) On the other hand, if Δp is small, the
rv
40. Calculate the de-Broglie wavelength of an electron
that has been accelerated from rest through a momentum of the particle is measured more
potential difference of 1kV. accurately, Δx would be large i.e., there would
se
41. The kinetic energy of a subatomic particle is 5.85 be large uncertainty with regard to the
× 10–25 J. Calculate the frequency of the particle position of the particle.
wave.
re
In other words, if the position of a particle is
42. What must be the velocity of a beam of electrons if measured accurately, there will be more error in the
they are to display a de-Broglie wavelength of measurement of momentum. Conversely, if momentum
100 Å?
ts
is measured more accurately, the position will not be
43. Calculate the wavelength associated with a moving accurately known.
electron having kinetic energy of 1.375 × 10–25 J.
gh
Since momentum p = mv, therefore, Δp = mΔv
(mass of e = 9.1 × 10–31 kg, h = 6.63 × 10–34 kg m2 s–1).
because mass is constant. The above relation may also
be written as :
ri
h
Δx × m(Δv) ≥
34. 6.626 × 10–34m ll 4π
35. 6.139 × 10–33 kg h
.A
or Δx × Δv ≥
36. 6.63 × 10–24 kg m s–1 4 πm
37. 2.30 × 10–19 J This also means that the position and velocity
of an object cannot be simultaneously known
rs
So far, we have been believing that anything can the object is large, its position and velocity will not
be specified to any desired degree of accuracy. change by the impact of the striking photons. Thus, it
n
However, Heisenberg in 1927, put forward a principle will be possible to determine both the position and
known as Heisenberg’s uncertainty principle. It velocity of the object simultaneously.
er
both the position and momentum (or velocity) with the particle will cause appreciable displacement
of a microscopic particle, with absolute of the particle from its normal path. As a result of this,
accuracy. the particle undergoes a considerable change in its
M
Mathematically, this law may be expressed as : path and velocity (or momentum) due to the impact of
h a single photon used to observe it. Thus, the very act
Δx × Δp ≥
of measuring the position of the microscopic particle
©
4π
where Δx = uncertainty in position causes a change in its momentum. Fig. 28 shows as to
Δp = uncertainty in momentum how the position of an electron changes by the impact
The sign > means that the product of Δx and Δp of photon. The collision of photon with the electron
can be either greater than or equal to h/4π. It can never sends it in an unpredictable direction.
.
ed
It should be borne in mind that the uncertainty
is not due to lack of sufficiently refined techniques but
it is due to the fact that we cannot observe microscopic
rv
things without disturbing them. No instrument can
observe the position of an electron without affecting
its motion. In other words, uncertainty principle is
se
the fundamental limitation of nature. Thus, we
cannot design an experiment to obtain an accurate
re
Fig. 28. The change in the path of electron by value of both the position and momentum for
the impact of striking photon. microscopic objects.
Thus, it is not possible to determine simultaneously However, in our daily life, this principle has no
ts
the exact position and momentum of the electron or significance. This is because we come across only large
objects, i.e., the objects which we can observe with naked
of any other microscopic object.
eye without altering their motion. The position and
gh
Let us consider a thought experiment for velocity of these objects can be determined accurately
because in these cases, during the interaction between
determining the position of an electron by using
the object and the measuring device, the changes in
ri
photons of light. According to principles of optics, if position and velocity are negligible.
we use light of wavelength λ, then the position of This may also be illustrated as follows :
electron cannot be located more accurately than +λ.
ll If uncertainty principle is applied to a microscopic
The shorter the wavelength, the greater is the
.A
object like an electron, Δx × Δv comes out to be of the
accuracy. Therefore, to observe the position of the order of 10–5 m2 s–1 as :
electron accurately, light of appropriately small h
Δx × Δv =
rs
be transferred to the electron at the time of collision. position of the electron, say to an uncertainty of about
This will change the velocity of the electron and 1Å (1 × 10–8 m), then uncertainty in velocity, Δv would
ub
change in momentum we have to use light having This is very large. On the other hand, if
photons with small values of p. This will require uncertainty principle is applied to an object of mass
n
radiations of longer wavelengths (λ = h/p). Now, if say about a milligram (10–6 kg), then
er
change appreciably but we will not be able to measure 6.626 × 10−34 kg m 2 s−1
=
position accurately with larger wavelength. Therefore, 4 × 3.1416 × 10−6 kg
uncertainty in position will increase. Thus, we cannot = 5.27 × 10–29 m2 s–1
M
simultaneously measure the position and momentum The value of Δx. Δv for milligram sized or heavier
of a small moving object like electron accurately. objects is extremely small. Therefore, uncertainty of
©
.
4π 4π electron = 300 m s–1).
ed
h Solution : Mass of electron = 9.1 × 10–31 kg,
∴ Δx ≥ h = 6.6 × 10–34 J s
4πmΔv
rv
Δv = 5.7 × 105 m s–1, m = 9.1 × 10–31 kg, (i) When Δv = 0.001% of velocity of electron
h = 6.6 × 10–34 J s 0.001 × 300
= = 3.0 × 10–3 m s–1
se
100
6.6 × 10−34 kg m 2 s−1 h
∴ Δx = Now, Δx × m Δv =
4 × 3.142 × (9.1 × 10−31 kg) × (5.7 × 105 m s−1 ) 4π
re
= 1.012 × 10–10 m. ∴ Δx =
h
4π m Δv
Example 41.
Calculate the uncertainty in the velocity of a wagon 6.6 ×10–34 kg m2 s–1
ts
=
of mass 2000 kg whose position is known to an 4 × 3.142 × (9.1 ×10–31 kg) × (3.0 ×10–3 ms–1 )
accuracy of ± 10 m. = 1.92 × 10–2 m.
gh
Solution : Mass of wagon = 2000 kg (ii) When Δv = 0
Uncertainty in position, Δx = ± 10 m Δx =
h
4π m Δv
ri
According to Heisenberg uncertainty principle
If Δv is zero, then denominator in the above expression
h h
Δx × Δp = or Δx × Δv = becomes zero and, therefore, uncertainty in position is
ll
4π 4πm infinity.
h
Example 44.
.A
or Δv =
4πmΔx
An electron has a speed of 500 m s –1 with
6.626 × 10−34 kg m 2 s−1 uncertainty of 0.02%. What is the uncertainty in
=
4 × 3.1416 × (2000 kg) × (10 m) locating its position ?
rs
Example 42.
On the basis of Heisenberg uncertainty principle, 500 × 0.02
=
show that electron cannot exist within the atomic 100
lis
Δx × Δp > h
4π or Δx ≥
4πmΔv
er
h 7
or Δv > ≥ 5.77 × 10–4 m
4π mΔx
Mass of electron, m = 9.1 × 10–31 kg, ∴Uncertainty in position = 5.77 × 10–4 m.
M
4 × 3 .1 4 × 9 .1 × 1 0 − 3 1 × 1 × 1 0 − 1 5
calculate the uncertainty in position.
= 5.77 × 1010 m s–1. Solution : The uncertainty in speed,
The value of uncertainty in velocity, Δv is much higher 2
than the velocity of light (3.0 × 108 m s–1) and therefore, Δv = 45 × = 0.9 ms–1
100
it is not possible. Hence an electron cannot be found
m = 40 × 10–3 kg
within the atomic nucleus.
h
Δx =
4 πmΔv
44. 5.79 × 106 m s–1.
6.626 × 10−34 kg m 2 s−1
= 45. 3.6 m s–1, 1.46 × 10–33 m.
4 × 3.14 × 40 × 10−3 kg × 0.9 m s−1
46. 1.59 × 104 m s–1.
.
= 1.46 × 10–33m
ed
Example 46. 47. 0.10 kg.
If the position of the electron is measured within 48. 1.06 × 10–13 m.
an accuracy of ± 0.002 nm, calculate the
49. 2.11 × 10–13 m.
rv
uncertainty in the momentum of the electron. If
suppose the momentum of the electron is Hints & Solutions on page 68
se
h
4 π × 0.05 nm QUANTUM MECHANICAL MODEL OF ATOM
is there any problem in defining this value? AND CONCEPT OF ATOMIC ORBITAL
re
The wave character of an electron and uncertainty
Solution : According to uncertainty principle, in its position and momentum gave a serious blow to
ts
h Bohr’s model of an atom. According to Bohr, the
Δx × Δp =
4π electrons revolve around the nucleus in certain well
Δx = ± 0.002 nm = ± 0.002 × 10–9 m defined circular orbits. But the idea of uncertainty
gh
h 6.626 × 10−34 kg m 2s−1 in position and velocity overruled the Bohr’s
Δp = = picture of fixed orbits. Thus, the classical mechanics
4πΔx 4 × 22 × 0.002 × 10−9 ms−1
ri
7 could not describe the behaviour of electrons in atoms
= 2.63 × 10–23 kg m s–1. correctly. Therefore, the scientists started looking for
a model which could incorporate the dual character of
ll
Momentum of electron = h
matter and uncertainty principle. This resulted in a
.A
4 π × 0.05 nm
new approach called quantum mechanics or wave
6.626 ×10 –34 kg m 2s –1
mechanics.
= 4 × 22 × 0.05 ×10 –9 m
Quantum mechanics was developed independently
rs
7
= 1.05 × 10–24 kg m s–1. in 1926 by Werner Heisenberg and Erwin Schrodinger.
Since the actual momentum of electron is smaller than On the basis of quantum mechanics, Schrodinger
he
the uncertainty, it cannot be defined. proposed a model of an atom which considered wave-
particle duality of matter and Heisenberg uncertainty
principle. This model was known as quantum
lis
0.1 Å. What is the uncertainty involved in the equation known as Schrodinger wave equation.
measurement of its velocity ? (mass of electron =
The Schrodinger wave equation is :
9.11 × 10–31 kg, h = 6.626 × 10–34 Js)
P
46. Calculate the minimum uncertainty in velocity of a the coordinates of the position of electron, E is the
particle of mass 1.1 × 10–27 kg if uncertainty in its total energy of the electron, V is the potential energy,
position is 3 × 10–10 cm (h = 6.6 × 10–34 kg m2 s–1).
od
Although the Schrodinger wave equation was motion, the square of the wave function, ψ2 may be
difficult to accept at first, it has now gained wide taken as intensity of electron at any point. In other
acceptance. This is because the results obtained from words, ψ2 determines the probability of finding the
Schrodinger wave equation are in excellent agreement moving electron in a given region i.e. it gives the
with the experimental findings. One of the major probability density. Thus, ψ2 has been called the
.
achievement of Schrodinger wave equation is that it probability density and ψ the probability
ed
successfully interpreted the experimental information amplitude. Hence, the solutions of Schrodinger wave
about atoms and molecules. equation replace the discrete energy levels or orbits
proposed by Bohr and led to the concept of most
rv
KEY NOTE
probable regions in space in terms of ψ2. A large value
Operators of ψ2 means a high probability of finding the electron
se
While studying the state of a system, we make at that place and a small value of ψ2 means low
various measurements of its properties such as mass, probability. If ψ2 is almost zero at a particular point, it
volume, momentum, position and energy. Each individual
re
means that the probability of finding the electron at
property is called observable. In order to determine the that point is negligible.
value of the observable property, we have to perform certain
mathematical operations. This operation is represented
Energy of electron in an atom
ts
by an operator. Therefore, operator is a mathematical When the Schrodinger wave equation was solved
command or instruction which acts on a for hydrogen atom, it gave the following expression
gh
mathematical function. For example, in the equation for the energy of electron :
4 × 5 = 20, the operation is multiplication and the operator
is '×'. We can also express the multiplication operation 2π2 me e4
En = –
ri
^ ^
with some symbols, designated as M . For example, M n2 h2
^
(4 × 5) = 20. In this case, M is multiplication operator, The above expression is same as Bohr’s equation
where the ^ signifies an operator.
ll
for the energy of electron in a hydrogen atom.
.A
Substituting the values of me and h, the above
Schrodinger Wave Equation for Hydrogen expression becomes :
Atom
When Schrodinger wave equation was solved 2.18 × 10 −18
rs
En = – J per atom
for hydrogen atom, the solution gives the possible n2
energy states (or energy levels) that an electron can
he
energy states are characterized by a set of three The important features of the quantum
quantum numbers (principal quantum number n, mechanical model of an atom are summed up below :
azimuthal quantum number l and magnetic quantum 1. The energy of electrons in atoms is quantized
P
number m l). These numbers arise as a natural i.e., they can have only certain specific values.
consequence in the solution of the Schrodinger wave 2. The existence of quantized electronic energy
n
equation. This quantum mechanical model of the levels is a direct result of the wave like properties of
er
hydrogen atom successfully predicts all aspects of electrons and are the allowed solutions of
hydrogen atom spectrum and other phenomena which Schrodinger equation.
od
could not be explained by Bohr model. 3. This gives the most probable regions in an
The Schrodinger wave equation cannot be atom where the probability of finding the electrons
solved exactly for multielectron atoms. However, is maximum. Therefore, this model is in accordance
M
approximate methods can be used to get solutions of with the Heisenberg uncertainty principle and does
reasonable accuracy. not specify the exact position and momentum of the
electron. But it talks about the most probable regions
©
wave functions for an electron in an atom are also Differences between Orbit and Orbital
called orbital wave functions or simply atomic The main differences between orbit and orbital
orbitals. Since many wave functions are possible for are summed up below :
an electron, there are many atomic orbitals in an Orbit Orbital
atom. In each orbital, an electron has definite energy.
1. Orbit is a well-defined 1. It represents the region in
.
Thus, all the information about the electron in an
ed
circular path around space around the nucleus
atom is provided by its orbital wave function ψ. the nucleus in which an in which the probability of
5. The probability of finding the electron at a electron revolves. finding the electron is
point within an atom is proportional to the square of maximum.
rv
the wave function i.e., |ψ|2 at that point. Therefore, 2. It represents the 2. It represents the three-
planar motion of an dimensional motion of an
|ψ|2 is known as probability density and is always
se
electron. electron around the
positive. nucleus.
6. The probability of finding the electron in a 3. Orbit gives a definite 3. Orbital does not specify
re
region having volume dV can be obtained as ψ2. dV, path of an electron and definite path and accor-
where dV is the infinitesimally small volume this concept is not in ding to this concept,
accordance with the electron may be any-where
(dx . dy . dz). in this region. This concept
uncertainty principle.
ts
7. The quantum mechanical model gave three is in accordance with the
constants, known as quantum numbers which are uncertainty principle.
required to specify the position and energy of the
gh
electron in an atom. These are discussed later. 4. All orbits are circular. 4. Orbitals have different
shapes. For example,
s-orbital is spherical,
ri
Probability Picture of an Electron : Concept of while p-orbital is dumb
atomic orbital. bell shaped.
5. Orbits do not have
ll 5. Except s-orbitals, all other
The solution of Schrodinger wave equation led to the directional characte- orbitals have
concept of most probable regions in place of well-
.A
ristics. directional characte-
defined circular paths proposed by Bohr. According to ristics.
this approach, we cannot say simply that the electron 6. An orbit can 6. An orbital cannot
exists at a particular point, but we talk about certain accommodate electrons accommodate more than
rs
regions in space around the nucleus where the equal to 2n2 where n two electrons.
probability (chances) of finding the electron is represents the
he
where there is high probability (90–95%) of finding the finding the electron is maximum. A large number of
electron, the figure obtained, gives the general picture electron orbitals are possible in atom. These can be
of orbital. For the sake of simplicity, it may be drawn distinguished by their size, shape and orientation.
P
as shown in Fig. 29 (a). The orbital, here is shown by To describe each electron in an atom in different
dotted figure representing electron cloud. The orbitals, we need a set of three numbers known as
n
intensity of dots gives the relative probability of finding quantum numbers. These are designated as n, l and
the electron in that particular region. It may be noted
er
1 such as
n = 1, 2, 3, 4 ....
This quantum number also identifies as shell.
The shell with n = 1 is called the first shell. The shell
Fig. 29. (a) Electron cloud representation of an with n = 2 is called the second shell and so on. The
orbital (b) Bohr’s orbit. various shells are also called K, L, M, N as :
.
(i) It gives the average distance of the electron
The letters s, p, d and f originate from the
ed
from the nucleus. If n = 1 (first shell), it is closest to
terms sharp, principal, diffuse and fundamental,
the nucleus and has lowest energy. As the value of n
which were used in the atomic emission spectra.
increases, the distance of the electron from the nucleus
rv
For l = 4 and higher values, the letters follow
increases. alphabetical order after f, i.e., l = 4 is designated
(ii) It determines the energy of the electron, as g; l = 5 is designated as h and so on.
se
according to the formula,
Thus, for n = 1, l = 0
2 π 2 me4 This means that the first principal energy level
re
En = –
n2 h2 consists of only one sublevel termed as s.
Similarly, for n = 2, l = 0, l i.e., the second
1311.8
ts
= – kJ mol–1 principal energy level consists of two sublevels,
n2 s (l = 0) and p (l = 1).
For example, the energy of electron in K-shell For n = 3, l = 0, 1, 2 i.e., third principal energy
gh
(n = 1), is level consists of three sublevels s (l = 0), p (l = 1) and
1311.8 d (l = 2).
E1 = – = –1311.8 kJ mol–1
ri
12 The different subshells or sublevels are
represented by first writing the value of n (1, 2, 3...)
Similarly, the energy of the electron in L shell
and then the letter designation for the value of l (s, p,
ll
(n = 2), E2, M-shell (n = 3), E3....... are respectively,
d, f...). For example, an orbital with n = 1 and l = 0 is
.A
E2 = –327.9 kJ mol–1 E3 = –145.7 kJ mol–1 denoted as ls, an orbital with n = 3, l = 2 is denoted as
Thus, principal quantum number gives the 3d. The designations of subshells for n = 1 to n = 4 are
average distance of the electron from the nucleus (size given below :
rs
{
3 0 3s
subshell or sublevel in a given principal energy shell
ub
1 3p Three
to which an electron belongs. It can have positive 2 3d
{
integer values ranging from zero to (n –1) where n is 4 0 4s
P
3 4f
following informations:
er
(i) It gives the number of subshells present in a (ii) It gives the relative energies of various shells.
principal shell. The value of ‘l’ depends upon the value
od
For n = 2, l has two values : l = 0, 1 i.e., two means that within a given shell, the s-subshell
subshells. (l = 0) has lowest energy, p-subshell (l = 1) has next
to lowest, followed by d, then f and so on. For
©
(iii) This quantum number gives the energy of the If l = 2, ml may be –2, –1, 0, +1, +2.
electron due to the angular momentum of the electron. i.e., d-subshell contains five orbitals called
The angular momentum of the electron is related d-orbitals.
to l as : Similarly,
h
Angular momentum = l ( l + 1) if l = 3, ml may be –3, –2, –1, 0, +1, +2, +3.
2π
.
i.e., f-subshell contains seven orbitals called
ed
KEY NOTE
f-orbitals.
• Some books express angular momentum as : The number of orbitals in a given subshell are
given below :
rv
Angular momentum = l (l + 1)
The use of is a shorthand way of writing h/2π and is Subshell s p d f g
extensively used in Quantum mechanics.
se
No. of orbitals 1 3 5 7 9
• It may be noted that angular momentum depends only on
the value of l and not on the value of n. This means that 1s,
2s, 3s, etc., electrons will have same angular momentum. It can be generalised that there are (2l + 1)
re
Similarly, all p-electrons or d-electrons or f-electrons will orbitals (or m values) for each value of l (or subshell).
have same angular momentum. By working out different combinations of these
Thus, azimuthal quantum number determines the quantum numbers, it can be easily calculated that there
ts
subshell in a given principal shell, angular momentum is one orbital for n = 1 (1s), four for n = 2 (one 2s and
of the electron present in the subshell and relative three 2p), nine for n = 3 (one 3s, three 3p, five 3d).
gh
energies of various subshells. These are given above in Table 4.
3. Magnetic quantum number (ml). This Thus, magnetic quantum number determines the
quantum number describes the behaviour of electron
ri
number of orbitals present in a given subshell.
in a magnetic field. We know that the movement of
electrical charge is always associated with magnetic Table 4. Permitted combinations of n, l and ml.
field. Since the revolving electron possesses angular ll
Value Value Value Subshell No. of
momentum, it will give rise to a very small magnetic
.A
field which will interact with the external magnetic of n of l of ml orbitals
field of the earth. Under the influence of external 1 0 0 1s 1
magnetic field, the electrons in a given subshell orient
{
rs
{
he
each value of l.
that the electron in an atom is not only revolving
This quantum number gives the number of
around the nucleus but is also spinning around its own
orbitals in a subshell. For example,
axis. In 1925, George Uhlenbeck and Samuel Goudsmit
P
If l = 0, m has only one value, i.e., ml = 0 proposed fourth quantum number known as electron
i.e., s subshell has only one orbital called spin quantum number. This quantum number
n
orbitals. However, it may be noted that for a pz-orbital, the Thus, an electron has also spin angular
value of m has been conventionally taken to be equal to
momentum commonly called spin. The magnitude of
spin angular momentum of an electron is given as :
zero, i.e., m = 0. But a px or a py-orbital does not designate
only one value of m but a linear combination of orbitals h
Spin angular momentum = s ( s + 1)
with m = +1 and m = –1. Therefore, it is not correct to 2π
designate single value of m for a px orbital as +1 or –1. For example, for spin quantum number, s = 1
2
.
Thus, the four quantum numbers describe the
ed
position of an electron in an atom by specifying its main
shell (n), subshell (l), the orientation of the
orbital (ml) and direction of its spin (ms). In other
rv
words, these quantum numbers serve as an address for
Fig. 30. Two possible orientations of the spin of an an electron.
se
electron ms = +½ and ms = –½
The various permitted values of the quantum
To sum up the four quantum numbers provide numbers are summarized below.
re
the following informations about orbitals : Name Symbol Information Permitted
1. n describes the shell, determines the size of provided values
the orbital and also to the large extent the energy of Principal n shell 1, 2, 3, 4...
ts
the orbital. Azimuthal l subshell 0, 1, 2, 3 ... (n – 1)
2. l determines the subshell and shape of the
Magnetic ml orbital – l ... 0 ... + l
orbital. There are n subshells in nth shell. To some
gh
extent, l also determines the energy of the orbital in Spin ms spin + ½, – ½
a multi-electron atom.
The number of subshells and orbitals in first three
3. ml designates the orientation of the orbital.
ri
For a given value of l, ml can have (2l + 1) values or shells (K, L and M) are shown in Fig. 31.
ll
.A
rs
he
lis
ub
P
Fig. 31. Number of subshells and orbitals in first three shells (K, L and M).
n
PAULI’S EXCLUSION PRINCIPLE electrons provided their spin quantum numbers are
After setting the rules for the possible values of different. For example, consider K shell where n = 1.
er
quantum numbers, we can calculate the maximum It will have one value of l (as l = 0) and one value of m
number of electrons that can be added in each shell (as m = 0) but it can have either of the two values of s
od
From the above discussion, it can be concluded (b) An atomic orbital has n = 3, what are the
that possible values of l ?
s-subshell (containing only one orbital) can (c) An atomic orbital has l = 3, what are the possible
have a maximum of 2 electrons values of ml ?
p-subshell (containing three orbitals) can have Solution : (a) If l = 2, the permitted values of m are :
maximum of 6 electrons
.
ml = –2, –1, 0, + 1, + 2
ed
d-subshell (containing five orbitals) can have (b) For n = 3, l may have the value l = 0, 1, 2
maximum of 10 electrons (c) For l = 3, m l may have the values
f-subshell (containing seven orbitals) can have ml = –3, –2, –1, 0, + 1, + 2, + 3.
rv
maximum of 14 electrons
Example 48.
Thus, it may be concluded that List all the values of l and m for n = 2.
se
No. of subshells in nth shell = n Solution : When n = 2, l can have values 0 and 1
No. of orbitals in a subshell = (2l + 1) For l = 0, ml = 0
For l = 1, ml = – 1, 0, + 1.
re
No. of electrons in a subshell = 2(2l + 1) = 4l + 2
Example 49.
It may be noted that the number of electrons in a Using the s, p, d notations, describe the orbital
subshell do not depend upon the value of principal with the following quantum numbers:
ts
quantum number. From this we can calculate the (a) n = 1, l = 0 (b) n = 3, l = 2
number of electrons in various shells as given below : (c) n = 3, l = 1 (d) n = 2, l = 1
gh
(e) n = 4, l = 3 (f) n = 4, l = 2
(n = 1) (l = 0) (m = 0)
K-shell 1s-subshell one orbital 2 electrons
ri
(n = 2) (l = 0) (m = 0) 2 electrons Solution :
L-shell 2s-subshell one orbital (a) n = 1, l = 0 : 1s-orbital
(l = 1) (m = – 1, 0, +1) 6 electrons ll (b) n = 3, l = 2 : 3d-orbital
2p-subshell three orbitals (c) n = 3, l = 1 : 3p-orbital
.A
4 orbitals 8 electrons (d) n = 2, l = 1 : 2p-orbital
(n = 3) (l = 0) (m = 0) 2 electrons (e) n = 4, l = 3 : 4f-orbital
M-shell 3s-subshell one orbital (f ) n = 4, l = 2 : 4d-orbital
rs
16 orbitals 32 electrons
Explain giving reasons, which of the following sets
er
Therefore, it may be concluded that the maximum of quantum numbers are not possible
number of orbitals in each shell is n2 and maximum (a) n = 0, l = 0, ml = 0, ms = + 1/2
number of electrons is 2n2 as shown below :
od
(b) n = 1, l = 0, ml = 0, ms = – 1/2
Shell symbol K L M N (c) n = 1, l = 1, ml= –0, ms = + 1/2
Shell number (n) 1 2 3 4 (d) n = 2, l = 1, ml = 0, ms = – 1/2
M
(d) Possible However, for 2s-orbital [Fig. 32 (b)] , the wave function
(e) Not possible because for n = 3, l cannot have 3 decreases in the beginning with increase in r, becomes
value. zero at a particular distance and then becomes negative.
(f) Possible The wave function for 2p-orbital [Fig. 32(c)] rises to
maximum and then decreases. It is very interesting to
note that the 2s-wave function can be positive or negative
.
depending upon the distance.
ed
50. Give all possible values of l, ml and ms for electrons
when n = 3. NOTE
51. How many electrons in a given atom can have the It may be noted that the positive and negative
rv
following quantum numbers ? signs refer to the sign of the wave function and have no
(a) n = 3, l = 1 (b) n = 3, l = 2, ml = 0 (c) n = 3, l = 2, ml connection to the positive and negative charges.
1
se
= + 2, ms = + (d) n = 3. As we here seen for 2s orbtial, at a point, the
2
52. If the quantum number ‘n’ has a value of 3, what are wave function also becomes zero. This point at which
the permitted values of the quantum number ‘l ’ ? radial wave function becomes zero is called radial
re
53. (i) An atomic orbital has n = 3, What are the possible nodal surface or simply node. At the node, the value
values of l and ml? of wave function changes from positive to negative. In
(ii) List the quantum numbers (ml and l) of electrons general, it has been found that ns orbitals have (n – 1)
nodes. Similarly, np orbitals have (n – 2) nodes and so
ts
for 3d-orbital.
(iii) Which of the following orbital are possible ? on. For example, the radial wave function for 2p-orbital
1p, 2s, 2p and 3f. has no node as shown in Fig. 32.
gh
In all cases, Ψ approaches zero as r approaches
infinity.
It may be noted that like Ψ, the plots of Ψ also do
ri
not have physical meaning. However, the importance
50. l = 0, m l = 0; l = 1, m l = –1, 0, + 1; of these plots lies in the fact that they give information
1 about how the radial wave function changes with
ll
l = 2, ml = –2, –1, 0, +1, +2 and ms = + and
2 distance r and about the presence of nodes where the
.A
1 sign of Ψ changes. These have particular importance
– for each value of ml.
2 in understanding the chemical bonding.
51. (a) 6 (b) 2 (c) 1 (d) 18
rs
52. 0, 1, 2.
53. (i) l = 0, ml = 0 ; l = 1, m = – 1, 0, + 1 ; l = 2,
ml = – 2 – 1, 0, + 1, + 2 (ii) l = 2, ml = –2, –1, 0 + 1, + 2,
he
(iii) 2s, 2p
lis
The shapes of orbitals are obtained from the According to the German physicst, Max Born, the
variation of wave function Ψ as a function of r (distance square of the wave function, Ψ2 at a point gives the
from the nucleus). This is also called radial dependence probability density of finding the electron at that point.
©
or radial wave function. This can be shown in a These variations of Ψ2 as a function of r are
simple method by plotting a graph between wave obtained by plotting ψ2 against r.
function (ψ) and distance (r) from the nucleus. The plots for 1s-, 2s- and 2p orbitals are shown in
These graphs are shown for 1s, 2s and 2p orbitals Fig. 33 ahead. These diagrams look similar to Fig. 32
in Fig. 32. It is clear from Fig. 32 (a) that wave function except that these become positive throughout (square
for 1s-orbital continuously decreases with increase in r. of negative quantity is always positive). These graphs
are called probability density graphs.
.
ed
rv
Fig. 33. Variation of ψ2 with distance from the nucleus for (a) 1s-orbital (b) 2s-orbital and (c) 2p-orbital.
se
It is clear from Fig. 32 that for s-orbitals (1s and 2s), the maximum electron density is at the nucleus
and for p-orbitals (2p), it has zero electron density at the nucleus. It may be noted that all orbitals
except s-orbitals have zero electron density at the nucleus.
re
Radial Probability Functions (4πr2Ψ2)
The Ψ2 versus r plots give the probability density for the electron around the nucleus. However, in order to
determine the total probability in an infinitesimally small region, we have to multiply probability density (Ψ2) by
ts
the volume of region, i.e.
Probability = Ψ2 × dV
gh
where dV is the volume of the region. Since the atoms have spherical symmetry, it is more useful to discuss the
probability of finding the electron in a spherical shell between the spheres of radius (r + dr) and r. To understand
this, consider the space around the nucleus to be divided into infinite number of concentric shells. The volume
ri
of such a shell of extremely small thickness, dr, is 4πr2 dr* so that
Probability = Ψ2 × 4πr2 dr = 4πr2 dr Ψ2
ll
This gives the total probability of finding the electron at a particular distance (r). This is called radial probability.
.A
The plots of probability (4πr2 Ψ2) as a function of distance from the nucleus (r) are called radial probability
distribution function (r.d.f.) graphs.
rs
It is clear that the radial probability distribution graphs depend not only upon the probability density but
also on the volume of the shell. The probability density (Ψ2) for 1s is maximum near the nucleus and it goes on
he
decreasing with distance (Fig. 33). However, the volume of the shell goes on increasing with increase in distance
as shown in Fig. 34 (a). The product of probability density and volume of shell gives the radial probability (4πr2Ψ2)
and is plotted against the distance from the nucleus [Fig. 34 (b)].
lis
ub
P
n
er
od
4
* The volume of the shell is 4πr2dr and it should not be confused with the volume of sphere which is πr3. It can be
3
calculated as :
⎡ Volume of sphere ⎤ ⎡ Volume of sphere ⎤ 4 4
©
Volume of shell = ⎢ with radius (r + dr)⎥ − ⎢ with radius r ⎥ = π(r + dr)3 – πr3
⎣ ⎦ ⎣ ⎦ 3 3
4 4 3 4 4
= π[r + 3r dr + 3rdr + dr ] – πr = π[r + 3r2dr] – πr3
3 2 2 3 3
3 3 3 3
(As dr represents an extremely small thickness, the higher powers of dr such as dr2 and dr3 may
be neglected)
4
Volume of shell = π × 3r2dr = 4πr2dr
3
The graph for 1s-orbital shows that the probability of finding the electron is zero at nucleus, it keeps
on increasing and becomes maximum at a particular distance from the nucleus and then gradually decreases. The
peak of the curve gives the distance from the nucleus where the probability of finding the electron is maximum.
This is called the radius of maximum probability. For hydrogen atom, this distance has been found to 52.9 pm
(or 0.529 Å). This value agrees well with the value calculated by Bohr for the radius of first orbit.
.
ed
It is important to note that Bohr predicted that the electron will always be at r = 52.9 pm for H atom.
However, according to wave mechanics, the electron is most likely to be found at this distance but there is
probability of finding the electron at distances shorter and larger than 52.9 pm.
rv
The radial probability distribution function for 2s orbital is shown in Fig. 35(b). The curve shows that there
are two regions of high probability (maxima). In between the regions of maximum probabilities, there is a
se
region where the probability of finding the electron is zero. It is called node. The radial probability function for
2p orbital is also shown in Fig. 35(c).
re
The radial probability functions versus distance r from the nucleus for 1s, 2s and 2p orbitals are shown in
Fig. 35.
ts
1s-orbital
gh
ri
ll
.A
Fig. 35. Radial probability functions as a function of distance, r from the nucleus
rs
NOTE
It may be noted that the distance of maximum probability for a 2p orbital is slightly less than that for a 2s
lis
electron. However, in contrast to a 2p-curve, there is a small additional maxima in the 2s curve. This indicates
that the electron in 2s orbital spends some of its time near the nucleus. In other words, the 2s electron
penetrates a little closer to the nucleus than the 2p-electron. As a result, 2s electron is attracted more
ub
strongly by the nucleus than a 2p-electron. That is why, 2s electron is more stable and, hence, has lower
energy than a 2p-electron.
P
a boundary surface is drawn for an orbital representation which encloses maximum probability (about 90%) of
finding the electron. In other words, boundary surface diagrams give the most probable regions.
er
s-orbitals are non-directional and spherically symmetrical. This means that the probability of finding the
electron is same in all directions at a particular distance from the nucleus. The 1s-orbital is shown in
M
Fig. 36 ahead. It is observed that density of charge cloud is maximum at the nucleus and decreases with increase
in distance from the nucleus.
The 2s-orbital is also non-directional and spherically symmetrical. In this case, the probability density is
©
maximum at the nucleus and becomes small at large distances. However, the effective volume or size of 2s is
larger than 1s-orbital. An important feature of 2s-orbital is that there is a spherical shell within 2s-orbital
(region without dots) where the probability of finding the electron is practically zero. This is called a node or a
nodal surface. Thus, a 2s-orbital differs from 1s-orbital in being larger in size and having a nodal surface.
For the sake of simplicity, the boundary surface diagram for 1s orbital is shown in Fig 36 (c) instead of
giving its charge cloud.
.
ed
rv
se
re
Fig. 36. Shapes of 1s and 2s orbitals. Fig. 37. Shape of 3s orbital. It has two nodes.
ts
The higher s-orbitals have also spherical shapes and number of nodal surfaces in s-orbital for any given
energy level is n – 1, where n represents energy level. For example, the shape of 3s orbital is shown in Fig. 37.
gh
It has two nodes.
It is also observed that the size of s-orbital increases with increase in value of n i.e., 4s > 3s > 2s > 1s and
the electron is located further away from the nucleus as the principal quantum number increases.
ri
2. Shapes of p-orbitals
For p-orbitals (l = 1), there are three possible orientations corresponding to ml = – 1, 0, + 1 values. This
ll
means that there are three p-orbitals in each p-subshell. These are designated as px, py and pz; for example, 2px,
.A
2py and 2pz. These three orbitals are equal in energy but differ in their orientations. Each orbital consists of two
lobes symmetrical about a particular axis. Depending upon the orientation of the lobes, these are designated as
2px, 2py and 2pz according as they are symmetrical about x, y and z-axes respectively. That is, 2px orbital has two
rs
lobes symmetrical around x-axis and 2py orbital has two lobes symmetrical around y-axis while the lobes of 2pz
orbital are symmetrical around z-axis (Fig. 38). The shape of the orbital is called dumb bell shape. For the sake
he
of simplicity a 2p orbital (e.g., 2px) is shown by simple boundary surface as given in Fig. 38 (d).
lis
ub
P
n
er
od
It may be noted that the two lobes of a p-orbital are separated by a plane having zero electron density. This
is called nodal plane. The nodal planes for 2px, 2py and 2pz orbitals are shown in Fig. 39 ahead.
©
It should be noted that the probability of finding the electron in a particular p-orbital is equal in both lobes.
The p-orbitals of higher energy levels (n = 3, 4, 5....etc.) have similar shapes although their sizes are bigger. The
size of p-orbital increases with increase in value of n as 4p > 3p > 2p.
Like s-orbitals, p-orbital also passes through zero probability region and number of nodes in p-orbitals are
given by n – 2 i.e, for 3p-orbital number of radial nodes is one, two for 4p orbital and so on.
.
ed
rv
se
Fig. 39. Nodal planes in 2p orbitals.
3. Shapes of d-orbitals
For d-orbital (l = 2), there are five possible orientations corresponding to ml = – 2, – 1, 0, + 1, + 2. This
re
means that there are five orbitals in each d-subshell. For 3d-subshell, these are designated as 3dxy, 3dyz, 3dzx,
3dx − y and 3d 2. These five orbitals are equal in energy but differ in their orientations. The shapes of these
2 2
z
ts
orbitals are described below :
(i) The three orbitals dxy, dyz and dzx are similar and each consists of four lobes of high electron density
lying in xy, yz and zx planes respectively. These lobes lie in between the principal axes. For example, in
gh
case of dxy orbital, the four lobes lie in xy plane in between the x and y-axes.
(ii) The d x2 − y2 orbital has also four of high electron density along the principal axes x and y. It may be noted that this
orbital is exactly like dxy orbital except that it is rotated through 45° around the z-axis.
ri
(iii) The dz2 orbital consists of two lobes along the z-axis with a ring of high electron density in the xy plane.
The shapes of 3d-orbitals are shown in Fig. 40. ll
.A
rs
he
Nodes and Nodal Planes passing through the nucleus. For example, 3dxy
A spherial surface within an orbital on which orbital has two nodal planes, passing through the
the probability of finding the electron is zero is called origin and bisecting the xy plane containing z-axis.
ub
nodes nodes
planes passing through the nucleus. A plane
passing through the nucleus on which the 1s 0 3d 0
2s 1 4d 1
©
.
d 2 planes for each orbital* is given in Fig. 41. It is clear from the figure that the
ed
*It may be noted that dz2 orbital has one conical surface
which is counted as two angular nodes.
rv
NOTE
se
It may be noted that the shapes of orbitals
represent plots of probability density (ψ2) and are
therefore, always positive. But sometimes sketches
re
of wave function (ψ) are also plotted. These have +ve
and –ve lobes. The signs in the lobes represent
Fig. 41. Energies of different orbitals in a hydrogen atom
the sign of the wave function in different
ts
directions. The wave functions may have +ve or –ve energies of various orbitals in a hydrogen atom depend
signs. These graphs of wave functions are very only upon the value of principal quantum number (n)
important in understanding the bonding between and independent of the value of l. In other words, all
gh
atoms (discussed in unit 4). These +ve and –ve signs the subshells in a given principal shell i.e., s, p, d and
have nothing to do with positive or negative charge. f have same energies. For example, 2s- and 2p-orbitals
have same energies. Similarly, all the orbitals of third
ri
It may be remembered that
shell (n = 3) i.e., 3s, 3p and 3d have same energies and
s-orbitals are always positive.
all the orbitals of fourth shell (n = 4), i.e., 4s, 4p, 4d
The opposite lobes of p-orbitals have opposite ll
and 4f have same energies.
signs. 1s < 2s = 2p < 3s = 3p = 3d < 4s = 4p = 4d = 4f
.A
The opposite lobes of d-orbitals have same sign (two The orbitals having the same energy are called
opposite lobes have +ve sign while the other two degenerate orbitals. The electron in 1s orbital in
opposite lobes have –ve sign). These are shown below: hydrogen atom corresponds to most stable state and is
rs
4d and 4f-subshell have different energies as shown in energy shell. For example, the energy of
Fig. 42. The main reason for having different energies 4s-orbital is less than that of a 3d-orbital.
of the subshells is the mutual repulsion among Similarly, 5s-orbital is lower in energy than
electrons in a multielectron atom. In case of hydrogen 4d-orbital.
atom, the only interaction present is the attraction (iv) The energy of various orbitals increases in
between the negatively charged electron and the the order :
.
positively charged nucleus. However, in multielectron 1s, 2s, 2p, 3s, 3p, 4s, 3d, 4p, 5s, 4d, 5p, 6s,
ed
atoms, besides the presence of attraction between the 4f, 5d, 6p, 7s.
electron and the nucleus, there are repulsion terms
between every electron and other electrons present ELECTRONIC CONFIGURATION OF ATOMS
rv
in the atom. The filling of orbitals in an atom is a hypothetical
Cause of different energy patterns of process in which the atom is built up by feeding
se
hydrogen atom and multielectron atoms : The electrons in orbitals, one at a time and by placing
energy on an electron in an orbital depends upon the each new electron in the lowest available energy
following factors :
re
orbital. The distribution of electrons in different
(i) nuclear charge
orbitals is known as electronic configuration of
(ii) principle energy level,
the atom. This characterises each electron in an
(iii) the presence of electrons in the lower energy
ts
levels. atom. For the sake of presentation, the following
In a multielectron atom, the electrons occupying symbols are commonly used :
gh
the inner energy levels tend to reduce the effect of
nuclear charge on the electrons present in the
outermost energy level. In other words, the electrons
ri
in the inner shells screen or shield the outermost
electrons form the nucleus. This effect is known as
screening or shielding effect. This is different in ll
different types of orbitals. The net positive charge
experienced by the electron from the nuclear is called A box for an orbital (square or circular); an arrow
.A
effective nuclear charge (Zeff). for an electron, the direction of the arrow giving the
For example, being spherical in shape, s-orbital orientation of its spin. Two arrows are shown for two
spends more time closer to the nucleus in comparison electrons with opposite orientations of spin.
rs
to p-orbital and p-orbital spends more time near the Alternatively, electronic configuration is
nucleus in comparison to d-orbital. In other words,
expressed by indicating the principal quantum number
he
(Zeff) experienced by the orbital decreases with increase indicates that there is one electron in px orbital of
of azimuthal quantum number (l). This means the third principal shell.
s-orbitals will be more tightly bound to the nucleus
ub
energies of subshells for n = 3 are in the 1. Aufbau principle. The Aufbau principle
order:
states that
3s (n = 3, l = 0) < 3p (n = 3, l = 1) < 3d
©
The word aufbau in German means building up. as (n + l) rule or Bohr Bury’s rule. According to
The building up of the orbitals means the filling up of this :
orbitals with electrons. From an energy level diagram (i) Orbitals fill in the order of increasing
(Fig. 42) for multi-electron atoms, the following value of n + l. For example, 3s-orbital
sequence is observed for orbitals in the increasing (n + l = 3 + 0 = 3) will be filled before 4s
order of energy:
.
(n + l = 4 + 0 = 4) orbital. Similarly, out of
ed
1s, 2s, 2p, 3s, 3p, 4s, 3d, 4p, 5s, 4d, 5p, 6s, 4f, 5d, 3d and 4s, the 4s (n + l = 4 + 0 = 4) orbital will
6p, 7s be filled before 3d (n + l = 3 + 2 = 5) orbital.
According to Aufbau principle, the orbital should (ii) If the two orbitals have same value of (n + l),
rv
be filled in the above sequence. then the orbital with lower value of n will be
It is very important to remember that the filled first. For example, 2p-orbital (n + l = 2
se
sequence of energy levels pertains up to 3p and + 1 = 3) and 3s-orbital (n + l = 3 + 0 = 3) have
then 4s-orbital comes first instead of 3d. In fact, the the same (n + l) value but 2p-orbital has lower
value of n and therefore, it will be filled first.
re
energy of an orbital is determined by the quantum
numbers n and l with the help of important rule known This is shown below in Table 5.
ts
Orbital Value Value of Value of
gh
of n l (n + l)
1s 1 0 1+0=1
ri
2s 2 0 2+0=2
2p 2 1 2+1=3
UV 2p (n = 2) has lower energy
3s 3 0 3+0=3
ll W than 3s (n = 3)
UV
.A
3p (n = 3) has lower energy
3p 3 1 3+1=4
4s 4 0 4+0=4 W than 4s (n = 4)
3d 3 2 3+2=5
UV 3d (n = 3) has lower energy
W
rs
4p 4 1 4+1=5 than 4p (n = 4)
he
2. Pauli’s exclusion principle. According to this Fig. 43. Schematic diagram to remember sequence of
principle, an orbital can accommodate maximum filling atomic orbitals.
M
and f orbitals, which have more than one kind of orbitals. For example, we know that there are three p-orbitals
(px, py and pz) of the p-subshell in a principal energy level. According to Hund’s rule, each of the three p-orbitals
must get one electron of parallel spin before any one of them receives the second electron of opposite spin.
Electronic Configurations of Atoms
Based upon the above rules and the sequence of energy levels, let us write the electronic configurations of
.
ed
some atoms.
Hydrogen (At. No. = 1). Since hydrogen has only one electron, it must go to 1s-orbital which has lowest energy.
1s
rv
H : (Z = 1) 1s1 or ↑
se
Helium (At. No. = 2). In helium atom, the second electron can also go into 1s-orbital. The two electrons
must have opposite spins (Pauli’s exclusion principle).
re
1s
He : (Z = 2) 1s2 or ↑↓
Lithium (At. No. = 3). Since 1s-orbital is filled with two electrons, it cannot have any more. Therefore,
ts
the third electron goes to the next lowest energy orbital, namely 2s-orbital.
1s 2s
gh
Li : (Z = 3) 1s22s1 or ↑↓ ↑
Beryllium (At. No. = 4). The fourth electron in beryllium fills 2s-orbital.
ri
1s 2s
Be : (Z = 4) 1s2 2s2 or ↑↓ ↑↓
ll
Boron (At. No. = 5). In this case, the fifth electron goes into any one of the 2p orbitals (say 2px) :
.A
1s 2s 2px2py2pz
B : (Z = 5) 1s22s22px1 or ↑↓ ↑↓ ↑
rs
It may be remembered here that the fifth electron can enter any of the three 2px, 2py or 2pz orbitals because all
he
are of the same energy. It is only a matter of convention that we fill the orbitals in the order of 2px, 2py and 2pz.
Carbon (At. No. = 6). In carbon atom, the sixth electron is also to go into the 2p-orbitals because it can
accommodate six electrons. Here Hund’s rule applies, i.e., the electrons enter the orbitals of same energy with
lis
parallel spin until all are singly filled. Therefore, the sixth electron cannot enter the 2px orbital, rather it can go
into either 2py or 2pz in accordance with Hund’s rule :
ub
Nitrogen (At. No. = 7). Applying Hund’s rule, nitrogen atom has three unpaired electrons in 2p-orbitals
as :
n
Oxygen (Z = 8), fluorine (Z = 9) and neon (Z = 10). Beginning with oxygen, the
2p-orbitals start getting filled by second electron till each of these is completely filled.
1s 2s 2px 2py 2pz
M
Sodium (Z = 11) to Argon (Z = 18). The electronic configurations of these atoms are written exactly in
the same manner as discussed above. Here, 3s- and 3p-orbitals are filled as did 2s-and 2p-orbitals in Li to Ne.
Furthermore, for simplicity a common convention is used. In this, the detailed electronic configuration of
the noble gas core preceding the valence shell is represented by the symbol of the noble gas in square brackets.
Then, the configuration of the valence shell is written after the symbol.
.
For example, the electronic configuration of sodium may be written as Na (Z = 11) : [Ne]3s1.
ed
The electronic configurations of these elements are given below :
Sodium, Na (Z = 11) [Ne] 3s1
Magnesium, Mg (Z = 12) [Ne] 3s2
rv
Aluminium, Al (Z = 13) [Ne] 3s2 3px1
Silicon, Si (Z = 14) [Ne] 3s2 3px1 3py1
se
Phosphorus, P (Z = 15) [Ne] 3s2 3px1 3py1 3pz1
Sulphur, S (Z = 16) [Ne] 3s2 3px2 3py1 3pz1
re
Chlorine, Cl (Z = 17) [Ne] 3s2 3px2 3py2 3pz1
Argon, Ar (Z = 18) [Ne] 3s2 3px2 3py2 3pz2
Potassium (Z = 19) and calcium (Z = 20). In these atoms, electrons enter the 4s-orbital in accordance
ts
with the fact that 4s-orbital is slightly lower in energy than 3d-orbital.
4s 3d
gh
Potassium, K (Z = 19) : [Ar] 4s 1 ↑
4s 3d
ri
Calcium, Ca (Z = 20) : [Ar] 4s2 ↑↓
ll
Scandium (Z = 21) to zinc (Z = 30). With scandium (Z = 21), 3d-subshell comes into use and from
.A
scandium to zinc, this subshell is gradually filled up. The electronic configurations of the elements from scandium
(Z = 21) to zinc (Z = 30) are given below :
3d 4s
rs
From the above configurations, it may be noted that there are two irregularities in the general trend. The
electronic configurations of Cr and Cu are different than what we expected. The electronic configurations for
these atoms are expected to be
Cr : [Ar] 3d4 4s2 and Cu : [Ar] 3d9 4s2
However the actual configurations are
Cr : [Ar] 3d5 4s1 and Cu : [Ar] 3d10 4s1
This is attributed to the fact that half-filled (containing one electron per orbital) and completely filled
(containing two electrons per orbital) electronic configurations have lower energy and, therefore, extra stability.
Consequently, the configurations, d5, d10, f 7 and f 14 which are either half-filled or completely-filled, are more
stable. Thus, to acquire increased stability, one of the 4s-electrons goes into the nearby 3d-orbitals so that 3d-
orbitals get half-filled in Cr and completely-filled in Cu.
.
ed
The extra stability of half-filled and completely-filled electronic configurations is due to their (i) symmetrical
arrangement and (ii) large exchange energy as explained below :
(i) Stability on the basis of symmetrical distribution of electrons. The symmetry leads to stability.
rv
Therefore, the electronic configurations in which all the orbitals of the same subshell are either completely-
filled or exactly half-filled are more stable because of symmetrical distribution of electrons. For example, the
se
expected configuration of Cr (Z = 24) is :
3d4 4s2
re
↑ ↑ ↑ ↑ ↑↓
But if one electron is shifted from 4s- to 3d-subshell, the distribution of electrons become more symmetrical
ts
and more stable.
3d5 4s1
gh
↑ ↑ ↑ ↑ ↑ ↑
Similarly, in case of copper, when an electron is shifted from 4s- to 3d-subshell, the 3d-subshell becomes
ri
more stable due to symmetrical arrangement.
3d10 4s1
↑↓ ↑↓ ↑↓ ↑↓ ↑↓
ll ↑
.A
(ii) Stability due to exchange energy. The half-filled and completely-filled electronic configurations are
also stable due to large exchange of energy of stabilization.
The exchange means the shifting of electrons from one orbital to another in the same subshell. This can be
rs
3d 4s
Cr (Z = 24) ↑ ↑ ↑ ↑ ↑↓
lis
1 2 3 4
(i) The first electron (number 1) can exchange its positions with electrons numbered 2, 3 and 4 i.e., in 3 ways.
ub
(ii) The second electron (number 2) can exchange its positions in two ways with electrons 3 and 4 only
because the exchange of position between 1 and 2 has already been considered in step (i).
(iii) The electron (number 3) can exchange only in one way i.e. with electron 4.
P
n
↑ ↑ ↑ ↑ ↑ ↑ ↑ ↑ ↑ ↑ ↑ ↑
1 2 3
er
Thus, in 3d4 arrangement, electrons can exchange in 3 + 2 + 1 = 6 ways or there are six possible arrangements
with parallel spins in 3d4 configuration.
od
On the other hand, in 3d5 configuration, the electron can exchange in 10 ways as shown below :
M
↑ ↑ ↑ ↑ ↑ ↑ ↑ ↑ ↑ ↑
©
1 2
↑ ↑ ↑ ↑ ↑ ↑ ↑ ↑ ↑ ↑
3 4
It is clear that the electron 1 can exchange in four ways, the electron 2 in three ways, electron 3 in two
ways and electron 4 in one way. Thus, total number of ways of exchanging electron in 3d5 configuration are
4 + 3 + 2 + 1 = 10. Since the number of exchanges in 3d5 4s1 configuration is more than that in 3d4 4s2
configuration, therefore, the greater exchange is responsible for the extra stability of this configuration.
Similarly, it can be seen that the exchange energy for d10 electronic configuration is more than for d9 configuration.
Thus, the greater exchange for half-filled and completely-filled configurations gives them extra stability.
.
ed
Gallium (Z = 31) to Krypton (Z = 36). With gallium onwards, 4p-orbitals get filled up as :
Gallium Ga (Z = 31) : [Ar] 3d10 4s2 4px1
rv
Germanium Ge (Z = 32) : [Ar] 3d10 4s2 4px1 4py1
Arsenic As (Z = 33) : [Ar] 3d10 4s2 4px1 4py1 4pz1
se
Selenium Se (Z = 34) : [Ar] 3d10 4s2 4px2 4py1 4pz1
Bromine Br (Z = 35) : [Ar] 3d10 4s2 4px2 4py2 4pz1
re
Krypton Kr (Z = 36) : [Ar] 3d10 4s2 4p6
Note. To write the electronic configuration of an atom, it is better if we remember the atomic number of noble gases
ts
and the orbitals which follow the noble gas. Then the next orbitals to be filled can be seen from the general sequence. The
noble gases with their atomic numbers and orbitals which follow these noble gas cores are given below :
gh
[He]2 2s; [Ne]10 3s; [Ar]18 4s; [Kr]36 5s; [Xe]54 6s; [Rn]86 7s.
For example, let us try to write the electronic configuration of I(Z = 53). The noble gas having atomic
number less than 53 is [Kr]36 and orbital which follows this is 5s. After 5s the orbitals with increasing
ri
energies are 4d, 5p ..... so that
I (Z = 53) : [Kr] 4d10 5s2 5p5
ll
In the next eighteen elements from rubidium (Rb) to Xenon (Xe), the orbitals which are being gradually
.A
filled are 5s, 4d and 4p similar to that of 4s, 3d and 4p orbitals as discussed above. Then 6s orbitals are filled for
Cs and Ba with one and two electrons respectively. Then from lanthanum (La) to mercury (Hg), the filling up of
electrons takes place in 4f and 5d orbitals. After this, the filling of 6p, then 7d and finally 5f and 6d orbitals takes
rs
place. However, it may be noted that there are many anomalous configurations due to stability of half filled and
completely filled orbitals.
he
The electronic configurations of all elements are given in Table 6. It may be noted that it is very important to
know the electronic configuration of atoms. The chemical behaviour of an atom depends on the distribution of
electrons in various shells, subshells or orbitals. The tendency of atoms to take part in chemical combination also
lis
depends on the electrons in the outermost shell (called valence shell). Therefore, a detailed understanding of the
electronic structure of atoms is very essential for getting an insight into the various aspects of modern chemical
ub
knowledge.
Table 6. Electronic configuration of different elements.
P
.
ed
34 Se – 3d10 4s2 4p4 75 Re – 4f 14 5d5 6s2
35 Br – 3d10 4s2 4p5 76 Os – 4f 14 5d6 6s2
36 Kr – 3d10 4s2 4p6 77 Ir – 4f 14 5d7 6s2
37 Rb – [Kr]36 5s1 78 Pt* – 4f 14 5d9 6s1
rv
38 Sr – 5s2 79 Au* – 4f 14 5d10 6s1
39 Y – 4d1 5s2 80 Hg – 4f 14 5d10 6s2
se
40 Zr – 4d2 5s2 81 Tl – 4f 14 5d10 6s2 6p1
41 Nb* – 4d4 5s1 82 Pb – 4f14 5d10 6s2 6p2
42 Mo* – 4d5 5s1 83 Bi – 4f14 5d10 6s2 6p3
re
43 Tc – 4d5 5s2 84 Po – 4f14 5d10 6s2 6p4
44 Ru* – 4d7 5s1 85 At – 4f14 5d10 6s2 6p5
45 Rh* – 4d8 5s1 86 Rn – 4f14 5d10 6s2 6p6
ts
46 Pd* – 4d10 87 Fr – [Rn]86 7s1
47 Ag* – 4d10 5s1 88 Ra – 7s2
48 Cd – 4d10 5s2 89 Ac – 6d1 7s2
gh
49 In – 4d10 5s2p1 90 Th – 6d2 7s2
50 Sn – 4d10 5s2p2 91 Pa – 5f26d17s2
51 Sb – 4d10 5s2p3 92 U – 5f36d17s2
ri
52 Te – 4d10 5s2p4 93 Np – 5f46d17s2
53 I – 4d10 5s2p5 94 Pu – 5f67s2
54
55
Xe
Cs
– 4d10 5s2p6
– [Xe]54 6s1
ll 95
96
Am
Cm
– 5f77s2
– 5f76d17s2
.A
56 Ba – 6s2 97 Bk – 5f97s2
57 La* – 5d1 6s2 98 Cf – 5f107s2
58 Ce* – 4f 2 6s2 99 Es – 5f117s2
rs
71 Lu – 4f 14 5d1 6s2
112 Cn – 5f146d107s2
n
written almost similar to those of the atoms. In case more than one subshell of the valence shell are
of negatively charged ions, the extra electrons equal occupied, the one with the highest value of l loses the
to the charge are added to the appropriate orbitals. electron first. For example,
M
Cl– (Z = 17) : 1s2 2s2 2p6 3s2 3p 2x 3p 2y 3p 2z Fe (Z = 26) : 1s2 2s2 2p6 3s2 3p6 3d6 4s2
S (Z = 16) : 1s2 2s2 2p6 3s2 3p 2x 3p y 3p1z
1
Fe2+ (Z = 26) : 1s2 2s2 2p6 3s2 3p6 3d6
S2– (Z = 16) : 1s2 2s2 2p6 3s2 3p 2x 3p 2y 3p 2z It is very important to remember here that in case
In case of positively charged ions, the electrons of Fe2+ ion, the electrons are removed from 4s-orbital
equal to the charge on the ion are removed from the first (highest value of n) and not from 3d-orbitals.
Ni (Z = 28) : 1s2 2s2 2p6 3s2 3p6 3d8 4s2 Example 55.
Ni2+ (Z = 28) : 1s2 2s2 2p6 3s2 3p6 3d8 What atoms are indicated by the following
Co (Z = 27) : 1s2 2s2 2p6 3s2 3p6 3d7 4s2 configuration ? Are they in the ground state or
excited state ?
Co3+ (Z = 27) : 1s2 2s2 2p6 3s2 3p6 3d6
(a) 1s2 2s2 2p2x 2p y 2p1z
2
.
REMEMBER
ed
While writing the electronic configuration of (b) 1s2 2s1 2p1x 2p y 2p1z
1
cations, the electronic configuration of the atom
should be written first. The atomic number Z, which
rv
(c) 1s2 2s2 2p6 3s2 3p1x 3p y
1
se
number of electrons equal to the units of positive
charge should be removed from the outermost shell. (e) [Ar] 3d5 4s2
re
2
Solution : (a) 1s2 2s2 2p2x 2p y 2p1z . Atomic number is
9 and it is configuration of fluorine in ground state.
1
(b) 1s2 2s1 2p1x 2p y 2p1z . Atomic number is 6 and it is
ts
Example 52.
configuration of carbon in excited state.
Which of the following are isoelectronic species :
1
gh
Na+, K+, Mg2+, Ca2+, S2–, Ar ? (c) 1s2 2s2 2p6 3s2 3p1x 3p y Atomic number is 14 and it
Solution : Isoelectronic species are those which is configuration of silicon in ground state.
contain same number of electrons
ri
(d) 1s2 2s2 2p6 3s1 3p 1x 3p y 3p1z 3d1. Atomic number
1
Na+, Mg2+ are isoelectronic (contain 10 electrons)
K+, Ca 2+ S 2– and Ar are isolectronic (contain 18 is 15 and it is configuration of phosphorus in
electrons). ll excited state.
Example 53.
(e) [Ar] 3d 5 4s 2 . Atomic number is 25 and it is
.A
Write the electronic configuration of elements of configuration of manganese in ground state.
atomic numbers 10, 17, 25, 29 and 37.
Solution : Electronic configuration of elements : Example 56.
rs
Atomic No. 10 : 1s2 2s2 2p6 Give the symbol of the atom whose ground state
2
Atomic No. 17 : 1s2 2s2 2p6 3s2 3p 2x 3p y 3p1z corresponds to each of the following
he
configurations:
Atomic No. 25 : [Ar] 3d5 4s2
Atomic No. 29 : [Ar] 3d10 4s1 (i) 1s2 2s2 2p6
Atomic No. 37 : [Kr] 5s1 (ii) 1s2 2s2 2p6 3s2 3p6
lis
ground state of (i) P (Z = 15) (ii) Fe2+ (Z = 26) (iii) Solution : (i) 1s2 2s2 2p6. Atomic number is 10 and it
Cl– (Z = 17) ? corresponds to Ne (neon).
Solution : The number of unpaired electrons can be
The ions corresponding to this configuration are :
P
(ii) Fe2+ (Z = 26). The electronic configuration of Fe is The ions corresponding to this configuration are :
1s2 2s2 2p6 3s2 3p6 3d6 4s2. For Fe2+, two electrons
are removed from 4s-orbtial so that the S2–, Cl–, K+, Ca2+.
od
.
ed
1
(1) n = 4, l = 2, ml = –2, ms = − (a) Atomic number
2
(b) Total number of principal quantum numbers
1
(c) Total number of sublevels
rv
(2) n = 3, l = 2, ml = 1, ms = +
2
(d) Total number of s-orbitals
se
1
(3) n = 4, l = 1, ml = 0, ms = + (e) Total number of p-electrons.
2
Solution : The electronic configuration of the atom is :
re
1
(4) n = 3, l = 2, ml = –2, ms = − 1s2 2s2 2p6 3s2
2
(a) Atomic number = 2 + 2 + 6 + 2 = 12
1
(5) n = 3, l = 1, ml = –1, ms = + (b) Number of principal quantum numbers = 3
ts
2
(c) Number of sub-levels = 4 (1s, 2s, 2p, 3s)
1
(6) n = 4, l = 1, ml = 0, ms = +
gh
2 (d) Number of s-orbitals = 3 (1s, 2s, 3s)
(e) Total number of p-electrons = 6.
ri
Example 62.
Solution : 5(3p) < 2 (3d) = 4 (3d) < 6 (4p)
= 3 (4p) < 1 (4d) ll What is the maximum number of unpaired
Example 59. electrons in Cu (Z = 29), Br – (Z = 35) and
.A
Indicate the number of unpaired electrons in K+ (Z = 19) ?
(i) P (ii) Si (iii) Cr (iv) Fe (v) Kr Solution :
rs
unpaired electrons = 3 Br (Z = 35) : 1s2 2s2 2p6 3s2 3p6 3d10 4s2 4p5
(ii) Si (Z = 16) : [Ne] 3s 3p2
2
∴ Br– (36e) : 1s2 2s2 2p6 3s2 3p6 3d10 4s2 4p6
unpaired electrons = 2
lis
Unpaired electrons = 0
(iii) Cr (Z = 24) : [Ar] 3d5 4s1
K (Z = 19) : 1s2 2s2 2p6 3s2 3p6 4s1
unpaired electrons = 6
ub
unpaired electrons = 0.
Example 60.
n
Which of the following do and which do not make ground state of phosphorus (Z = 15) ?
sense ? 55. Name the elements which correspond to each of
7p, 2d, 3s3, 3py3, 4f.
od
.
60.
56. (a) n = 3, l = 0 (b) n = 4, l = 1
How many electrons in zinc (Z = 30) have n + l
ed
value equal to 4 ? (c) n = 4, l = 3 (d) n = 3, l = 2
61. Write the electronic configuration of chromium 57. n = 3, l = 0, m = 0, s = + ½
(Z = 24) and predict in it 58. Ten.
rv
(i) number of subshells 59. n = 4, l = 0, m = 0, s = + ½
(ii) number of electrons in subshell having l = 0 60. 8 (electrons in 4s and 3p-subshells)
se
(iii) number of electrons having n + l value equal to 3 61. (i) 7 (ii) 7 (iii) 8 (iv) 1
(iv) number of electrons in highest value of n. 62. 6
re
62. How many electrons in p-subshell of argon (Z = 18)
have same spin ? Hints & Solutions on page 68
ts
gh
2
ri
Q. 1. What will happen to the wavelength associated with a moving particle if its velocity is reduced to
half? ll h 1
Ans. Wavelength becomes double of the original value because λ = or λ ∝ .
.A
mv v
Q. 2. Can we apply Heisenberg’s uncertainty principle to a stationary state ?
Ans. No, because velocity is zero and position can be measured accurately.
rs
Q. 3. An electron beam after hitting a nickel crystal produces a diffraction pattern. What do you
conclude ?
he
h 1
Ans. λ= or λ ∝ .
mv m
ub
Q. 5. What is the sequence of energies of 4s, 4p, 4d and 4f-orbitals in (i) a hydrogen atom and (ii) a zinc
atom?
n
Q. 7. The 4f subshell of an atom contains 12 electrons. What is the maximum number of electrons
having same spin in it ?
Ans. Seven.
M
Q.8. Which of the following sets of quantum numbers for orbitals in hydrogen atom has larger
energy ?
©
n = 3, l = 2, ml = +1, n = 3, l = 2, ml = – 1
Ans. Both are orbitals of the same subshell and, therefore, have same energy.
Q.9. How many nodes are present in 3p-orbital ?
Ans. No. of nodes in an orbital = (n – l – 1)
∴ No. of nodes in 3p-orbital = 3 – 1 – 1 = 1.
Q.10. What is the difference in the angular momentum of an electron present in 2p and that present
in 3p-orbital ?
Ans. Angular momentum is same because it depends upon the quantum number l and not on n as :
h
Angular momentum = l(l + 1) .
2π
.
ed
Q.11. What will be the maximum number of electrons present in an atom having n + l = 4 ?
Ans. The subshells which can have n + l = 4 are 4 s (4 + 0) and 3p ( 3 + 1). Therefore, these will accommodate
maximum of 2 + 6 = 8 electrons.
rv
Q.12. How many spherical nodes do you expect in a 4s-orbital ?
Ans. Three nodes.
Q.13. What is the maximum number of electrons which can be accommodated in an atom in which
se
the highest principal quantum number value is 4 ?
Ans. 36 corresponding to Kr (4s2 4p6).
Q.14. How many quantum numbers are required to specify an orbital ?
re
Ans. Three (n, l and ml)
Q.15. What are the quantum numbers of the valence electrons in potassium atom (Z = 19) in ground
state ?
ts
Ans. For 3p-orbital,
1
n = 4, l = 0, ml = 0, ms = +
.
gh
2
Q.16. In which atom, the outermost electron can have the following set of quantum numbers ?
1
ri
n = 3, l = 0, ml = 0 ms = –
2
Ans. Sodium. ll
Q.17. What is the value of orbital angular momentum for an electron in 2s-orbital ?
.A
h
Ans. Angular momentum is given as l (l + 1)
2π
Since for 2s orbital, l = 0 ∴ Angular momentum = 0
rs
6
Q. 20. Explain the meaning of the symbol 4d .
Ans. It means that 4d subshell has 6 electrons. 4 represents fourth energy shell and d is a subshell and 6
electrons are present in d orbitals of subshell.
ub
Ans. Fourth
Q. 23. Which quantum number does not follow from the solutions of Schrodinger wave equation ?
n
5h
Ans. .
2π
od
.
Ans. (a) n (b) n (c) l (d) ml.
ed
Q. 30. What physical meaning is attributed to the square of the absolute value of wave function
2
Ψ |?
|Ψ
rv
Ans. It measures the electron probability density at a point in an atom.
Q. 31. Discuss the similarities and differences between a 1s and a 2s orbital.
Ans. Similarities. (i) Both the orbitals have spherical shape.
se
(ii) Both have same angular momentum (equal to zero).
Differences. (i) Size of 2s orbital is larger than that of 1s orbital
(ii) 1s has no node while 2s has one node.
re
(iii) Energy of 2s orbital is more than that of 1s.
Q. 32. What is the difference between the notation l and L ?
Ans. The notation l represents azimuthal quantum number, which can have values 0, 1, 2, ... etc., whereas L
ts
represents second Bohr orbit for which n = 2.
Q. 33. How many electrons in an atom may have the following quantum number ?
gh
(i) n = 4, ms = +½ (ii) n = 3 , l = 0
Ans. (i) n = 4, ms = +½ = 16 electrons
(ii) n = 3 , l = 0 = 2 electrons
ri
Q. 34. For each of the following pair of hydrogen orbitals, indicate which is higher in energy :
(i) 1s, 2s (ii) 2p, 3p
(iii) 3dxy, 3dyz (iv) 3s, 3d
ll
(v) 4f, 5s.
.A
Ans. (i) 2s (ii) 3p (iii) both have same energy (iii) both have same energy (v) 5s.
Q. 35. Which orbital in each of the following pairs is lower in energy in a many electron atom ?
rs
.
ciated with proton ? (Mass of electron = 9.1095 × 10–31 kg, h
But λ =
ed
mass of proton = 1.6725 × 10–27 kg). mv
Solution According to de-Broglie equation, 2πr 2πr . mv
No. of waves = =
h h / mv h
rv
λ = 2π (mvr)
mv or =
h h
For electron, λe =
se
The angular momentum of Bohr's 3rd orbit is
meve
3h
For proton, λp =
h mvr =
2π
re
m pvp
2π 3h
But λe = λp ∴ No. of waves = × =3
h h h 2π
∴ = No. of waves in Bohr's 3rd orbit = 3.
meve m pvp
ts
or m e v e = m pv p Problem 12. An electron in a H-atom in its ground
m pvp state absorbs 1.50 times as much energy as the
gh
or ve = minimum energy required for it to escape from the
me
atom. What is the wavelength of the emitted electron?
1.6725 × 10 –27
= × vp = 1836 vp
ri
9.1095 × 10−31 Solution The energy required to remove an electron
Velocity of electron = 1836 velocity of proton.
from H atom = Ionisation energy of H atom
Problem 10. Calculate the ratio between the ll = 2.18 × 10−18 J atom−1
wavelength of an electron and a proton if the proton
.A
is moving with half the velocity of electron. The amount of energy absorbed = 1.50 × 2.18 × 10−18 J
(mass of electron = 9.11 × 10–31 kg, mass of proton = 1.67 = 3.27 × 10−18 J
× 10–27 kg). Amount of energy converted to kinetic energy
rs
1 2
λe m pv p 1 v 1 K.E = m ν
= , vp = ve or p = 2
λp meve 2 ve 2
2 × (1.09 × 10 −18 J)
lis
2 K.E
λe 1 mp 1 1.67 × 10−27 ν= =
= = × = 916.6 . m (9.1 × 10 −31 kg)
λp 2 me 2 9.11 × 10−31
= 1.55 × 106 m s−1
ub
λ=
Solution In general, the number of waves in any mν
orbit is 6.63 × 10 −34 Js
n
=
Number of waves =
Circumference of orbit (9.1 × 10 −31 kg) × (1.55 × 106 ms −1 )
er
Wavelength
= 4.70 × 10−10m
od
Practice Problems
©
.
ed
No. of protons = x – 1 = 3.98 × 10–15 J
11.1 (ii) E = 6.63 × 10–34 Js × 3 × 1015 s–1
No. of neutrons = x + x × = 1.111 x
100 = 1.98 × 10–18 J
Now, x – 1 + 1.111 x = 37
rv
16. λ = 25.0 μm = 25.0 × 10–6 m
or 2.111x = 38
x = 18 c 3.0×108
ν = = = 1.2 × 1013
se
No. of e = 18 λ 25.0×10−6
No. of p = 18 – 1 = 17 Energy of photon,
No. of n = 35 – 17 = 18 E = hν = 6.626 × 10–34 × 1.2 × 1013
re
Symbol 35
17 Cl
– = 7.96 × 10–21 J
17. Energy of photon,
c
9. ν = hc
λ E =
ts
3.0×108 λ
= = 6.52 × 1014 sec–1 (6.63×10−34 Js) × (3×108 ms−1 )
460 ×10–9 ∴ E =
gh
400 ×10−9 m
10. Frequency, ν = 980 kHz = 980 × 103 s–1 = 4.97 × 10–19 J
c No. of photons in 1J
Wavelength λ =
ri
ν
3.0 × 108 m s −1 1
= = 2.01 × 1018 photons
∴ λ = = 306 m. 4.97 ×10−19
980 × 103s −1 ll
ν 18. Wavelength of light,
.A
11. Wave number, ν =
c λ = 250 nm = 250 × 10–9 m
4 ×1014 s−1
∴ ν = 6.63×10−34 × 3.0×108
3.0 × 108 m s−1
rs
E =
= 1.33 × 106 m–1. 250×10−9
12. Frequency, ν =
c = 7.956 × 10–19 J
he
λ
1 19. K.E. = hν – hν0 = h (ν – ν0)
=
480 ×10−9
h = 6.63 × 10–34 J s,
P
5800 × 10 −10 m
= 1.724 × 106 m–1 = 6.63 × 10–34 × 3.0 × 1014 = 1.99 × 10–19 J
er
= 1.2 × 1013 Hz ∴ ν = =
1 1 λ 6800 × 10−10 m
Wave number, ν = = = 4.41 × 1014 s–1
λ 25.0 × 10 −6 m
= 4.0 × 104 m Threshold frequency = 4.41 × 1014 s–1
hc Work function = hν0 = 6.626 × 10–34 × 4.41 × 1014
15. (i) E = hν =
λ
= 2.92 × 10–19 J
1 ⎛ 1 1 ⎞ 6.63×10−34 × 3.0×108
21. = 109678 ⎜⎜ 2 − 2 ⎟⎟ cm–1 Now, E =
9.11×10−8
.
λ ⎝ n1 n2 ⎠
ed
For Balmer series, = 2.183 × 10–18 J
n1 = 2, 24. For Balmer series, the longest wavelength
corresponds to transition from n = 3 to n = 2
rv
1 ⎛1 1 ⎞ −1
= 109678 ⎜ 2 − 2 ⎟ cm ⎛ 1 1 ⎞
λ ⎝2 n ⎠ − 2 ⎟ cm–1
ν = 109678 ⎜
se
2
For the first line, ⎝ 2 n ⎠
n2 = 3 so that ⎛ 1 1 ⎞
= 109678 ⎜ 2 − 2 ⎟
⎝2 3 ⎠
re
1 ⎛1 1⎞
= 109678 ⎜ − ⎟ cm–1 = 15233.0 cm–1
λ ⎝4 9⎠
or = 1.523 × 106 m–1
5 25. No. of lines produced when an electron in 6th shell
ts
= 109678 ×
36 drops to ground state
(n2 − n1 ) ( n2 − n1 + 1) (6 − 1) × ( 6 − 1 + 1)
gh
36
or λ = cm = 6.56 × 10–5 cm =
5× 109678 2 2
= 656 nm 5×6
= = 15 lines
ri
For the limiting line, 2
n2 = ∞ These lines correspond to
∴
1 ⎛ 1 1 ⎞
= 109678 ⎜⎝ 2 − 2 ⎟⎠ cm–1
ll 6 → 5, 6 → 4, 6 → 3, 6 → 2, 6 → 1 (5 lines)
.A
λ 2 ∞ 5 → 4, 5 → 3, 5 → 2, 5 → 1 (4 lines)
1 4 → 3, 4 → 2, 4 → 1 (3 lines)
= 109678 × 3 → 2, 3 → 1 (2 lines)
rs
4
2 → 1 (1 line)
4
or λ = cm = 3.647 × 10–5 cm
he
1 3.0×108 msec−1
22. = 109678 ⎜⎜ n2 − n 2 ⎟⎟ cm–1 c
λ ⎝ 1 2 ⎠ Now, λ = = = 3.0 × 10–7 m
ν 1×1015 sec−1
ub
1 ⎛1 1 ⎞ 8 = 300 nm
∴ = 109678 ⎜ 2 − 2 ⎟ cm–1 = 109678 ×
λ ⎝1 3 ⎠ 9
2.18 × 10−18
27. En = – J atom–1
P
9 n2
or λ = cm = 102.6 × 10–7 cm
109678 × 8 For ionization from 5th orbit,
n
= 102.6 nm n1 = 5 and n2 = ∞
er
λ ⎝ n1 n2 ⎠
Here n1 = 1, n2 = ∞ ⎛ 1 1⎞
= 2.18 × 10–18 ⎜ 2 − ⎟ = 8.72 × 10–20 J
⎝5 ∞⎠
M
1 ⎛1 1 ⎞
∴ = 109678 ⎜ 2 − 2 ⎟
λ ⎝ 1 ∞ ⎠
For ionization of H atom,
©
1 n1 = 1, n2 = ∞
= 109678
λ
⎛1 1⎞
Δ E = 2.18 × 10–18 ⎜ 2 − ⎟
1 ⎝1 ∞⎠
or λ = = 9.11 × 10–6 cm
109678 = 2.18 × 10–18 J
or = 9.11 × 10–8 m
1 ⎛ 1 1 ⎞ ⎛1 1 ⎞
28. = R ⎜⎜ 2 − 2 ⎟⎟ 31. ΔE = 2.18 × 10–18 ⎜ 2 − 2 ⎟ = 2.09 × 10–18J
.
λ ⎝ n1 n2 ⎠ ⎝1 5 ⎠
ed
For spectral line of electron jump from n = 5 to 6.626 × 10−34 × 3.0 × 108
hc
n = 3, λ = =
ΔE 2.09 × 10−18
rv
1
= R ⎛⎜ − ⎞⎟ = R ⎛⎜
1 1 16 ⎞ = 9.51 × 10–8 m = 951 Å
⎟ ...(i)
1281.8 ⎝3
2
52 ⎠ ⎝ 225 ⎠
se
For spectral line of electron jump from n = 3 to ⎛ 1 1 ⎞
n=1
32. ν = 109678 ⎜⎜ n2 − n2 ⎟⎟
⎝ 1 2 ⎠
re
1 ⎛1 1 ⎞ ⎛8⎞ For longest wavelength, n2 = 3, n1 = 2
= R⎜ 2 − 2 ⎟ = R⎜ ⎟ ...(ii)
λ ⎝ 1 3 ⎠ ⎝9⎠ ⎛ 1 1 ⎞
ν = 109678 ⎜ 2 − 2 ⎟
Dividing (i) by (ii) ⎝2 3 ⎠
ts
λ 16 9 5
= × = 109678 × = 1.523 × 104 cm–1
gh
1281.8 225 8 36
= 1.523 × 106 m–1
16 × 9
or λ = × 1281.8 = 102.5 nm
225× 8 2.18 × 10−18
ri
33. Ei = – = 5.45 × 10–19 J
22
1 ⎛ 1 1 ⎞ Ef = 0
29. ν = = RH Z2 ⎜ −
⎜ n 2 n 2 ⎟⎟
λ ⎝ 1 2 ⎠
ll ΔE = 0 – ( – 5.45 × 10–19) = 5.45 × 10–19 J
For He+ spectrum, Z = 2, n2 = 4, n1 = 2
.A
3 RH = 3.647 × 10–7
or 3647 Å
1
For hydrogen spectrum, =
λ 4 6.626 × 10−34 kg m 2 s−1
he
1 ⎛ 1 34. λ =
1 ⎞ 0.1 kg × 10 m s−1
∴ ν = λ = RH × 1 ⎜⎜ 2 − 2 ⎟⎟
⎝ n1 n2 ⎠ = 6.626 × 10–34m
lis
⎛ 1 1 ⎞ 3R H
RH ⎜ − =
⎜ n 2 n 2 ⎟⎟
h h
⎝ 1 2 ⎠
4 35. λ = or m =
mv λv
ub
1 1 3
− = 6.63×10−34
n12 n22 4 = = 6.139 × 10–33 kg
∴ n1 = 1, n2 = 2 3.6×10−10 × 3.0 × 108
P
n 2h 2 4
Since n = 1 37. Mass of α-particle = = 6.64 × 10−24 g
6.022 × 1023
2π2me 4
∴ I.E. = = 13.6 eV
M
I. E = mv mλ
h2 λ = 12 pm = 12 × 10–12 m
I.E. for He+
ion (Z = 2) = I.E. for H – atom × (2)2
h = 6.63 × 10–34 Js
= 13.6 × 4 = 54.4 eV
Similarly, I.E. for Li 2+ ion (Z = 3) = I.E. for 6.63 × 10−34
H-atom × (3)2 v =
6.64 × 10−27 × 12 × 10−12
= 13.6 × 9 = 122.4 eV
.
1 1 (9.1 × 10−31 kg) × (1.876 × 107 ms−1 )
ed
K.E. = mv2 = × 6.64 × 10 −27 × (8.32 × 103 )2
2 2
= 3.88 × 10–11 m
= 2.30 × 10–19 J
mv2
rv
v v h
h 41. λ = or = ∴ν=
38. λ = ν ν mv h
mv
se
1
mv2 = 5.85 × 10–25
6.63 × 10−34 2
= = 1.52 × 10–38m
0.1 × 4.37 × 10−5 mv2 = 2 × 5.85 × 10–25
re
39. The kinetic energy of the electron is provided by 2 × 5.85 × 10 −25
accelerating potential ∴ ν =
6.63 × 10 −34
ts
1
= 1.76 × 109 s–1
1 ⎛ 2 Ve ⎞ 2
mv2 = eV or v = ⎜ ⎟
⎝ m ⎠ h
gh
2 42. λ =
mv
h h h
∴ λ = = = h
1 1 or v =
ri
mv
(
m 2Ve
m )
2 (2Vem) 2 mλ
λ2 × 2 × e × m
2
λ = 9 pm = 9 × 10–12 m,
2K.E.
he
e = 1.6 × 10–19 C, or v2 =
m
m = 9.1 × 10–31 kg
h = 6.63 × 10–34 Js 2 × 1.375 × 10 −25
=
lis
9.1 × 10 −31
(6.63 × 10−34 Js)2
V= −12 = 0.3022 × 106 m2s–1
(9 × 10 m) × 2 × (1.6 × 10−19 C) × 9.1 × 10−31 kg
2
ub
= 1.602 × 10–16 J
er
1
mv2 = 1.602 × 10–16 J h
2 44. Δv = Δx = 0.1 Å or = 0.1 × 10–10 m
4πm Δx
od
1
× 9.1 × 10−31 v2 = 1.602 × 10–16 J
2 6.626 × 10−34
Δv =
4 × 3.14 × 9.11 × 10−31 × 0.1 × 10−10
M
2 × 1.602 × 10−16
v2 = = 3.521 × 1014
9.1 × 10 −31 = 5.79 × 106 m s–1
v = 1.876 × 107 m s–1 45. Uncertainty in the speed of ball
©
h 90 × 4
Now, λ = = = 3.6 m s–1
mv 100
.
4 × 3.14 × 10 × 10−3 kg × 36 m s−1 6.626 × 10−34
ed
=
4 × 3.14 × 1.66 × 10−27 × 3.0 × 105
= 1.46 × 10–33 m
= 1.06 × 10–13 m
rv
h 6.626 × 10−34
46. Δv = = 3 × 107 × 0.5
4 πmΔx 4 × 3.14 × 1.1 × 10−27 × 3 × 10−12 49. Δv = = 1.5 × 105 m s–1
100
se
= 1.59 × 104 m s–1.
h
h Δx =
47. m = 4πmΔv
4πΔx . Δv
re
6.626 × 10−34
6.626 × 10 −34 =
= 4 × 3.14 × 1.66 × 10−27 × 1.5 × 105
4 × 3.14 × 10−10 × 5.27 × 10−24
ts
= 0.10 kg = 2.11 × 10–13 m
gh
ri
ll
.A
Electromagnetic spectrum is the arrangement of different types of electromagnetic radiations in the increasing
rs
of small packets of energy called quantum or photon (for light), each having energy = hν.
de Broglie principle. All microscopic material particles in motion have dual character i.e., particle and wave
characteristics.
lis
h h
λ= or
mv p
ub
Dual nature of electrons was experimentally verified by Davisson and Germer (1927) and G. Thomson (1928).
Heisenberg uncertainty principle states that it is impossible to measure simultaneously both the position and
P
momentum (or velocity) with absolute accuracy for a microscopic particle. Mathematically, it may be expressed as:
h
Δx × Δp ≥
n
4π
er
Schrodinger wave equation is a mathematical equation to describe behaviour of electron in an atom. Schrodinger
wave equation may be expressed as :
od
Ĥ ψ = Eψ, where ψ gives the amplitude of electron wave and ψ2 is the probability density.
Electronic configuration gives the distribution of electrons in various orbitals in an atom.
Quantum numbers. A set of four numbers which give complete information about any electron in an atom.
M
Aufbau rule states that the orbitals are filled in the order of increasing energy, starting with the orbital of lowest
energy.
©
Pauli’s exclusion principle. No two electrons in an atom can have same set of all the four quantum numbers. This
means that an orbital can accommodate only two electrons having opposite spins.
Hund’s rule. The pairing of electrons in orbitals of a same subshell cannot take place until all orbitals are singly
occupied with parallel spin.
.
Einstein equation for photoelectron emission constant.
ed
nh
1 Angular momentum quantized, mvr =
mv2 = hν − hνo 2π
2 Energy of electron in an orbit
↓
rv
work function 2π2me e4Z2 2.18 × 10−18 Z 2
=– =– J per atom
ν0 = threshold frequency 2 2
n h n2
se
1.312 × 106 Z 2 13.6 Z2
Rydberg equation =– J per mol = – eV atom–1
⎛ 1 n 2 n2
1 1 ⎞
re
= ν(cm −1 ) = R ⎜ 2 − 2 ⎟ Bohr radius of an orbit
λ ⎝ n1 n2 ⎠
n2 > n1 , R = 109677 0.059 n2
rn = Å
Z
ts
Spectral lines of H-atom
Lyman n1 = 1, n2 = 2, 3, 4, 5, ⇒ ultraviolet e.g., For H atom, r1 = 0.059 Å, r2 = 2.12 Å
Balmer n1 = 2, n2 = 3, 4, 5, 6, ⇒ visible
r3 = 4.77 Å
gh
Paschen n1 = 3, n2 = 4, 5, 6 ⎤ +
Brackett n1 = 4, n2 = 5, 6, 7 ⎥ ⇒ Infrared For He , r1 = 0.0295 Å, r2 = 1.06 Å
Pfund n1 = 5, n2 = 6, 7, 8 ⎦ rn (H atom)
rn(H-like) =
ri
Z
h 2.19 × 106 Z
de-Broglie equation, λ = Velocity v =
ll = m s −1
mv n
Relationship between K.E. and λ of moving particle 2πmvZe2
.A
No. of revolutions =
h n2 h2
λ=
2 × K.E. × m
Schrodinger wave equation
Heisenberg uncertainly principle
rs
∂ 2ψ ∂ 2ψ ∂ 2ψ 8π 2m
h h + 2 + 2 + (E − V) ψ = 0
∆x × ∆p ≥ ; ∆x × ∆v ≥ ∂x 2 ∂y ∂z h2
4π 4 πm
he
or H ˆ ψ = Eψ
Quantum Numbers
lis
electron.
Name Symbol Information Permitted
er
r r
2
4pr y
2s orbital
4pr y
No. of nodes = n – l – 1
2
.
ed
rv
NCERT Textbook Exercises
se
Q. 1. (i) Calculate the number of electrons which = 6.022 × 1023 × 8 neutrons
re
will together weigh one gram. = 4.8176 × 1024 neutrons
(ii) Calculate the mass and charge of one mole (a) 14 g of 14C contain = 4.8176 × 1024 neutrons
of electrons. 7mg (7 × 10–3 g) of 14C contain
ts
Ans. (i) Mass of one electron = 9.11 × 10–31 kg
24
∴ 9.11 × 10–31 kg = 1 electron 4.8176 × 10
= × 7 × 10–3 neutrons
gh
14
1
1 g or 10–3 kg = × 10–3 electrons
9.11 × 10−31 = 2.4088 × 1021 neutrons
ri
(b) Mass of 1 neutron = 1.675 × 10–27 kg
= 1.098 × 1027 electrons
∴ Mass of 2.4088 × 1021 neutrons
(ii) Mass of one electron = 9.11 × 10–31 kg ll = 1.675 × 10–27 × 2.4088 × 1021kg
Mass of 1 mol of electrons = 9.11 × 10–31 × 6.022 × 1023
.A
= 4.035 × 10–6 kg.
= 5.486 × 10–7 kg.
Charge on one electron (iii) 1 mole of NH3 = 17g NH3 = 6.022 × 1023 molecules
of NH3
rs
= 1.602 × 10–19 C
1 molecule of NH3 contains protons = 7 + 3 = 10
Charge of 1 mol of electrons (a) 17g of NH3 contain molecules = 6.022 × 1023
he
(iii) Find (a) the total number and (b) the total
∴ Mass of 1.204 × 1022 protons
mass of protons in 34 mg of NH3 at STP.
= 1.6726 × 10–27 × 1.204 × 1022
Will the answer change if the temperature
n
Ans. (i) 1 molecule of methane (CH4) contains electrons There is no effect of temperature and pressure.
= 6 + 4 = 10 Q. 3. How many neutrons and protons are there in the
od
(ii) In C
13C 6 13 6 13 – 6 = 7
No. of neutrons + No. of Protons = 14 6
16O 8 16 8 16 – 8 = 8
No. of protons = 6 8
©
∴ No. of neutrons + 6 = 14 24 Mg
12 12 24 12 24 – 12 = 12
∴ No. of neutrons = 14 – 6 = 8 56Fe 26 56 26 56 – 26 = 30
26
Now 1 mol of 14C = 14 g C 88Sr 38 88 38 88 – 38 = 50
38
or = 6.022 × 1023 atoms of 14C
Q. 4. Write the complete symbol for the atom with the emission, and (iii) the velocity of the
given atomic number (Z) and atomic mass (A) photoelectron (1 eV = 1.602 × 10–19J)
(i) Z = 17, A = 35 Ans. (i)Energy of photon
(ii) Z = 92, A = 233 hc
E = hν =
(iii) Z = 4, A = 9 λ
.
35
Ans. (i) 17Cl (ii) 233U
9
(iii) 4Be λ = 4.0 × 10–7m, c = 3.0 × 108 m s–1
ed
92
Q. 5. Yellow light emitted from a sodium lamp has a h = 6.626 × 10–34 J s,
wavelength (λ λ ) of 580 nm. Calculate the
ν ) and wavenumber (ν–) of the ( 6.626 × 10−34 Js) × (3.0 × 108 m)
rv
frequency (ν ∴ E = −7
yellow light. (4.0 × 10 m)
Ans. Refer Solved Example 6 (Page 13).
se
= 4.97 × 10–19 J
Q. 6. Find energy of each of the photons which :
-19
(i) correspond to light of frequency 3 × 1015 Hz. 4.97 × 10
or = = 3.10 eV.
re
-19
(ii) have wavelength of 0.50 Å 1.6020 × 10
Ans. (i) E = hν (ii) Kinetic energy of emission
= (6.626 × 10–34 Js) × (3 × 1015 s–1) K.E. = hν – hν0
ts
= 1.988 × 10–18 J hν = 3.10 eV and hν0 (work function) = 2.13 eV
hc ∴ K.E. = 3.10 – 2.13 = 0.97 eV
gh
(ii) E = hν =
λ 1
(iii) Now, K.E. = mv2
(6.626 × 10 Js) × (3 × 10 m s )
−34 8 −1 2
ri
= 1
or mv2 = 0.97 eV = 0.97 × 1.602 × 10–19 J
(0.50 × 10 m ) −10
2
Q. 7.
= 3.98 × 10–15 s
Calculate the wavelength, frequency and
ll or
1
2
× (9.11 × 10–31kg) v2 = 0.97 × 1.602 × 10–19 J
.A
wavenumber of a light wave whose period is 2 × 0.97 × 1.602 × 10
−19 2 −2
kg m s
2.0 × 10–10s. v2 =
−31
9.11 × 10 kg
1
rs
1
Ans. (i) Frequency = = = 5 × 109 s–1 = 34.12 × 10 m s 10 2 –2
Period 2.0 × 10−10s
∴ v = 5.84 × 105 m s–1
he
8 −1
c 3 × 10 m s
(ii) Wavelength, λ = = = 6.0 × 10–2 m Q. 10. Electromagnetic radiation of wavelength 242
ν 5 × 109 s−1
nm is just sufficient to ionise the sodium atom.
lis
wavelength of 4000 pm that provide 1J of wavelength 242 nm will be equal to ionization energy,
energy ? hc
Ans. Wavelength of light, λ = 4000 pm E = hν =
P
λ
= 4000 × 10–12 m = 4 × 10–9 m h = 6.626 × 10–34 J s
Energy of 1 photon, c = 3.0 × 108 m, λ = 242 × 10–9 m
n
hc
(6.626 × 10 −34 J s) × (3.0 × 108 m s−1)
er
E = ∴ E =
λ
(242 × 10−9 m)
h = 6.626 × 10–34 J s, c = 3.0 × 108 m s–1
10–19
od
= 8.21 × J
6.626 × 10–34 × 3.0 × 108
∴ E = Ionization energy per mol = 8.21 × 10–19 J × 6.023 × 1023
4 × 10–9 = 494 × 103 J = 494 kJ mol–1.
M
4.97 × 10 –17
Ans. Energy of one photon,
Q. 9. A photon of wavelength 4 × 10–7 m strikes on
metal surface, the work function of the metal hc
E = hν =
being 2.13 eV. Calculate (i) the energy of the λ
photon (eV), (ii) the kinetic energy of the h = 6.626 × 10–34 J s
c = 3.0 × 108 m s–1, λ = 0.57 μm
= 0.57 × 10–6 m s–1 Thus, the energy required to remove electron from Ist
−34 8 orbit is 25 times than that required to remove electron
6.63 × 10 × 3.0 × 10
E = −6
from 5th orbit.
0.57 × 10
Q. 15. What is the maximum number of emission lines
= 3.487 × 10–19 J when the excited electron of a H-atom in n = 6
.
Now, 25 Watt = 25 J/s drops to the ground state ?
ed
25
∴ No. of photons emitted per second = Ans. Refer Solution to Practice Problems 25 (Page 70).
3.487 ×10−19
= 7.17 × 1019s–1. Q. 16. (i) The energy associated with the first orbit in
rv
Q. 12. Electrons are emitted with zero velocity from a the hydrogen atom is – 2.18 × 10–18 J atom–1. What
metal surface when it is exposed to radiation of is the energy asociated with the fifth orbit ?
se
wavelength 6800 Å. Calculate threshold (ii) Calculate the radius of Bohr’s fifth orbit for
ν0) and work function (W0) of the
frequency (ν hydrogen atom.
re
metal. Ans. Refer Solved Example 22 (Page 28).
Ans. K.E. = hν –hν0 Q. 17. Calculate the wavenumber for the longest
If the velocity is zero, K.E. = 0 wavelength transition in the Balmer series of
ts
∴ 0 = hν – hν0 atomic hydrogen.
or h ν = hν0 or ν = ν0 ⎛ 1 1 ⎞
gh
−1
Ans. ν– = R ⎜⎜ 2 − 2 ⎟⎟
c 3.0 × 10 m s 8
⎝ n1 n2 ⎠
∴ ν = =
λ 6800 × 10 −10 m For Balmer series n1 = 2, for longest wavelength, ν–
ri
= 4.41 × 1014 s–1 should be minimum so that n2= 3.
Threshold frequency = 4.41 × 1014 s–1 ⎛ 1 1 ⎞
Work function = hν0 = 6.626 × 10–34 × 4.41 × 1014
ll ν– = (1.097 × 107m–1) ⎜ 2 − 2 ⎟
⎝2 3 ⎠
= 2.92 × 10–19 J.
.A
5
Q. 13. What is the wavelength of light emitted when = 1.097 × 107 ×
36
the electron in a hydrogen atom undergoes
= 1.523 × 106 m–1.
rs
H-atom if the electron occupies n = 5 orbit ? returns to the ground state ? The ground state
Compare your answer with the ionization electron energy is –2.18 × 10–11 ergs.
enthalpy of H-atom (energy required to remove
Ans. The ground state energy is –2.18 × 10–11 ergs.,
ub
2
n
For ionization from 5th orbit, n1 = 5, n2 = ∞ ⎛ 1 1 ⎞
ΔE = E5 – E1 = – 2.18 × 10–11 ⎜ 2 − 2 ⎟
⎝5 1 ⎠
n
⎛ 1 1 ⎞
ΔE = E2– E1 = – 21.8 × 10–19 ⎜⎜ 2 − 2 ⎟⎟ 24
er
⎛ 1 ⎞
= – 21.8 × 10–19 × ⎜ − ⎟ When electron returns to ground state (i.e., n = 1),
⎝ 25 ⎠ energy emitted = 2.093 × 10–18 J
M
= 8.72 × 10–20 J
For ionization from Ist orbit, n1 = 1, n2 = ∞ hc hc
Now, ΔE = or λ =
⎛1 1 ⎞ λ ΔE
©
Q. 19. The electron energy in hydrogen atom is given Ans. (i) No. of protons = 29
by En = (–2.18 × 10–18)/n2J. Calculate the energy (ii) Atomic number = 29
required to remove an electron completely from
Electronic configuration : [Ar]18 3d104s1.
the n = 2 orbit. What is the longest wavelength of
light in cm that can be used to cause this Q. 27. Give the number of electrons in the species H+2,
H2 and O+2 .
.
transition ?
ed
Ans. ΔE = E∞ – E2 Ans. H+2 : 1 , H2 = 2, O+2 = 15.
Q. 28. (i) An atomic orbital has n = 3. What are the
⎛ 2.18 × 10−18 J atom −1 ⎞
= 0 – ⎜− ⎟ possible values of l and ml ?
rv
⎝ 22 ⎠ (ii) List the quantum numbers (ml and l) of elec-
= 5.45 × 10–19 J atom–1 trons for 3d orbital.
se
hc hc (iii) Which of the following orbitals are possible ?
ΔE = or λ = 1p, 2s, 2p and 3f
λ ΔE
Ans. (i) For n = 3, l = 0, 1, 2
re
∴ λ =
(6.626 × 10 −34
) (
Js × 3.0 × 108 ms−1 ) When l = 0, ml = 0
−19
5.45 × 10 J atom -1
When l = 1, ml = –1, 0, + 1
ts
= 3.647 × 10–7m When l = 2, ml = –2, –1, 0, +1, + 2
or = 3.647 × 10–5 cm. (ii) n = 3, l = 2 and for l =2, ml = –2, –1, 0, +1, +2
gh
Q. 20. Calculate the wavelength of an electron moving (iii) 2s, 2p are possible.
with a velocity of 2.05 × 107 ms–1 Q. 29. Using s, p, d notations, describe the orbital with
Ans. Refer Solved Example 31 (Page 38). the following quantum numbers.
ri
Q. 21. The mass of an electron is 9.1 × 10–31 kg. If its K.E. (a) n = 1, l = 0; (b) n = 3; l = 1
is 3.0 × 10–25 J, calculate its wavelength. (c) n = 4; l = 2; (d) n = 4; l = 3
Ans. (i) Refer Solved Example 57 (Page 63). for the hydrogen atom is an integral multiple of
er
h
Q. 25. An electron is in one of the 3d orbitals. Give the According to de Broglie equation, λ = .....(ii)
mv
possible values of n, l and ml for this electron. From (i) and (ii) we get
©
.
λ 2 1.2 × 10−10
n 22 Radius of carbon atom =
ed
1
2
Now, n1 = 2, n2 = 4 and Z = 2
= 0.6 × 10–10 m
LM 1 OP = 3 R = 0.06 × 10–9 m
rv
1 1
= R × (2)2 ...(i)
λ N22
−
4 2
Q 4 = 0.06 nm
Q. 37. The diameter of zinc atom is 2.6 Å. Calculate
LM 1 OP
se
1 1 (a) radius of zinc atom in pm and (b) number of
−
For H atom
λ
= R
MN n
1
2
n 22 PQ ...(ii) atoms present in a length of 1.6 cm if the zinc
atoms are arranged side by side lengthwise.
re
Since λ is the same, equating equations (i) and (ii) Ans. (i) Diameter of zinc atom = 2.6 Å = 2.6 × 10–10m
1 1 3 2.6 ×10−10
− = Radius =
n 12 2
ts
n2 4 2
–10 –12
= 1.3 × 10 m = 130 × 10 m
Now, if n1 = 1 and n2 = 2.
or = 130 pm
gh
Therefore, the transition from n = 2 to n = 1 in H atom (ii) No. of Zn atoms present on a 1.6 cm length =
will have the same wavelength as the transition from
1.6
n = 4 to n = 2 in He+. = 6.154 × 107
ri
Q. 34. Calculate the energy required for the process 2.6 × 10−10 × 102
He+(g) ⎯⎯→ He2+ (g) + e– Q. 38. A certain particle carries 2.5 × 10–16C of static
The ionization energy for the H atom in the
ll electric charge. Calculate the number of
electrons present in it.
ground state is 2.18 × 10 –18 J atom –1.
.A
Ans. Charge of one electron = 1.602 × 10–19 C
−2 π 2 m Z 2 e 4
Ans. For H like species, En =
n2 h2 2.5 × 10 −16
No. of electrons present = = 1.56 × 103
rs
⎛ 2π2me4 2 2 ⎞
( ) ⎟ = 4 × 2π m e
2 4
Q. 40. In Rutherford’s experiment, generally the
= 0 – ⎜−
⎜⎝ (
12 × h2 ⎟⎠ ) h2 thin foil of heavy atoms, like gold, platinum, etc.,
have been used to be bombarded by the
P
Q. 35. If the diameter of a carbon atom is 0.15 nm, be observed from the above results ?
calculate the number of carbon atoms which Ans. In Rutherford’s experiment, heavy atoms have heavy
er
can be placed side by side in a straight line across nucleus carrying a large amount of positive charge.
length of scale of length 20 cm long. Therefore, some α-particles even got deflected back
od
Ans. A carbon atom covers length = diameter of atom = after hitting the nucleus. Because of large positive
0.15 nm charge on the heavy nucleus, some α-particles are
= 0.15 × 10–9 × 102 cm deflected through small angles which passed closer
M
∴ No. of carbon atoms which can be placed on 20 cm to the nucleus because of repulsion. However, if
length lighter atoms are used, their nuclei will be light and
they will have small positive charge on the nucleus.
20
©
Ans. An atom will cover length equal to its diameter Answer briefly.
Ans. Atomic number of an element (say Br) is fixed but its Ans. Energy of 1 photon,
mass number is not fixed depending upon the isotope. hc
E=
Hence, it is necessary to indicate mass number. λ
Q. 42. An element with mass number 81 contains 31.7% h = 6.626 ×10–34Js, c = 3.0 ×108 ms–1,
more neutrons as compared to protons. Assign λ = 600nm = 600 × 10–9m
the atomic symbol.
.
−34
ed
8
Ans. Refer Solved Example 4 (Page 10). 6.626 × 10 × 3.0 × 10
E =
−9
Q. 43. An ion with mass number 37 possesses one unit 600 × 10 m
of negative charge. If the ion contains 11.1%
= 3.31 × 10–19 J
rv
more neutrons than the electrons, find the
symbol of the ion Total energy received = 3.15 × 10–18 J
−18
Ans. Solutions to Practice Problem 8 (Page 69). 3.15 × 10
se
No. of photons = = 9.52 10
Q. 44. An ion with mass number 56 contains 3 units of 3.31 × 10
−19
re
electrons. Assign the symbol to this ion.
are often measured by using pulsed radiation
Ans. Refer Solved Example 5 (Page 10). source of duration nearly in the nano second
Q. 45. Arrange the following type of radiations in range. If the radiation source has the duration of
increasing order of frequency : (a) radiation
ts
2 ns and the number of photons emitted during
from microwave oven (b) amber light from traffic the pulse source is 2.5 × 1015, calculate the energy
signal (c) radiation from FM radio (d) cosmic of the source.
gh
rays from outer space and (e) X-rays.
1
Ans. Cosmic rays < X-rays < amber light < microwave Ans. Frequency = = 0.5 × 109 s–1
< FM. 2 × 10−9
ri
Energy = Nhν = (2.5 × 1015) × (6.62 × 10–34 Js)
Q. 46. Nitrogen laser produces a radiation at a
× (0.5 × 109 s–1) = 8.275 × 10–10J.
wavelength of 337.1 nm. If the number of photons
Q. 50. The longest wavelength doublet absorption
ll
emitted is 5.6 × 1024, calculate the power of this
transition is observed at 589 and 589.6 nm.
laser.
Calculate the frequency of each transition and
.A
Ans. E = N hν =
Nhc energy difference between two excited states.
λ Ans. λ1 = 589 nm = 589 × 10–9 m
( ) ( ) (
5.6 × 1024 × 6.62 × 10−34 Js × 3.0 × 108 ms−1 )
rs
λ2 589.6 × 10−9 m
this radiation in 30 s (c) energy of quantum and
(d) number of quanta present if it produces 2 J ΔE = h (ν2 –ν1)
of energy. = (6.626 × 10–34 Js) (5.093 – 5.088) × 1014 s–1
ub
λ −9
616 × 10 m (b) the threshold frequency of the radiation. If
= 4.87 × 1014 s–1 the caesium element is irradiated with a
wavelength 500 nm, calculate the kinetic energy
n
hc
Now, energy of radiated light, E = 123.17 × 10−21 × 10−9 × 400 × 526
λ h=
3.0 × 108 × 126
6.626 × 10 −34 × 3.0 × 108 = 6.84 × 10–34 Js.
= = 3.98 × 10–19 J
500 × 10 −9 Q. 53. The ejection of the photoelectron from the silver
∴ K.E. of ejected electron = 3.98 × 10–19 – 3.04 × 10–19 metal in the photoelectric effect experiment
.
= 9.4 × 10–20 J can be stopped by applying the voltage of 0.35 V
ed
1 when the radiation 256.7 nm is used. Calculate
K.E. = mv2 = 9.4 × 10–20 J the work function for silver metal.
2
Ans. Energy of incident radiation
rv
9.4 × 10−20 × 2
v2 = = 20.66 × 1010 = Work function + K.E. of photoelectrons
9.1 × 10−31 Energy of incident radiation,
se
∴ v = (20.66 × 1010)½ = 4.54 × 105 m s–1
hc
Q. 52. Following results are observed when sodium E = hν =
metal is irradiated with different wavelengths. λ
re
Calculate (a) threshold wavelength and,
(b) Planck’s constant.
=
(6.626 × 10 Js ) × ( 3.0 × 10
−34 8
ms −1 )
λ(nm)
v × 10–5 (cm s–1)
500
2.55
450
4.35
400
5.20
( 256.7 × 10 m )
−9
ts
Ans. Let threshold wavelength be λ0 nm = λ × 10–9 m = 7.74 × 10–19 J
1
h(ν – ν0) = mv2 7.74 × 10−19 J
gh
2 or = = 4.83 eV
1.602 × 10 −19
⎛c c ⎞ 1 2 The potential energy gives the kinetic energy to the
h ⎜ λ − λ ⎟ = 2 mv
ri
⎝ 0⎠ electrons.
Hence K.E. of the electron = 0.35 eV
⎛1 1 ⎞ 1 2 ll Work function = 4.83 eV – 0.35 eV = 4.48 eV
or hc ⎜ λ − λ ⎟ = 2 mv
⎝ 0⎠ Q. 54. If the photon of the wavelength 150 pm strikes
.A
an atom and one of its inner bound electrons is
⎛ 1 1 ⎞ 1 2 ejected out with a velocity of 1.5 × 107 m s–1,
hc ⎜⎜ − ⎟ = mv
500 × 10 −9
λ × 10 −9 ⎟
2 calculate the energy with which it is bound to
⎝ 0 ⎠
the nucleus.
rs
=
Similarly, λ
hc ⎛ 1
−9 ⎜
1 ⎞
− ⎟ =
1
m (4.35 × 105)2 ....(ii) =
(6.626 × 10 Js ) × (3.0 × 10 ms )
−34 8 −1
lis
λ
10 ⎝ 450 0⎠ 2
(150 × 10 m )
−12
hc ⎛ 1 1 ⎞ 1 = 1.325 × 10–15
J
⎜ − ⎟ = m (5.20 × 105)2 ....(iii)
10−9 ⎝ 400 λ0 ⎠ Energy of ejected electron
ub
2
Dividing eq. (ii) by (i) 1
= mv2
2 2
λ0 − 450 500λ0 ⎛ 4.35 ⎞
P
× = 1
450λ0 λ0 − 500 ⎜⎝ 2.55 ⎟⎠ = × (9.11 × 10–31 kg) × (1.5 × 107ms–1)2
2
n
2 = 1.025 × 10–16J
λ0 − 450 450 ⎛ 4.35 ⎞ Energy with which the electron was bound to the
= ×
er
1.91 λ0 = 1005
∴ λ0 = 526.2 nm or = 526 nm Q. 55. Emission transitions in the Paschen series end at
orbit n = 3 and start from orbit n and can be
Substituting the value of λ0 in eqn. (iii), we get
represented as ν = 3.29 × 1015 (Hz) [1/32 – 1/n2]
©
( )
h × 3.0 × 108 ⎛ 1 1 ⎞ 1 –31 5 2
Calculate the value of n if the transition is
⎜ 400 − 526 ⎟ = 2 × 9.11 × 10 × (5.20 × 10 ) observed at 1285 nm. Find the region of the
10−9 ⎝ ⎠ spectrum.
( )
h × 3.0 × 108 ⎛ 526 − 400 ⎞
–21
⎛ 1 1 ⎞
ν = 3.29 × 10 15 ⎜ 2 − 2 ⎟ s–1
⎜ 400 × 526 ⎟ = 123.17 × 10 Ans.
10−9 ⎝ ⎠ ⎝3 n ⎠
λ = 1285 nm
h 6.626 × 10−34 kg m 2s−1
Ans. λ= =
( ) ( )
8 −1
c 3.0 × 10 ms mv 9.11 × 10−31 kg × 1.6 × 106 ms−1
∴ ν = = −9
λ 1285 × 10 m = 4.55 × 10–10m
= 2.33 × 1014 s–1 or = 455 pm.
Q. 58. Similar to electron diffraction, neutron
.
⎛1 1 ⎞
ed
2.33 × 1014 = 3.29 × 1015 ⎜ − 2 ⎟ diffraction microscope is also used for the
⎝9 n ⎠ determination of the structure of molecules. If
14 the wavelength used here is 800 pm calculate
1 1 2.33 × 10 the characteristic velocity associated with the
rv
or − 2 = = 0.0708
9 n 3.29 × 10
15
neutron.
h h
se
1 1 Ans. λ = = or ν =
− 2
= 0.0708 – = –0.0403 mv mλ
n 9
6.626 × 10−34 kg m2s−1
1 =
re
2 = 0.0403 1.675 × 10−27 kg × 800 × 10−12m
n
= 4.94 × 102 ms–1
or n2 = 24.82 Q. 59. If the velocity of the electron in Bohr’s first orbit
ts
∴ n = 5 is 2.19 × 106 ms–1, calculate the de Broglie
wavelength associated with it.
Transition is obtained in infrared region.
gh
Ans. Refer Solved Example 35 (Page 38).
Q. 56. Calculate the wavelength for the emission
Q. 60. The velocity associated with a proton moving in
transition if it starts from the orbit having radius
1.3225 nm and ends at 211.6 pm. Name the series a potential difference of 1000 V is 4.37 × 10 5
ri
to which this transition belongs and the region ms–1. If the hockey ball of mass 0.1 kg is moving
of the spectrum. with this velocity, calculate the wavelength
0.529 n2
ll associated with this velocity.
Ans. Radius of nth orbit of H like species = Å h
.A
Z Ans. λ =
mv
52.9n2 6.626 × 10 −34 kg m 2s −1
or = pm =
Z
(
( 0.1 kg ) × 4.37 × 105 ms −1 )
rs
52.9 n12
r1 = 1.3225 nm = 1322.5 pm =
Z = 1.516 × 10–28 m
he
n22 n2 4 π × 0.05 nm
this value.
If n2 = 2, n1 = 5, This transition corresponds to tran-
sition from 5th orbit to 2nd orbit. Hence, it belongs Ans. Refer Solved Example 46 (Page 43).
P
21 –1
= 1.097 × 107 × m (a) n = 4, l = 2, ml = –2, ms = – 1/2
100
(b) n = 3, l = 2, ml = 1, ms = + 1/2
od
1 1
∴ λ = = (c) n = 2, l = 0, ml = 0, ms = + 1/2
ν 23.037 × 105 m −1
(d) n = 2, l = 0, ml = 0, ms = – 1/2
= 434 × 10–9m or 434 nm
M
(e) n = 3, l = 3, ml = 0, ms = + 1/2
It lies in the visible region. (f) n = 3, l = 1, ml = 0, ms = + 1/2
Q. 57. Dual behaviour of matter proposed by Ans. Refer Solved Example 58 (Page 64).
©
de Broglie led to the discovery of electron Q. 63. The bromine atom possesses 35 electrons. It
microscope often used for the highly magnified contains 6 electrons in 2p orbital, 6 electrons in
images of biological molecules and other type of 3p orbital and 5 electron in 4p orbital. Which of
material. If the velocity of the electron in this
these electron experiences the lowest effective
microscope is 1.6 × 10 6 ms –1 , calculate
de Broglie wavelength associated with this nuclear charge ?
electron.
.
(iii) 3d and 3p. n = 4 ? (b) How many electrons will be present in
ed
Ans. (i) 2s (ii) 4d (iii) 3p. the sub-shell having m s value of –1/2 for
Q. 65. The unpaired electrons in Al and Si are present n=4?
in 3p orbital. Which electrons will experience Ans. Sub-shells in n = 4 are 4 i.e., 4s, 4p, 4d and 4f
rv
more effective nuclear charge from the
nucleus ? No. of electrons having ms = −1 2 for n = 4 will be 16.
se
NCERT Exemplar Problems
re
(a) They start from the cathode and move towards the
anode.
(b) They travel in straight line in the absence of an
ts
1. Which one of the following conclusions could not be
derived from Rutherford’s α-paticle scattering external electrical or magnetic field.
experiments ? (c) Characteristics of cathode rays do not depend upon
gh
(a) Most of the space in the atom is empty. the material of electrodes in cathode ray tube.
(d) Characteristics of cathode rays depend upon the
(b) The radius of the atom is about 10–10 m while that
nature of gas present in the cathode ray tube.
of nucleus is 10–15 m.
ri
(c) Electrons move in a circular path of fixed energy 5. Which of the following statement about the electron is
called orbits. incorrect ?
(d) Electrons and the nucleus are held together by
ll (a) It is a negatively charged particle
(b) The mass of electron is equal to the mass of neutron
electrostatic forces of attraction.
.A
(c) It is a basic constituent of all atoms
2. Which of the following options does not represent ground
(d) It is a constituent of cathode rays
state electronic configuration of an atom ?
6. Which of the following properties of atom could be
rs
given in Fig. 1 :
7. Two atoms are said to be isobars if
(a) they have same atomic number but different mass
ub
number.
(b) they have same number of electrons but different
number of neutrons.
P
11. Total number of orbitals associated with third shell 15. For the electrons of oxygen atom, which of the f ollowing
will be _______ statements is correct ?
(a) 2 (b) 4 (a) Zeff for an electron in a 2s orbital is the same as Zeff
(c) 9 (d) 3 for an electron in a 2p orbital.
12. Orbital angular momentum depends on _______ . (b) An electron in the 2s orbital has the same energy
(a) l (b) n and l as an electron in the 2p orbital.
.
(c) Zeff for an electron in 1s orbital is the same as Zeff
ed
(c) n and m (d) m and s
13. Chlorine exists in two isotopic forms, Cl-37 and Cl-35 for an electron in a 2s orbital.
but its atomic mass is 35.5. This indicates the ratio of (d) The two electrons present in the 2s orbital have
spin quantum numbers ms but of opposite sign.
rv
Cl-37 and Cl-35 is approximately
(a) 1:2 (b) 1:1 16. If travelling at same speeds, which of the following
matter waves have the shortest wavelength ?
(c) 1:3 (d) 3:1
se
(a) Electron (b) Alpha particle (He2+)
14. The pair of ions having same electronic configuration is
(a) Cr3+, Fe3+ (b) Fe3+, Mn2+ (c) Neutron (d) Proton
re
(c) Fe3+, Co3+ (d) Sc3+, Cr3+
ANSWERS / HINTS
ts
MCQs Type-I
1. (c) The concept of circular paths of fixed energy was 9. (c) 10. (b)
gh
proposed by Bohr and not derived from Rutherford’s
11. (c) No. of orbitals in 3rd shell (n = 3) = n2 = 32 = 9.
scattering experiment.
2. (b) Correct configuration in ground state should be 12. (a)
ri
1s2 2s2 2p6 3s2 3p6 3d10 4s1. 13. (c) If Cl-37 and Cl-35 exist in the ratio of 1:3, then
3. (d) The probability density of electrons in 2s orbit first ll average atomic mass comes out to be 35.5.
increases, then decreases and after that it begins to
14. (b) Cr3+ : [Ar]3d3, Fe3+ : [Ar]3d5 ; Mn2+ : [Ar]3d5
.A
increase again.
4. (d) 5. (b) Thus, Mn2+ and Fe3+ have same electronic configuration.
6. (a) Thomson model of atom could explain only the 15. (d)
rs
m
8. (d) No. of radial nodes in 3p = n – l – 1
has shortest wavelength.
or 3–1–1=1
lis
6 7
18. Out of the following pairs of electrons, identify the (d) 3 4 –2
pairs of electrons present in degenerate orbitals :
n
2
1 21. Which of the following statements concerning the quan-
(b) (i) n = 3, l = 1, ml = 1, ms = + tum numbers are correct ?
2
(a) Angular quantum number determines the three
M
1
(ii) n = 3, l = 2, ml = 1 ms = +
2 dimensional shape of the orbital.
(c) (i) n = 4, l = 1, ml = 1, ms = + 1 (b) The principal quantum number determines the
©
ANSWERS / HINTS
MCQs Type-II
17. (c, d) are not isotopes because they have different atomic Al3+ = 13 – 3 = 10 e–, O2– = 8 + 1 = 9 e– i.e. do not have
numbers. same no. of electrons.
.
18. (a, d) : Degenerate orbitals have same n and same Na+ = 11 – 1 = 10 e–, O2– = 8 + 2 = 10 e– i.e. have same
ed
l values. no. of electrons.
19. (b, c) N3– = 7 + 3 = 10 e–, Cl– = 17 + 1 = 18 e– i.e. do not have
rv
20. (a, c) same no. of electrons.
Na+ = 11 – 1 = 10 e–, Mg2+ = 12 – 2 = 10e–, i.e. have same 21. (a, d)
se
no. of electrons.
re
highest energy ?
Q.22. Arrange s, p and d sub-shells of a shell in the 5p, 5d, 5f, 6s, 6p
increasing order of effective nuclear charge
Ans. Refer Conceptual Qs. 2 Q.39 (Page 67).
ts
(Zeff) experienced by the electron present in
them. Q. 28. Which of the following will not show deflection
gh
Ans. s-orbitals shield the electrons from the nucleus more from the path on passing through an electric
effectively than p-orbitals which in turn shield more field ?
effectively than d-orbitals. Hence, the subshells in Proton, cathode rays, electron, neutron.
ri
the increasing order of effective nuclear charge is :
Ans. Neutron is a neutral particle. Hence, it will not show
d<p<s
deflection from the path on passing through an
Q.23. Show the distribution of electrons in oxygen ll electric field.
atom (atomic number 8) using orbital diagram.
.A
2p
Q. 29. An atom having atomic mass number 13 has
1s 2s
Ans. O (Z = 8) : 7 neutrons. What is the atomic number of the atom ?
↑↓ ↑↓ ↑↓ ↑ ↑
Ans. Atomic mass, A = 13, n = 7
Q.24. Nickel atom can lose two electrons to form Ni2+
rs
n + p = 13 ∴ p = 13 – 7 = 6
ion. The atomic number of nickel is 28. From
Hence, atomic number, Z = p = 6
which orbital will nickel lose two electrons.
he
λ(C) = 3 nm λ(D) = 30 Å
3dxy, 4dxy , 3 dz 2 , 3dyz, 4dyz, 4dz 2 Arrange these radiations in the increasing order
of their energies.
ub
Q.27. The arrangement of orbitals on the basis of λ(C) = 3 × 10–9 m, λ(D) = 30 × 10–10 m = 3 × 10–9 m
er
(c) 5p, 4d, 5d, 4f, 6s (d) 5f, 6d, 7s, 7p are completely filled and hence this configuration is
II. Based upon the above information, solve the more stable.
questions given below : Q. 32. The Balmer series in the hydrogen spectrum
(a) Which of the following orbtials has the corresponds to the transition from n1 = 2, to n2 =
lowest energy ? 3, 4, .........., This series lies in the visible region.
4d, 4f, 5s, 5p Calculate the wave number of line associated
with the transition in Balmer series when the Ans. Wavelength is the distance between the successive
electron moves to n = 4 orbit. crests.
(RH = 109677 cm–1) ∴ Wavelength, λ = 4 × 2.16 = 8.64 pm
⎛ 1 1⎞ −1 Q.37. Chlorophyll present in green leaves of plants
Ans. ν = 109677 ⎜ 2 − 2 ⎟ cm
absorbs light at 4.620 × 1014 Hz. Calculate the
.
⎝ n1 n2 ⎠
ed
⎛1 1 ⎞ wavelength of radiation in nanometer. Which
= 109677 ⎛⎜ 1 − 1 ⎞⎟ = 109677 ⎜ − ⎟ part of the electromagnetic spectrum does it
⎝ 22 4 2 ⎠ ⎝ 4 16 ⎠
belong to ?
rv
3
= 109677 × = 20564.4 cm–1 c
16 Ans. λ=
ν
se
Q.33. According to de Broglie, matter should exhibit
3.0 × 108
dual behaviour, that is both particle and wave ∴ λ = = 0.6494 × 10–6
4.620 × 1014
like properties. However, a cricket ball of mass
re
100 g does not move like a wave when it is thrown = 649.4 × 10–9 m = 649.4 nm
by a bowler at a speed of 100 km/h. Calculate the
It belongs to visible light.
wavelength of the ball and explain why it does
Q.38. What is the difference between the terms ‘orbit
ts
not show wave nature.
and orbital’ ?
Ans. m = 100 g = 0.1 kg
Ans. Refer Text (Page 45).
gh
100 × 1000
v = 100 km/h = Q.39. Table-tennis ball has a mass 10 g and a speed of
60 × 60
90 m/s. If speed can be measured within an
1000
ri
= ms −1 accuracy of 4% what will be the uncertainty in
36 speed and position ?
h 6.63 × 10−34 kg m 2s−1 ll
λ= = 90 × 4
mv (0.1 kg) (1000 / 36 ms−1 ) Ans. Uncertainty in speed of ball = = 3.6 ms–1
.A
100
= 2.387 × 10–34 m
h
Since the wavelength is very small, the wave nature Uncertainty in position, Δx =
4 πm Δv
rs
cannot be detected.
Q.34. What is the experimental evidence in support of 6.626 × 10 −34 Js
=
he
wavelength are obtained. These lines correspond to Q.40. The effect of uncertainty principle is
electronic transitions between fixed energy levels. significant only for motion of microscopic
Thus, the electrons in these energy levels have definite particles and is negligible for the macroscopic
ub
h
Ans. λ = . For the same value of λ, the electron will have repulsion between electrons is absent. However,
er
Q.42. Match the following species with their corresponding ground state electronic configuration.
Atom / Ion Electronic configuration
(i) Cu (a) 1s2 2s2 2p6 3s2 3p6 3d10
.
ed
(ii) Cu2+ (b) 1s2 2s2 2p6 3s2 3p6 3d10 4s2
(iii) Zn2+
(c) 1s2 2s2 2p6 3s2 3p6 3d10 4s1
(iv) Cr3+ (d) 1s2 2s2 2p6 3s2 3p6 3d9
rv
(e) 1s2 2s2 2p6 3s2 3p6 3d3
se
Q.43. Match the quantum numbers with the information provided by these.
Quantum number Information provided
(i) Principal quantum number (a) orientation of the orbital
re
(ii) Azimuthal quantum number (b) energy and size of orbital
(iii) Magnetic quantum number (c) spin of electron
ts
(iv) Spin quantum number (d) shape of the orbital
gh
Q.44. Match the following rules with their statements :
Column I Column II
ri
(i) Hund’s Rule (a) No two electrons in an atom can have the same set of four quantum numbers.
(ii) Aufbau Principle (b) Half-filled and completely filled orbitals have extra stability.
ll
(iii) Pauli Exclusion Principle (c) Pairing of electrons in the orbitals belonging to the same subshell does
.A
not take place until each orbital is singly occupied.
(iv) Heisenberg’s Uncertainty Principle (d) It is impossible to determine the exact position and exact momentum
of a subatomic particle simultaneously.
rs
(e) In the ground state of atoms, orbitals are filled in the order of their
increasing energies.
he
ANSWERS / HINTS
M
Matching Type
42. : (i) – (c) (ii) – (d) (iii) – (a) (iv) – (e) has also particle as well as wave nature. ψ2 represents
probability density and is always positive. Principle
©
.
(a) Both A and R are true and R is the correct explanation
ed
of A. Reason (R) : The frequency of radiation emitted
(b) Both A and R are true but R is not the correct by a body goes from a lower
explanation of A. frequency to higher frequency with
rv
(c) A is true but R is false. an increase in temperature.
(d) Both A and R are false. Q.50. Assertion (A) : It is impossible to determine the
se
exact position and exact momentum
Q.48. Assertion (A) : All isotopes of a given element show of an electron simultaneously.
the same type of chemical behaviour. Reason (R) : The path of an electron in an atom
re
is clearly defined.
ts
ANSWERS / HINTS
Assertion Reason Type
gh
48. (a) 49. (b) 50. (c)
ri
hydrogen gas, the hydrogen molecules dissociate
Q.51. What is photoelectric effect? State the result to produce excited hydrogen atoms. These
of photoelectric effect experiment that could
ll excited atoms emit electromagnetic radiation
of discrete frequencies which can be given by
not be explained on the basis of laws of
.A
classical physics. Explain this effect on the basis the general formula.
of quantum theory of electromagnetic ⎡1 1 ⎤
radiations. ν = 109677 ⎢ 2 − 2 ⎥
rs
⎢ ni nf ⎥
Ans. Refer Text (Page 14–16). ⎣ ⎦
Q.52. Threshold frequency, ν 0 is the minimum What points of Bohr’s model of an atom can be
he
frequency which a photon must possess to used to arrive at this formula? Based on these
eject an electron from a metal. It is different points derive the above formula giving
for different metals. When a photon of description of each step and each term.
lis
frequency 1.0 × 1015 s–1 was allowed to hit a Ans. The following two points of Bohr’s model can be used
metal surface, an electron having 1.988 × 10–19J to derive the given formula :
of kinetic energy was emitted. Calculate the
(i) Electrons revolve around the nucleus in definite
ub
mu2
2 nth level is given as
er
1
or hν0 = hν – mu2 2 π 2 me4
2 En = −
n2 h2
od
600 × 10–9 m
2π2me4 ⎛ 2π2me4 ⎞
3.0 × 108 ms −1 = − −⎜− 2 2 ⎟
ν = = 5.0 × 1014 s–1 n22 h2 ⎜ n1 h ⎟⎠
600 × 10 −9 m ⎝
Since frequency of photon is less than v0, electron
2 π2 me4 ⎛ 1 1 ⎞
will not be emitted. = ⎜ 2 − 2⎟
h 2 ⎜ ⎟
⎝ n1 n2 ⎠
c ΔE = hν
Now, ΔE = hν = h = hc ν hc
λ = = hcν
λ
ΔE 2π 2 me4 ⎛ 1 1 ⎞ = (6.626 × 10–34 Js) (3.0 × 1010 cm s–1)
or ν = = ⎜ 2 − 2⎟
hc ch3 ⎝ n1 n2 ⎠ × (15232.9 cm–1)
.
= 3.028 × 10 J–19
ed
Substituting the values of c, h, π, m, e in CGS units, ν = cν
we get = (3.0 × 1010 cm s–1) × 15239.9 cm–1
⎛
−1 1 1⎞ = 4.57 × 1014s–1
rv
ν = 109677 cm ⎜ 2 − 2 ⎟
⎝ n1 n2 ⎠ Q.55. Why was a change in the Bohr model of atom
required? Due to which important development
se
Q.54. Calculate the energy and frequency of the
(s), concept of movement of an electron in an
radiation emitted when an electron jumps from orbit was replaced by the concept of probability
n = 3 to n = 2 in a hydrogen atom. of finding electron in an orbital ? What is the
re
⎛ 1 1⎞ name given to the changed model of atom ?
−1
Ans. ν = 109677 ⎜ 2 − 2 ⎟ cm Ans. According to Bohr’s model, electrons move along fixed
⎝ n1 n2 ⎠
circular paths called orbits. Therefore, position and
ts
For n1 = 2, n2 = 3 velocity of electron can be well defined. This was
contradicted by de Broglie concept of dual nature of
⎛ 1 1⎞ electron and Heisenberg’s uncertainty principle.
ν = 109677 ⎜ 2 − 2 ⎟
gh
⎝2 3 ⎠ Therefore, to incorporate the two principles, the well
5 defined orbits were replaced by regions of maximum
= 109677 × = 15232.9 cm–1 probability called orbitals. The changed concept
ri
36
formed the basis of wave mechanical model of atom.
ll
.A
rs
he
hydrogen like species: II. Read the following passage and answer questions
2π2 m e 4 Z 2 6–10 that follow:
n
This helps to calculate the radius of an orbit, various permitted values of quantum numbers are:
0.529 n 2 ° principal, n = 1, 2, 3, 4 .....
rn = A
M
Z azimuthal, l = 0, 1, 2 ..... (n – 1)
Bohr’s model also explains the occurrence of different magnetic, ml = –l .... 0..... + l
spectral lines. The wavelengths of different lines can be spin, ms = +1/2 and –1/2
©
9. What is the difference in angular momentum of an 7. The limiting line of any spectral series in the hydrogen
electron present in 2p and 3p orbital? spectrum is the line when n2 in the Rydberg’s formula
10. How many electrons are possible in all sub-shells is .................
with n + l = 4? 8. Lyman series of hydrogen spectrum lies in .................
III. Read the following passage and answer questions region.
11–15 that follow:
.
9. The quantum number which tells about the orientation
ed
The shapes of orbitals may be represented by boundary of different orbitals of an atom is called ................. .
surface diagrams. These boundary surface diagrams
10. The quantum number which describes the subshells
give the most probable regions. s-orbitals are
present in any main shell is called ................. .
rv
non-directional while p-, d-and f-orbitals have
different orientations given by ml values. These Assertion Reason Questions
boundary surfaces also have spherical nodes or radial
se
nodes and nodal planes or angular nodes which depend Note : In the following questions a statement of assertion
upon the .values of n and l. followed by a statement of reason is given. Choose
the correct answer out of the following choices.
re
11. How many orbitals are possible for l = 2 subshell?
(a) Assertion and reason both are correct statements and
12. How many spherical nodes are present in 3p-orbital?
reason is correct explanation for assertion.
13. How do 3s and 4s orbitals differ in terms of nodes (b) Assertion and reason both are correct statements but
ts
present in these? reason is not correct explanation for assertion.
14. Does dz2 orbital has zero electron density in xy plane? (c) Assertion is correct statement but reason is wrong
statement.
gh
15. How many angular nodes are present in 3dyz orbital?
(d) Assertion is wrong statement but reason is correct
True or False Questions statement.
Predict which of the following statements are true or false.
ri
1. Assertion : Hydrogen has one electron in its orbit but
1. 2s orbital has one node.
it produces several spectral lines.
2. Phosphorus (Z = 15) has three unpaired electrons. ll Reason : There are many excited energy levels
2. Fe3+ ion has four unpaired electrons. available.
4. Number of radial nodes in 5f orbital is four.
.A
2. Assertion : The 19th electron in potassium atom enters
5. It takes less energy to ionize (or remove) an electron into 4s-orbital and not in the 3d-orbital.
from first excited state than the ground state of Reason : (n + l) rule is followed for determining the
H-atom. orbital of lowest energy state.
rs
6. Paschen, Brackett and Pfund series fall in infra red (A.I.I.M.S. 1999)
region. 3. Assertion : The energy of an electron is largely
he
7. The splitting of spectral lines in electrical field is determined by its principal quantum
called Zeeman effect. number.
8. Electronic energy is negative because electron has Reason : The principal quantum number (n) is a
negative charge. measure of the probable distance of finding
lis
11. 3s and 4p orbitals have same number of radial orbital angular momentum is zero.
nodes. Reason : For 3s electron , l = 0 and orbital angular
12. Angular momentum for 2p and 3p orbital is same. momentum is zero.
P
13. Copper (I) is diamagnetic. 5. Assertion : The configuration of C cannot be 1s2 2s2 2p2x.
14. Spin quantum number is not originated from Reason : According to Pauli exclusion principle an
n
gen atom is larger than that of deuterium atom if Reason : Number of nodes in an orbital is equal to
both are travelling at the same speed. (n – l – 1) value.
7. Assertion : All microscopic bodies in motion have wave
od
electron simultaneously.
4. Dual character of electrons was verified by ................. Reason : The path of an electron in an atom is clearly
and ................. . defined.
5. The series in hydrogen spectrum falling in visible 10. Assertion : Photoelectric effect is easily given by
region is ................. series. cesium metal.
Reason : Photoelectric effect is easily given by the
6. The number of unpaired electrons in Fe2+ is ............ . metals having high ionization enthalpy.
.
5. Name the three quantum numbers which are
ed
One Word/Very Short Sentence Answer necessary to describe an orbital. What are the
permitted values for these ?
1. What is the difference between a quantum and a
photon? 6. What is the maximum number of electrons in :
rv
2. Can an electron have the quantum number values (i) a principal quantum number
as n = 2, l = 2 and m = + 2 ? (ii) an orbital
se
3. How many sub-levels are there in M shell ? What (iii) p-subshell
are their designations ?
(iv) s, p and d-subshells in an atom ?
4. What quantum numbers n and l are assigned to a
re
3p-orbital ? 7. Give the number of orbitals in :
5. Write the electronic configuration of chromium (i) a p-subshell
(Z = 24). (ii) a d-subshell
ts
6. An atom of an element has 19 electrons. What is (iii) second shell.
the total number of p-electrons ? 8. Describe the orbital with the following quantum
gh
7. What is the sequence of energies of 3s, 3p and numbers :
3d-orbitals in (i) n = 1, l = 0 (ii) n = 2, l = 1, m = 0
(i) a hydrogen atom, and (iii) n = 3, l = 2 (iv) n = 4, l = 1
ri
(ii) a multielectron atom ? (v) n = 3, l = 0, m = 0 (vi) n = 3, l = 1
8. According to which principle an atom cannot have 9. Give the electronic configurations of :
more than two electrons ? ll (i) Scandium (Z = 21)
9. What designations are given to the following (ii) Chromium (Z = 24).
.A
subshells having:
10. How many orbitals and how many electrons are
(i) n = 4, l = 3 there in each of the first two principal quantum
(ii) n = 3, l = 2 numbers ?
rs
10. If n = 3, what are values of quantum number l? 11. Give the electronic configurations of the following
11. What is the relation between the shapes of 3 dxy ions :
he
–
and 3dx 2 − y2 orbitals? (i) H (ii) Na+
–
12. How many electrons are present in 3d orbitals in (iii) F (iv) Mg2+.
chromium (Z = 24)? 12. Fill in the blanks :
lis
13. If the quantum number ‘l ’ has a value of 2, what are (i) The size of an orbital is dependent on the value
the permitted values of quantum number m ? of ......
ub
14. How many electrons s and p-subshell can (ii) The orbitals having the same energies are
accommodate ? called ......
15. Which energy levels do not have p-orbital ? (iii) The number of unpaired electrons in carbon is
P
16. What do you mean by saying that energy of electron ...... and in nitrogen is ......
is quantized ? (iv) The shape of 1s-orbital is .............. .
n
17. Out of electron and proton which one will have, a (v) ...... filled orbitals have extra stability.
higher velocity to produce matter waves of the same
er
19. Write the electronic configurations of the following 2. What are quantum numbers ? What permitted
elements : values can these have ? Give the significance of each
Carbon (Z = 6), neon (Z = 10), magnesium quantum number.
(Z = 12), chlorine (Z = 17), calcium (Z = 20), chromium
(Z = 24), iron (Z = 26) and rubidium 3. What is photoelectric effect ? State the result of
(Z = 37). photoelectric effect experiment that could not be
.
20. Identify the atoms that have the following ground explained on the basis of laws of classical physics.
ed
state electronic configurations : Explain this effect on the basis of quantum theory
(i) 1s2 2s2 2p6 3s2 of electromagnetic radiations.
rv
(ii) 1s2 2s2 2p5 4. Why was a change in the Bohr Model of atom
(iii) 1s2 2s2 2p6 3s2 3p6 4s1 3d10. required? Due to which important development (s),
se
21. In building up of the atoms, the filling of 4s-orbital concept of movement of an electron in an orbit was
takes place before the 3d-orbital. Explain. replaced by, the concept of probability of finding
22. The expected electronic configuration of copper is electron in an orbital ? What is the name given to
re
[Ar] 3d9 4s2 though actually it is [Ar] 3d10 4s1. Give the changed model of atom?
reasons. 5. (a) What is radial probability distribution curve?
23. An atom of an element has 19 electrons. Find out : Draw radial probability distribution curves for
ts
(a) its atomic number 1s and 2s orbitals.
(b) total number of s-electrons (b) Discuss the similarities and differences between
gh
(c) total number of p-electrons. 1s and 2s orbitals.
24. Give the shapes of s and p-orbitals.
(c) How many nodes are present in 1s and 2s
25. What is Hund’s rule of maximum multiplicity ?
ri
orbitals?
Illustrate this by taking the example of carbon.
26. Explain why half-filled and completely-filled 6. (a) How many orbitals are possible for a d-subshell?
orbitals have extra stability.
ll (b) Draw the shapes of dxy and dx2 − y2 orbitals?
27. What is Aufbau principle ? Write the electronic
.A
configurations of the elements of atomic numbers What is common between these and what is
16, 20, 24 and 35. difference between these orbitals? What is the
28. Draw the shape of an orbital which has l = 0. angle between the lobes of these orbitals?
rs
29. State Aufbau principle. Write electronic (c) Name a 3d orbital which has electron density
configurations of the elements with atomic numbers along all the three axes.
he
17 and 24.
30. Why in the building of the atom, the filling of
lis
34. What is the experimental evidence in support of transition from 4th energy level to the 2nd energy
the idea that electronic energies in an atom are level.
er
for motion of microscopic particles and is negligible 656·4 nm was obtained. Calculate the number of
for the macroscopic particles. Justify the statement higher orbit from which the electron drops to
with the help of a suitable example. produce this line.
M
(ii) origin of spectral lines in hydrogen atom ? 6. Calculate the value of Δx · Δv for an electron (mass
of electron = 9.1 × 10–31 kg).
.
Passage Based Questions 13. True
ed
13.6 14. True
1. En = − eV
n2 15. True.
rv
Energy of first excited state (n = 2),
Fill in the Blanks Questions
13.6
E2 = − eV = −3.4 eV
se
22 1. 2f 2. Nine 3. Five
2. Balmer series.
4. Davisson and Germer, Thomson
0.529 n2 °
3. rn = A 5. Balmer 6. 4 7. 6
re
Z
0.529 × 16 8. uv
r4 (H) = 9. magnetic quantum number
1
ts
0.529 × 9 10. azimuthal quantum number.
2+
r3 (Li ) = (∵ Z = 3)
3 Assertion Reason Questions
gh
r4 (H) 16
= 1. (a) 2. (a) 3. (a) 4. (a) 5. (b)
r3 (Li 2+ ) 3
6. (a) 7. (c) 8. (c) 9. (c) 10. (c)
4. Between 4th orbit to 1st orbit.
ri
5. Line spectra Very Short Answer Questions
6. Possible
7.
8.
5
Nine
ll 1. The smallest packet of energy of any radiation is
called a quantum while that of light is called photon.
.A
9. No difference because angular momentum depends
only upon value of l. 2. No
h 3. Three, 3s, 3p, 3d
rs
Angular momentum = l (l + 1)
2π 4. n = 3, l = 1
5. 1s2 2s2 2p6 3s2 3p6 4s1 3d5
The value of l (1) is same for 2p and 3p orbital.
he
6. 12
10. Subshells with n + l = 4 are 4s and 3p. Hence
7. (i) 3s = 3p = 3d
electrons present in these subshells are 2 + 6 = 8.
(ii) 3s < 3p < 3d
lis
11. Five
12. Nodes = n – l – 1 8. Pauli’s exclusion principle
=3–1–1=1 9. (i) 4f (ii) 3d
ub
3. False. Fe 3+
ion has five unpaired electrons.
13. m = – 2, – 1, 0, +1, +2
4. False. (n – l – 1) i.e. (5 – 3 – 1) = 1 node. 14. 2, 6
od
.
17. (i) 1s (ii) 2pz (iii) 3d (iv) 4p (v) 3s (vi) 3p For Balmer series n1 = 2, λ = 656.4 × 10–9 m
ed
18. (i) 1s2 2s2 2p6 3s2 3p6 3d1 4s2 = 656.4 × 10–7 cm
(ii) 1s2 2s2 2s6 3s2 3p6 3d5 4s1
rv
20. (i) 1s2 (ii) 1s2 2s2 2p6 1 ⎡1 1 ⎤
−7 = 109678 ⎢ − ⎥ cm −1
(iii) 1s2 2s2 2p6 656.4 ×10 cm ⎢⎣ 2
2
n22 ⎥⎦
se
21. (i) principal quantum number
(ii) degenerate 1 1 1
or − =
(iii) two, three 4 n22 109678 × 656.4 ×10−7
re
(iv) spherical
1 1 1
(v) half-filled and completely. or − =
4 n22 7.20
22. 2s, 2p
ts
27. [Ar] 3d10 4s1 ; one. 1 1 1
or – = –
28. 1s2 2s2 2p2, 1s2 2s2 2p6, 1s2 2s2 2p6 3s2, 1s2 2s2 2p6 3s2 n22 7.20 4
gh
3p5,1s2 2s2 2p6 3s2 3p6 4s2,
4 − 7.20 3.20
1s2 2s2 2p6 3s2 3p6 3d5 4s1, 1s2 2s2 2p6 3s2 3p6 3d6 4s2, = =−
ri
1s2 2s2 2p6 3p6 3d10 4s2 4p6 5s1. 28.80 28.80
29. (i) Magnesium 1 3.20 28.80
ll or = or n22 = =9
(ii) fluorine n2 2 28.80 3.20
(iii) copper.
.A
n2 = 9 = 3. Third energy level.
32. (a) 19 (b) 7 (c) 12.
36. 1s2 2s2 2p6 3s2 3p4, 1s2 2s2 2p6 3s2 3p6 4s2, 1s2 2s2 2p6 4. For transition from ni = 6 to nf = 4,
rs
4s1 3d5, 1s2 2s2 2p6 3s2 3p6 4s2 3d10 4p5. ⎛1 1 ⎞
ΔE = 2.18 × 10–18 J ⎜⎝ 2 − 2 ⎟⎠
40. 1s2 2s2 2p6 3s2 3p5, 1s2 2s2 2p6 3s2 3p6 3d5 4s1 6 4
he
= –7.57 × 10–20 J
1. According to Rydberg equation, It is emission energy. The frequency of the photon
(taking magnitude) of energy,
ub
1 ⎛ 1 1 ⎞ −1
= 109678 ⎜⎜ 2 − 2 ⎟⎟ cm
λ n
⎝ 1 n2 ⎠ ∆E 7.57 × 10−20
ν = = = 1.142 × 1014 s −1
P
∴ ⎜ 2 ⎟ or λ = = = 2.63 × 10–6 m
λ ⎝1 42 ⎠ ν 1.142 × 1014
er
15 or = 2.63 × 103 nm
= 109678 × cm −1
16
od
h
16 5. λ=
or λ = = 9.72 × 10 cm –6 mν
109678 ×15cm−1
6.626 × 10−34 Js
M
= 97.2 × 10–9 m =
(1.676 × 10−27 kg) × (8 m s −1 )
= 97.2 nm
= 4.94 × 10–8 m = 49.4 nm
©
1 ⎛ 1 1 ⎞
2. ν = = 109678 ⎜⎜ 2 − 2 ⎟⎟ cm–1
h 6.626 × 10−34
λ ⎝ n1 n2 ⎠ 6. Δx · Δv = =
4πm 4 × 3.14 × 9.1 × 10−31
Here n1 = 2, n2 = 4
1 = 5.797 × 10–5 m2 s–2
⎛ 1 1 ⎞
∴ ν = λ = 109678 ⎜ 2 − 2 ⎟ cm–1
⎝2 4 ⎠
.
overruled the circular orbits proposed by Bohr ? h
ed
λ =
Ans. The wave mechanical model of an atom was proposed p
by Schrodinger in terms of mathematical equation known as
But, λ = Δx (given)
Schrodinger wave equation. The solution of this equation
rv
gives wave function ψ and the square of the wave function, i.e., h
∴ Δx =
ψ2 measures the probability density of finding the electron. p
se
So, according to this model, it is possible to locate the regions
around the nucleus where the probability of finding the electron According to Heisenberg uncertainty principle,
is maximum. These regions where the probability of finding h
re
Δx . Δp ≥
the electron is maximum are called orbitals. Thus, wave 4π
mechanical model leads to the concept of orbital.
h h
This concept overruled the idea of circular orbits proposed .Δp ≥
ts
by Bohr. According to Bohr’s model of an atom, the electrons p 4 π
revolve around the nucleus in certain well defined circular h h
gh
paths called orbits. This means that electron remains only in or . m Δv ≥
mv 4π
the circular orbits at a definite distance from the nucleus.
However, according to wave mechanical model, there are Δv 1
or ≥
ri
regions where the probability of finding the electrons is v 4π
maximum. The electron may be even outside this regions. v
ll or Δv ≥
Q.2. Which of the following sets of orbitals are 4π
degenerate and why ? Thus, uncertainty in velocity is so large that its velocity
.A
(i) 1s, 2s, and 3s in Mg atom is uncertain.
(ii) 2px, 2py and 2pz in C atom Q.5. Show that ground state energy of an electron
(iii) 3s, 3px and 3d orbitals in H atom . in H-atom is equal to the first excited state energy of
rs
Ans. (i) 1s, 2s and 3s orbitals in Mg atom are not electron in He+ ion (assuming their Rydberg’s constants
degenerate because these have different values to be equal).
he
n
(iii) 3s, 3px and 3d orbitals in H atom are degenerate
Z2
because for H atom, the subshells having same = −13.60
eV
n2
ub
2
(ii) It takes more energy to ionize (remove) the
Q.6. What is the difference in the orbital angular
er
electron from n = 4 than in the ground state. momentum of 2p and 3p electron ? Explain.
(iii) The wavelength of light emitted when the Ans. Orbital angular momentum is given as :
od
going from n = 1 to n = 4 is the same as emitted when Since it is independent of the principal quantum number
it goes from n = 4 to n = 1. ‘n’ and depends only on the azimuthal quantum number ‘l’, it
(v) The electron is farther from the nucleus (on will be same for 2p and 3p electron. Hence, there will be no
©
.
associated with a moving particle if its velocity is
ed
doubled ?
Ans. Wavelength becomes half of the original value
rv
h
(λ = ).
mv
se
Q.9. Cu 2+ is more stable than Cu + in aqueous Q.12. For a multielectron atom, the maximum of
solution. Explain. 2p-orbital in radial probability distribution graph is
Ans. Cu+ has outermost electronic configuration as 3d10 nearer the nucleus than that of 2s-orbital. Therefore,
re
and therefore, should be most common and most stable state 2p-orbital should be closer to the nucleus and lower in
energy than 2s-orbital. But 2s-orbital has lower energy
because of extra stability associated with completely filled
than 2p-orbital. Explain.
d-subshell (d10). However, this is not true and Cu2+ with outer
ts
Ans. The radial probability distribution graphs for 2s-
electronic configuration 3d9 is more stable than Cu+. This is
and 2p- orbitals are shown below :
due to high hydration energy in aqueous solution and
gh
high lattice energy in solid state of Cu2+ as compared to
Cu+. The equilibrium :
ri
2 Cu+ Cu2+ + Cu
⎡Cu 2+ ⎤ ll
K= ⎣ ⎦ = 1.6 × 106
2
⎡Cu+ ⎤
.A
⎣ ⎦
The constant for disproportionation of Cu+ in aqueous
solution shows that Cu2+ is very stable in aqueous solution
rs
as compared to Cu+. It is clear from the figure that in case of 2s-orbital, there
is a small additional peak or hump. This indicates that a
Q.10. How do dx 2 − y 2 and dxy orbitals differ in their
he
the lobes of dxy lie in between X and Y axis (i.e. at an angle of nuclear charge. As a result, a 2s-electron is attracted more
strongly by the nucleus than a 2p-electron. Thus, 2s has lower
45°). So, d x 2 − y 2 orbital is exactly like dxy except that it is energy than a 2p-electron.
ub
rotated through 45° around Z-axis. 13. For which hydrogen like ion the wavelength
difference between the first lines of the Lyman and
Q.11. For H atom the Bohr radius for first orbit is Balmer series is equal to 59.3 nm ?
P
0.529Å and the radius of maximum probability for Ans. For a spectral line,
H-atom according to wave mechanical model is also
LM 1 OP
n
found moving around the nucleus in a circular path of radius For Lyman line,
0.529 Å. According to his model, electron cannot be found at LM 1 − 1 OP
od
1 3RZ 2
distance less than or more than 0.529 Å. However according λ Lyman
= RZ2
N1 2 Q
2 2 =
4
to wave mechanical model, electron is most likely to be found
M
λBalmer – λLyman =
36
–
4
FG 22 IJ × e9.1 × 10 j × e4.8 × 10 j
2
−10 4
H7K
−28
2 2 4×
5 RZ 3 RZ
=
1 LM 36 − 4 OP 33 × (6.626 × 10 −24 )3
⇒ 59.3 × 10–7 cm =
RZ 2 N 5 3Q = 2.42 × 1014 .
.
ed
1
⎡ 108 − 20 ⎤ 16. Calculate the energy emitted when electrons
⇒ 59.3 × 10–7 = ⎢ 2 ⎥ of 1.0 g atom of hydrogen undergo transition giving the
109677.8 Z ⎣ 15 ⎦
spectral line of largest energy in the visible region of
= 59.3 × 10–7
rv
its atomic spectrum. (RH = 1.1 × 107 m–1, c = 3 × 108 m s–1,
1 88 h = 6.62 × 10–34 J s)
or Z2 = × =9
109677.8 15 × 59.3 × 10−7
se
Ans. The spectral line in visible region corresponds to
or Z = 3. This corresponds to Li2+ ion. Balmer series i.e. n1 = 2 and n2 = 3 for lowest energy.
Q. 14. Calculate the wavelength of radiation
F1 I
re
emitted producing a line in Lyman series when an
electron falls from fourth energy level in hydrogen Now, ν = RH GH n 1
2
−
1
n22
JK
atom. (RH = 1.1 × 107 m–1)
FG 1 − 1 IJ m
ts
Ans. According to Rydberg equation,
F 1 − 1I = 1.1 × 107 H2 3 K
2 2
–1
1
GH n n JK
gh
= RH
FG 1 − 1IJ
2 2
λ 1 2
For Lyman series, n1 = 1 and n2 = 4 = 1.1 × 107 H 4 9K
ri
1 ⎛1 1 ⎞ −1
∴ = 1.1 × 107 ⎜⎝ 2 − 2 ⎟⎠ m 5
λ 1 4 = 1.1 × 107 ×
ll 36
1 7 15
= 1 .1 × 10 ×
.A
λ 16 36
∴ λ =
. × 107 × 5
11
1 × 16
or λ = = 9.70 × 10–8 m = 6.55 × 10–7 m
1.1 × 107 × 15
rs
or λ = 97.0 nm. hc
Now, E = hν =
Q.15. Calculate the velocity (cm s–1) of an electron λ
he
Now, e = 4.8 × 10–10 e.s.u., n = 3, h = 6.63 × 10–27 erg sec ∴ Energy corresponding to 1 gram atom of
H = 3.03 × 10–19 × 6.02 × 1023
P
2 × 22 × (4.8 × 10 −10 )2
∴ v = = 18.25 × 104 J
7 × 3 × 6.63 × 10 −27
n
= 182.5 kJ
= 7.27 × 107 cm s–1
er
Circumference
Ans. Radius of nth orbit of H-atom is given as :
v
= rn = 0.529 × n2 Å
2πr
M
Ans. According to Heisenberg’s uncertainty principle, K.E. of electrons emitted by using λ = 3 × 103Å or 3000Å
h ⎛ 1 1 ⎞
Δx × Δp ≥ (K.E.)1 = hc ⎜ − (... λ0 = 6000 Å)
4π ⎝ 3000 6000 ⎟⎠
h
or Δx × m Δ v ≥ (... m is constant) K.E. of electrons by using wavelength λ (to be calculated)
4π
.
ed
(K.E.)2 = hc ⎛⎜ −
1 1 ⎞
⎝ λ 6000 ⎟⎠
h
or Δx × Δv ≥
4 mπ
Now, (K.E.)2 = 2 (K.E.)1
rv
m = 9.1 × 10–31 kg, h = 6.6 × 10–34kg m2 s–1, π = 3.14
⎛1 1 ⎞ ⎛ 1 1 ⎞
hc ⎜ − = 2hc ⎜ −
6.6 × 10 −34 kg m 2s−1 ⎝ λ 6000 ⎠⎟ ⎝ 3000 6000 ⎟⎠
se
∴ Δx × Δv ≥
( )
4 × 9.1 × 10−31 kg × 3.14
1 2 1 1 1
6.6 or = − + =
re
or Δx × Δv ≥ × 10 −3 m 2s −1 λ 3000 3000 6000 2000
4 × 9.1 × 3.14
∴ λ = 2000Å
or Δx × Δv ≥ 5.77 × 10–5 m2 s–1
Q.21. The angular momentum of an electron in
ts
Thus, product of uncertainties in position and velocity
Bohr’s orbit of H-atom is 3.02 × 10–34 kg m2 s–1. Calculate
of an electron is equal to or greater than 5.77 × 10–5 m2 s–1.
the wavelength of the spectral line emitted when the
gh
Q.19. Which state of the triple ionized beryllium electron jumps from this level to the next lower level.
(Be3+) has the same orbit radius as that of the ground Ans. Angular momentum (mvr)
state of hydrogen atom ?
ri
h
Ans. For H-atom, radius of ground state is = n = 3.02 × 10 −34 kg m 2 s −1
2π
r1 =
h2
....(i)
ll n = 3.02 × 10 −34 ×
2π
4 π 2me2
.A
h
For hydrogen like atom,
3.02 × 10−34 × 2 × 3.14
n2 h2 = =3
6.3 × 10−34
rs
r′n = ....(ii)
4 π 2mZe2
When the electron jumps from n = 3 to n = 2, the
Dividing Eq. (ii) by Eq. (i)
he
λ ⎝ 2 n1 ⎠
For Be3+ ion, Z = 4
⎛ 1 1 ⎞
= 109677 ⎜ 2 − 2 ⎟
ub
rn′ n2 ⎝2 3 ⎠
∴ =
r1 4
1 5
Now, r′n = ri = 109677 ×
P
λ 36
∴ n2 = 4 or n = 2
n
36
Thus, the second orbit of Be3+ has the same radius as or λ =
5 × 109677
the Bohr’s radius of hydrogen atom.
er
Q.20. The threshold wavelength for emitting = 6.56 × 10–5 cm or 656 nm.
photons from a metal is 6.0 × 103Å. What would be the Q.22. An electron in certain Bohr orbit has velocity
od
wavelength of radiation to produce photoelectrons 1/275 of the velocity of light. Calculate the orbit in which
having twice the kinetic energy of those produced by the electron is revolving.
radiation of wavelength 3 × 103 Å ?
M
hc hc 275
K.E. = hν − hν0 = −
λ λ0 = 1.09 × 108 cm s–1
.
n ∴ n = 2
ed
rv
se
re
ts
BOHR MODEL: RADIUS OF ORBIT, ENERGY AND n2 h2
VELOCITY OF AN ELECTRON rn = ...(v)
4 π 2me2
gh
According to Bohr model, an electron with charge
Energy of Electron
–e revolves around the nucleus in an orbit of radius r. Let
the charge on the nucleus be +Ze. For an electron to remain The total energy, E of electron revolving in the
nth orbit is equal to the sum of the kinetic
ri
in its orbit, the electrostatic attraction between the electron
and the nucleus which tends to pull the electron towards ⎛1 ⎞
energy ⎜ mv2 ⎟ and the potential energy
the nucleus must be equal to the centrifugal force which ll ⎝2 ⎠
tends to throw the electron out of its orbit. ⎛ − Ze2 ⎞
mv2 ⎟⎟ . Thus,
.A
⎜⎜
Centrifugal force = ⎝ rn ⎠
r
Coulombic force of attraction between nucleus of charge 1 Ze2
+Ze and electron of charge –e is E = mv2 – ...(vi)
rs
2 rn
2
Ze × e Ze From eq. (i)
Coulombic force of attraction = =
r 2
r2
he
r r2
Ze2 Ze2 Ze2
Ze2 E = – = − ...(vii)
Hence, v2 = ...(i) 2rn rn 2rn
mr
ub
Radius of Orbit, Substituting the value of rn from eq. (v) in eq. (vii), we
get
According to Bohr’s model of an atom, angular
P
nh
or v = ...(ii)
2πmr 2π 2me4 Z 2
En = – ...(viii)
n2 h2 n 2 h2
od
Ze2 n2 h2 He , Li , etc.)
=
mr 4 π 2 m2 r 2 Velocity of Electron in an Orbit
n2 h2 Substituting the value of r from eq. (iv) in eq. (ii),
©
or Ze2 = 2 we get
4 π mr
nh 4π2me2 Z
n2 h2 v = ×
or r = ...(iv) 2 πm n2h2
4 π 2me2 Z
Since for hydrogen atom, Z = 1, hence the radius of nth 2πe2 Z
or v = ...(ix)
orbit of H-atom is nh
.
ise MULTIPLE CHOICE QUESTIONS
ed
A10. The ratio of the difference in energy between the first
icw and the second Bohr orbit to that between second and
Top third Bohr orbit is
rv
1 1
(a) (b)
Select the Correct Answers : 2 3
27 4
se
Bohr’s Model and its Limitations (c) (d)
5 9
A1. In the Bohr’s orbit, the ratio of total kinetic energy and A11. Which of the following transitions will have minimum
the total energy of the electron is wavelength ?
re
(a) –2 (b) –1 (a) n2 ⎯→ n1 (b) n3 ⎯→ n1
(c) +2 (d) 0 (c) n4 ⎯→ n2 (d) n4 ⎯→ n1
A2. The spectral line in hydrogen spectrum obtained when A12. In hydrogen atom, energy of the first excited state is
ts
the electron jumps from n = 5 to n = 2 energy level
–3.04 eV. Then find out the K.E. of the same orbit of
belongs to :
H-atom.
(a) Lyman series (b) Balmer series
gh
(a) + 3.4 eV (b) + 6.8 eV
(c) Paschen series (d) Pfund series.
(c) – 13.6 eV (d) + 13.6 eV
A3. The energy of the electron in the nth orbit of hydrogen A13. The ratio of radii of the first three Bohr orbits of H-atom
ri
atom is given as : is
1311.8
En = – kJ mol–1 (a) 1 : 2 : 3 (b) 1 : 4 : 9
n2 ll (c) 1 : 3 : 27 (d) 1 : 2 : 3
What is the energy emitted per atom when an electron
A14. The ratio of the frequency corresponding to the third line
.A
jumps from third energy level to second energy level ?
in Lyman series of hydrogen atomic spectrum to that of
(a) 329·7 kJ (b) 3·03 × 10–19 J the first line in Balmer series of Li2+ spectrum is
(c) 182·2 kJ (d) 145·7 kJ.
4 5
rs
E1
A6. The radius of first orbit of hydrogen is 0.53 Å. The
Which one of the following relationship is correct?
radius of second orbit would be :
(a) λ3 = λ1 + λ2 (b) λ1 + λ2 + λ3 = 0
P
(a) 5 (b) 10
Numbers
(c) 15 (d) 4
A8. In Bohr’s theory, the radius, r of orbit is proportional to A16. The de-Broglie wavelength of an electron is 600 nm.
od
A1. (b) A2. (b) A3. (b) A4. (a) A5. (d) A6. (d) A7. (b) A8. (b) A9. (c) A10. (c)
A11. (d) A12. (a) A13. (b) A14. (d) A15. (c) A16. (d) A17. (b)
A18. If uncertainty in the position of an electron is zero, the A29. The orbital angular momentum for an electron revolving
uncertainty in its momentum would be in an orbit for an s-electron is
.
h
(a) zero (b) ≥
h (a) zero (b)
ed
2π
4π
h h 1. h
(c) < (d) infinite (c) 2 (d)
4π 2π 2 2π
rv
A19. Which of the following orbitals does not make sense ? A30. The following quantum numbers are possible for how
(a) 6s (b) 3p
many orbitals ?
se
(c) 2d (d) 4f.
A20. Two electrons occupying the same orbital are n = 3, l = 2, ml = +2
distinguished by : (a) 1 (b) 2
re
(a) spin quantum number
(c) 3 (d) 4
(b) azimuthal quantum number
(c) magnetic quantum number Shapes of Orbitals and Electronic Configurations
ts
(d) principal quantum number.
A21. The designation of an orbital with n = 4 and A31. The correct ground state electronic configuration of
l = 3 is : chromium atom (Z = 24) is :
gh
(a) 4s (b) 4p (a) [Ar] 3d5 4s1 (b) [Ar] 3d4 4s2
(c) 4d (d) 4f. 6
(c) [Ar] 3d 4s 0 (d) [Ar] 4s1 4p5.
A22. For each value of l, the number of ms values are :
ri
A32. In manganese atom, Mn (Z = 25), the total number of
(a) 2l (b) nl orbitals populated by one or more electrons (in ground
(c) 2l + 1 (d) n – l. ll state) is :
A23. The maximum number of 4d-electrons having spin (a) 15 (b) 14
1
.A
quantum number s = + are : (c) 12 (d) 10.
2
(a) 10 (b) 7 A33. Which of the following has maximum number of
(c) 1 (d) 5. unpaired electrons ?
rs
A24. The maximum number of electrons in a subshell is (a) Mg2+ (b) Ti3+
given by the expression : (c) V3+ (d) Fe3+.
(a) 4l – 2 (b) 4l + 2
he
(c) 2l + 1 (d) 2n2. A34. Azimuthal quantum number for the last electron in
Na atom is :
A25. A subshell with n = 6, l = 2 can accommodate a
maximum of : (a) 1 (b) 0
lis
numbers an electron will have the highest energy : (c) n and m (d) m and s
n l m s A36. How many electrons in Argon have m = 0 ?
(a) 3 2 1 1/2
P
(a) 12 (b) 10
(b) 4 1 0 –1/2
(c) 8 (d) 6.
(c) 4 2 –1 1/2
n
2
(a) 3s (b) 3p A38. Consider the following ions :
(c) 4d (d) 4f 1. Ni2+ 2. Co2+ 3. Cr2+ 4. Fe3+
A28. Which of the following orbital designations is not (Atomic number : Cr = 24, Fe = 26, Co = 27 and
M
(c) n = 4, l = 3 ⎯⎯→ 4f
(a) 1, 2, 3, 4 (b) 4, 2, 3, 1.
(d) n = 7, l = 2 ⎯⎯→ 7p
(c) 1, 3, 2, 4 (d) 3, 4, 2, 1
A18. (d) A19. (c) A20. (a) A21. (d) A22. (c) A23. (d) A24. (b) A25. (a) A26. (c) A27. (d)
A28. (d) A29. (a) A30. (a) A31. (a) A32. (a) A33. (d) A34. (b) A35. (a) A36. (b) A37. (a)
A38. (a)
A39. If the value of (n + l) is more than 3 and less than 6, A43. The number of radial and angular nodes in 3p orbital
what will be the possible number of orbitals are
.
(a) 6 (b) 9 (a) 1, 0 (b) 2, 1
ed
(c) 10 (d) 13 (c) 1, 1 (d) 2, 0
A40. In an atom, the signs of lobes indicate the A44. Consider the ground state of Cr atom (Z = 24). The
number of electrons with azimuthal quantum numbers
rv
(a) sign of charges
l = 1 and 2 are respectively
(b) sign of probability distribution
(a) 16 and 4 (b) 12 and 5
se
(c) sign of wave function
(c) 12 and 4 (d) 16 and 5
(d) presence or absence of electron
A45. Which of the following 3d-orbital has electron density
A41. The number of radial nodes possible for 3d orbital is
re
in all the three axes?
(a) 3 (b) 1 (a) 3 dxy (b) 3 dx 2 − y2
(c) 2 (d) 0 (c) 3dz2 (d) 3dyz
ts
A42. The radial part of wave function depends on the A46. How many spherical nodes are present in 4s orbital in
quantum numbers a hydrogen atom?
(a) n, l (b) n only
gh
(a) 0 (b) 2
(c) l, ml (d) l only (c) 3 (d) 4
ri
A39. (d) A40. (c) A41. (d) A42. (a) A43. (c) A44. (b)
ll A45. (c) A46. (c)
.A
1 h h
(a) (b)
B MULTIPLE CHOICE QUESTIONS 2m π 2π
from competitive examinations
1 h h
rs
(c) (d)
AIPMT, NEET & Other State Boards’ m π π
Medical Entrance (C.B.S.E. P.M.T. 2008)
he
(a) A, B, C and D (b) B, D and E of particle A, then the ratio of de-Broglie wavelength
(c) A and C (d) B, C and D associated with particles A and B is
n
(a) 2 : 5 (b) 3 : 4
(C.B.S.E. Med. 2007)
(c) 6 : 4 (d) 4 : 3
er
B2. The maximum kinetic energy of photoelectrons ejected (e) 5 : 2 (Kerala PMT 2009)
from a metal, when it is irradiated with radiation of
B6. Maximum number of electrons in a subshell of an atom
od
B8. Which one of the following ion has electronic configura- (d) Larger the value of n, the larger is the orbit radius.
tion [Ar] 3d6 ? (NEET 2013)
(a) Fe3+ (b) Co3+
.
B16. The value of Planck's constant is 6.63 × 10–34 J s. The
ed
(c) Ni3+ (d) Mn3+
speed of light is 3 × 1017 nm s–1. Which value is closest
(C.B.S.E. Med. 2010)
to the wavelength in nanometer of a quantum of light
B9. The total number of atomic orbitals in fourth energy
with frequency of 6 × 1015 s–1 ?
rv
level of an atom is
(a) 8 (b) 16 (a) 50 (b) 75
(c) 32 (d) 4 (AIPMT 2011) (c) 10 (d) 25 (NEET 2013)
se
B10. The energies E1 and E2 of two radiations are 25 eV and B17. The ratio of de Broglie wavelengths of a deuterium
50 eV respectively. The relation between their atom to that of an α-particle, when the velocity of the
former is five times greater than that of the latter, is
re
wavelengths i.e., λ1 and λ2 will be
(a) λ1 = λ2 (b) λ1 = 2λ2 (a) 4 (b) 0.2
(c) 2 (d) 0.4
1
(c) λ1 = 4λ2 (d) λ1 = λ 2 (AIPMT 2011) (e) 5 (Kerala PMT 2014)
ts
2
B18. The uncertainty in the velocity of a particle of mass
B11. If n = 6, the correct sequence for filling of electrons will be
6.626 × 10–31 kg is 1 × 106 m s–1. What is the uncertainty
gh
(a) ns → (n – 2)f → (n – 1)d → np in its position (in nm)? (h = 6.626 × 10–34 J s)
(b) ns → (n – 1)d → (n – 2)f → np
⎛ 1⎞ ⎛ 2.5 ⎞
(c) ns → (n – 2)f → np → (n – 1)d (a) ⎜ ⎟ (b) ⎜
⎝ π ⎟⎠
ri
⎝ 2π ⎠
(d) ns → np(n – 1)d → (n – 2)f (AIPMT 2011)
⎛ 1⎞
(c) ⎛⎜ ⎞⎟
B12. Maximum number of electrons in a subshell with 4
(d) ⎜ ⎟
ll ⎝ π⎠ ⎝ 4π ⎠
= 3 and n = 4 is
⎛ 5⎞
.A
(a) 14 (b) 16 (c) 10 (d) 12
(e) ⎜⎝ ⎟⎠ (Kerala PMT 2014)
(A.I.P.M.T. 2012) π
B19. What is the maximum number of orbitals that can be
B13. The correct set of four quantum numbers for the identified with the following quantum numbers?
rs
wavelength 45 nm.
be associated with the following set of quantum (Planck’s constant, h = 6.63 × 10–34 J s, speed of light,
numbers ? c = 3 × 108 m s–1)
ub
⎜n ⎟
⎝ ⎠ (a) n = 3, l = 3, ml = −3, ms = +1/2
conclusions are written. Which of them is not correct ?
er
(b) n = 2, l = 1, ml = 2, ms = −1/2
(a) Equation can be used to calculate the change in
energy when the electron changes orbit. (c) n = 2, l = 0, ml = 0, ms = +1/2
od
(d) n = 1, l = 0, ml = 0, ms = 0
(b) For n = 1, the electron has a more negative energy
than it does for n = 6 which means that the electron (e) n = 3, l = 2, ml = 3, ms = −1/2
is more loosely bound in the smallest allowed orbit. (Kerala PMT 2015)
M
(c) The negative sign in equation simply means that B22. The angular momentum of electron in 'd' orbital is
the energy of electron bound to the nucleus is lower equal to ?
©
B8. (b) B9. (b) B10. (b) B11. (a) B12. (a) B13. (c) B14. (b) B15. (b) B16. (a) B17. (d)
B18. (d) B19. (a) B20. (d) B21. (c) B22. (c)
B23. Which is the correct order of increasing energy of the B30. 4d, 5p, 5f and 6p orbitals are arranged in the order of
listed orbitals in the atom of titanium ?
decreasing energy. The correct option is
(At. no. = 22)
.
(a) 5f > 6p > 4d > 5p (b) 5f > 6p > 5p > 4d
ed
(a) 3s 3p 3d 4s (b) 3s 3p 4s 3d
(c) 6p > 5f > 5p > 4d (d) 6p > 5f > 4d > 5p
(c) 3s 4s 3p 3d (d) 4s 3s 3p 3d
(AIPMT 2015) (NEET 2019)
rv
B24. The number of d–electrons in Fe2+ (Z = 26) is not B31. Which of the following series of transitions in the
equal to the number of electrons in which one of the spectrum of hydrogen atom falls in visible region?
following? (a) Brackett series (b) Lyman series
se
(a) d-electrons in Fe (Z = 26) (c) Balmer series (d) Paschen series
(b) p-electrons in Ne (Z = 10) (NEET 2019)
(c) s-electrons in Mg (Z = 12)
re
(d) p-electrons in Cl (Z = 17) (AIPMT 2015) JEE (Main) & Other State Boards’
Engineering Entrance
B25. The electrons occupying the same orbital are
distinguished by B32. Which of the following sets of quantum numbers
ts
(a) azimuthal quantum number represents the highest energy of an atom ?
(b) spin quantum number (a) n = 3, l = 2 , m = 1, s = +1/2
gh
(c) principal quantum number (b) n = 4, l = 0 , m = 0, s = +1/2
(d) magnetic quantum number (NEET 2016) (c) n = 3, l = 0 , m = 0, s = +1/2
B26. How many electrons can fit in the orbital for which (d) n = 3, l = 1 , m = 1, s = +1/2 (A.I.E.E.E. 2007)
ri
n = 3 and l = 1? B33. The ionization enthalpy of hydrogen atom is 1.312 × 106
(a) 2 (b) 6 ll J mol–1. The energy required to excite the electron in the
(c) 10 (d) 14 (NEET 2016) atom from n = 1 to n = 2 is
.A
B27. Which of the following pairs of d-orbitals will have (a) 9.84 × 105 J mol–1 (b) 8.51 × 105 J mol–1
electron density along the axes?
(c) 6.56 × 105 J mol–1 (d) 7.56 × 105 J mol–1
(a) dz2 , dxz (b) dxz, dyz (A.I.E.E.E. 2008)
rs
(c) dz2 , dx2 − y2 (d) dzy , dx2 − y2 (NEET 2016) B34. In an atom, an electron is moving with a speed of 600
m/s with an accuracy of 0.005%. Certainty with which
B28. Which one is the wrong statement?
he
stability due to greater exchange energy, greater (c) 1.92 × 10–3 m (d) 3.84 × 10–3 m
symmetry and more balanced arrangement. (A.I.E.E.E. 2009)
(c) The energy of 2s-orbital is less than the energy of
ub
(NEET 2017)
B29. Which one is a wrong statement? B36. What is the maximum number of emission lines
er
(a) Total orbital angular momentum of electron in obtained when the excited electrons of a hydrogen atom
s-orbital is equal to zero. in n = 5 drop to ground state ?
od
(a) n = 3 to n = 1 (b) n = 10 to n = 1
(d) The value of m for dz2 is zero. (NEET 2018) (c) n = 9 to n = 1 (d) n = 2 to n = 1
(Karnataka CET 2010)
B23. (a) B24. (d) B25. (b) B26. (a) B27. (c) B28. (c) B29. (c) B30. (b) B31. (c) B32. (a)
B33. (a) B34. (c) B35. (b) B36. (a) B37. (a)
B38. Ionisation energy of He+ is 19.6 × 10–18 J atom–1. The B44. The electronic configuration of Cu2+ ion is
energy of first stationary state (n =1) of Li2+ is (a) [Ar] 3d84s1 (b) [Ar] 3d9 4s0
(a) – 2.2. × 10–15 J atom–1
.
(c) [Ar] 3d74s2 (d) [Ar] 3d8 4s0
ed
(b) 8.82 × 10–17 J atom–1
(c) 4.41 × 10–16 J atom–1 (Karnataka CET 2013)
(d) – 4.41 × 10–17 J atom–1 (A.I.E.E.E. 2010) B45. The ionization enthalpy of He+ ion is 19.60 × 10–18 J
rv
B39. Which transition in the hydrogen atomic spectrum will atom–1. The ionization enthalpy of Li2+ ion will be
have the same wavelength as the transition, n = 4 to
(a) 84.2 × 10–18 J atom–1
n = 2 of He+ spectrum ?
se
(a) n = 4 to n = 3 (b) n = 3 to n = 2 (b) 44.10 × 10–18 J atom–1
(c) n = 4 to n = 2 (d) n = 3 to n = 1 (c) 63.20 × 10–18 J atom–1
re
(e) n = 2 to n = 1 (Kerala PET 2011) (d) 21.20 × 10–18 J atom–1
B40. For Balmer series in the spectrum of atomic hydrogen,
the wave number of each lines is given by (e) 2.17 × 10–19 J atom–1 (Kerala P.E.T. 2013)
B46. Energy of an electron is given by
ts
⎛ 1 1 ⎞
ν = R H ⎜ 2 − 2 ⎟ where R is a constant and n and
⎜n n ⎟ H 1 ⎛ Z2 ⎞
⎝ 1 2 ⎠ E = – 2.178 × 10–18 J ⎜ 2 ⎟ . Wavelength of light required
⎜n ⎟
gh
n2 are integers. Which of the following statement(s) is ⎝ ⎠
(are) correct? to excite an electron in a hydrogen atom from level
1. As wavelength decreases, the lines in the series n = 1 to n = 2 will be (h = 6.62 × 10–34 J s and c = 3.0 × 108
ri
converge. m s–1)
2. The integer n1 is equal to 2 (a) 8.500 × 10–7 m (b) 1.214 × 10–7 m
3. The ionization energy of hydrogen can be calculated ll
from the wave number of these lines (c) 2.816 × 10–7 m (d) 6.500 × 10–7 m
.A
4. The line of longest wavelength corresponds to (JEE Main 2013)
n2 = 3.
B47. As per de Broglie’s formula, a macroscopic particle of
(a) 1, 2 and 3 (b) 2, 3 and 4 mass 100 g and moving at a velocity of 100 cm s–1 will
(c) 1, 2 and 4 (d) 2 and 4 only
rs
have a wavelength of
(e) 2 only (Kerala PET 2011)
B41. The frequency of light emitted for the transition n = 4 (a) 6.6 × 10–29 cm (b) 6.6 × 10–30 cm
he
to n = 2 of He+ is equal to the transition in H-atom (c) 6.6 × 10–31 cm (d) 6.6 × 10–32 cm
corresponding to which of following ? (WB JEE 2014)
(a) n = 2 to n = 1 (b) n = 3 to n = 2 B48. The shortest wavelength of the line in hydrogen atomic
lis
line in Lyman series of hydrogen atomic spectrum to (c) 1127.30 Å (d) 911.7 Å
that of the first line in Balmer series of Li2+ spectrum is (e) 1234.7 Å (Kerala PET 2014)
4 5 B49. The correct set of four quantum numbers for the valence
P
(a) (b)
5 4 electrons of rubidium atom (Z = 37) is
4 3 1 1
n
3 1 1
(e) (Kerala P.E.T. 2012) (c) 5, 1, 0, + (d) 5, 1, 1, +
8 2 2
od
(c) (1) < (3) < (2) < (4) (d) (3) < (4) < (2) < (1) (JEE Main 2015)
(A.I. E.E.E. 2012)
B38. (d) B39. (e) B40. (c) B41. (a) B42. (d) B43. (a) B44. (b) B45. (b) B46. (b) B47. (c)
B48. (d) B49. (b) B50. (a)
B51. Consider the following sets of quantum numbers. (Planck’s constant, h = 6.6262 × 10 –34Js; mass of
Which of the below setting is not permissible electron = 9.1091 × 10 –31 kg; charge of electron
e = 1.60210 × 10–19 C; permittivity of vaccum
.
arrangement of electrons in an atom?
ed
n l m s ∈0 = 8.854185 × 10–12 kg–1 m–3 A2)
1 (a) 1.65 Å (b) 4.76 Å
(a) 4 0 0 −
2 (c) 0.529 Å (d) 2.12 Å
rv
1 (JEE Main 2017)
(b) 5 3 0 +
2 B58. Which of the following set of quantum numbers is not
se
1 possible?
(c) 3 2 –2 −
2 (a) n = 3, l = 0, m = 0 (b) n = 3, l = 1, m = –1
1 (c) n = 2, l = 0, m = –1 (d) n = 2, l = 1, m = 0
(d) 3 + (Kerala PET 2016)
re
2 –3
2 (WB JEE 2018)
B52. A stream of electrons from a heated filament was B59. The number of unpaired electrons in Ni (atomic
passed between two charged plates kept at a potential number = 28) are
(a) 0 (b) 2
ts
difference V e.s.u. If e and m are charge and mass of an
electron respectively, then the value of h/λ (where λ is (c) 4 (d) 8 (WB JEE 2018)
wavelength associated with electron wave) is given by B60. With respect to atomic spectrum, each line in the
gh
Lyman series is due to electrons returning
(a) meV (b) 2meV (a) from a particular higher energy level to n = 3
(c) meV (d) 2meV (b) from a particular higher energy level to n = 2
ri
(JEE Main 2016) (c) from a particular higher energy level to n = 1
B53. If the given four electronic configurations (d) from a particular higher energy level to n = 4
(a) n = 4, l = 1
(c) n = 3, l = 2
(b) n = 4, l = 0
(d) n = 3, l = 1
ll
B61. The orbital nearest to the nucleus is
(J.K. CET 2018)
.A
are arranged in order of increasing energy, then the (a) 4f (b) 5d
order will be (c) 4s (d) 7p
(a) (iv) < (ii) < (iii) < (i) (Karnataka CET 2018)
rs
(b) (ii) < (iv) < (i) < (iii) B62. What is the work function of the metal if the light of
(c) (i) < (iii) < (ii) < (iv) wavelength 4000 Å generates photoelectrons of
he
(d) (iii) < (i) < (iv) < (ii) (WB JEE 2017) velocity 6 × 105 ms–1 from it?
B54. Which of the following set of quantum numbers (Mass of electron = 9 × 10–31 kg
represents the 19th electron of Cr (Z = 24)? Velocity of light = 3 × 108 ms–1
lis
B56. The energy of an electron in the 3s orbital (excited wavelength 550 nm is _______, if 100 V and 1A is
state) of H-atom is supplied for one hour.
(a) – 1.5 eV (b) – 13.6 eV (a) 1 × 1024 (b) 5 × 1024
M
B57. The radius of the second Bohr orbit for hydrogen atom
is :
B51. (d) B52. (d) B53. (a) B54. (b) B55. (d) B56. (a) B57. (d) B58. (c) B59. (b) B60. (c)
B61. (c) B62. (b) B63. (d) B64. (a)
B76. The number of radial nodes of 3s and 2p-orbitals are thickness dr, at a distance r from the nucleus. The
respectively : volume of this shell is 4πr2dr. The qualitative sketch
(a) 2, 0 (b) 0, 2 of the dependence of P on r is?
.
ed
(c) 1, 2 (d) 2, 1
(I.I.T. Screening 2005)
B77. The kinetic energy of an electron in the second Bohr
rv
orbit of a hydrogen atom [a0 is Bohr radius]
h2 (a) (b)
h2
se
(a) (b) 2
4π 2
ma02 16π ma02
2
h h2
re
(c) (d)
32π 2 ma02 64 π 2 ma02
(I.I.T. J.E.E. 2012)
(c) (d)
B78. P is the probability of finding the 1s electron of
ts
hydrogen atom in a spherical shell of infinitesimal (JEE Advance 2016)
gh
ri
B76. (a) B77. (c) B78. (c)
(c) 3p orbital has two spherical nodes (d) n = 3, l = 2 and n = 2, l = 1 (J.K. C.E.T. 2006)
(d) 3d orbital has no spherical node
C7. Identify the correct statement(s).
C2. Which of the following are correct notations for the
lis
orbitals ? The findings from the Bohr model for H-atom are
(a) n = 3, l = 2 ⎯→ 3d (a) angular momentum of the electron is expressed as
ub
(b) n = 4, l = 1 ⎯⎯→ 4p h
integral multiples of
(c) n = 4, l = 3 ⎯→ 4d 2π
(b) the first Bohr radius is 0.529 Å
(d) n = 6, l = 1 ⎯⎯→ 6f
P
1
C3. The orbitals having the same number of spherical (c) the energy of the nth level, En is proportional to 2
n
nodes are :
(d) the spacing between adjacent levels increases with
n
(a) It is a 4d state.
unpaired electrons ? (b) The nuclear charge experienced by the electron in
(a) Copper (Z = 29) this state is less than 2e, where e is the magnitude
©
C1. (b, d) C2. (a, b) C3. (b, c) C4. (b, c) C5. (a, b) C6. (a, d) C7. (a, b, c) C8. (a, d)
.
2.0 × 104 m s–1 is
ed
(a) 4.99 pm (b) 49.9 pm
Passage I.
(c) 499 nm (d) 499 pm
In 1924, de-Broglie proposed that every particle
rv
possesses wave properties with a wavelength, λ given by Passage II.
h The position and energy of an electron is specified with
λ = where m is the mass of the particle, v is its
se
mv the help of four quantum numbers namely, principal
velocity and h is Planck’s constant. The de-Broglie quantum number (n), azimuthal quantum number (l),
prediction was confirmed experimentally when it was magnetic quantum number (m l ) and spin quantum
number (ms). The permissible values of these are :
re
found that an electron beam undergoes diffraction, a
phenomenon characteristic of waves. The de-Broglie n = 1, 2
wavelength can be estimated by measuring kinetic energy l = 0, 1, ......(n – 1)
of an electron accelerating by a potential V as :
ts
m l = – l....... 0, ..... + l
1
mv2 = eV where 1 eV = 1.6 × 10–19C, h = 6.6 × 10–34 J. 1 1
gh
2 ms = + and – for each value of ml.
2 2
Answer the following questions : The angular momentum of electron is given as
ri
D1. The mass of a photon moving with velocity of light h
l ( l + 1).
having wavelength same as that of an α-particle 2π
(mass = 6.6 × 10–27 kg) moving with velocity of ll h
2.5 × 102 m s–1 is While spin angular momentum is given as s ( s + 1) .
2π
.A
(a) 7.92 × 10–21 kg 1
where s =
2
(b) 5.5 × 10–33 kg
The electrons having the same value of n, l and ml are
(c) 5.65 × 10–31 kg
rs
h × 10 −4 angular momentum is
(a) 104 (2 me)1/2h (b) h
(2 me)1 / 2 3h
(a) (b) 6
ub
π 2π
10−3 h h × 104
(c) (d) h
(2 me)1 / 2 (2 me)−1 / 2 (c) zero (d) 2 3
π
P
(a) 32 (b) 18
1 1
er
2 2π possible.
(c) (d)
πλ λ (d) The orbitals 2d, 3f and 4g are not possible.
Passage I. D1. (b) D2. (b) D3. (d) D4. (c) D5. (a) Passage II. D6. (c) D7. (b) D8. (c)
Passage III. D10. Energy of the state S1 in units of the hydrogen atom
ground state energy is
The hydrogen like species Li2+ is in a spherically
.
symmetrical state S1 with one radial node. Upon absorbing (a) 0.75
ed
light the ion undergoes transition to a state S2. The state (b) 1.50
S2 has one radial node and its energy is equal to the ground (c) 2.25
(d) 4.50
rv
state of the hydrogen atom.
D11. The orbital angular momentum quantum number of
Answer the following questions :
the state S2 is
se
D9. The state S1 is
(a) 0 (b) 1
(a) 1s (b) 2s
(c) 2p (d) 3s (c) 2 (d) 3 (I.I.T. 2010)
re
Passage IV.
The wave function, ψn, l, m is a mathematical function whose value depends upon spherical polar coordinates
l
(r, θ, φ,) of the electron and characterized by the quantum numbers n, l and ml. Here r is distance from nucleus, θ is
ts
colatitude and φ is azimuth. In the mathematical functions given in the table, Z is atomic number and a0 is Bohr radius.
Using this information which is available in three columns of the table, answer the following questions:
gh
Column 1 Column 2 Column 3
3
ri
⎛Z⎞ (r)
l
Ψn, l, m
2
(I) 1s orbital (i) Ψ n,l,ml ∝ ⎜ ⎟ e− (Zr / a0 ) (P)
⎝ a0 ⎠
ll 0
r/a0
1
.A
(II) 2s orbital (ii) One radial node (Q) Probability density at nucleus ∝
a03
5 ⎛ Zr ⎞
⎛ Z ⎞2 −⎜ ⎟
rs
⎝ 2 a0 ⎠
(III) 2pz orbital (iii) ψn, l, m ∝ ⎜ ⎟ re cos θ (R) Probability density is maximum at nucleus.
l ⎝ a0 ⎠
he
(IV) 3dz2 orbital (iv) xy-plane is a nodal plane (S) Energy needed to excite electron from
27
n = 2 state to n = 4 state is 32 times the
energy needed to excite electron from n = 2
lis
state to n = 6 state
D12. For He+ ion, the only incorrect combination is List-I List-II
ub
(U) ∝ n1/2
(JEE Advance 2017)
D15. Which of the following options has the correct
Passage V.
combination considering List-I and List-II?
Consider the Bohr’s model of a one-electron atom
M
of the atom and List-II contains options showing how they D16. Which of the following options has the correct
depend on n. combination considering List-I and List-II?
(a) (II), (R) (b) (II), (Q)
(c) (I), (P) (d) (I), (T)
(JEE Advance 2019)
Passage III. D9. (b) D10. (c) D11. (b) Passage IV. D12. (d) D13. (d) D14. (d) D15. (c) D16. (d)
.
given below the lists:
ed
1. Match the orbital given in List-I with its description
given in List-II and select the correct answer using the List I List II
code given below the lists:
(P) discovery of proton 1. Chadwick
rv
List-I List-II (Q) discovery of neutron 2. Goldstein
(P) 3pz 1. total number of nodes = 2 (R) determination of atomic 3. Moseley
se
(Q) 4s 2. has 3 radial nodes and 0 angular number
node (S) determination of charge/ 4. J.J. Thomson
re
(R) 3dx2–y2 3. corresponds to n = 3, l = 0, ml = 0 mass of electron
ts
Code (a) 4 2 1 3
P Q R S (b) 4 1 2 3
gh
(a) 2 4 3 1 (c) 2 1 4 3
(b) 4 2 3 1 (d) 2 1 3 4
(c) 4 1 3 2
ri
(d) 3 2 4 1
ll
.A
1. (a) 2. (d)
rs
I may have one or more than one correct option from Column
II. The answers to these questions have to be appropriately
bubbled as illustrated in the following example.
P
1. Match the spectral series in Column I with its 2. Match the quantum number given in Column I with
characteristics in Column II.
n
(A) Balmer series (p) falls in infra red region (A) n (p) designates the orientation of
od
orbital
(B) Pfund series (q) involve jumps from higher
orbits to n = 3 (B) l (q) orientation of the spin of the
electron
M
(1) : (A) – (r) (B) – (p), (s) (C) – (p) (D) – (p), (q) (2) : (A) – (s) (B) – (r) (C) – (p) (D) – (q)
3. Match the entries in Column I with the correctly related quantum number(s) in ColumnII.
.
Column I Column II
ed
(A) Orbital angular momentum of the electron in (p) Principal quantum number
a hydrogen-like atomic orbital.
(B) A hydrogen-like one-electron wave function (q) Azimuthal quantum number
rv
obeying Pauli principle
(C) Shape, size and orientation of hydrogen like (r) Magnetic quantum number
se
atomic orbitals.
(D) Probability density of electron at the nucleus (s) Electron spin quantum number
re
in hydrogen-like atom.
ts
(3) : (A)– (q) (B)– (p), (q), (r), (s) (C) – (p), (q) (r) (D) – (p), (q)
gh
Integer Type and Numerical Value Type Questions 7. Number of times radius of the 3rd shell of the
ri
H-atom as compared to that of radius of first shell is
Integer Type: The answers to each of the following question ll 8. The atomic masses of He and Ne are 4 and 20 a.m.u.,
is a single digit integer ranging from 0 to 9. respectively. The value of the de Broglie wavelength of
1. The maximum number of electrons that can have He gas at –73°C is M times that of the de Broglie
.A
principal quantum number, n = 3 and spin quantum wavelength of Ne at 727°C. M is
number, ms = +1/2 is (I.I.T. 2011)
(JEE Advanced 2013)
2. How many times is the ionization energy of He+ ion
rs
2π
P
1. (9) 2. (4) 3. (7) 4. (4) 5. (6) 6. (6) 7. (9) 8. (5). 9. (3). 10. (3).
n
ions for
er
s & Explanat
Hint Difficult Objective Type Questions
od
1 2 1 2
∴ Total energy = mv + (– mv2) = – mv
2 2
M
2 1
1 mv2
mv , P.E. = − Ze
2
A1. (b) K.E. = K.E.
2 r ∴ = 2 = –1
Total energy 1
and electrostatic force = centrifugal force − mv2
©
2 2 2
Ze mv
2 = 1311.8 ⎛ 1311.8⎞ −1
r r A3. (b) Energy emitted = − −⎜− ⎟ kJ mol
Ze
2
3 2 ⎝ 22 ⎠
or = mv2
r ⎛ 1 1 ⎞
or PE = –mv2 or = 1311.8 ⎜ 2 − 2 ⎟ kJ mol–1
⎝2 3 ⎠
⎛1 1⎞ ΔE1 3 36 27
= 1311.8 ⎜ − ⎟ Ratio, = × =
⎝4 9⎠ ΔE 2 4 5 5
.
5
= 1311.8 × kJ mol–1
ed
hc hc
36 A11. (d) ΔE = hν = = or λ =
1311.8 × 103 × 5 λ ΔE
∴ Energy emitted per atom = J 1
6.02 × 1023 × 36 i.e., λ ∝
rv
= 3.03 × 10–19 J ΔE
+
A4. (a) For transition in He (Z = 2) ΔE = E4 – E1 will be maximum and hence λ will be
se
minimum.
1 ⎛ 1 1 ⎞
= RZ2 ⎜⎜ 2 − 2 ⎟⎟ A12. (a) Energy in the excited state is same as the K.E.
λ ⎝ n1 n2 ⎠
A13. (b) rn = 0.059 n2
re
⎛1 1 ⎞ 3
=R×4 ⎜ − ⎟ = R r1 = 0.059, r2 = 0.059 × 4, r3 = 0.059 × 9
⎝ 4 16 ⎠ 4
For H atom (Z = 1) ∴ r1 : r2 : r3 = 1 : 4 : 9
ts
1 ⎛ 1 1 ⎞ A14. (d) Frequency of a line in spectral series can be
= R⎜ 2 − 2 ⎟
λ ⎜n n ⎟ calculated as
⎝ 1 2 ⎠
gh
⎛ 1 1 ⎞ 3 c ⎛ 1 1 ⎞
=R ⎜ 2 − 2⎟= R ν= = cR H (Z) 2 ⎜ 2 − 2 ⎟
⎜n ⎟
⎝ 1 n2 ⎠ 4 λ n
⎝ 1 n 2 ⎠
ri
1 1 3 For third line in Lyman series,
2
− 2
=
n1 n2 4
ll n1 = 1, n2 = 4
∴ n2 = 2, n1 = 1,
1 1 15
νH = c.R H (1) 2 ⎡⎢ 2 − 2 ⎤⎥ = R HC
.A
A5. (d) I.E.Li2+ = Z2 × I.E. (H) ⎣1 4 ⎦ 16
= 9 × 13.6 eV = 122.4 eV For first line in Balmer series for Li2+
A6. (d) rn = 0.53 × n2 n1 = 2, n2 = 3
rs
∴ r2 = 0.53 × 4 = 2.12 Å
⎡1 1 ⎤
( n2 − n1 )( n2 − n1 + 1) νLi2+ = cR H (Z )2 ⎢ − 2⎥
2
A7. (b) No. of spectral lines = n
⎣⎢ 1 n 2⎥
he
⎦
2
( 5 − 1) ( 5 − 1 + 1) 2⎡ 1 1⎤
= =
4×5
= 10 = cR H (3) ⎢ 2 − 2 ⎥
⎣2 3 ⎦
lis
2 2
A8. (b) rn = 0.529 n2Å 5 5
= c × RH × 9 × = cR H
A9. (c) For H atom, first Balmer line in the series is 36 4
ub
νH 15 4 3
E1 (H) E (H) 5E1 ( H ) = × =
E3 – E2 = – 1 = νLi2+ 16 5 4
9 4 36
A15. (c) Energies are additive so that
P
hc hc hc
6 2
42 = +
λ3 λ1 λ2
er
⎛ 16 − 36 ⎞
= – E1(H) × 22 ⎜ ⎟ λ + λ2
⎝ 16 × 36 ⎠ 1
=
1
+
1
= 1
or λ3 λ1 λ2 λ1 λ 2
od
4 × 20 5E1 ( H )
= E1(H) =
16 × 36 36 λ1 λ 2
λ3 =
λ1 + λ 2
M
1.316 × 106
A10. (c) En = – h h
n2 A16. (d) λ = or v =
mv λm
©
.
A25. (a) It corresponds to 6d-orbitals and, therefore, can 4π
ed
accommodate maximum of 10 electrons. h
A26. (c) 4d has highest energy. Δp . Δp =
4π
A28. (d) n = 7, l = 2 corresponds to 7d and not 7p h
rv
A29. (a) Because l = 0 for s electrons. Δp2 =
4π
h h
Orbital angular momentum = l ( l + 1) .
=0
se
Δp =
2π 4π
A30. (a) One set of quantum numbers represents only one
h
orbital. mΔv =
re
4π
A32.(a) Electronic configuration of Mn (Z = 25) is
1 h
1s22s22p63s23p63d54s2 Δv =
2m π
Total orbitals populated
ts
= 1 + 1 + 3 + 1 + 3 + 5 + 1 = 15 B4. (a) Δp = mΔv = 1 × 10–18 g cm s–1
A33. (d) Fe3+ has 3d5 configuration and, therefore, has 5 1 × 10−18 g cm s−1
gh
Δv =
(maximum) number of electrons while Mg2+ has 9 × 10−28 g
zero, Ti3+ has 1(3d1) and V3+ has 2(3d2). ∴ = 1 ×109 cm s–1
A34. (b) The last electron in Na atom is present in 3s h
ri
subshell i.e., l = 0 B5. (e) λ =
mv
A36. (b) The electronic configuration of Ar is : 1s22s22p63s23p6 mA = x vA = 0.1 mAvA = 0.1 x
No. of electrons having m = 0 are
= 2(1s) + 2 (2s) + 2(2pz) + 2(3s) + 2(3pz) = 10
ll mB = 5x vB = 0.05 mBvB = 0.25 x
.A
λA mBvB
=
A38. (a) No. of unpaired electrons : λB mA vA
1. Ni2+ (Z = 28) : [Ar] 3d8 = 2 0.25 x 25 5
= = =
rs
Angular nodes = l = 1 hc hc
B10. (b) E1 = , E2 =
A44. (b) Electronic configuration of Cr (Z = 24): λ1 λ2
P
6.63 × 10–20 kg m2s–2 B15. (b) The electron is more tightly bound in the smallest
allowed orbit.
= 6.63 × 10–34 kg m2 s–1 (2 × 1014 – ν0)
c c
6.63 × 10−20 B16. (a) ν = or λ =
= 2 × 1014 –ν0 λ ν
6.63 × 10−34
3 × 1017 nm s −1
1 × 1014 – 2 × 1014 = – ν0 or or ν0 = 1 × 1014 s–1 λ = = 50 nm .
6 × 1015 s −1
B17. (d) Deuterium has 1 proton and 1 neutron B28. (c) The energy of 2s-orbital and 2p-orbital is equal in
α-particle has 2 protons and 2 neutrons hydrogen like atoms.
.
∴ Mass of α-particle (mα) is twice that of deuterium B29. (c) Option (c) is wrong according to Hund’s rule.
ed
(mD) i.e. mα = 2mD
1.312 × 106 ⎛ 1.312 × 106 ⎞
Also, velocity of deuterium (vD) B33. (a) ΔE = E2 – E1 = – – ⎜⎜ −
22
12 ⎟⎟
⎝ ⎠
rv
= 5 times velocity of α-particle (vα)
i.e. vD = 5vα or vα = vD/5 = 9.85 × 105 J mol–1.
h
se
λD = and 0.005
mDvD B34. (c) Δv = × 600 = 0.03
100
h h
λα = =
re
h
Δx × mΔv =
mα vα
( 2mD ) ⎛⎜⎝ vD ⎞⎟⎠ 4π
5
h
λD h (2mD ) (vD ) Δx =
or = × = 0.4 4 πm Δ v
ts
λα mDvD 5h
B18. (d) m = 6.626 × 10–31 kg 6.6 × 10 −34
Δ v = 1 × 106 ms–1 =
gh
4 × 3.14 × 9.1 × 10−31 × 0.03
Δx = ?
h = 1.92 × 10–3 m
Δ x. mΔ v =
4π
ri
h 6.63 × 10−34
h 6.626 × 10−34 B35. (b) λ = =
or Δx = = mv 1.67 × 10−27 × 1 × 103
4 πmΔv 4 × π × 6.626 × 10−31 × 106 ll = 3.9 × 10–10 m
1 × 10−9 m
.A
= ≈ 4 × 10–10 m = 0.40 nm
4π
B36. (a) Number of spectral lines emitted when electron
⎛ 1⎞
= ⎜ ⎟ nm jumps from n2 to n1
⎝ 4π ⎠
rs
(n2 − n1 )(n2 − n1 + 1)
B19. (a) It represents 3pZ orbital. =
2
hc 6.626 × 10−34 × 3 × 108
he
2 B37. (a) ν = RH ⎢ 2 − 2 ⎥ = 8 RH
are not possible. ⎣ n1 n2 ⎦ 9
B22. (c) Orbital angular momentum = l (l + 1) 1 1 8
∴ − =
ub
3d 4s 19.6 × 10−18
∴ I.E of H atom =
B24. (d) Fe2+ : [Ar]3d6
od
4
Number of d–electrons = 6 = 4.9 × 10–18 J atom–1
Fe : [Ar] 3d6 4s2 (No. of d-electrons = 6) I.E of Li2+ ion = I.E of H atom × (3)2
Ne : 1s2 2s2 2p6 (No. of p-electrons = 6)
M
.
4 ∴ E1 = − k = −
⎝ 1 n2 ⎠
ed
2
1 1 3 n 12
or 2
− 2 = = – 44.1 × 10 J atom–1
–18
n n 4
1 2 I.E. of Li = E∞ – E1 = 0 – (–44.1 × 10–18)
2+
rv
B40. (c) 1. Beyond a certain wavelength, line spectrum
12
appears as band spectrum. B46. (b) E1 = –2.178 × 10–18 × = –2.178 × 10–18 J
se
2. For Balmer series n1 = 2. 12
4. For line of longest wavelength (lowest energy) 12
in this series, n2 = 3. E2 = –2.178 × 10–18 × = –5.445 × 10–19 J
re
22
B41. (a) ν (H) = ν (He+)
Energy required to excite electron from n = 1 to
2⎛ 1 1⎞ 2⎛ 1 1 ⎞ n = 2,
RZ ⎜ 2 − 2 ⎟ = RZ ⎜ 2 − 2 ⎟ ΔE = – 5.445 × 10–19 – (–2.178 × 10–18)
ts
⎝ n1 n2 ⎠ H ⎜ ⎟
⎝ n1 n2 ⎠ He+
= 16.335 × 10–19 J
1 1 ⎛1 1 ⎞
gh
− = 4⎜ − ⎟ hc
= 16.335 × 10–19
2
n1
2
n2 ⎝ 4 16 ⎠ λ
1 1 1 3 6.63 × 10−34 × 3.0 × 108
− = 1− = λ =
ri
n1
2
n2
2
4 4 16.335 × 10−19
= 1.217 × 10–7 m
∴ n1 = 1 and n2 = 2 ll
B42. (d) For third line in Lyman series, h
B47. (c) λ =
.A
n1 = 1, n2 = 4 mv
m = 100 g = 0.1 kg v = 100 cm s–1 = 1 m s–1
c ⎛ 1 1⎞
νH = = c.R H Z2 ⎜ 2 − 2 ⎟
λ ⎝ n1 n2 ⎠ 6.6 × 10 −34
rs
⎢⎣ 12 4 2 ⎥⎦ 16 H 1 ⎡ 1 1 ⎤
B48. (d) = RH ⎢ 2 − 2 ⎥
For first line in Balmer series for Li2+ λ n
⎣ 1 n2 ⎦
n1 = 2, n2 = 3
Shortest wavelength line in hydrogen spectrum of
lis
⎡ 1 1 ⎤ 5 5 λ ⎣ (1) ∞ ⎦
= cRH (3)2 ⎢ 2 − 2 ⎥ = c × R H × 9 × = cR H
⎣2 3 ⎦ 36 4 1
or λ = cm = 9.11 × 10–6 cm
νH
P
109678
= 15 × 4 = 3
νLi2+ 16 5 4 = 911 × 10–10 m or 911 Å
B43. (a) The orbitals are (1) 4p (2) 4s (3) 3d (4) 3p. B49. (b) The electronic configuration of Rb (Z = 37) is
n
Increasing order of energies is : 1s2 2s2 2p6 3s2 3p6 3d10 4s2 4p6 5s1
er
n = 5, l = 0, m = 0, s = + ½
2π2 me4
B45. (b) En = − (Z2 ) B50. (a) For first excited state of hydrogen (n = 2)
n2 h2
M
1
mv2 = eV −9
B63. (d) λ A = 33.33 nm = 33.33 × 10 m
2
1
.
2 eV pB = pA
ed
or v= 3
m
According to de-Broglie, h
λA = ...(i)
pA
rv
h h
λ= = h h 3h
mv 2eV λB = = = ...(ii)
m pB pA /3 pA
m
se
h From eq. (i) and (ii)
or λ=
2meV λA 1
re
h =
∴ = 2meV λB 3
λ
B53. (a) (i) 4p orbital (ii) 4s orbital (iii) 3d-orbital ∴ λB = 3 × λA = 3 × 33.33 × 10–9 m
= 1.0 × 10–7 m
ts
(iv) 3p-orbital
Order of increasing energy hc
B64. (a) Energy of a photon =
3p (iv) < 4s (ii) < 3d (iii) < 4p (i) λ
gh
B54. (b) Electronic configuration of Cr (Z = 24) 6.62 × 10−34 × 3.0 × 108
= = 3.6 × 10−19 J
Cr : 1s2 2s2 2p6 3s2 3p6 4s1 3d5 550 × 10−9
ri
19th electron is present in 4s-orbital Energy absorbed per sec by bulb = V × I × t
For 4s orbital; n = 4, l = 0, m = 0, s = + ½ = 100 × 1 × 3600
B55. (d) Cl (Z = 17) : 1s2 2s2 2p6 3s2 3p5
The unpaired electron of Cl is in 3p orbtial.
ll n=
100 × 3600
= 1 × 1024
3.6 × 10 −19
.A
Hence, quantum numbers for unpaired electron are: 1
B65. (b) hν = w0 + mv2
1 1 2
n = 3, l = 1, m = + 1, 0, –1, s = + or –
2 2 1
rs
w0 = hν − mv2
13.6 2
B56. (a) En = – 2
eV
6.626 × 10−34 × 3.0 × 108 1
he
n
For 3s-orbital, n = 3 = − × 9 × 10−31 × (6 × 105 ) 2
4000 × 10−10 2
En = –
13.6
= –1.51eV = 4.96 × 10–19 –1.62 × 10–19
lis
3
2
= 3.34 × 10–19 J
2
B57. (d) r=
0.529 n 3.34 × 10−19
or =
Z 1.6 × 10−19
ub
Z =1, n = 2 = 2.1 eV
2
0.529 × (2) B66. (d) For 71st electron :
r=
[Xe]54 4f14 6s2
P
1 Z = 70
= 2.12Å Z = 71 [Xe] 4f14 5d1 6s2
B58. (c) For n = 2, l = 0, m cannot be –1 because m has values B67. (b) Ratio of the spectral lines,
n
–l ....0 ...... + l 1 ⎛1 1 ⎞
= R H ⎜ 2 − 2 ⎟ Z2
er
λ1 m
⎝ 1 m 2 ⎠
to n = 1.
B62. (b) For shortest wavelengths, n2 = ∝ and m2 = ∝
1 λ1 9 m12
M
hv = w0 + mv2 ∴ λ = 1 = n2
2 2 1
1 Now, if m1 = 3 and n1 = 1, the ratio is justified.
wo = hv − mv2
©
.
ν max = R H ⎜ 2 − 2 ⎟ = R H
⎝1 ∝ ⎠
ed
Z
⎛1 1 ⎞ 3 For H atom Z = 1
ν min = R H ⎜ 2 − 2 ⎟ = R H 2
r1 = 0.529 n Å
⎝1 2 ⎠ 4 3+
For Be , Z = 4
rv
3 R
ΔνLyman = R H − R H = H 2
4 4 r 3+ = 0.529n Å
se
For Balmer series Be 4
Given that r = r 3+
⎛1 1 ⎞ R H Be
ν max = RH ⎜ 2 − 2 ⎟ = H ∴ It is equal when n = 2 for Be
3+
re
⎝ 2 ∝ ⎠ 4
3+
⎛1 1 ⎞ 5 ∴ r2 (Be ) = r1(H)
ν min = RH ⎜ 2 − 2 ⎟ = RH
⎝ 2 3 ⎠ 36 B77. (c) For Bohr orbit, angular momentum is
ts
R 5R H nh nh
ΔνBalmer = H− mvrn = or v=
4 36 2π 2πmrn
gh
4R H
= 1
36 Kinetic energy, KE = mv2
RH 2
=
ri
9 1 n2 h2 n2 h2
ΔνLyman R /4 9 or K.E. = m× 2 2 2 = 2 2
= H = or 9 : 4 2 4 π m rn 8π mrn
ΔνBalmer R H /9 4 ll For n = 2, rn = a0 × n2 = 4a0 (a0 = Bohr radius)
.A
B72. (b) de-Broglie wavelength
h 22 h 2 h2
λ= K.E. = =
2 K.E. × m 8 π 2 m (4 a0 ) 2 32 π 2 ma02
rs
KE = hν – hν0 C3. (b, c) : (b) 3p and 4d have 1 node (c) 4s and 5p have
3 nodes.
h
λ= C4. (b, c) : No. of unpaired electrons : Cr3+(3), Fe2+(4), Cu+(0),
lis
∴
2m (hν − hν 0 ) Zn2+(0), Mn2+(5), Fe3+(5), Sc3+(0), Ti3+(1).
h C5. (a, b) : No. of unpaired electrons : Cu (1), Sc (1), Mn (5),
or λ=
ub
2mh (ν − ν 0 ) Cr (6).
1 C6. (a, d) : (a) 4d > 5s (b) 3d = 3p (c) 3p = 3d (d) 3d > 2p.
or λ∝ 1
(ν − ν 0 ) 2
C7. (a, b, c) : (d) is not correct. The spacing between adjacent
P
ν = RH × ⎜ 2 − 2 ⎟ (Z = 1 for H) −13.6 × Z 2
⎜ nf 8 ⎟ n2 = = 16 (∵ Z = 2)
⎝ ⎠
−3.4
od
1 R n=4
ν = RH × − H
n f 2 64 l=2
1 ∴ It corresponds to 4 d state.
M
1
B74. (c) hν = hν0 + mv2
2
1
mv2 (K.E.) = hν – hν0 D1. (b) Wavelength of α-particles
2
Plot of K.E. vs ν cannot have intercept equal to h h
zero. λ (α-particle) = = −27 2
mv 6.6 × 10 × 2.5 × 10
.
D12. (d) The 1s-orbital is non-directional and its wave
Since λ(α-particle) = λ (photon)
ed
function is independent of cos θ. Hence (d) is incorrect.
h h D13. (d) For hydrogen like species, only (d) is correct.
=
6.6 × 10−27 × 2.5 × 102 m × 3.0 × 108 In (a) 1s orbital has no radial node.
rv
6.6 × 10−27 × 2.5 × 102 In (b) for 3dz2 , xy is not a nodal plane.
m = In (c) 2pz has no radial node.
se
3.0 × 108
D14. (d) Energy needed when electron is excited from n = 2
= 5.5 × 10–33 kg to n = 4 state.
D2. (b) Wavelength of electron accelerated by potential of
re
V volt is ⎛1 1 ⎞ 3
E4 – E2 = R ⎜⎝ 2 − 2 ⎟⎠ = R
2 4 16
h h h × 10 −4
λ= = = Energy needed when electron is excited from n = 2 to
ts
1/ 2 6 1/ 2
(2 Vem) (2 × 100 × 10 em) (2 me)1 / 2
n = 6 state.
h
D3. (d) λp = ,
gh
⎛1 1⎞ 8
(2π Ve mp )1/ 2 E6 – E2 = R ⎜⎝ 2 − 2 ⎟⎠ = R
2 6 36
h h
λ He2+ = =
ri
(2π Ve m 1/ 2
(2πVe 4mp )1 / 2 E4 – E2 3 36 27
2+ ) × =
He
E6 – E2 = 16 8 32
λHe2+ (2π Ve mp ) 2 1
1 ll 27
∴ = = Hence E4–E2 = × (E6 – E2)
.A
λp 1 2 32
(2π Ve 4mp ) 2
n2 h2
D15–D16. (I) Radius, rn = or Z ∝ n2
h 4h 4 π2Zme2
rs
6.022 × 1023 1 1 Ze 2
−34
(III) Kinetic energy, K.E. = mv2 =
6.6 × 10 2 2 rn
λ= = 4.99 × 10–12m
6.64 × 10−27 × 2.0 × 104 Since, rn ∝ n2
ub
= 4.99 pm 1
K.E. ∝
D6. (c) For 3s electron, l = 0 so that orbital angular n2
P
h
momentum l(l + 1) is zero. (III) – (P)
2π
−Ze 2
n
1
D8. (c) For 4f electron, n = 4, l = 3, ml = 0 and s = + is
2 1
possible. ∴ P.E. ∝
n2
od
9
= − × 13.6 eV
4
1 (9) : No. of orbitals in a shell in n = 3 will be = n2
EH = – 13.6 eV (ground state) i.e., 32 = 9. Each orbital can have maximum of
E = − 9 × EH = 2.25 EH two electrons, one with ms = +
1
and the second
4 2
1 h
with ms = – . Hence number of electrons with λNe ∝
2
.
2m Ne TNe
ed
1
ms = + will be 9.
2 λ He m Ne TNe
∝
2 (4) : Ionization energy of He+ ion = Z2 × I.E. (H) = 22 λ Ne m He THe
rv
× I.E. (H) = 4 times
3 (7) : For f-subshell, l = 3 and hence it has (2 × 3 + 1) T H = 273 – 73 = 200 K,
= 7 times
se
T Ne = 727 + 273 = 1000
4 (4) : Fe2+ : [Ar] 3d6. Hence, it has four unpaired
electrons. λ He 20 × 1000
=
5 (6) : It corresponds to p-subshell and hence has
re
λ Ne 4 × 200
maximum of 6 electrons.
λ He
h = 5
6 (6) : Angular momentum is l(l + 1) λ Ne
ts
2π
Since l = 2 for 3d orbital ∴ λHe = 5 λNe
gh
h h 9 (3) : For n = 4 ; l = 0, 1, 2, 3.
Angular momentum = 2(2 + 1) = 6 .
2π 2π Except l = 0, all other ‘l’ values can have ml = 1.
Therefore, three orbitals of n = 4 can have ml
n2 = 1. Each of these orbitals can accommodate
ri
7 (9) : rn = a0
Z 1
For H, Z = 1 and for n = 3 one electron with ms = − . Thus, the total
ll 2
r3 = a0 × 32 = 9a0. number of electrons is three.
10 (3) : In 'H' atom, there is only 1 electron (1s1).
.A
h
8 (5) : λ = Therefore, there is no screening effect. Due to
mv
this, energy of all subshells in a given shell is
1 same. Thus, order of energy of various sub-
rs
mv2 = KE shells is
2
mv2 = 2KE 1s < 2s = 2p < 3 s = 3 p = 3 d < 4s = 4p = 4d = 4f
he
.
ed
Time Allowed: 1½ Hr. Maximum Marks: 25
rv
1. Maximum number of electrons in a subshell of an atom is determined by
(a) 2 l + 1 (b) 2n2
se
(c) 4l+2 (d) 4l – 2 (1)
2. The orbital angular momentum of a p-electron is given as :
re
h 2h
(a) (b)
2π π
ts
3h
(c) (d) 0 (1)
2π
gh
3. Which of the following d-orbital has electron density in all the three axes
ri
(a) 3 dxy (b) 3 dx2 − y2
(c) 3 dz 2 (d)
ll 3 dyz (1)
(a) Assertion and reason both are correct statements and reason is correct explanation for assertion.
(b) Assertion and reason both are correct statements but reason is not correct explanation for assertion.
he
doubled.
Reason: Wavelength associated with a moving object is inversely proportional to its velocity. (1)
P
8. Calculate the wavelength associated with an electron (mass = 9.1 × 10–31 kg) having kinetic
er
(ii) Cr (Z = 24)
(ii) Fe (Z = 26) (2)
M
13. (a) What observations in scattering experiment led Rutherford to make the following conclusions ?
(i) The most of the space in an atom is empty.
(ii) The whole of the mass of the atom is present in the centre of the nucleus.
(iii) Nucleus has positive charge.
.
(b) What is the value of orbital angular momentum for an electron in 2s orbital ?
ed
(c) How many electrons in an atom may have n = 4 and ms = +1/2 ?
(d) What physical meaning is attributed to the square of the absolute value of wave
rv
function Ψ2 ? (5)
se
re
To check your performance, see HINTS and SOLUTIONS to some questions at the end of
Part I of the book.
ts
gh
ri
ll
.A
rs
he
lis
ub
P
n
er
od
M
©
.
3
ed
rv
se
re
N
O
cte r B
C hara L
tallic E
-me
ts
Non cter
CLASSIFICATION OF
G
hara
ic C A
M etall S
E
gh
S
ELEMENTS AND
ri
PERIODICITY ll
.A
IN PROPERTIES
rs
Scientists like to find patterns. About 200 years ago, scientists were discovering lots of new elements. However, they
he
were struggling hard to bring order and pattern to the vast amount of information they were discovering about the elements
and their properties. The periodic table is an important triumph for the nineteenth century chemists and has become an
important landmark in the history of chemistry. It is used as a sorting system for chemical elements. It has become the
lis
everyday support for students and teachers and provides a concise organization of the whole of chemistry.
In the present unit, we will study the historical developments of the periodic table. We will also learn how the periodic
ub
classification of elements follows as a logical consequence of the electronic configuration of atoms. We shall also study
some of the important periodic trends in the physical and chemical properties of elements.
P
OBJECTIVES
n
Objective Questions 68
Answers/Hints 46
3/1
.
century, the pace of discovery of new elements classified the elements in groups of three elements
ed
quickened. Between 1800 to 1869, the number of called triads. The elements in a triad had similar
identified elements had become nearly double to 63. properties and the atomic weight of the middle
With such a large number of elements, it became member of each triad is very close to the arithmetic
rv
difficult to study individually the chemistry of all these mean (average) of the other two elements. The
elements and their innumerable compounds. At this common triads of Dobereiner classification were :
se
stage, it was realised that there should be some simple lithium, sodium and potassium; calcium, strontium
way to study and remember the numerous properties and barium; chlorine, bromine and iodine; etc.
of the elements and their compounds. This gave rise to For example, in the triad of lithium, sodium and
re
necessity of classification of the elements into various potassium,
groups having similar properties. This has been done Atomic weight of middle element (sodium) = 23
by arranging the elements in such a way that similar Mean of atomic mass of lithium and potassium
ts
elements are placed together while dissimilar elements 7 + 39
are separated from one another. This is known as = = 23.
2
gh
classification of elements. Such a classification of The Dobereiner’s relationship was also referred
the elements has resulted in the formulation of the to as law of triads. However, it seemed to work only
periodic table. Periodic table may be defined as for a few elements. It was dismissed as coincidence
ri
the arrangement of the known elements because all the known elements could not be arranged
according to their properties in a tabular form. in triads.
At present, about 118 elements are known. Of
ll 2. De Chancourtois Classification
these, the recently discovered elements are man
.A
made. Efforts to synthesize new elements are The next reported attempt was made by a French
continuing. The periodic classification of the elements geologist, A.E.B.de Chancourtois in 1862. He arranged
has extremely simplified their study. Not only the the known elements in order of increasing atomic
rs
periodic classification rationalizes the known chemical weights and proposed a cylindrical table of elements
facts about elements, but it also helps to predict new to display the periodic recurrence of properties. He
he
ones for undertaking further study. observed that the elements with similar properties fell
in a vertical line from the centre of the spiral. However,
HISTORICAL DEVELOPMENT OF THE this did not attract much attention.
PERIODIC TABLE*
lis
elements having similar properties. Lavoisier classified proposed a new system of grouping elements of similar
the elements simply as metals and non-metals. properties. According to him, when the elements are
arranged in the increasing order of atomic
P
Lithium 7 Mean of 1st and 3rd relation as the law of octaves due to similarity with
er
Strontium 88 = 88.5
2
Barium 137 Table 2. Newlands’s law of octaves
Element Li Be B C N O F
©
M
K
In the above table, sodium (Na), the eighth elements with similar properties recur at regular
element from lithium (Li) is similar to Li and similarly intervals or periodically. As a result of this, the elements
the next eighth element potassium is similar to Na. fall in certain groups or families. The elements in each
The same is true for magnesium resembling beryllium, group were similar to each other in many properties.
aluminium resembling boron, etc. The properties repeated periodically. Mendeleev’s
.
4. Lother Meyer’s arrangement periodic table published in 1905 is given in Table 3
ed
ahead. The horizontal rows in the periodic table are
In 1869, a German chemist, Lother Meyer used
called periods and the vertical columns are called
the physical properties such as atomic volume, melting
groups.
rv
point, boiling point, etc., to arrive at his table of
elements. He showed that when the properties of the Mendeleev’s system of classifying elements was
more elaborate than the earlier attempts. He realized
se
elements such as atomic volume, melting point, boiling
point, etc., are plotted as a function of their atomic that some of the elements did not fit in very well with
weights, the elements with similar properties occupied his scheme of classification if the order of atomic
re
almost similar positions. On this basis, Lother Meyer weight was strictly followed. He showed courage to
proposed that when the elements are arranged in ignore the order of atomic weights, thinking that the
atomic weight measurements might be incorrect. He
the increasing order of their atomic weights,
ts
placed the elements with similar properties together.
similarities in physical and chemical properties
For example, iodine has lower atomic weight than that
appear at regular intervals.
gh
of tellurium (of Group VI) but he placed iodine in Group
All these attempts provided same clue that there VII alongwith fluorine, chlorine and bromine because
are certain regularities among the elements. The first of similarities in their properties.
ri
most successful attempt in classification of elements
The Mendeleev’s classification gave him so much
was made by a Russian chemist Dmitri Mendeleev.
confidence that he boldly left certain spaces or gaps for
ll
5. Mendeleev’s Periodic Law undiscovered elements. By considering the properties
In 1869, Mendeleev made a remarkable of the adjacent elements in his table, he predicted the
.A
contribution to the classification of elements. On the properties of the undiscovered elements. Later on,
basis of physical and chemical properties of the when these elements were discovered, their properties
elements, he gave a law known as the periodic law.
rs
This means that when the elements are arranged of the elements but he estimated their properties. He
in the order of their increasing atomic weights, the tentatively named these elements as eka-aluminium
elements with similar properties recur at regular and eka-silicon (from the Sanskrit word eka meaning
ub
intervals. Such orderly recurring properties in a cyclic ‘next’) because he believed that these would be similar
fashion are said to be occurring periodically. This is to aluminium and silicon respectively. When chemists
P
responsible for the name periodic law or periodic table. discovered these elements, Mendeleev’s prediction of
Mendeleev’s Periodic Table their properties proved to be remarkably correct.
n
On the basis of his periodic law, Mendeleev Some of the properties predicted by Mendeleev for
these elements and those found experimentally are
er
Table 4. Comparison of the properties predicted for eka-aluminium and eka-silicon by Mendeleev with those
observed for gallium and germanium.
Property Eka-aluminium predicted Gallium Eka-silicon predicted Germanium
M
M
K
Table 3. Mendeleev’s periodic table published in 1905 showing elements and their atomic weights.
4
od
SERIES GROUPS OF ELEMENTS
0 I II III IV V VI VII VIII
er
Hydrogen
H
—
1.008
n — — — — — —
1 Helium Beryllium Boron Carbon Nitrogen Oxygen Fluorine
He Lithium Be
P Br C N O F
2 4.0 Li 9.1 11.0 12.0 14.04 16.00 19.0
Neon 7.03 Magnesium Aluminium Silicon Phosphorus Sulphur Chlorine
Ne Sodium Mg Al Si P S Cl
3 Na
19.9 24.3 27.0 28.4 31.0 31.0 35.45
ub
23.5
Argon Potassium Calcium Scandium Titanium Vanadium Chromium Manganese Iron Cobalt Nickel
Ar K Ca Sc Ti V Cr Mn Fe Co Ni (Cu)
4 38 39.1 40.1 44.1 48.1 51.4 52.1 55.0 55.9 59 59
lis
— Copper Zinc Gallium Germanium Arsenic Selenium Bromine
5 Cu Zn Ga Ge As Se Br
63.6 65.4 70.0 72.3 75 79 79.95
he
Krypton Rubidium Strontium Yttrium Zirconium Niobium Molybdenum — Ruthenium Rhodium Palladium
M
M
6 Kr Rb Sr Y Zr Nb Mo Ru Rh Pd (Ag)
rs
K
K
81.8 85.4 87.6 89.0 90.6 94.0 96.0 101.7 103.0 106.5
— Silver Cadmium Indium Tin Antimony Tellurium Iodine
7 Ag Cd In Sn Sb Te I
107.9 112.4 114.0 119.0 120.0 127 127
.A
8 Xenon Cesium Barium Lanthanum Cerium
Xe Cs Ba La Ce
ll
9 128 132.9 137.4 139 140
— — Ytterbium — Tantalum Tungsten — Osmium Iridium Platinum (Au)
ri
10 Gold Mercury Yb Lead Ta W Os Ir Pt
Au Hg 173 Pb 183 184 191 193 194.9
11 Thallium Bismuth
197.2 200.0 Tl 206.9 Bi — —
gh
1/11/2020, 2:10 PM
204.1 208
— — Radium — Thorium — Uranium
12
ts
Ra Th U
224 232 239
ed
.
M
K
The success of Mendeleev’s bold quantitative 3. Position of isotopes. Isotopes are the atoms
predictions convinced Chemists of the usefulness of of the same element having different atomic masses
Mendeleev’s periodic table and led to its wide acceptance. but same atomic number. Therefore, according to
Important Contributions of Mendeleev’s Periodic Mendeleev’s classification, these should be placed at
Table different places depending upon their atomic masses.
.
For example, isotopes of hydrogen with atomic masses
ed
Mendeleev’s periodic table was one of the greatest
1, 2 and 3 should be placed at three places. However,
achievements in the development of chemistry. Some of
isotopes have not been given separate places in the
the important contributions of his periodic table are :
rv
periodic table.
1. Systematic study of the elements. The
4. Some similar elements are separated and
Mendeleev’s periodic table simplified the study of
se
dissimilar elements are grouped together. In the
chemistry of elements. Knowing the properties of one
Mendeleev’s periodic table, some similar elements
element in a group, the properties of other elements in
were placed in different groups while some dissimilar
re
the group can be easily guessed. Thus, it became very
elements had been grouped together. For example,
useful in studying and remembering the properties of
copper and mercury resembled in their properties but
a large number of elements.
they had been placed in different groups. At the same
ts
2. Correction of atomic masses. The time, elements of group I A such as Li, Na and K were
Mendeleev’s periodic table helped in correcting the grouped with copper (Cu), silver (Ag) and gold (Au),
gh
atomic masses of some elements based on their positions though their properties are quite different.
in the table. For example, atomic mass of beryllium
5. Cause of periodicity. Mendeleev did not
was corrected from 13.5 to 9. Similarly, with the help
ri
explain the cause of periodicity among the elements.
of this table, atomic masses of indium, gold, platinum
6. Position of lanthanoids (or lanthanides)
etc., were corrected. ll
and actinoids (or actinides). The fourteen elements
3. Prediction of new elements. At the time of
following lanthanum (known as lanthanoids, from
.A
Mendeleev, only 56 elements were known. While atomic number 58–71) and the fourteen elements
arranging these elements, he left some gaps. These following actinium (known as actinoids, from atomic
gaps represented the undiscovered elements. Mendeleev
rs
In spite of many advantages, the Mendeleev’s discovered the relationship between X-ray spectra and
periodic table had certain defects also. Some of these the atomic number of the elements. When high energy
electrons were focused on a target made of the
P
In these cases, Mendeleev placed elements according where ν is the frequency of the emitted X-ray and a
to similarities in their properties and not in increasing and b are constants that are same for all the elements.
order of their atomic masses. For example, argon (Ar, A plot of ν versus atomic number (Z) gives a straight
©
atomic mass 39.9) is placed before potassium (K, atomic line. This led Moseley to conclude that atomic
mass 39.1). Similarly, cobalt (Co, atomic mass 58.9) is number and not atomic weight is the
placed before nickel (Ni, atomic mas 58.6) and tellurium fundamental property of the atoms. He, therefore,
(Te, atomic mass 127.6) is placed before iodine (I, suggested that atomic number instead of atomic weight
atomic mass 126.9). These positions were not justified. should be the basis of classification of elements.
M
K
The acceptance of atomic number, as the International Union of Pure and Applied Chemistry
important characteristic of an atom, led to the modern (IUPAC), the groups are numbered from 1 to 18 (Fig. 1).
periodic law. Modern periodic law may be stated as These are discussed below :
the physical and chemical properties of the Periods
elements are periodic functions of their A horizontal row in the periodic table is
.
atomic numbers. called period.
ed
This law implies that the physical and chemical In terms of electronic structure of the atom, a
properties of the elements depend on atomic number period constitutes a series of elements whose atoms
rv
and this dependency shows periodicity. Consequently, have the same number of electron shells i.e. principal
when the elements are arranged in the order of their quantum number (n). Thus, the period indicates the
se
increasing atomic numbers, it is observed that the value of n for the outermost or valence shell of the
atom of the element. Each successive period in the
elements of similar properties recur at regular
periodic table is associated with the filling of the next
intervals or periodically. As a result of this, the
re
higher principal energy level (n=1, n=2, n=3, etc.) There
elements fall in certain groups and lead to an
are seven periods and each period starts with a
arrangement called the periodic table.
different principal quantum number.
ts
With the modern periodic law as the guiding It can be seen that the number of elements in
principle and the modern theory of atomic structure, each period is twice the number of atomic orbitals
gh
many new forms of the periodic table have been available in the energy level that is being filled.
proposed from time to time. Some forms emphasise Electronic Configurations of Elements in Periods
chemical reactions and valence, whereas other forms Elements of First Period
ri
stress the electronic configurations of elements. The first period corresponding to n = 1 is unique
However, the general plan of the table has remained because it contains only two elements. This is not
ll
the same as proposed by Mendeleev. Furthermore, it surprising because first energy shell (K) has only one
.A
is now recognized that the periodic law is essentially orbital (i.e. 1s) which can accommodate only two
the consequence of the periodic variation in electronic electrons. This means that there can be only two
configurations of atoms, which determine the physical elements in which one and two electrons are present
rs
and chemical properties of the elements and their in first energy level. The first period contains hydrogen
compounds. (1s1) and helium (1s2).
he
arranged in the order of increasing atomic numbers. period has eight elements in it. It starts with lithium
Long form of the periodic table is given in Fig. 1 ahead. (Z = 3) in which one electron enters the 2s-orbital.
Structural Features of the Long Form of the The period ends with neon (Z = 10) in which the second
P
horizontal rows called periods and vertical columns In third period corresponding to n = 3, there
er
called groups. There are altogether seven periods. are nine orbitals : one 3s, three 3p and five 3d.
These are numbered as 1, 2, 3, 4, 5, 6 and 7. The first However, we know from energy level diagram for
od
period contains 2 elements and the subsequent periods multielectron atoms (Unit 2) that 3d-orbitals are higher
consist of 8, 8, 18, 18, 32 and 32 elements respectively. in energy than 4s-orbitals. Consequently, 3d-orbitals
Fourteen elements of both sixth and seventh periods are filled after filling 4s-orbital. Hence, this period
M
(called lanthanoids and actinoids respectively) are involves the filling of only four orbitals (3s and 3p) and
placed in separate columns at the bottom of the periodic contains eight elements from sodium
©
M
K
7
od
⎯⎯→ IA IIA IIIB IVB VB VIB VIIIB VIIIB IB IIB IIIA IVA VA VIA VIIA VIIIA
GROUP = (1) (2) (3) (4) (5) (6) (7) (8) (9) (10) (11) (12) (13) (14) (15) (16) (17) (18)
M
7 87 Fr 88 Ra 89 Ac** 104 Rf 105 Db 106 Sg 107 Bh 108 Hs 109 Mt 110 Ds 111Rg 112Cn 113 Nh 114 Fl 115 Mc 116 Lv 117 Ts 118 Og
rs
K
K
7s1 7s2 6d 1 7s2 6d 2 7s2 6d 3 7s2 6d 4 7s2 7s2 6d 5 6d 6 7s2 6d7 7s2 6d8 7s2 6d10 7s1 6d10 7s2 7s2 7p1 7s2 7p2 7s2 7p3 7s2 7p4 7s2 7p5 7s2 7p6
⎯⎯⎯⎯⎯⎯⎯⎯⎯⎯⎯⎯⎯⎯⎯⎯⎯⎯→
CLASSIFICATION OF ELEMENTS AND PERIODICITY IN PROPERTIES
1/11/2020, 2:10 PM
Actinoids** 5f ° 5f 2 5f 3 5f 4 5f 6 5f 7 5f 7 5f 9 5f 10 5f 11 5f 12 5f 13 5f 14 5f 14
5f1–146d0-27s2 6d 2 7s2 6d 1 7s2 6d 1 7s2 6d1 7s2 6d 0 7s2 6d 0 7s2 6d 1 7s2 6d 0 7s2 6d 0 7s2 6d 0 7s2 6d 0 7s2 6d 0 7s2 6d 0 7s2 6d 1 7s2
ts
←⎯⎯⎯⎯⎯⎯⎯⎯⎯⎯⎯⎯⎯⎯⎯⎯⎯⎯⎯⎯⎯ Inner Transition Elements ⎯⎯⎯⎯⎯⎯⎯⎯⎯⎯⎯⎯⎯⎯⎯⎯⎯⎯⎯⎯⎯⎯⎯→
∗ Discovery of these elements have been claimed
re
se
Fig 1. Long form of the periodic table.
3/7
rv
ed
.
M
K
and are filled later). In between 4s-and 4p-orbitals, five This also provides a theoretical justification for
3d-orbitals are filled which have energies in between periodicity occurring at regular intervals of 2, 8, 8,
these orbitals. Thus, in all nine orbitals are to be filled 18, 18 and 32. These numbers i.e., 2, 8, 18 and 32 are
and, therefore, there are eighteen elements in fourth also called magic numbers. The number of elements
period from potassium (Z = 19) to krypton (Z = 36). In and the corresponding orbitals being filled are given
.
this period, we come across elements which involve in Table 5.
ed
the filling of 3d orbitals. These are known as transition The first three periods containing 2, 8 and 8
series of elements. This starts from scandium (Z = elements respectively are called short periods, the
rv
21) which has electronic configuration 3d1 4s2 and ends next three periods containing 18, 18 and 32 elements
at zinc (Z = 30) with 3d orbitals completely filled having respectively are called long periods.
Groups
se
electronic configuration 3d10 4s2.
A vertical column in the periodic table is
Elements of Fifth Period known as group.
The fifth period, like the fourth period also In terms of electronic structure of the atom, a
re
consists of 18 elements. It begins with rubidium group constitutes a series of elements whose atoms
(Z = 37) with filling of 5s-orbital and ends at xenon have the same outermost electronic configurations.
There are 18 groups in the long form of the periodic
ts
(Z = 54) with the filling up of the 5p-orbitals. This period
also contains ten elements of 4d transition series table. According to the new recommendations of
International Union of Pure and Applied Chemistry
gh
starting at yttrium (Z = 39).
(IUPAC), the groups are numbered from 1 to 18.
Elements of Sixth Period Previously, these were numbered only from I to VIII
The sixth period contains 32 elements (Z = 55 as A and B groups. Both the systems of numbering the
ri
to 86) and successive electrons enter into 6s, 4f, 5d groups is given in the periodic table. However, the old
and 6p-orbitals, in that order. It starts with cesium convention is still used in many books.
ll
(Z = 55) and ends at radon (Z = 86). In addition to ten The first two groups on the extreme left and last
six groups on the extreme right involve the filling of
.A
elements of 5d transition series, this period contains
s-and p-orbitals, respectively. These groups represent
14 elements which involve filling up of 4f orbitals
the main groups of the periodic table and are numbered
beginning from cerium (Z = 58) to lutetium (Z = 71). as 1, 2, 13, 14, 15, 16, 17 and 18 corresponding to
rs
The series of elements are called inner transition general configurations of ns1, ns2, ns2 np1, ns2 np2, ns2
series or lanthanoid series. np3, ns2 np4, ns2 np5, ns2 np6, respectively (in the old
he
Elements of Seventh Period system these were denoted as IA, IIA, IIIA, IVA, VA,
VIA, VIIA and VIIIA (or zero) respectively corresponding
The seventh period, is similar to the sixth
to general configurations of ns1, ns2, ns2 np1,..........ns2
lis
been recently established and named. This period also 3d-subshells also and this results into ten more groups.
contains 14 elements which involve the filling of 5f These ten groups lie in between first two and last six
orbitals starting from actinium (Z = 89). This is also groups, (see periodic table) i.e., between group 2 (IIA)
n
called 5f– inner transition series or actinoid series. and group 13 (IIIA) because 3d-orbitals are filled after
er
First (1) 1s 2 =2
Second (2) 2s 2p 2+6 =8
©
M
K
4s-orbitals but before 4p-orbitals. These are numbered a systematic nomenclature for naming the new
from 3 to 12 (in the old system these were usually elements until their names are officially recognized.
labelled as B groups i.e., IB, IIB, IIIB,...etc.). The This nomenclature is based on the Latin words for
elements present in these groups are called transition their numbers.
elements. The name is derived from the fact that they
The IUPAC Rules for Nomenclature of
represent transition (change) in character from reactive
.
Elements with Z > 100 are :
metals (elements of groups 1 and 2) on one side to the
ed
(i) The name of the element should be derived
non-metals (elements of group 13 to 18) on the other
directly from the atomic number using the numerical
side.
roots for their numbers. For example, nil for 0, un for
There are two more rows at the bottom of the
rv
periodic table. These rows consist of fourteen elements 1, bi for 2, tri for 3 and so on as given in Table 6.
after lanthanum (Z = 57) and fourteen elements which Table 6. Notations for numbers (word root) used by
se
follow actinium (Z = 89). These are placed separately in IUPAC Nomenclature of Elements.
the periodic table to save space and avoid undue Digit Root Name Abbreviation
sidewise expansion of the periodic table. This
0 nil n
re
arrangement also helps in keeping elements with
similar properties in a single column. The elements in 1 un u
the first row, starting from cerium are called 2 bi b
lanthanoids (or lanthanides) and the elements 3 tri t
ts
present in the second row starting from thorium are 4 quad q
called actinoids (or actinides). These lanthanoids 5 pent p
gh
and actinoids together are called inner transition 6 hex h
elements or rare earth metals and these are built up 7 sept s
by filling of f-orbitals. 8 oct o
ri
NOMENCLATURE OF ELEMENTS WITH 9 enn e
ATOMIC NUMBER MORE THAN 100 ll (ii) The names are derived by combining together
Today, we have elements upto 118. These the roots in the order of digits which make up the
elements were earlier named traditionally by its atomic number and ium is added at the end.
.A
discoverer or discoverers. However, in recent years, (iii) In certain cases, the names are shortened.
disputes regarding the original discoveries of some For example, if enn occurs before nil, the second n
elements of atomic number 104 and above have arisen. should be dropped and written as ennil. Similarly, the
rs
This is because the new elements with very high final i of bi and tri should be dropped when these occur
atomic numbers are very unstable and only minute before ium as bi + ium ⇒ bium, tri + ium = trium
he
quantities of elements are obtained. This has led to (iv) The symbol of the element is obtained from
controversy in deciding the claims of more than one the first letters of the numerical roots (abbreviations)
of the digit which make up the atomic number of the
scientist for the discovery and names assigned by
lis
element.
them for the new element discovered.
For example, for atomic number 104, the name
For example, both American and Soviet scientists may be written as :
claimed credit for discovery of element 104. The
ub
M
K
.
**118 Ununoctium Uuo Oganesson Og
ed
* The element 112 has been named as copernicium with chemical symbol Cn. It is named in honour of scientist astronomer
Nicolaus Copernicus. The element 114 is named Flerovium with chemcial symbol Fl to honour the Flerov Laboratory of
rv
Nuclear Reactions and the element 116 is named Livermorium with chemical symbol Lv to honour the Lawrence
Livermore National Laboratory.
** Recently, IUPAC have confirmed the discovery of four new elements with atomic numbers 113, 115, 117 and 118 and named
se
them. Therefore, the seventh row of the periodic table is now complete.
re
f-BLOCKS The general characteristics of s-block elements are :
The long form of the periodic table can be divided (i) They are soft metals having low melting and
into four main blocks. These are s, p, d and f-blocks. boiling points.
ts
This division of elements is based upon the electronic (ii) They have low ionisation enthalpies.
configurations of the atoms. In this division, the (iii) They are very reactive and readily form
gh
elements which involve the filling of a particular univalent (alkali metals) or bivalent (alkaline
orbital (i.e., s, p, d or f) are grouped together as shown earth metals) ions by losing one or two valence
in Fig. 2. (These are also shown in Long form of the electrons respectively. Because of their high
ri
Periodic Table. reactivity, they are never found pure in nature.
(iv) They act as strong reducing agents.
ll (v) Most of them impart characteristic colours to
the flame.
.A
(vi) They mostly form ionic compounds except
N lithium and beryllium.
O (ii) p-block elements
rs
B
L The elements in which the last electron enters the
E p-orbital of their outermost energy level are called
he
G
ns1–2 1–6
ns2 np A p-block elements. The elements of groups 13 to 18
1–10 0–2
S
E
involving addition of one (ns2 np1), two (ns2 np2), three
(n – 1)d ns
S (ns2 np3), four (ns2 np4), five (ns2 np5), and six (ns2 np6)
lis
1–14 0–2 2
block may be written as :
(n – 2)f (n – 1)d ns
General electronic configuration of p-block
P
M
K
.
to lose or gain electrons. Therefore, the noble gases The general characteristics of d-block elements are :
ed
exhibit very low reactivity. Preceding the noble gas (i) They are metals having high melting and
family are two chemically important groups of non- boiling points.
metals. These are halogens (group 17) and (ii) Most of them form coloured compounds.
rv
chalcogens (group 16). These two groups of elements (iii) They have a good tendency to form complex
can readily accept one and two electrons respectively compounds.
se
to attain noble gas configurations and form univalent (iv) Their compounds are generally paramagnetic.
and divalent negative ions. In these elements, the non- (v) They exhibit several oxidation states.
metallic character increases as we move from left to (vi) Most of the transition elements such as Mn,
re
right across a period and metallic character increases Ni, Co, Cr, V, Pt and their compounds are
as we go down the group. used as catalysts.
(iii) d-block elements (iv) f-block elements
ts
The elements in which the last electron enters The elements in which the last electron enters
the d-orbitals of their last but one (called penultimate) the f-orbitals of their atoms are called f-block
gh
energy level constitute d-block elements. This block elements. In these elements, the last electron is added
consists of the elements lying between s and p blocks to the third to the outermost (called antipenultimate)
starting from fourth period and onwards. They
ri
energy level; (n – 2) f. These consist of two series of
constitute groups 3 to 12 in the periodic table. In these elements placed at the bottom of the periodic table :
elements the outermost shell contains one or two ll (i) The first series follows lanthanum, La (Z=57)
electrons in their s-orbital (ns) but the last electron and the elements present in this series (58Ce – 71Lu)
.A
enters the last but one d-subshell i.e., (n – 1)d. The are called lanthanoids or lanthanides. These are
general electronic configuration for the atoms of d- also called rare earth elements.
block may be written as :
(ii) The second series follows actinium,
rs
General electronic configuration : (n – 1)d1–10 ns0–2 Ac (Z = 89) and the elements present in this series
(90Th – 103Lr) are called actinoids or actinides. These
he
active metals of s-block elements and the less active For example, for lanthanoids, the general
elements of Groups 13 and 14. Therefore, they electronic configuration is 4f1—14 5d0—1 6s2 while for
P
represent transition (change) in behaviour and take actinoids, it is 5f1—14 6d0 – 2 7s2.
their familiar name “transition elements.” The elements included in these two series are
The d-block comprises three complete rows of
n
These series are also called transition series. The general characteristics of f-block elements are :
First transition series (i) They are heavy metals.
Scandium (Z = 21) to Zinc (Z = 30) (ii) They generally have high melting and boiling
M
M
K
and their chemistry is not fully studied. The in the Periodic Table. It may be noted that the change
elements after uranium are called from metallic to non-metallic character is not abrupt
transuranium elements. as shown by zig-zag line in Fig 3 ahead. The elements
On the basis of above four blocks, the elements bordering this line (silicon, germanium, arsenic,
can be classified into the following four general types : antimony, tellurium, etc.) show characteristic
.
properties of both metals and non-metals. These are
ed
Classification of Elements into four General called metalloids or semimetals.
Types
Advantages of the Long Form of the Periodic
1. Noble gases. The noble gases are found at Table
rv
the end of each period in group 18. Except for helium, The important advantages of the long form of
the elements have completely filled s- and p-orbitals the periodic table are given below :
se
of the outermost shell i.e., ns2 np6. Helium has 1s2 (i) This classification is based on the atomic number
configuration. All these elements are highly stable and which is a more fundamental property of the
chemically inert under ordinary conditions.
re
elements.
2. Representative elements (s and p-block (ii) Since this classification is based on the atomic
elements). All the elements of s and p-block with number and not on the atomic mass, the position
ts
exception of noble gases are called representative of placing isotopes at one place is fully justified.
elements. They represent two groups 1 (alkali metals) (iii) The position of elements in the periodic table is
and 2 (alkaline earth metals) on the extreme left and
gh
governed by the electronic configurations, which
five groups from group 13 to 17 on the right hand side determine their properties.
of the periodic table.
(iv) It is easy to remember and reproduce.
ri
3. Transition elements. The elements of d- (v) The systematic grouping of elements into four
block elements are called transition elements. These blocks; s, p, d and f has made the study of the
include elements of group 3 to 12 lying in between the ll elements more simple.
representative elements (between s and p-blocks). (vi) The position of some elements which were misfit
.A
4. Inner transition elements. The elements on the basis of atomic mass is now justified on
of f-block are called inner transition elements. These the basis of atomic number. For example, argon
comprise two series of 14 elements called lanthanoids proceeds potassium because argon has atomic
rs
In addition to classifying the elements into properties different from other groups are placed
s-, p-, d- and f–block, the elements can be classified on separately at the bottom of the periodic table.
the basis of their properties as :
lis
elements and appear on the left hand side of the elements, yet it has certain defects. The main defects
Periodic Table. They have high melting and boiling of this table are :
points and are generally solid at room temperature
P
melting points, 303 K and 302 K respectively. They (ii) Lanthanoids and actinoids have not been
accommodated in the main body of the periodic
er
usually solids or gases at room temperature with low predicted from the electronic configuration of
melting and boiling points (boron and carbon are the elements.
exceptions). The only liquid non-metal is bromine. Most
1. Period of the element corresponds to the
©
M
K
13
od
er
n
Group 1 P 18
Ist Period 1 2
H 2 13 14 15 16 17 He
3 4 5 6 7 8 9 10
ub
2nd Period
Li Be Transition Elements B C N O F Ne
3rd Period 11 12 13 14 15 16 17 18
lis
Na Mg 3 4 5 6 7 8 9 10 11 12 Al Si P S Cl Ar
4th Period 19 20 21 22 23 24 25 26 27 28 29 30 31 32 33 34 35 36
he
K Ca Sc Ti V Cr Mn Fe Co Ni Cu Zn Ga Ge As Se Br Kr
5th Period 37 38 39 40 41 42 43 44 45 46 47 48 49 50 51 52 53 54
M
M
rs
K
K
Rb Sr Y Zr Nb Mo Tc Ru Rh Pd Ag Cd In Sn Sb Te I Xe
55 56 57 72 73 74 75 76 77 78 79 80 81 82 83 84 85 86
CLASSIFICATION OF ELEMENTS AND PERIODICITY IN PROPERTIES
6th Period
Cs Ba La* Hf Ta W Re Os Ir Pt Au Hg Tl Pb Bi Po At Rn
.A
7th Period 87 88 89 104 105 106 107 108 109 111 110 112 113 114 115 116 117 118
1/11/2020, 2:10 PM
Lanthanoids
Metalloids Ce Pr Nd Pm Sm Eu Gd Tb Dy Ho Er Tm Yb Lu
90 91 92 93 94 95 96 97 98 99 100 101 102 103
ts
Actinoids
Non-metals
Th Pa U Np Pu Am Cm Bk Cf Es Fm Md No Lr
*Discoveries of these elements have been claimed
re
se
Fig 3. Position of metals, non-metals and metalloids in the periodic table.
rv
3/13
ed
.
M
K
3. Group is predicted from the number of Solution: For n = 4, the electronic configuration is
3d14s2. The element belongs to d-block elements.
electrons in the outermost shell or
penultimate (last but one, i.e., n–1) shell as Group number = No. of electrons in (n – 1)d subshell
follows : + No. of electrons in nth shell
For s-block element, group number is equal to number of =1+2=3
.
valence electrons (ns electrons). Example 6.
ed
For p-block element, group number is equal to (a) Elements A, B, C and D have atomic numbers
10 + number of valence electrons (ns and np). 12, 19, 29 and 36 respectively. On the basis of
rv
For d-block element, group number is equal to total electronic configuration, write to which group
number of electrons in (n–1)d and ns subshells. of the periodic table each element belongs.
se
(b) Predict the blocks to which these elements can
be classified. Also predict their periods and
groups.
re
Example 1. (c) Which of these are representative elements ?
What would be the IUPAC name and symbol of the Solution: (a)
element with atomic number 120 ?
ts
Element At. No. Electronic Group
Solution: The roots for 1, 2 and 0 are un, bi and nil configuration
gh
respectively.
A 12 1s2 2s2 2p6 3s2 2
∴ Name of element : Unbinilium
B 19 1s2 2s2 2p6 3s2 3p6 4s1 1
ri
Symbol : Ubn
C 29 1s2 2s2 2p6 3s2 3p6 3d10 4s1 11
Example 2.
ll D 36 1s2 2s2 2p6 3s2 3p6 3d10 4s2 4p6 18
How would you justify the presence of 18 elements
[According to old system, these elements belong to
.A
in the 5th period of the periodic table ?
groups : IIA (A), IA (B), IB (C), VIIIA or zero (D)]
Solution: The 5th period involves the filling of 5th (b) Prediction of blocks
shell (n=5). This period has 4d, 5s and 5p available orbitals.
rs
The total number of available orbitals are 9 and therefore (i) A receives the last electron in 3s-subshell, therefore, it
the maximum number of electrons that can be accommodated belongs to s-block.
is 18. Thus, 5th period contains 18 elements.
he
Period = 3rd
Example 3. Group = No. of valence electrons = 2
The elements Z = 107, 108 and Z = 109 have been (ii) B receives the last electron in 4s-orbital, therefore, it
lis
Solution: We know that metallic character increases (c) The elements A and B are representative elements.
down a group and decreases along a period as we move
from left to right. Example 7.
©
M
K
.
(i) The third alkali metal.
ed
Name the elements. Out of these which
(ii) Second transition element.
1. is an alkaline earth metal 2. has lowest chemical
(iii) The fourth noble gas.
reactivity 3. belong to group 15 of the periodic table
rv
4. is a transition element 5. is halogen 6. belong to (iv) Fourth element in the second period.
p-block 7. belong to second period. 2. The outer electronic configurations of some elements
se
Solution: are :
Names of elements: (i) 3s2 3p4 (ii) 3d10 4s2
re
(i) Sodium (Na) (ii) Scandium (Sc) (iii) 3s2 3p6 4s2 (iv) 6s2 4f 3
(iii) Nitrogen (N) (iv) Calcium (Ca)
State to which block in the periodic table each of
(v) Chlorine (Cl) (vi) Phosphorus (P) these elements belong.
ts
(vii) Neon (Ne) (viii) Sulphur (S).
3. How many elements are present in the third period
1. Calcium (iv) is an alkaline earth metal.
of the p-block elements ?
gh
2. Neon (vii) has lowest chemical reactivity.
4. Complete the following statements :
3. Nitrogen (iii) and phosphorus (vi) belong to group
15 of the periodic table. (a) There are.........................periods in the long form of
ri
4. Scandium (ii) is a transition element. the periodic table.
5. Chlorine (v) is halogen. ll (b) The s block element having highest atomic mass
6. Nitrogen (iii), chlorine (v), phosphorus (vi), neon is ..................... .
(vii) and sulphur (viii) belong to p-block.
.A
(c) The elements Cu, Ag and Au are called
7. Nitrogen (iii) and neon (vii) belong to second ....................... metals.
period of the periodic table.
5. An element X belongs to the third period of the
rs
in the periodic table ?
order of non-metallic character :
Solution: We know that the elements having similar
B, C, Si, N and F.
n
electrons in the valence shell belong to the same group i.e., what will be the atomic number of the next noble
group 13 of the periodic table. gas to be discovered. Write its symbol.
od
M
1. (i), K, 19 (ii) Ti, 22 (iii) Kr, 36 (iv) C, 6. 6. Oxygen family. This family is also called chalcogens.
©
M
K
.
ed
Q.1. On the basis of quantum numbers, justify that the sixth period of the periodic table should have 32 elements.
rv
Ans. The sixth period of the periodic table begins with n=6. It involves the filling of 6s, 4f, 5d and 6p subshell in the increasing
order of energy. The total number of orbitals available are 16 and therefore, the maximum number of electrons that
se
can be accommodated is 32. Thus, the sixth period of the periodic table should have 32 elements.
Q.2. In terms of period and group where would you locate the element with Z = 114 ?
re
Ans. The electronic configuration of the element with Z=114 would be [Rn] 5f14 6d10 7s2 7p2. Since it has n=7 for the valence
shell, it belongs to 7th period. It receives the last electron in p-orbital. Therefore, it belongs to p-block. The group
number will be 10 + 4 (No. of electrons in ns and np orbitals) = 14
ts
So, it belongs to 7th period and 14th group.
gh
Q.3. The elements Z = 117 and 118 have been recently discovered and named. In which family group would you
place these elements and also give electronic configuration in each case. Name these elements.
Ans. The electronic configuration of the element with Z = 117 would be [Rn] 4f 14 5d10 7s2 7p5.
ri
It has outermost ns 2np 5 configuration and therefore, it belongs to halogen family or Group 17.
IUPAC name: Tennessine, Ts ll
The electronic configuration of the element with Z=118 would be [Rn] 4f 14 5d10 7s2 7p6. It has outermost ns2np6
.A
configuration and therefore, belongs to noble gases or group 18. IUPAC name: Oganesson, Og.
Q.4. Write the atomic number of the element present in the third period and sixteenth group of the periodic table.
rs
= 16. It is sulphur.
Q.5. Give the general electronic configurations of
lis
Q.6. An element ‘X’ with atomic number 112 has been recently predicted. Its electronic configuration is :
[Rn] 5f 14 6d10 7s2. Predict
(i) group and (ii) block in which this element would be placed (iii) IUPAC name and symbol.
P
(i) It belongs to 12th group. (ii) It belongs to d block. (iii) IUPAC name is : Copernicium; Symbol : Cn.
Q.7. The element 119 has not been discovered so far. What would be the IUPAC name and the symbol for this
od
element ? On the basis of periodic table, predict the electronic configuration of this element and the formula
of its most stable chloride and oxide. (NCERT Exemplar Problem)
Ans. IUPAC name : Ununennium
M
Symbol : Uue
Electronic : 1s2 2s2 2p6 3s2 3p6 3d10 4s2 4p6
©
configuration 4d10 4f14 5s2 5p6 5d10 5f14 6s2 6p6 6d10 7s2 7p6 8s1 or [At] 8s1
Formula : Chloride : ACl
: Oxide : A2O
M
K
Q.8. Eka-aluminium and eka-silicon were named given by Mendeleev for the then unknown elements gallium
and germanium respectively. A recently discovered element has first named as eka-mercury. What is its
atomic number? Write its group number, electronic configuration, IUPAC name, official name and symbol.
Ans. The element which comes after mercury in the periodic table is eka-mercury. Its atomic number = 80 + 32 = 112
.
Electronic configuration: [Rn] 5f14 6d10 7s2
ed
IUPAC name : Uub
Official name: Copernicium, symbol : Cn
rv
Q.9. Write the IUPAC names, official names and symbols for the elements having atomic numbers 108, 114 and
116.
se
Ans. Z = 108 Unniloctium Uno ; Hassium Hs
re
Z = 114 Ununquadium Uuq ; Flerovium Fl
Z = 116 Ununhexium Uuh ; Livermorium Lv
ts
CAUSES OF PERIODICITY Table 8. Electronic configurations of alkali metals.
gh
The recurrence of similar properties of the Element At. No. Electronic configuration
elements after certain regular intervals when
Li 3 1s22s1
they are arranged in the order of increasing
ri
Na 11 1s22s2 2p6 3s1
atomic numbers is called periodicity.
The classification of the elements in certain ll K 19 1s22s2 2p6 3s2 3p6 4s1
families was based upon the observed similarities Rb 37 1s22s2 2p6 3s2 3p6 3d10 4s2 4p6 5s1
.A
in the physical and chemical properties. But, why Cs 55 1s22s2 2p6 3s2 3p6 3d10 4s2 4p6 4d10 5s25p6
do certain elements exhibit similar properties and 6s1
why do the similar properties recur at regular
rs
the nuclei of the atoms do not undergo any change and, configuration of their atoms in the outermost
therefore, it is the distribution of the electrons in the energy shell (or valence shell) after certain
ub
various shells of their atoms that determine the physical regular intervals.
For example, all the alkali metals have a great
and chemical properties of the elements. Further, it
tendency to lose the single electron in order to acquire
has been also found that the properties of atoms do not
P
inner shells. Thus, it must primarily be the Similarly, all the members of the halogen family
arrangement of electrons in the outermost shell (called have two electrons in the s-orbital and five electrons
er
the valence shell) that determines the properties of the in the p-orbital of the valence shell (Table 9). The
atoms. For example, the electronic configurations of general electronic configuration of halogens may be
od
the atoms of alkali metals group (Table 8) shows that expressed as ns2 np5.
they all have one electron in the s-orbital of the Table 9. Electronic configurations of halogens.
M
M
K
.
ELEMENTS
ed
Internuclear distance between
There are numerous physical properties of rcovalent =
two bonded atoms
elements such as melting points, boiling points, enthalpy 2
of fusion, enthalpy of vaporization, enthalpy of
rv
atomization, density, etc., which show periodic For example, as shown in Fig. 4., the internuclear
variations. These are indirectly related to electronic distance between two hydrogen atoms in H2 molecule
se
configurations of atoms. However, some physical is 74 pm.
properties such as atomic size, ionization enthalpy,
electron gain enthalpy, electronegativity, valency, etc.,
re
are directly related to the electronic configuration of
atoms. Let us discuss some of the important properties
and their periodic trends.
ts
ATOMIC RADIUS
gh
The size of atom is very important property
because many physical and chemical properties are
related to it. If the atom is assumed to be spherical, the
ri
atomic size is given by the radius of the sphere and is
called atomic radius. Generally, the term atomic
radius means the distance from the centre of the
ll
nucleus to the outermost shell of electrons. However,
.A
it is difficult to determine the exact radius of the atom
because of the following reasons :
Fig. 4. Atomic radius of hydrogen atom.
(i) The size of an atom is (approximately 1.2Å or
rs
an atom does not have well defined boundary. The Similarly, the atomic radii of chlorine and bromine
probability of finding the electron is never zero even at are 99 pm and 114 pm because the internuclear
lis
large distances from the nucleus. distances in chlorine Cl—Cl and Br—Br are 198 pm
(iii) It is not possible to isolate an atom and and 228 pm respectively.
In the case of molecules containing different
measure its radius. The atomic radius of an atom is
ub
as the effective size which means the distance of are added, the resulting value agrees fairly well with
closest approach of one atom to another atom in a given the experimentally determined internuclear distance
od
distance) in a covalent molecule by X-ray diffraction, In case of metals, a large number of atoms are
electron diffraction or other spectroscopic techniques. closely packed and are held together by means of
As shown in Fig. 4., this internuclear distance metallic bonds. The close packing of the metal atoms
©
corresponds to twice the radius of an atom and, is known as metallic lattice. The metallic radius is
therefore, half of this distance gives the atomic radius. taken as
Atomic radii may be assigned different names such one half of the internuclear distance between
as covalent or metallic depending upon the type of
the two neighbouring atoms of a metal in a
bonding between the atoms.
metallic lattice.
M
K
For example, the distance between two adjacent This may be explained on the basis of increasing
copper atoms in metallic copper is 256 pm. Therefore, nuclear charge along a period. With the increase in
metallic radius of copper is 256/2 i.e., 128 pm. atomic number from lithium to fluorine, the magnitude
Thus, the covalent radius gives the size of a non- of the nuclear charge increases progressively by one
.
metallic element when the atoms are bound together unit while the corresponding addition of electron takes
ed
by a single covalent bond in a covalent molecule. For place in the same principal shell (second). Since, the
metals, the metallic radius gives the size of a metallic
element in metals held by metallic bonds. electrons in the same shell do not screen each other
rv
from the nucleus, the increase in nuclear charge is
It may be noted that for simplicity, we may use the not neutralised by the extra valence electron. As a
se
term atomic radius for both covalent or metallic radius result, electrons are pulled closer to the nucleus by
depending on whether the element is a non-metal or a
the increased effective nuclear charge and thereby,
metal. However, covalent radii or metallic radii are
re
cause a decrease in the size of the atom. In this way,
always shorter than the atomic radii in the
uncombined atoms. This is so because the covalent the atomic size goes on decreasing across the period.
bond is formed by the overlapping of atomic orbitals
ts
It may be noted from Fig. 5 and Table 10 that in
and the overlap region becomes common between the second period atomic size decreases from Li to N. After
two atoms.
gh
nitrogen atomic size increases for oxygen and then
Variation of Atomic Radii in the Periodic Table
decreases for fluorine. This anomalous behaviour of
The following periodic trends in atomic radii have been
oxygen and fluorine may be explained as follows.
ri
observed.
(a) Variation in a period. In general, the atomic radii In nitrogen, all the three 2p-orbitals have one
decrease with increase in atomic number (going from left to
right) in a period.
ll
electron each; 2s2 2px1 2py1 2pz1. When we move from
.A
nitrogen to oxygen, the nuclear charge increases by
For example, in the second period, the atomic radii
one. But at the same time, one of the 2p-orbital has two
decrease from Li to Ne through Be, B, C, N, O and F as
given in Table 10. The variation of atomic radius with atomic electrons in oxygen; 2s2 2px2 2py1 2pz1. The electrons in
rs
number in a second period is also shown in Fig. 5. same orbital repel each other. In case of oxygen, the
interelectronic repulsions outweigh the effect of increased
he
M
K
.
ed
its atomic no.
rv
(pm)
se
Thus, it may be concluded that in general
(except for noble gases)
• the alkali metals which are at the extreme
re
left of the periodic table have the largest size
Fig. 6. Variation of atomic radius with atomic number
in a period.
for alkali metals and halogens.
ts
• the halogens which are present at the
extreme right of the periodic table have the
Van der Waals Radius
gh
smallest size in the period.
We observe from Table 12, that atomic radius
• the atomic size decreases along the period
abruptly increases in case of noble gas element, Ne.
from left to right.
ri
This is because of the reason that the values for other
elements are for covalent radii while the value of Ne
(b) Variation in a group. The atomic radii of is not covalent radius because neon cannot form neon
ll
elements increase from top to bottom in a group. In (Ne2) molecule. In case of noble gas elements, we
.A
moving down a group, the nuclear charge is increasing measure van der Waals radius because these atoms
with increase in atomic number and we expect that the are held together by weak van der Waals forces in
size of atom should decrease. However, while going solid state. Like covalent radius, van der Waals radius
rs
from one atom to another, there is increase in the may also be defined as :
principal quantum level, although the number of
one half of the internuclear distance between
he
charge. Consequently, the distance of the outermost molecules in the solid state.
electron from the nucleus gradually increases down a For example, the internuclear distance between
group. In other words, the size of the atom goes on
ub
the alkali metals group (group 1) and halogens group radius of chlorine = = 180 pm or 1.80Å.
(group 17) are given in Table 12 and are graphically 2
represented in Fig. 6.
n
Group 1 Group 17
od
Na 186 Cl 99
K 231 Br 114
Rb 244 I 133
©
Cs 262 At 140
Fig. 7. Van der Waals radius of chlorine.
M
K
.
donot form chemical compounds and therefore, their
ed
atomic radii are usually expressed in terms of van der
Waals radii. These are also called inert gas radii.
The van der Waals forces existing between atoms
rv
in the solid state are weak and the atoms are held at
larger distances. Therefore, the internuclear distances Fig. 8. Internuclear distance and ionic radii.
se
in case of atoms held by van der Waals forces are larger Several methods have been proposed to fix the
than that between covalently bonded atoms. absolute value of atleast one ion (the details of these
Consequently, van der Waals radii are always
re
methods are beyond the scope of the present chapter).
larger than covalent radii. The radii of all other ions can easily be calculated by
Variation of van der Waals radii. Like subtracting the value of ionic radius of known ion from
ts
covalent radii, van der Waals radii also decrease measured internuclear distance in its compound. Out
as we move from left to right in a period and of the various methods, Pauling’s method is the most
increase as we move down the group. For example, widely accepted and the values given here are on the
gh
• van der Waals radii of N, O and F decrease in basis of this method.
the period: The study of ionic radii leads to two important
ri
generalizations :
N O F
150 pm 140 pm 135 pm (i) The radius of positive ion (cation) is always
• van der Waals radii increase down the group:
ll smaller than that of the parent atom.
.A
F Cl Br I (ii) The radius of negative ion (anion) is larger
135 pm 180 pm 195 pm 215 pm than that of the parent atom.
• van der Waals radii of noble gases also increase (i) The radius of cation is smaller than that
rs
down the group: of the atom. A cation is formed by the loss of one or
He Ne Ar Kr Xe more electrons from the gaseous atom. Generally, the
he
120 pm 160 pm 191 pm 200 pm 220pm whole of the outermost shell of electrons is removed so
that the resulting cation is smaller in size. For example,
Ionic Radius in sodium atom, there is only one electron in the
lis
The ionic radii correspond to the radii of ions in outermost 3s-subshell. As sodium atom changes to Na+
ionic crystals. The ions are formed as a result of ion, the outermost 3s-subshell disappears completely.
addition or removal of electrons from the outermost This disappearance of outermost 3s-subshell results in
ub
shells of atoms. The ions formed by the loss of electrons a decrease in size, and therefore, cation is smaller than
acquire positive charge and are called cations while the atom from which it is formed. It has been observed
the ions formed by gain of electrons, get negative that the size of Na atom is 186 pm whereas that of
P
charge and are called anions. Ionic radius may be corresponding Na+ ion is only 95 pm.
defined as Na ⎯⎯⎯→ Na+ + e–
n
the effective distance from the centre of the 1s2 2s22p63s1 (1s22s22p6)
er
nucleus of the ion upto which it has an (186 pm) (95 pm)
influence in the ionic bond. Similarly, in the formation of Mg2+ ion, both the
od
internuclear distance may be taken equal to the sum of (1s2 2s2 2p6 3s2) (1s2 2s2 2p6)
the radius of the cation and the radius of the anion. For (160 pm) (72 pm)
©
example, the internuclear distance of Na+Cl– is Moreover, with the removal of electrons from an
276 pm which corresponds to the sum of the ionic radii atom the magnitude of the nuclear charge remains
of Na+ and Cl– ions. But the problem of assigning same while the number of electrons decreases. For
different values to constituting ions is not simple. example, in the formation of sodium ion from sodium
Unless the value of any one of the ions is known, we atom, the nuclear charge remains + 11 while the
cannot calculate the radius of other ion. number of electrons decreases from 11 to 10.
M
K
− e−
Variation of Ionic Radii in a Group
Na ⎯⎯⎯→ Na+
The ionic radii in a particular group increase in
Atom Cation moving from top to bottom. The reason for the increase
Electrons 11 10 is the increase in the principal quantum level though
the number of electrons in the valence shell remains
.
Nuclear charge + 11 + 11
the same (similar to those discussed in case of atomic
ed
Size 186 pm 95 pm radii). For example, the ionic radii of alkali metal ions
As a result, the same nuclear charge now acts on increase from Li+ to Cs+.
less number of electrons. In other words, the effective
rv
Table 15. Ionic radii of alkali metal ions.
nuclear charge per electron increases and the
Ion Ionic radius (pm)
electrons are more strongly attracted and are
se
pulled towards the nucleus. This causes a Li +
60
Na+ 95
decrease in the size of the ion.
K+ 133
re
Table 13 illustrates the comparative size of certain Rb+ 149
atoms and the positive ions formed from them. Cs + 170
Table 13. Atomic and cationic radii of atoms (in pm).
Variation of Size in an Isoelectronic Series
ts
Li Na K Be Mg Ca A1 Isoelectronic ions are ions of different elements
152 186 231 111 160 197 143 which have same number of electrons but differ from
gh
Li+ Na+ K+ Be2+ Mg2+ Ca2+ A13+ one another in magnitude of the nuclear charge. A set
60 95 133 39 72 100 50 of species having the same number of electrons is
ri
known as isoelectronic series. One such type of series
(ii) The negative ion is always larger than is shown below in which all the species have ten
that of the corresponding atom. The negative ion electrons. As we move from one ion to another the
ll
is formed by the gain of one or more electrons in the nuclear charge increases and therefore, the force of
.A
neutral atom and the number of electrons increases attraction by the nucleus on the outermost electrons
while the magnitude of nuclear charge remains same. also increases. Consequently, the electrons are pulled
Therefore, the size of an anion will be larger than that more and more strongly and thus size decreases.
rs
the electrons and a decrease in effective nuclear charge. Ions N3– O2– F– Na+ Mg2+ Al3+
For example, in the formation of Cl– ion from Cl atom, Nuclear charge +7 +8 +9 +11 +12 +13
the number of electrons increases from 17 to 18 while Radius (pm) 171 140 136 95 72 50
lis
the nuclear charge remains same (+17). * All ions have 10 electrons.
– Thus, the ionic radii in the the above isoelectronic
Cl + e− Cl
⎯⎯⎯→
ub
series decreases as :
Atom Anion
Electrons 17 18 Al3+ < Mg2+ < Na+ < F– < O2– < N3–
Nuclear charge + 17 + 17
P
Size 99 pm 181 pm
Example 9.
n
neutral atom. In other words, effective nuclear Give the formula of a species that will be
charge per electron is reduced and the electron isoelectronic with the following atoms or ions :
od
–
cloud is held less tightly by the nucleus. This (i) F (ii) Ar
causes increase in the size of the ion. (iii) Mg2+ (iv) Rb+
Thus, as shown in Table 14, the anions are larger (b) Which of the following species are isoelectronic ?
M
in size than the corresponding atoms. (i) O2– (ii) Mg2+ (iii) Na (iv) F
Table 14. Covalent and ionic radii of some atoms (v) Cl– (vi) Al3+ (vii) Ne (viii) S2–
©
181 196 220 142 171 (iii) Mg2+ (10e–) : Na+ (iv) Rb+ (36e–) : Kr
M
K
.
(iv) F = 9
ed
(v) Cl– 17 + 1 = 18 in size :
(vi) Al3+ 13 – 3 = 10 (a) Li, F (b) O, Se (c) Fe2+, Fe3+ (d) Br, Br–
(vii) Ne = 10 (e) Na+, F– (f) K, K+
rv
(viii) S2– 16 + 2 = 18 13. Account for the difference in size of Na+ [0.095 nm]
(ix) Ca2+ 20 – 2 = 18 and Mg2+ [0.065 nm] both of which have the same
se
(x) K = 19 noble gas configuration.
Thus, O2–, Mg2+, Al3+ and Ne are isoelectronic and Cl–, S2– 14. Arrange each pair of ions in order of increasing ionic
and Ca2+ are isoelectronic. radius :
re
Example 10. (a) Mg2+ and Al3+ (b) O2- and S2– (c) O2– and F–
Which of the following species will have the largest 15. A boy has reported the radii of Cu, Cu+ and Cu2+ as
and the smallest size? 0.096 nm, 0.122 nm and 0.072 nm respectively.
ts
However, it has been noticed that he interchanged the
Mg, Mg2+, Al, Al3+
values by mistake. Assign the correct values to
Solution: Atomic radii decrease across a period. So, different species.
gh
Mg will have larger size than Al. Cations are smaller than 16. Arrange the following ions in the order of increasing
their parent atoms. So size is : Mg2+ < Mg and Al3+ < Al. Now, size :
among isoelectronic ions, the ion with the larger positive
ri
nuclear charge will have a smaller radius i.e., size is : Al3+ < Be2+, Cl–, S2–, Na+, Mg2+, Br–
Al. 17. Select from each group the species which has the
Hence, the largest species is Mg ll smallest radius:
the smallest species is Al3+ (a) O, O–, O2– (b) K+, Sr2+, Ar (c) Si, P, Cl.
.A
Example 11.
Which of the following atoms and ions are
isoelectronic ? 11. (i) I+ < I < I– (ii) O < N < P (iii) F < Cl < Br.
rs
–
(i) Al3+ (ii) F (iii) Cl 12. (a) Li (b) Se (c) Fe2+ (d) Br– (e) F– (f) K.
–
(iv) O2 (v) Na (vi) Mg2+
13. The nuclear charge in Mg2+ is more and therefore, its
he
Arrange the isoelectronic ions in the decreasing electrons are more strongly attracted.
order of their size. 14. (a) Al3+ < Mg2+ (b) O2– < S2– (c) F– < O2–
Solution: The number of electrons in these atoms or
lis
No. of Electrons 10 9 18 10 11 10
Thus Al3+, O2– and Mg2+ are isoelectronic ions because
all the three ions have ten electrons. Now nuclear
IONIZATION ENTHALPY
charge in Al3+ is + 13, in O2– is +8 and in Mg2+ is + 12.
P
With increase in nuclear charge (electrons remain The electrons in an atom are attracted by the
same), size will decrease. Consequently, the size positively charged nucleus. In order to remove an
n
follows the order : electron from an atom, energy has to be supplied. The
O2– > Mg2+ > Al3+
er
Solution: Be2+ < Mg2+ < Na+ < Cl– < S2– < Br– of the element in its ground state.
Example 13. Ionization enthalpy is also known as ionization
In each of the following pairs, which species has a potential because it is the minimum potential
©
M
K
1 eV per atom = 96.49 kJ mol–1 is because the attractive force between the nucleus
or = 23.06 kcal mol–1 and the electron increases with the increase in nuclear
Successive Ionization Enthalpies charge. The force of attraction is directly proportional
The atom may not only lose one electron but can to the product of charges on the nucleus and the
lose more than one electrons also. It may be noted electron. Therefore, with the increase in nuclear charge,
.
that if the gaseous atom is to lose more than one it becomes more difficult to remove an electron and
ed
electron, these are removed one after the other. The ionization enthalpy increases.
energies required to remove subsequent electrons from 3. Screening effect of the inner electrons. The
rv
an atom in the gaseous state are known as successive ionization enthalpy decreases with increase in screening
ionization enthalpies. effect of the inner electrons. In multielectron atoms,
se
Thus, the first ionization enthalpy, IE1 is the energy the outermost electrons are shielded or screened from
required to remove the most loosely bound electron of the nucleus by the inner electrons. This is known as
the neutral atom and the second ionization enthalpy, shielding or screening effect. As a result of this,
re
IE2 is the energy required to remove the second
the outermost electron does not feel the full charge of
electron from the resulting positive ion and so on.
the nucleus. The actual charge felt by an electron is
Thus, first ionization enthalpy of an element (X) may
termed as effective nuclear charge. Therefore, the
ts
be defined as the enthalpy change (ΔiH) for the reaction
represented as : effective nuclear charge (Zeff) is
+
gh
X (g) ⎯⎯→ X (g) + e– (g) (Zeff) = Total nuclear charge (Z) – Screening constant (s)
In other words, first ionization enthalpy is
where screening constant takes into account the
the enthalpy change when most loosely
screening effect of the inner electrons. If the number
ri
bound electron is removed from an isolated
of electrons in the inner shell is large, the screening
gaseous atom.
effect will be large. As a result, the nuclear electron
ll
Similarly, we can define second ionization
attraction will be less. Consequently, ionization
enthalpy as the energy required or enthalpy
.A
enthalpy will decrease.
change to remove the second most loosely bound
electron. In other words, it is the enthalpy change 4. Penetration effect of electrons. The
for the reaction : ionization enthalpy increases with increase in
rs
+ 2+
X (g) ⎯⎯→ X (g) + e– (g) penetration power of the electrons. It is well known
Since energy is required to remove electrons that in case of multielectron atoms, the electrons in
he
from an atom and therefore, ionization enthalpies are the s-orbital have the maximum probability of being
always positive. found near the nucleus and this probability goes on
lis
Thus, the ionization enthalpy gives the ease with decreasing in case of p, d and f-orbitals. In other words,
which electron can be removed from an atom. s-electrons are more penetrating towards the nucleus
Evidently, the smaller the value of ionization than p-electrons and the penetration power decreases
ub
enthalpy, the easier it is to remove the electron in a given shell (same value of n) in the order
from the atom. s>p>d>f
Factors Governing Ionization Enthalpy
P
decreases with increase in size of an atom. This is be more to remove a s-electron than the energy required
because the attractive force between the electron and to remove a p-electron, which in turn will be more than
od
by the nucleus. As a result, it becomes easier to remove enthalpy also depends upon the electronic configuration
of the atom. It has been observed that certain electronic
the electron and therefore, ionization enthalpy decreases
configurations are more stable than others. For
©
M
K
configurations and ionization enthalpy is high. This may It can be seen that in each period, the maxima
be illustrated by the following examples : are found at the noble gases while minima are found
(i) The noble gases have the most stable electronic at alkali metals. Thus, the metals of group-I with one
configurations (ns2 np6) in each period and electron in outermost s-orbital are easy to ionise while
therefore, have high ionization enthalpies. the noble gases (group 18) with ns2 np6 configuration
.
(ii) The elements like Be (1s2 2s2) and Mg (1s2 2s2 2p6 are the most difficult to ionise. The following periodic
ed
3s2) have completely filled orbitals and their trends have been observed :
ionization enthalpies are large. (a) Variation along a period. In general, the
(iii) The elements like N (1s2 2s2 2px1 2py1 2pz1) and ionization enthalpy increases with increasing atomic
rv
P (1s2 2s 2 2p6 3s2 3px1 3py1 3pz1) have the number in a period. This is quite clear from the values
configurations in which the orbitals of same of ionization enthalpy of the second period elements
se
subshell are exactly half-filled and are also stable. as given in Fig. 10.
Hence, they need large energy to remove the
electron i.e., their ionization enthalpies are high.
re
Thus, the more stable the electronic configuration,
the greater is the ionization enthalpy.
ts
Variation of Ionization Enthalpy in the Periodic
Table
Ionization enthalpy provides another example for
gh
understanding periodicity among the elements. The
ionization enthalpies of some elements are given in
Table 16.
ri
The variation of first ionization enthalpy of
elements with atomic number (up to atomic number ll
60) is shown in Fig. 9. Fig. 10. Variation of ionization enthalpy with
.A
atomic number in second period.
keeps on increasing.
However, some irregularities in the general trend
n
Li Be B C N O F Ne
520 899 801 1086 1402 1314 1681 2080
Na Mg Al Si P S Cl Ar
©
M
K
.
Be to B. Although the nuclear charge of B is more
ed
than Be, yet there is slight decrease in ionization
enthalpy from Be to B. This is due to the fact that
(i) the electronic configuration of B (1s2 2s2 2p1) is
rv
less stable than that of Be (1s2 2s2 ) which has
completely-filled orbitals.
se
(ii) When we consider the same principal quantum
shell, an s-electron is attracted to the nucleus
more than a p-electron. In Be, the electron
re
removed during ionization is a 2s-electron
whereas the electron removed during ionization Fig. 11. The variation of ionization enthalpy with atomic
of B is a 2p-electron. Thus we know that the number in group 1 (alkali metals) of the periodic table.
penetration of a 2s-electron to the nucleus is
ts
more than that of a 2p-electron and therefore, 2p (iii) There is increase in shielding effect on the
electron of boron is more shielded from the inner outermost electron due to increase in the
gh
core of electrons than the 2s electron of Be. number of inner electrons.
Hence, it is easier to remove the 2p electron from The effect of increase in atomic size and the
B compared to the removal of a 2s electron of Be.
ri
shielding effect is much more than the effect of increase
Thus, B has smaller ionization enthalpy than Be.
in nuclear charge. As a result, the electron becomes
As a result the 2p-electron of B is not tightly held
less and less firmly held to the nucleus as we move
ll
by the nucleus as 2s-electron of Be and hence ionization
enthalpy of B is less than that of Be. down the group. Hence, there is a gradual decrease in
.A
B to C to N. As we move from B to C to N, the ionization enthalpies in a group.
ionization enthalpy keeps on increasing due to Variation in Successive Ionization Enthalpies
increasing nuclear charge and decreasing atomic size. The energies required to remove subsequent
rs
N to O. Oxygen, the element next to nitrogen electrons from the atom in the gaseous state, are known
has slightly smaller ionization enthalpy as compared as successive ionization enthalpies. The term first,
he
to that of nitrogen. It is due to the fact that the second, third. .......... ionisation enthalpy refers to the
electronic configuration of nitrogen atom (1s2, 2s2 2px1 removal of first, second, third..............electron
2py1 2pz1), in which 2p-orbital is half-filled, is more respectively. These changes may be represented as
lis
the maximum ionization enthalpy in the period due to Here , IE1, IE2 and IE3 represent first, second and
the stable (ns2 np6) electronic configuration. third ionization enthalpies respectively. For example,
n
Similar variation in ionization enthalpy of the Li (g) ⎯⎯→ Li+ (g) + e– IE1 = 520 kJ mol–1
er
elements of third period has also been observed. Li+ (g) ⎯⎯→ Li2+ (g) + e– IE2 = 7298 kJ mol–1
(b) Variation down a group. Within a group, Similarly, the successive ionization enthalpies of
there is a gradual decrease in ionization enthalpy in
od
aluminium are :
moving from top to bottom. This is clear from the
Al (g) ⎯⎯→ Al+ (g) + e– IE1 = 577 kJ mol–1
ionization energy values of the elements of the first
group as shown in Fig. 11. Al+ (g) ⎯⎯→ Al2+ (g) + e– IE2 = 1795 kJ mol–1
M
The decrease in ionization enthalpy down a group Al2+(g) ⎯⎯→ Al3+ (g) + e– IE3 = 2758 kJ mol–1
can be explained in terms of net effect of the following Successive ionization enthalpies are higher
As can be seen, the second ionization enthalpies are
©
factors :
(i) In going from top to bottom in a group, the higher than the first ionization enthalpies. This is mainly
nuclear charge increases. due to the fact that after the removal of the first electron,
the atom changes into monovalent positive ion. In the
(ii) There is a gradual increase in atomic size due to
ion, the number of electrons decreases but the nuclear
an additional main energy shell (n).
M
K
charge remains the same. As a result of this, the remaining understood in terms of the electronic configurations
electrons are held more tightly by the nucleus and it of atoms and ions as explained below :
becomes difficult to remove the second electron. Hence, IE1 IE2
the value of second ionization enthalpy (IE2) is greater Li 520 kJ mol –1 7298 kJ mol–1
than that of the first (IE1). Be 899 kJ mol –1 1758 kJ mol–1
.
In the same way, the removal of the second electron The electronic configuration of Li is 1s2 2s1 and
ed
will result in the formation of dipositive ion and attraction that of Be is 1s2 2s2. Since the configuration of Be is
between the nucleus and remaining electrons increases more stable (being completely-filled) than of Li, the
further. This results into higher value of third ionization IE1 of Be is more than IE1 of Li. After the loss of an
rv
enthalpy (IE 3) than second (IE 2). The successive electron from Li atom, it acquires the electronic
ionization enthalpies of some elements of first and second configuration of noble gas i.e., 1s2. On the other hand,
se
period are given in Table 17. in case of Be atom, the electronic configuration
becomes 1s2 2s1 after the loss of one electron. Thus,
Successive ionization enthalpy and
the electronic configuration of Li+ is more stable than
re
electronic configurations
Be+ and, therefore, IE2 of lithium is much greater than
The comparison of successive ionization IE2 for beryllium.
enthalpies of Li and Be gives very interesting results.
ts
IE IE
It can be seen from the successive ionization enthalpies Li (1s2 2s1) ⎯⎯
⎯1
→ Li+ (1s2) ⎯⎯
⎯2
→ Li2+ (1s1)
of Li and Be that the first ionization enthalpy (IE1) of ⎯→ Be+ (1s2 2s1) ⎯ ⎯
IE1
⎯
IE
→ Be2+ (1s2)
Be (1s2 2s2) ⎯ ⎯
2
gh
Li is smaller than for Be while the second ionization
The IE3 of beryllium is also very high because
enthalpy (IE2) of Li is much greater than for Be. Why the third electron is to be removed from stable
is it so, is an important question? This can be easily configuration (1s2).
ri
Table. 17. Successive ionization enthalpies of some elements (kJ mol–1).
H 1s1 1312 – – – – – – –
He 1s2 2372 – – – – – – –
rs
= 495 × 1 × 10 – 4
P
= 0.0495 kJ or 49.5 J
Example 14. Example 15.
n
How much energy in joules must be needed to Which of the following pairs of elements would
convert all the atoms of sodium to sodium ions you expect to have lower ionization enthalpy?
er
M
K
(iv) Xe is expected to have lower ionization enthalpy In other words, electron gain enthalpy provides
because ionization enthalpy decreases down the a measure of the ease with which an atom adds an
group and Xe lies below Kr. electron to form an anion as :
Example 16. A (g) + e– (g) ⎯⎯→ A– (g) ΔH = ΔegH
The first ionization enthalpy (ΔiH) values of the It is represented as ΔegH and is expressed in
third period elements, Na, Mg, and Si are
.
kilojoules per mole (kJ mol–1). It may be noted that, in
ed
respectively 496, 737 and 786 kJ mol–1. Predict general, electron gain enthalpy may be regarded
whether the first ΔiH value for Al will be more close as enthalpy change when an electron is added
to 575 or 760 kJ mol–1 respectively. Justiy your to an isolated gaseous atom. This is because, the
rv
answer. electron gain process may be exothermic or
Solution: endothermic. For many elements, energy is released
se
It will be more close to 575 kJ mol–1 because the value when an electron is added to the atom and therefore,
of Al should be lower than that of Mg. This is due to electron gain enthalpy is negative. For example,
effective shielding of 3p electrons from the nucleus by elements of group 17 have very high negative electron
re
3s-electrons. gain enthalpy values because they have strong tendency
Example 17. to accept an electron to acquire stable noble gas
From each set, choose the atom which has the configurations. On the other hand, noble gases have
ts
largest ionization enthalpy : large positive electron gain enthalpies because they
(a) F, O, N (b) Mg, P, Ar (c) B, Al, Ga have no tendency to accept the electron. When they
gain an electron, it enters the next higher principal
gh
Solution:
(a) F (b) Ar (c) B quantum number leading to unstable electronic
configurations.
As the definition implies, the magnitude of the
ri
electron gain enthalpy measures the ability of an atom
18. The electronic configurations for some neutral atoms to hold an additional electron. If an atom has more
ll
are given below : tendency to accept an electron, large energy will be
.A
A : 1s2 2s2 B : 1s2 2s2 2p6 3s1 released. Consequently, electron gain enthalpy will be
2 2
C : 1s 2s 2p 4 D : 1s2 2s2 2p3 highly negative. On the other hand, if an atom has
Which of these electronic configurations would you less tendency to hold the electron, small amount of
energy will be released, leading to a small negative
rs
your answer.
attraction. Consequently, the electron gain enthalpy
er
18. (i) D (ii) B 19. (a) O (b) Cl (c) Na (d) K (e) Xe. having stable electronic configurations of half filled and
20. 13.60 eV per atom 21. Lowest : K, highest : Kr. completely filled valence subshells show very small
22. (i) C (ii) N (iii) Ne.
M
Just as energy is required to remove an electron Due to the lack of sufficient data, the changing
from an atom, energy is released when an electron is trends in electron gain enthalpies on moving down a
added to a neutral atom. This is called electron gain group along the period in the periodic table are less
enthalpy. Thus, electron gain enthalpy is well defined than those for ionisation enthalpies.
the energy released when electron is added However, it has been observed (Table 18) that the
to an isolated gaseous atom to form the electron gain enthalpy, in general, becomes more
gaseous negative ion. negative from left to right in a period and becomes less
M
K
.
Na Mg Al Si P S Cl Ar
ed
– 53 ≈0 – 50 – 119 – 74 – 200 – 349 + 97
K Ca Ga Ge As Se Br Kr
– 48 –2 – 36 – 116 – 77 – 195 – 325 + 96
rv
Rb Sr Te I Xe
– 47 –5 – 190 – 295 + 77
se
Cs Po At Rn
–46 –174 – 270 + 68
re
negative as we go from top to bottom in a group. These Similarly, the low (almost zero) electron gain
variations are discussed below. enthalpy values for Be, Mg, N and P can be explained
(a) Variation down a group. On moving down due to the extra stability of completely filled 2s-and3s-
ts
a group, the size and nuclear charge increases. But orbitals in Be (2s2 ) and Mg (3s2 ) respectively and of
the effect of increase in atomic size is much more half-filled 2p-and 3p-orbitals in N and P respectively.
Therefore, the configurations show little tendency to
gh
pronounced than that of nuclear charge and thus the
additional electron feels less attraction by the large gain any electron and hence their electron gain
atom. Consequently, electron gain enthalpy becomes enthalpies are very low (almost zero).
ri
less negative. This is clear from decrease of electron (iii) Electron gain enthalpy of fluorine is
gain enthalpy in going from chlorine to bromine and unexpectedly less negative than that of chlorine. As
to iodine. already explained, the electron gain enthalpy, in general,
ll
(b) Variation along a period. On moving across becomes less negative from top to bottom in a group.
.A
a period, the size of the atom decreases and nuclear However, it is observed that F-atom has unexpectedly less
charge increases. Both these factors result into greater negative electron gain enthalpy than Cl-atom. The electron
attraction for the incoming electron, therefore, gain enthalpy values for the halogens are shown in Fig. 12.
rs
electron gain enthalpy, in general, becomes more The less negative electron gain enthalpy value of F is due to
negative in a period from left to right. However, certain the very small size of F-atom. As a consequence of small
he
irregularities are observed in the general trend. These size, there are strong inter electronic repulsions in the
are mainly due to the stable electronic configurations relatively compact 2p -subshell of fluorine and thus the
of certain atoms. incoming electron does not feel much attraction.
lis
Important Trends in Electron Gain Enthalpies Hence, the element with most negative electron
There are some important features of electron gain enthalpy is chlorine.
gain enthalpies of elements. These are :
ub
M
K
.
–
The energy has to be supplied to force the second electron atoms of iodine in vapour state are converted to I
ed
–13
ions is 4.9 × 10 J according to the reaction:
into the anion. –
First electron gain enthalpy I(g) + e– ⎯⎯→ I (g)
Express the electron gain enthalpy of iodine in
rv
O (g) + e– ⎯⎯→ O – (g); ΔegH1 = – 141 kJ mol–1 terms of kJ mol–1 and eV per atom.
(Energy released) Solution: The amount of energy released for the
se
Second electron gain enthalpy conversion of 1 million, i.e., 1 × 106 atoms of iodine is
4.9 × 10–13 J according to the reaction,
O – (g) + e– ⎯⎯→ O2–(g) ΔegH2 = + 780 kJ mol–1 –
I(g) + e– ⎯⎯→ I (g)
re
(Energy required to add an electron) The amount of energy released for the conversion of
–
In other words, the second (and higher) electron one mole (6.02 × 1023) of atoms of iodine into I ions can
gain enthalpies would have positive values. For be calculated. This corresponds to electron gain
ts
example, the first and second electron gain enthalpy enthalpy. Thus,
Amount of energy released for 1 × 106 atoms of iodine
values of O, S and Se are given below.
= 4.9 × 10 –13 J
gh
Amount of energy released for 6.02 × 1023 atoms of
Element Electron gain enthalpy iodine
(kJ mol–1)
ri
4.9 × 10 –13
EA1 EA2 = × 6.02 × 1023
1 × 106
O – 141 + 780
ll Now
= 29.5 × 104 J = 295 kJ/mol.
1 eV/atom = 96.3 kJ mol–1
S – 200 + 590
.A
∴ Electron gain enthalpy = – 295 kJ mol–1
Se – 195 + 420
295
= = – 3.06 eV/atom
96.3
rs
Example 21.
Example 18. Which element will have the greatest negative
he
F. The less negative electron gain enthalpy of F is due Which of the following pairs would have a higher
to its very small size which has strong electron - electron negative electron gain enthalpy ?
n
M
K
.
to smaller 2p-orbital leads to greater inter-electronic
ed
repulsions than adding an electron to larger 3p-orbital. Hence, a scale of electronegativity. This scale was based on
Cl has more negative electron gain enthalpy than F. the bond enthalpies in heteronuclear bonds. He
Therefore, Cl has the most negative electron gain enthalpy suggested that if two atoms A and B had the same
and P has the least negative electron gain enthalpy.
rv
electronegativity values, then the bond enthalpy of A—
B bond would be equal to the geometric mean of the
bond enthalpies of A—A and B—B bonds. For example,
se
the bond enthalpy of A—B molecule may be written as :
23. Arrange the following electronic configurations in EAB = (EAA × EBB)1/2
re
order of increasing negative electron gain enthalpy : This relation is based on the assumption of pure
(i) 1s2 2s2 2p6 3s2 3p5 (ii) 1s2 2s2 2p5 covalent bonds in A2, B2 and AB molecules. However,
2 2
(iii) 1s 2s 2p 3 (iv) 1s2 2s2 2p6 3s1
Pauling observed that for most of the A—B bonds, the
ts
24. Which of the following pairs has higher negative
actual bond enthalpy is more than the geometric mean
electron gain enthalpy?
(i) O, S (ii) F, Cl (iii) O, O– (iv) O or F
of bond enthalpies of EAA and EBB. This means that the
gh
25. Arrange the following elements in the decreasing two participating atoms have different tendencies to
order of negative electron gain enthalpy : B, C, N, O attract the shared pair of electrons i.e., have different
electronegativities. The difference between the actual
ri
26. The electron enthalpies of halogens become less
negative in the order F > Cl > Br > I. Comment on the bond enthalpy of A—B(EAB) and the geometric mean
statement. bond enthalpy [(EAA × EBB)1/2] is known as excess bond
ll
enthalpy (ΔE). It is a measure of difference in
.A
electronegativities of two atoms A and B. By calculating
23. (iii), (iv), (ii), (i)
ΔE for different bonds, Pauling suggested the following
24. (i) S (ii) Cl (iii) O (iv) F
relationship between the electronegativities of two
rs
25. O, C, B, N
atoms χA and χB and the excess bond enthalpy, ΔE.
26. Statement is wrong, correct order is
χ A − χ B = 0.12(Δ E) 2 kJmol −1
1
he
Electronegativity provides a qualitative measure covalent (A–A) and (B–B) bonds, then there is large
of the ability of an atom in a chemical compound to electronegativity difference between the two atoms A
attract shared pair of electrons towards itself. We know and B. Assigning an arbitrary value of one of the
ub
that a covalent bond is formed by mutual sharing of element, the electronegativity values of other elements
electrons between two atoms. However, all covalent can be easily calculated. Pauling assigned the value
bonds are not similar. In order to understand the of 4.0 to the most electronegative element
P
nature of a covalent bond between atoms, a new fluorine. The Pauling electronegativity values are
concept known as electronegativity was introduced by given in Table 20 for normal elements. These are most
commonly used values.
n
attracting electrons and is the most electronegative of electronegativity in terms of ionization enthalpy and
element. It must be remembered that unlike other electron gain enthalpy. We know that the tendency of
atomic properties such as ionisation enthalpy, electron an atom to lose an electron is related to its ionization
©
gain enthalpy which are related to individual gaseous enthalpy and the tendency of an atom to gain an
atoms, the electronegativity is related to atoms in the electron is related to its electron gain enthalpy. This
bonded state. Electronegativity values of different means that an atom having a high ionization enthalpy
elements are not measured but are derived indirectly and a more negative electron gain enthalpy will attract
by different methods. Consequently, a number of electrons towards itself and hence will be more
relative scales of electronegativity such as Pauling scale, electronegative. On the other hand, an atom with low
M
K
ionization enthalpy and a less negative electron gain penetration power. Therefore, s-orbitals will have
enthalpy will have little tendency to attract electrons greater electron attracting power or electronegativity.
to itself and therefore, will have a low electronegativity. In other words, the electronegativity increases with the
Mulliken suggested that the tendency of an atom increasing s-character of the hybrid orbitals for carbon.
to attract the shared pair of electrons towards itself in
Hydrocarbon Type s-character Electro-
a bond is the average of the ionization enthalpy (ΔiH)
.
of negativity
ed
and electron gain enthalpy (ΔegH). Therefore, Mulliken hybridisation
electronegativity is given as :
Methane, CH4 sp3 25% 2.48
Δ i H − Δ eg H
rv
χM = Ethylene, C2H4 sp2 33% 2.75
2 Acetylene, C2H2 sp 50% 3.29
where, ΔiH = Ionization enthalpy
se
The s-character increases in the order CH4, C2H4
and ΔegH = Electron gain enthalpy.
and C2H2 and the electronegativity of carbon in these
The physical picture of Mulliken is reasonable compounds also increases. Similarly, for nitrogen atom,
re
because the tendency of an atom to attract the shared the values of the electronegativity are 3.68, 3.94 and
pair of electrons in a bond should be the average of the
4.67 for sp3, sp2 and sp hybridisation respectively.
tendency of an atom to hold its own electrons (ΔiH and
3. Nature of the substituents. The
ts
its tendency to attract the additional electron ΔegH).
electronegativity of group varies with the nature of
In addition, two more electronegativity scales;
the substituents. This is due to the inductive effect of
Allred and Rochow scale and Sanderson’s scale of
gh
electronegativity were proposed. The electronegativity the substituent group. For example,
values of Pauling are given in Table 19. CH3 = 2.30 CCl3 = 3.30 CF3 = 3.35
In these cases, the electronegativites of these groups
ri
Factors Affecting Electronegativity will be the electronegativity of carbon as it is adjusted
Electronegativity of an atom is not a fixed quantity by the presence of substituents (3H, 3Cl or 3F atoms).
but depends upon the following factors: ll Periodic Variation of Electronegativity
1. Oxidation state. In general, the electro- (a) In a period. The electronegativity generally
.A
negativity increases as the positive oxidation state of increases on moving across a period from left to right (e.g.,
the atom increases. This is because with the increase from Li to F). This is primarily due to decrease in atomic
in positive oxidation state, the tendency to attract the size and increase in effective nuclear charge. As a result
rs
electrons will increase. For example, Pauling of increase in effective nuclear charge, the attraction for
electronegativity value for Fe(II) is 1.83 whereas it is about the outer electrons and the nucleus increases in a period
1.96 for Fe(III). Some common examples are :
he
due to the fact that a more negatively charged ion will It is clear from Table 19 that the electronegativity
attract less electrons than a less negatively charged generally increases on moving across a period from left
(or neutral) ion. to right and decreases on moving down a group. These
P
2. Type of hybridisation. The type of trends are similar to that of ionization enthalpy.
hydridisation also affects the electronegativity of an Thus, the alkali metals of group I have the lowest
atom. We have learnt that s-orbitals are nearer to the
n
H
2.1
Li Be B C N O F
M
M
K
.
large extent the chemical behaviour. The main trends
ed
of the elements. Thus, the increase in electronegativity
along a period is accompanied by increase in non-metallic in periodic properties of elements are summed up in
properties or decrease in metallic properties. Similarly, Fig. 13. Most of these trends in chemical properties
rv
the decrease in electronegativity down a
group is accompanied by decrease in non-
metallic properties or increase in metallic
se
properties of the elements. Thus, fluorine
with the highest electronegativity of 4.0 is
the most non-metallic element while cesium
re
with the lowest electronegativity of 0.7 is
the most metallic element. The elements
with the high electronegativities on the right-
ts
hand side of the periodic table are non-
metallic elements while those having low
gh
electronegativities on the left hand side are
metallic elements.
In general, as an approximation,
ri
elements with electronegativity 2.0 or
greater are non–metals while those with
electronegativity less than 2.0 are metals.
The electronegativity helps in predicting
ll
.A
Fig. 13 Periodic trends of elements in the periodic table.
the polar or non-polar bonds in molecules
(discussed in unit 4).
Differences between electron gain enthalpy such as diagonal relationship, inert pair effect, effects of
rs
enthalpy and electronegativity are summed up below : Some very common properties such as valence
Electron gain enthalpy Electronegativity shown by elements and the anomalous properties of the
second period (from lithium to fluorine) are taken
lis
measured.
a relative number. called valence electrons and these electrons determine
5. It has units such as 5. It has no units. There are the valence of the atom.
od
constant. depends upon the equal to eight minus the number of valence electrons.
oxidation state of an atom, This is shown in Table 20 ahead.
hybridisation state of the
However, the transition elements exhibit variable
©
M
K
.
Variation of valency in the Periodic Table
ed
(a) Variation in a period. The number of valence electrons increases from 1 to 8 on moving across a period,
the valency of the elements with respect to hydrogen and chlorine increases from 1 to 4 and then decreases to zero.
This may be illustrated by taking the examples of elements of second and third periods as given in Table 21. The
rv
number in brackets represents the valency of the elements in the compound.
se
Table 21. Variation of valency of elements of second and third period.
Group 1 2 13 14 15 16 17
re
Elements of second period
Li Be B C N O F
Valence with respect to H
ts
LiH BeH2 BH3 or B2H6 CH4 NH3 H2O HF
(1) (2) (3) (4) (3) (2) (1)
Valence with respect to Cl
gh
LiCl BeCl2 BCl3 CCl4 NCl3 Cl2O ClF
(1) (2) (3) (4) (3) (2) (1)
Elements of third period
ri
Na Mg Al Si P S Cl
Valence with respect to H
NaH MgH2 AlH3 SiH4ll PH3 H2S HCl
(1) (2) (3) (4) (3) (2) (1)
.A
(b) Variation in a group. On moving down a group, the number of valence electrons remains same
and, therefore, all the elements in a group exhibit same valency. For example, all the elements of group
1 have valency one and those of group 2 have valency two.
rs
The periodic trends in valence of representative elements shown by the formulae of their hydrides and oxides
are given in Table 22 below.
he
Table 22. Periodic trends in valence of elements as shown by the formulae of their compounds.
Group 1 2 13 14 15 16 17
lis
Period
Formulae LiH B2H6 CH4 NH3 H2O HF
of NaH CaH2 AlH3 SiH4 PH3 H2S HCl
ub
There are many elements which exhibit variable atom of fluorine shares one electron with oxygen in
od
valence. This is particularly characteristic of transition OF2 molecule. Since fluorine is highest electronegative
elements and actinoids. These will be studied later. element, it is given oxidation state – 1. Now there are
Nowadays, the term oxidation state is preferred two fluorine atoms in the molecule, oxygen with outer
M
and frequently used for valence. Oxidation state of electronic configuration 2s22p4 share two electrons with
an element in a particular compound gives the charge fluorine atoms. Therefore, it shows oxidation state of
acquired by its atoms on the basis of electronegativity +2. However, in Na 2 O, oxygen being more
©
consideration from other atoms in the molecule. Let us electronegative accepts two electrons, one from each of
consider two oxygen containing compounds : OF2 and the two sodium atoms and thus exhibits oxidation state
Na2O. The order of electronegativity of the three atoms of –2. On the other hand, sodium with electronic
involved in these compounds is F > O > Na. Fluorine, configuration 3s1 loses one electron to oxygen and
has the electronic configuration 2s22p5 and each of the therefore, exhibits oxidation state of +1. The
M
K
determination of oxidation state is discussed in Unit 8. elements (specially second). The first member differs
The common oxidation states of representative from its succeeding members in some of the properties
elements are given in Table 23 ahead. as given below
It is clear from Table 23 that s-block elements (i) As we go down a group, the size goes on
show only one oxidation state either +1 (alkali metals) increasing, therefore, the first member of each
or +2 (alkaline earth metals). Hence, p-block elements group has the smallest size in its group.
.
show different oxidation states. (ii) Because of small size the first member has
ed
2. Anomalous Properties of Second Period largest ionization enthalpy and ionization
Elements enthalpy decreases down the group.
(iii) All the elements of second period have
rv
As we have learnt that the elements in a group
show similar physical and chemical properties. abnormally low negative electron gain enthalpy
However, the first element of each of the groups i.e., than the second member. For example, the
se
lithium (of group 1), beryllium (of group 2) and boron to electron gain enthalpy decreases as we move
fluorine (of groups 13–17) differ in many respects from down a group but the first member has
the other members of their respective groups. For abnormally lower electron gain enthalpy than
re
example, lithium shows an anomalous behaviour as the second because of its small size.
compared to sodium and rest of the family members of (iv) The small size of the atom results in relatively
the alkali metal family. It forms compounds with high cohesive properties associated with
ts
pronounced covalent character whereas other members relatively strong intermetallic bonding. On the
of the group predominantly form ionic compounds. other hand, large atoms usually form weak
Similarly, beryllium, the first member of alkaline earth bonds, therefore, the bond strengths of the
gh
metals family differ from other family members. It compounds decrease as we move down the
forms covalent compounds unlike other alkaline earth group. For example, lithium has relatively high
metals which form mainly ionic compounds. In fact, it enthalpy of atomization, melting and boiling
ri
has been observed that some elements of second period points, density and hardness.
show similarities with the elements of the third period ll (v) The first member has higher electronegativity
present diagonally to each other, though belonging to as compared to other members of the group.
different groups. For example, lithium resembles with Therefore, it has greater tendency to form
.A
magnesium (of group 2) and beryllium resembles with covalent bonds. For example, lithium halides,
aluminium (of group 3) and so on. This similarity in are covalent while halides, of other members of
properties of elements present diagonally is called group 1 are ionic in nature.
rs
diagonal relationship. (vi) The first member of the group has no vacant
d–orbitals in its valence shell. Therefore, it has
only four valence orbitals (2s and 2p) available
he
For example, as shown below, metallic radii and (second member of the group) have vacant
ionic radii of Li are close to Mg but these are quite 3d-orbitals in their valence shell. Therefore,
ub
different from sodium. these have nine valence orbitals (3s, 3p and 3d)
and therefore these elements can expand their
Property Element valence shell to accommodate more than four
P
Na Mg Al
elements of higher periods can extend their
186 160 143
er
The anomalous behaviour of first member of each members cannot form stable pπ – pπ multiple bonds.
group as compared to other group members is mainly
due to the following reasons : Periodic Trends and Chemical Reactivity
©
(i) Small size of the atom and its ion. We have learnt the periodic trends in some
(ii) Large charge / radius ratio. fundamental properties such as atomic and ionic radii,
(iii) High electronegativity. ionization enthalpy, electron gain enthalpy and,
(iv) Non availability of d-orbitals in their valence electronegativity and valence. The periodicity in these
shells. properties are related to electronic configurations. Since
The above factors have strong affects on the the chemical and physical properties of the elements
chemistry of first element as compared to other and compounds are a manifestation of the electronic
M
K
36
od
H He
+1 HCl
-1 Na+H–
er
II (2) n III (13) IV (14) V (15) VI (16) VII (17)
P N Ne
C -3 NH3
+2 NO O
Li Be B -4 CH4 -2 H2O F
+3 N2O3
+1 Li+Cl– +2 BeCl2 +3 BCl3 +4 CO2 -1 H2O2 -1 Na+F-
+4N2O4
ub
d- +2 CO2 +5 N2O5
S Cl
lis
P -2 H2S -1 Na+Cl- Ar
B Si -3 PH3 +1 HOCl
Na Mg Al +4 SO2
-4 SiH4 +3 PCl3 +3 HClO2
he
+1 Na+Cl– +2Mg2+(Cl–)2 +3 Al2Cl6 +6 SO3
+4 SiCl4 +5 PCl5 +5 HClO3
M
M
+7 HClO4
rs
L
K
K
As Br
Se Kr
Ge -3 AsH3 -1 Na+Br-
.A
-2 H2Se
K Ca Ga +4 GeCl4 +3 AsCl3 +1 HOBr +2 KrF2
O +4 SeO2
+1 K+Cl– +2 Ca2+(Cl)2 +3 Ga2Cl6 +2 GeCl2 +5 As4O10 +6 SeO3
+5 HBrO3
ll +7 HBrO4
ri
C Sb I
Sn Te
-3 SbH3 -1 Na+I- +2 XeF2
Rb Sr In
gh
-2 H2Te
1/11/2020, 2:12 PM
+4 SnCl4 +3 Sb3+(F–)3 +1 HOI +4 XeF4
+1 Rb+Cl– +2 Sr2+(Cl–)2 +3 InCl3 +2 SnCl2 +5 SbCl5
+4 TeO2
+3 HIO3 +6 XeF6
K +6 TeO3 +8 Na4XeO6
+5 HIO4
ts
Bi
re
Cs Ba Tl Pb -3 BiH3
+1 Cs+Cl– +2 Ba2(Cl–)2 +3 TlF3 +4 PbO2 Po At Rn
+3 Bi(F–)3
+1 Tl+Cl– +2 Pb+(Cl–)2 +4 PoO2
+5 Bi2O5
se
rv
MODERN'S abc + OF CHEMISTRY–XI
ed
.
M
K
configuration of elements, therefore, the chemical left to right across the period. For example, Li is
reactivity of the elements are also governed by these strongest metal while F is strongest non-metal.
fundamental properties.
Second period Li Be B C, N, O, F, Ne
The atomic and ionic radii generally decrease in
a period from left to right. As a result, the ionization Metals Metalloid Non-metals
enthalpies in general increase (with some exceptions)
.
and electron gain enthalpies become more negative Third period Na, Mg, Al Si S, Cl, Ar
ed
across a period. In other words, the ionization enthalpy Metals Metalloid Non-metals
of the extreme left element in a period is the least and
therefore, it will have highest tendency to lose electron
rv
and this tendency decreases as we move in the period
from left to right. Similarly, with the exception of noble
se
gases the electron gain enthalpy of the element on the
extreme right is the highest negative showing that it
has strongest tendency to gain electron and form
re
negative ion. Noble gas element is an exception because
it has filled shells and have rather positive electron The chemical reactivity of an element can best be
gain enthalpy values. Therefore, these are least reactive. shown by its reactions with oxygen and halogen.
ts
Thus, we see that there is high chemical reactivity
Elements on the two extremes of a period readily combine
at two extreme ends and the lowest in the centre.
with oxygen to form oxides. The normal oxide formed
Thus, the maximum chemical reactivity at the
gh
extreme left (among alkali metals) is exhibited by the by the element on the extreme left (most metallic) is
easy loss of electrons forming a cation and at the extreme most basic (Na2O) whereas, the oxide formed by the
right (among halogens) shown by gain of electrons element on the extreme right (most non–metallic) is
ri
forming an anion. This property of losing or gaining most acidic (e.g., Cl2O7). Oxides of the elements in the
electrons can be related with oxidising and reducing centre are amphoteric (e.g., Al2O3) or neutral (e.g., CO,
ll
behaviour of the elements. The elements which readily NO, N2O, etc.). The amphoteric oxides show the acidic
lose electrons act as strong reducing agents while those
.A
and basic character. They behave as acidic with bases
which readily accept electrons act as strong oxidising
agents. These you will learn in Unit 8. However, this and basic with acids. On the other hand, the neutral
tendency of an element to lose or gain electrons is also oxides have no acidic or basic properties. In general, on
rs
related to metallic or non-metallic character. The metals moving across a period from left to right, the basic
have strong tendency to lose electrons. Thus, the character of the oxides decreases while acidic character
metallic character of an element is highest at the
he
extreme right. In other words, the metallic character P2O5 is acidic, SO3 is strongly acidic and Cl2O7 is very
decreases and non-metallic character increases from strongly acidic.
ub
Element Na Mg Al Si P S Cl
Oxide Na2O MgO Al2O3 SiO2 P2O5 SO3 Cl2O7
P
As we go down the group, the basic character of the oxides increases or acidic character decreases. For
od
example, in group 13, B2O3 is acidic, Al2O3 and Ga2O3 are amphoteric while In2O3 is basic.
Group I Group II Group III
ACIDIC CHARACTER
BASIC CHARACTER
M
ACIDIC CHARACTER
M
K
It may be noted that among transition elements (ii) Aluminium and Sulphur
(3d series), the change in atomic radii is much smaller Solution: (i) Silicon belongs to group 14 having a
as compared to those of representative (s and p–block)
valence of 4 and bromine belongs to group 17 (halogen family)
elements across the period. This changes in atomic radii
with a valence of 1. Therefore, the formula of compound
is still smaller among inner transition metals (4f–series).
The ionization enthalpies of d– and f– block elements formed would be SiBr2.
.
are intermediate between those of s– and p–blocks. As (ii) Aluminium belongs to group 13 with a valence of 3
ed
a result, d-and f-block elements are less while sulphur belongs to group 16 with a valence of 2. Hence,
electropositive than group 1 and group 2 metals. the formulae of the compounds formed would be Al2S3.
In a group, the increase in atomic and ionic radii
rv
Example 25.
with increase in atomic number generally results in a
gradual decrease in ionization enthalpies and a regular Are the oxidation state and covalency of Al in
[AlCl(H2O)5] 2+ same ?
se
decrease (with some exceptions) in electron gain enthalpies
in case of main group elements. Thus, the metallic Solution: No, the oxidation state of Al is +3
character increases down the group and non-metallic covalency is 6
re
character decreases. For example, in 4th group, C is typical
metal, Si and Ge are non-metals while Sn and Pb are
Example 26.
typical metals. This bond can also be related with their Show by chemical reaction with water that Na2O
is a basic oxide and Cl2O7 is an acidic oxide.
ts
reducing and oxidising properties which you will learn
later. However, in case of transition elements, a reverse Solution: Na2O reacts with water and forms a strong
trend is observed which can be explained in terms of atomic base
gh
size and ionization enthalpy. Na2O + H2O ⎯⎯→ 2NaOH
Cl2O7 reacts with water to form a strong acid.
ri
Cl2O7 + H2O ⎯⎯→ 2HClO4
Example 24.
Their acidic and basic nature can be qualitatively
Predict the formulae of compounds which might tested with litmus paper. As we know acids turn blue litmus
ll
be formed by the following pairs of elements : paper red while bases turn red litmus paper blue.
.A
(i) Silicon and Bromine
rs
2
he
Q. 1. What is the significance of the terms “isolated gaseous atom” and “ground state” while defining
lis
possible for that atom. These terms are used for comparison purposes.
Q. 2. Energy of an electron in the ground state of the hydrogen atom is –2.18 × 10–18J. Calculate the
P
M
K
.
Q. 7. Arrange the following elements in order of increasing ionization enthalpy :
ed
B, C, N, O
Ans. B < C < O < N.
Q. 8. Consider the ground state electronic configurations given below :
rv
(A) 1s2 2s2 2p6 (B) 1s2 2s2 2p4
2 2
(C) 1s 2s 2p 3s 6 2 (D) 1s2 2s2 2p6 3s1
se
2 2
(E) 1s 2s 2p 5
(i) Which of the above configuration is associated with the lowest and which is associated with
highest ionization enthalpy ?
re
(ii) Arrange the above configurations in order of increasing negative electron gain enthalpy.
Ans. (i) Lowest ionisation enthalpy = D
Highest ionisation enthalpy = A
ts
(ii) Order of increasing negative electron gain enthalpy is :
A<C<D<B<E
gh
Q. 9. How would you explain the fact that the first ionization enthalpy of sodium is lower than that of
magnesium but its second ionization enthalpy is higher than that of magnesium ?
ri
Ans. The electronic configuration of Na is [Ne] 3s1 and that of Mg is [Ne] 3s2. The configuration of Mg is more
stable (being completely filled) than that of Na. Therefore, first ionization enthalpy of Mg is more than
ll
that of Na . After the loss of an electron from Na, it acquires the electronic configuration of noble gas, Ne,
.A
i.e., 1s22s22p6. On the other hand, in case of Mg atom, the electronic configuration becomes [Ne] 3s1. Thus,
the electronic configuration of Na+ is more stable than Mg+ and hence the second ionization enthalpy of
Na is much larger than that of Mg.
rs
Na([Ne]3s1) ⎯⎯⎯
IE1 + IE2
→ Na ([Ne]) ⎯⎯⎯
2+ 2 2 5
→ Na (1s 2s 2p )
Mg([Ne]3s2) ⎯⎯⎯ IE2 2+ 1
→ Mg ([Ne]3s ) ⎯⎯⎯
IE2 2+
→ Mg ([Ne])
he
IE1 (Na) < IE1 (Mg) IE2 (Na) > IE2 (Mg)
Q. 10. Among the second period elements the actual ionization enthalpies are in the order :
lis
configuration 1s22s22p1. The larger ΔiH of Be in comparision to B is due to the fact that
(a) The electronic configuration of Be is more stable (completely filled 2s orbitals) than that of B.
n
(b) In Be, the electron to be removed during the ionization is an s-electron while the electron to be removed
during ionization of B is a p–electron. The penetration of a 2s–electron to the nucleus is more than that
er
of a 2p–electron and hence 2p electron of B is more shielded from the nucleus by the inner core of electrons
than the 2s–electron of Be. As result, 2s-electron is attracted to the nucleus more than 2p-electron.
od
Therefore, it is difficult to remove a 2s–electron from Be than to remove the 2p–electron from B. Thus, Be
has higher ionization enthalpy than B.
(ii) Oxygen has four electrons in 2p orbitals and two of the four 2p electrons must occupy the same 2p-orbital
M
resulting in increased electron-electron repulsion. On the other hand, N has stable half filled configuration,
while F has greater nuclear charge. Therefore, O has ionization enthalpy less than N as well as F.
©
Q. 12. Arrange the following in the decreasing negative electron gain enthalpy :
B, C, N, O
Ans. O, C, B, N
Q.13. Which of the following has highest ionization enthalpy : C, N and O and why ?
Ans. N has highest ionisation enthalpy because of stable exactly half filled 2p orbitals.
.
Q.14. Arrange the following in the increasing order of their size : F–, Li+, Na+, Cl–
ed
– –
Ans. Li+ < Na+ < F < Cl .
Q.15. A student reported the radii of Al3+, Mg2+ and F– as 136 pm, 65 pm and 50 pm respectively. Is the
order correct ? Comment.
rv
Ans. The order is not correct. The correct order should be Al3+, Mg2+ and F– as 50 pm, 65 pm and 136 pm
respectively.
se
Q.16. Among the elements Li, K, Ca, S and Kr which one is expected to have the lowest first ionization
enthalpy and which one has the highest first ionization enthalpy ?
Ans. Lowest ionization enthalpy : K; Highest ionization enthalpy : Kr.
re
Q.17. The first (IE1) and second (IE2) ionization enthalpies (kJ mol–1) of three elements A, B
and C are given below :
A B C
ts
IE1 403 549 1142
IE2 2640 1060 2080
gh
Identify the element which is likely to be
(i) a non metal (ii) an alkali metal (iii) an alkaline earth metal
ri
Ans. (i) C is non metal
(ii) A is alkali metal
(iii) B is alkaline earth metal. ll
Q.18. Among the elements B, Al, C and Si
.A
(i) Which has the highest first ionization enthalpy ?
(ii) Which has the most negative electron gain enthalpy ?
(iii) Which has the largest atomic radius ?
rs
Q.19. Select from each group, the species having the smallest radius :
(i) K+, Sr2+, Ar (ii) Si, P, Cl (iii) O, O–, O2–
lis
Q.20. The first (IE1) and the second (IE2) ionization enthalpies (kJ mol–1) of a few elements designated
by Roman numerals are shown below :
ub
II 520 7300
III 900 1760
n
IV 1680 3380
er
Ans. (a) Element II (b) Element IV (c) Element I (d) Element III.
Q.21. Which of the elements Na, Mg, Si and P would have the greater difference between the first and
©
.
ed
Modern Periodic law. The physical and chemical properties of the elements are periodic function of their atomic
rv
numbers.
Period. A horizontal row in the periodic table. The elements in a period have same number of electron shells i.e.
se
principal quantum number (n). There are seven periods in the periodic table.
Group. A vertical column in the periodic table. It constitutes a series of elements whose atoms have the same
re
outermost (or valence) electronic configuration.
Periodicity. The recurrence of similar properties of the elements after certain regular intervals when they are
arranged in the order of increasing atomic numbers.
ts
Covalent radius is one half of the distance between the centres of nuclei of the similar atoms bonded by a single
covalent bond.
gh
Van der Waals radius is one half of the distance betwen two adjacent atoms belonging to the nearest neighbouring
atoms of the substance in the solid state.
ri
Ionization enthalpy. The energy required to remove the most loosely held electron from the gaseous isolated state
of an atom.
ll
Electron gain enthalpy. The energy released when an electron is added to the gaseous isolated state of an atom.
.A
Electronegativity is the tendency of an atom to attract the shared pair of electrons to itself in a bond.
rs
he
PERIODICITY
⇓
lis
Second period ⎯→ 2s, 2p ⎯→ 8 elements d-block refers to groups of elements which involve the filling of
Third period ⎯→ 3s, 3p ⎯→ 8 elements
d-orbitals of last but one shell.
General electronic configuration : (n – 1)d1–10 ns0–2
Fourth period ⎯→ 4s, 3d, 4p ⎯→ 18 elements
©
.
ed
rv
se
re
ts
gh
ri
ll
.A
rs
he
lis
ub
P
Q. 1. * What is the basic theme of organisation in the Q. 2. Which important property did Mendeleev use to
periodic table ? classify the elements in his periodic table and
Ans. The basic theme of organisation of elements in the did he stick to that ?
od
periodic table is to simplify and systematize the Ans. Mendeleev used atomic weight as the important
study of the numerous properties of all the elements property for the classification of elements. Mendeleev
and their compounds. This has been done by arranged all the known elements in the form of a
M
arranging the elements in such a way that similar table known as periodic table. He observed that
elements are placed together while dissimilar some of the elements did not fit in very will with his
elements are separated from one another. This has scheme of classification if the order of atomic weight
©
made the study simple to remember because the was strictly followed. He showed courage to ignore
properties of the elements are now studied in the the order of atomic weights thinking that the atomic
form of groups or families having similar properties weight measurements might be incorrect. He placed
rather than studying the elements individually. the elements with similar properties together. For
example, iodine has lower atomic weight than Ionic radius means the size of the ion i.e., a cation or
tellurium (of Group VI) but he placed iodine in anion. This gives the effective distance from the
Group VII alongwith fluorine, chlorine and bromine nucleus of the ion upto which it has an influence in
because of the similarities in their properties. He the ionic bond. The size of the cation is always
even left some spaces or gaps for some undiscovered smaller than that of the parent atom while the size
elements. By considering the properties of the of the anion is always larger than that of the parent
.
adjacent elements, he predicted the properties of the atom.
ed
undiscovered elements. Later on, when these Q. 10. How does atomic radius vary in a period and in
elements were discovered, their properties were a group ? How do you explain the variation ?
found to be exactly similar to those predicted by Ans. The atomic size decreases as we move from left to
rv
Mendeleev. For example, gallium and germanium right in a period. This is because when we move along
a period the nuclear charge increases and therefore,
were not discovered at that time, when Mendeleev
the attraction of the nucleus for the outer electrons
se
formulated his periodic table and therefore, he left
increases and hence the atomic size decreases.
gaps for these elements. He not only predicted the
Within a group, the atomic size increases down the
existence of the elements but he estimated their
group. This is because of addition of a new energy
re
properties. He tentatively named these elements as shell at each succeeding element while the number
eka-aluminium and eka-silicon. When these elements of valence electrons remain the same. For detail,
were discovered, the prediction of Mendeleev proved refer Text.
ts
to be remarkably correct. However, after the discovery Q. 11. What do you understand by isoelectronic
of atomic number as more fundamental property species ? Name a species that will be
than atomic weight by Moseley in 1913, the basis of isoelectronic with each of the following atoms
gh
classification was changed to atomic number. or ions.
Q. 3. What is the basic difference in approach between (i) F– (ii) Ar (iii) Mg2+ (iv) Rb+
the Mendeleev’s periodic law and the Modern Ans. Refer Solved Example 9 (Page 22).
ri
periodic law ? Q. 12. Consider the following species :
Ans. According to Mendeleev’s periodic law, the physical N3–, O2–, F–, Na+, Mg2+ and Al3+
and chemical properties of the elements are periodic
function of their atomic weight. On the other hand,
ll (a) What is common in them?
(b) Arrange them in the order of increasing
.A
according to modern periodic law, the properties are
ionic radii.
periodic function of their atomic numbers.
Ans. (a) All these ions have same number (10) of
Q. 4. On the basis of quantum numbers, justify that
electrons. Therefore, these are also called
rs
Al3+ < Mg2+ < Na+ < F– < O2– < N3–
Q. 6. Write the atomic number of the element present
Q. 13. Explain why cations are smaller and anions
in the third period and seventeenth group of
larger in radii than their parent atoms ?
the periodic table.
ub
Q. 7. Which element do you think have been named result, the same nuclear charge now acts on lower
by number of electrons and therefore, the effective nuclear
n
(i) Lawrence Berkeley Laboratory charge per electron increases. As a result, electrons
(ii) Seaborg’s group ? are more strongly attracted and are pulled towards
er
Ans. (i) Lawrencium (Z = 103) the nucleus and therefore, the size decreases.
(ii) Seaborgium (Z = 106) The anion is formed by the gain of one or more
od
Q. 8. Why do elements in the same group have similar electrons and therefore, the number of electrons
physical and chemical properties ? increases while the magnitude of nuclear charge
Ans. Elements in the same group have similar properties remains same. As a result, the electrons are less
tightly held by the nucleus and therefore, the size
M
mean to you ?
Ans. Atomic radius is one half of the distance between the the ionization enthalpy and electron gain
nuclei of two covalently bonded atoms of the same enthalpy ?
element in a molecule. In case of metals, the atomic Ans. Refer Conceptual Questions 2 , Q.1 (page 38)
radius is called metallic radius. It corresponds to Q. 15. Energy of an electron in the ground state of the
one half of the distance between two adjacent atoms hydrogen atom is –2.18 × 10–18 J. Calculate the
in a crystal lattice. ionization enthalpy of atomic hydrogen in
terms of J mol–1.
Ans. Refer Conceptual Questions 2 , Q.2 (page 38) Ans. The second electron gain enthalpy of oxygen would be
Q. 16. Among the second period elements the actual positive. This is because after the addition of one
ionization enthalpies are in the order Li < B < electron, it becomes negatively charged. The second
Be < C < O < N < F < Ne. electron is added to negatively charged ion and the
Explain why addition is opposed by coulombic repulsions.
(i) Be has higher ΔiH than B. Therefore, energy has to be supplied to force the
.
(ii) O has lower ΔiH than N and F ?. second electron into the anion and hence second
ed
Ans. Refer Conceptual Questions 2 , Q.10 (page 39) electron gain enthalpy would be positive.
Q. 17. How would you explain the fact that the first Q. 22. What is the basic difference between the terms
ionization enthalpy of sodium is lower than that electron gain enthalpy and electronegativity ?
rv
of magnesium but its second ionisation enthalpy Ans. Electron gain enthalpy refers to the tendency of an
is higher than that of magnesium ? atom in its gaseous isolated state to accept an
se
Ans. Refer Conceptual Questions 2 , Q.9 (page 39) additional electron to form a negative ion.
Q. 18. What are the various factors due to which the Electronegativity refers to the tendency of an atom
ionization enthalpy of the main group elements to attract the shared pair of electrons towards it in a
re
tends to decreases down a group ? covalent bond. Thus, electron gain enthalpy is the
Ans. Within the main group elements, the ionization property of isolated atoms whereas electronegativity
enthalpy decreases regularly as we move down the is the property of atoms in molecules.
group. This is due to the following factors :
ts
Q. 23. How would you react to the statement that
(i) Atomic size : On moving down the group, there
is a gradual increase in atomic size due to an electronegativity of N on Pauling scale is 3.0 in
gh
additional main energy shell (n). all its compounds ?
Ans. The electronegativity of nitrogen will not be 3.0 in
(ii) Shielding effect. There is increase in shielding
all its compounds. It depends upon the other atoms
effect on the outermost electron due to increase
attached to it. It also depends on the state of
ri
in the number of inner electrons.
hybridisation and the oxidation state of the element.
(iii) Nuclear charge. In going from top to bottom in
a group, the nuclear charge increases. Q. 24.
ll Describe the theory associated with the radius
of an atom as it
The effect of increase in atomic size and the
.A
shielding effect is much more than the effect of (a) gains an electron
increase in nuclear charge. As a result, the (b) loses an electron.
electron becomes less tightly held to the nucleus Ans. (a) When an atom gains one electron to form an
as we move down the group. Hence there is a
rs
Q. 19. The first ionization enthalpy values (in kJ (b) When an atom loses an electron, it forms a
mol–1) of group 13 elements are :
cation and its radius decreases. The cations are
B Al Ga In Tl always smaller in size than the corresponding
lis
801 577 579 558 589 atoms. For reason : Refer answer to Q. 16.
How would you explain this deviation from the Q. 25. Would you expect the first ionization enthalpies
general trend ?
ub
effectively than s– and p– electrons. As a result, the Q. 26. What are the major difference between metals
outer electron is held fairly strongly by the nucleus and non-metals ?
and therefore, ionization enthalpy increases slightly Ans. Elements which have strong tendency to lose
od
inspite of the increase in atomic size from Al to Ga. electrons to form cations are called metals whereas
The similar increase is observed from In to Tl, which those which have a strong tendency to accept electrons
is due to the presence of 14f- electrons in the inner to form anions are called non-metals. Thus, metals
M
electronic configuration of Tl which have very poor are strong reducing agents, they have low ionisation
shielding effect. enthalpies, low negative electron gain enthalpies,
Q. 20. Which of the following pairs of elements would low electronegativity, form basic oxides and ionic
©
.
electronic configuration is 4f75d16s2. Since the
ed
(d) Identify the group having metal, non-metal, electron goes to f-orbital, therefore, its belongs to
liquid as well as gas at the room f-block.
temperature.
All f-block elements belongs to group 3.
rv
Ans. (a) Chromium (Z = 24). It has five electrons in the
∴ Period = 6, Group = 3
outer 3d subshell.
Q. 31. The first (Δ ΔiH1) and the second (Δ ΔiH2) ionization
(b) Magnesium (Z = 12) can lose two electrons readily.
se
enthalpies (in kJ) and (Δ ΔegH) electron gain
(c) Oxygen (Z = 8) can gain two electrons. enthalpy (in kJ mol–1) of a few elements are
(d) Halogens (group 17). It has metal (iodine), non- given below :
metals (F, Cl, Br), liquid bromine and gases.
re
Elements Δ H1 Δ H2 Δeg H
Q. 28. The increasing order of reactivity among group I 520 7200 –60
1 elements is Li < Na < K < Rb < Cs whereas that
among group 17 elements is F > CI > Br > I. II 419 3051 –48
ts
Explain. III 1681 3374 –328
Ans. The elements of group 1 have only one electron in IV 1008 1846 –295
gh
their outermost shells and therefore, have strong V 2372 5251 +48
tendency to lose this electron. The tendency of these VI 738 1450 –40
elements to lose the valence electron depends upon Which of the above elements is likely to be
the ionization enthalpy. Since ionisation enthalpy
ri
decreases down the group, therefore, the reactivity of (a) the least reactive element.
group 1 elements increases in the same order (b) the most reactive metal.
Li < Na < K < Rb < Cs.
ll (c) the most reactive non–metal.
On the other hand, the elements of group 17 have (d) the least reactive non-metal.
.A
seven electrons in their respective valence shells (e) the metal which can form a stable binary
and therefore, they have strong tendency to accept halide of the formula MX2 (X = halogen).
one more electron. The tendency to accept additional
(f) the metal which can form predominantly
rs
Q. 29. Write the general electronic configuration of Q. 32. Predict the formula of the stable binary
s, p, d and f block elements. compounds that would be formed by the
Ans. s-block : ns1–2 following pairs of elements :
ub
= 10 + No. of electron in the valence shell (iii) Aluminium belongs to group 13 with a valence of
= 10 + 6 = 16 3 and iodine belongs to group 17 with a valence
∴ Period = 3, Group = 16 of 1. Hence, the formula of compound is AlI3.
(ii) For n = 4, the element belongs to fourth period. (iv) Silicon belongs to group 14 with valence of 4
The electronic configuration is 3d24s2. Since the and oxygen belongs to group 16 with a valency
d-subshell is incomplete, the element belongs to of 2. Hence, the formula of the compound is
d-block. SiO2.
(v) Phosphorus belongs to group 15 and it has (c) electron-electron interaction in the outer
valence 3 or 5, while fluorine belongs to group orbitals
17 with a valence of 1. Hence the formula of the (d) none of the factors because their size is the
compound may be PF3 or PF5. same.
(vi) Element 71 has three electrons in the valence Ans. (a) The size of isoelectronic species depends upon
shell (4f14 5d16s2) and therefore has valence 3 the nuclear charge (Z)
.
and fluorine has valence 1. Hence the formula Q. 37. Which one of the following statements is
ed
of the compound is EF3 (E is the element). incorrect in relation to ionization enthalpy ?
Q. 33. In the modern periodic table, the period indicates (a) Ionization enthalpy increases for each
the value of : successive electron.
rv
(a) atomic number (b) atomic mass (b) The greatest increase in ionization
(c) principal quantum number enthalpy is experienced on removal of
se
electron from core noble gas configuration.
(d) azimuthal quantum number.
(c) End of valence electrons is marked by a big
Ans. (c)
jump in ionization enthalpy.
re
Q. 34. Which of the following statements related to the
(d) Removal of electron from orbitals bearing
modern periodic table is incorrect ?
lower n value is easier than from orbital
(a) The p-block has 6 columns, because a having higher n value.
ts
maximum of 6 electrons can occupy all the
Ans. (d)
orbitals in a p-sub shell.
Q. 38. Considering the elements B, Al, Mg and K, the
(b) The d-block has 8 columns, because a
gh
correct order of their metallic character is :
maximum 8 electrons can occupy all the
orbitals in a d-subshell. (a) B > Al > Mg > K
(c) Each block contains a number of columns (b) Al > Mg > B > K
ri
equal to the number of electrons that can (c) Mg > Al > K > B
occupy that subshell. (d) K > Mg > Al > B
(d) The block indicates value of azimuthal
quantum number (l) for the last subshell
ll
Ans. (d) The metallic character decreases along a period
from left to right. Therefore, metallic character of Al,
.A
that received electrons in building up the Mg, K follows the order : K > Mg > Al. Within a group,
electronic configuration. metallic character increases from top to bottom so
Ans. (b) that B is less metallic than Al. So, the correct order
rs
Q. 35. Anything that influences the valence electrons is K > Mg > Al > B.
will affect the chemistry of the element. Which Q. 39. Considering the elements B, C, N, F, and Si, the
one of the following factors does not affect the correct order of their non-metallic character is:
he
valence shell ? (a) B > C > Si > N > F (b) Si > C > B > N > F
(a) Valence principal quantum number (n) (c) F > N > C > B > Si
(b) Nuclear charge (Z)
lis
(a) nuclear charge (Z) (c) Cl > F > O > N (d) O > F > N > Cl
(b) valence principal quantum number (n) Ans. (b)
n
er
(a) Na < Mg > Al < Si (d) O– ion has comparatively smaller size than
(b) Na > Mg > Al > Si oxygen atom.
(c) Na < Mg < Al < Si 12. Comprehension given below is followed by some
(d) Na > Mg > Al < Si multiple choice questions. Each question has one
5. The electronic configuration of gadolinium (Atomic
correct option. Choose the correct option.
.
number 64) is In the modern periodic table, elements are
ed
(a) [Xe] 4f3 5d5 6s2 (b) [Xe] 4f7 5d2 6s1 arranged in order of increasing atomic numbers
(c) [Xe] 4f7 5d1 6s2 (d) [Xe] 4f8 5d6 6s2 which is related to the electronic configuration.
Depending upon the type of orbitals receiving the
rv
6. The statement that is not correct for periodic
classification of elements is : last electron, the elements in the periodic table
have been divided into four blocks, viz, s, p, d and
se
(a) The properties of elements are periodic
f. The modern periodic table consists of 7 periods
function of their atomic numbers.
and 18 groups. Each period begins with the filling
(b) Non metallic elements are less in number
of a new energy shell. In accordance with the
re
than metallic elements.
Aufbau principle, the seven periods (1 to 7) have
(c) For transition elements, the 3d-orbitals are
filled with electrons after 3p-orbitals and 2, 8, 8, 18, 18, 32 and 32 elements respectively.
The seventh period is still incomplete. To avoid
ts
before 4s-orbitals.
(d) The first ionisation enthalpies of elements the periodic table being too long, the two series
generally increase with increase in atomic of f-block elements, called lanthanoids and
gh
number as we go along a period. actinoids are placed at the bottom of the main
7. Among halogens, the correct order of amount of body of the periodic table.
energy released in electron gain (electron gain (i) The element with atomic number 57 belongs to
ri
enthalpy) is : (a) s-block (b) p-block
(a) F > Cl > Br > I (b) F < Cl < Br < I ll (c) d-block (d) f-block
(c) F < Cl > Br > I (d) F < Cl < Br > I (ii) The last element of the p-block in 6th period
.A
8. The period number in the long form of the periodic is represented by the outermost electronic
table is equal to configuration.
(a) magnetic quantum number of any element (a) 7s2 7p6 (b) 5f 14 6d10 7s2 7p0
rs
(b) atomic number of any element of the period. (iii) Which of the elements whose atomic
he
(c) maximum principal quantum number of any numbers are given below, cannot be
element of the period. accommodated in the present set up of the
(d) maximum azimuthal quantum number of long form of the periodic table ?
lis
(a) actinoids (b) transition elements which is just above the element with atomic
(c) lanthanoids (d) halogens number 43 in the same group is
P
10. Which of the following is the correct order of size of the ______________.
given species : (a) 1s2 2s2 2p6 3s2 3p6 3d5 3s2
n
– + + –
(a) I > I > I (b) I > I > I (b) 1s2 2s2 2p6 3s2 3p6 3d5 4s3 4p6
+ – – +
(c) I > I > I (d) I > I > I
er
ANSWERS / HINTS
MCQs Type-I
.
5. (b) 6. (c) 7. (c) 8. (c) 9. (c)
ed
Mg2+(12) < Na+(11) < F– (10) < O2–(8)
2. (d) : Actinoids are elements with Z = 90 – 103. 10. (d) 11. (c)
Therefore, terbium (Z = 65) is not an actinoid. 12. (i)–(c), (ii)–(c), (iii)–(c), (iv)–(a), (v)–(b)
rv
13. (a)
se
19. Which of the following sets contain only isoelectronic
ions ?
re
Note : In the following questions two or more options (a) Zn2+, Ca2+, Ga3+, Al3+ (b) K+, Ca2+, Sc3+, Cl–
may be correct. 3– 2– –
(c) P , S , Cl , K+
(d) Ti4+, Ar, Cr3+, V5+
14. Which of the following elements can show covalency 20. In which of the following options order of arrangement
ts
greater than 4 ? does not agree with the variating of property indicated
(a) Be (b) P against it ?
gh
(c) S (d) B (a) Al3+ < Mg2+ < Na+ < F– (increasing ionic size)
15. Those elements impart colour to the flame on heating (b) B < C < N < O (increasing first ionisation
in it, the atoms of which require low energy for the enthalpy)
ri
ionisation (i.e., absorb energy in the visible region of
(c) I < Br < Cl < F (increasing electron gain enthalpy)
spectrum). The elements of which of the following
groups will impart colour to the flame ? (d) Li < Na < K < Rb (increasing metallic radius)
(a) 2 (b) 13
ll
21. Which of the following have no unit ?
.A
(c) 1 (d) 17 (a) Electronegativity
16. Which of the following sequences contain atomic (b) Electron gain enthalpy
numbers of only representative elements ? (c) Ionisation enthalpy
(a) 3, 33, 53, 87 (b) 2, 10, 22, 36 (d) Metallic character
rs
(c) 7, 17, 25, 37, 48 (d) 9, 35, 51, 88 22. Ionic radii vary in
17. Which of the following elements will gain one electron (a) inverse proportion to the effective nuclear charge.
he
more readily in comparison to other elements of their (b) inverse proportion to the square of effective
group ? nuclear charge.
(a) S (g) (b) Na (g) (c) direct proportion to the screening effect.
lis
(c) O (g) (d) Cl (g) (d) direct proportion to the square of screening effect.
18. Which of the following statements are correct ? 23. An element belongs to 3rd period and group-13 of the
ub
(a) Helium has the highest first ionisation enthalpy periodic table. Which of the following properties will
in the periodic table. be shown by the element ?
(b) Chlorine has less negative electron gain enthalpy (a) Good conductor of electricity
P
than fluorine.
(b) Liquid, metallic
(c) Mercury and bromine are liquids at room
temperature. (c) Solid, metallic
n
(d) In any period, atomic radius of alkali metal is (d) Solid, non-metallic.
er
the highest.
od
ANSWERS / HINTS
MCQs Type-II
M
14. (b, c) : P and S have d-orbitals in their valence shell and 16. (a, d) : Elements of s- and p-blocks are called
therefore, can accommodate more than 8 representative elements.
electrons in their respective valence shells. Hence 17. (a, d) : Chlorine has highest tendency to gain an electron
©
they show covalency more than 4. and its electron gain enthalpy (–ve) is high. O and
15. (a, c) : The elements of group 1(alkali metals) and S belong to group 16 but S has larger tendency to
group 2(alkaline earth metals) have low accept electron.
ionization enthalpies. Therefore, they impart 18. (a, c)
colour to flame.
19. (b, c) : (a) Zn2+ (30 – 2 = 28), Ca2+(20 – 2 = 18), Ga3+ 21. (a, d) : Electronegativity and metallic character has no
(31 – 3 = 28), Al3+(13 – 3 = 10) are not isoelectronic. units.
(b) K+ (19 – 1 = 18), Ca2+(20 – 2 = 18), Se3+ (21 – 3 = 18), 22. (a, c) : Ionic radius decreases as effective nuclear charge
Cl–(17 + 1 = 18) are isoelectronic. increases.
(c) P3– (15 + 3 = 18), S2– (16 + 2 = 18), Cl– (17 + 1 = 18), Ionic radius increases as the screening effect
.
ed
K+(19 – 1 = 18) are isoelectronic. increases.
23. (a, c) : The elements which belong to 3rd period and
(d) Ti4+ (22 – 4 = 18), Ar (18), Cr3+(24 – 3 = 21),
group 13 is aluminium. It is solid, metallic and
V5+ (23 – 5 = 18) are not isoelectronic.
rv
good conductor of electricity.
20. (b, c) : For (b) correct order is B < C < O < N
For (c) correct order is I < Br < F < Cl
se
electronic configuration of element having atomic
re
number 119 will be 8s1. Since it has one electron in
24. Explain why the electron gain enthalpy of the outermost s-orbital, its valency will be 1 and it
should belong to group 1 alongwith alkali metals.
ts
fluorine is less negative than that of chlorine.
The general formula of its oxide will be M2O where
Ans. In fluorine, the new electron to be added goes to 2p M represents the element.
gh
subshell while in chlorine, the added electron goes 27. Ionisation enthalpies of elements of second
to 3p subshell. Since the 2p-subshell is relatively period are given below :
small as compared to 3p-subshell, the added electron
Ionisation enthalpy/kJ mol–1 : 520, 899, 801, 1086,
ri
in small 2p subshell experiences strong inter
1402, 1314, 1681, 2080.
electronic repulsions in comparison to that in 3p
subshell in Cl. As a result, the incoming electron ll Match the correct enthalpy with the elements
does not feel much attraction from the nucleus and and complete the graph given in Fig. 1. Also
write symbols of elements with their atomic
.A
therefore, the electron gain enthalpy of F is less
negative than that of Cl. number.
25. All transition elements are d-block elements, 2500
rs
2000
⎯→
elements. Explain.
he
–1
Ans. All the elements in between s-and p-block i.e., between 1500
Δ H/kJ mol
elements should not be regarded as d-block elements. 28. Among the elements B, Al, C and Si,
However, the properties of these elements resemble (i) which element has the highest first ionisation
n
of periodic classification of elements more rational, (ii)which element has the most metallic
these are studied alongwith d-block elements. Thus, character ?
on the basis of properties, all the transition elements Justify your answer in each case.
od
are d-block elements but on the basis of electronic Ans. Arranging the elements into different groups and
configuration, all d-block elements are not transition periods :
elements. Group 13 14
M
29. Write four characteristic properties of p-block enthalpy of nitrogen is slightly positive. On the other
elements. hand, the outermost electronic configuration of O is
Ans. Refer Text; Page 10. 2s22px22py12pz1. It has higher positive charge (+8)
30. Choose the correct order of atomic radii of than nitrogen (+7) and lower atomic size than N.
fluorine and neon (in pm) out of the options Therefore, it has a tendency to accept an extra
given below and justify your answer. electron. Thus, electron gain enthalpy of O is
.
ed
(i) 72, 160 (ii) 160, 160 negative. However, oxygen has four electrons in the
2p subshell and can lose one electron to acquire
(iii) 72, 72 (iv) 160, 72
stable half filled configuration and therefore, it has
Ans. Atomic radius of F is smaller than that of neon.
rv
low ionization enthalpy. Because of stable
Therefore, option (i) 72, 160 is correct.
configuration of N, it cannot readily lose electron
31. Illustrate by taking examples of transition and therefore, its ionization enthalpy is higher than
se
elements and non-transition elements that that of O.
oxidation states of elements are largely based
on electronic configuration. 33. First member of each group of representative
elements (i.e., s and p-block elements) show
re
Ans. Oxidation state depends upon the tendency to lose
anomalous behaviour. Illustrate with two
or gain electrons to form negatively or positively
examples.
charged ions. It is the charge which an atom of the
Ans. Refer Text, Page 34–36.
ts
element has on its ion or appears to have when
present in the combined state with other atoms. The 34. p-block elements form acidic, basic and
actual sign and its magnitude depends upon the amphoteric oxides. Explain each property by
gh
electronic configuration of the atoms. For non- giving two examples and also write the
transition elements, the oxidation state is equal to reactions of these oxides with water.
the number of electrons present in the outermost Ans. The oxides of p-block elements show acidic, basic
ri
shell or eight minus the number of electrons present and amphoteric properties. The oxides formed by
in the outermost shell. For example, s-block the elements on the extreme left (most metallic) are
elements have either +1 (alkali metals) or +2 ll most basic (e.g., Na2O) whereas the oxides formed
(alkaline earth metals) because they have 1 and 2
by the elements on the extreme right (most non-
.A
electrons respectively in their outermost shells and
metallic) are most acidic (e.g., Cl2O7). Oxides of the
can lose these electrons. The p-block elements show
elements in the centre are amphoteric (e.g., Al2O3).
positive, negative or even zero oxidation states. For
The amphoteric oxides show acidic as well as basic
rs
oxidation state by losing only one electron from p- decreases while acidic character increases. For
orbitals. For example, Tl shows +1 oxidation state. example, or moving across the third period, the acidic
Similarly, group 14 elements can exhibit +4 and +2 and basic character of oxides is :
lis
oxidation states and group 15 elements can exhibit Na2O MgO Al2O3 SiO2 P2O5 SO3 Cl2O7
a minimum oxidation state of –3 and maximum Strongly Basic Amphoteric Weakly Acidic Strongly Very strongly
oxidation state of +5. basic acidic acidic acidic
ub
electronic configurations. But unlike p-block because of increasing electronegativity of the element
elements, the variable oxidation states of transition as we move along a period.
n
and inner transition elements arise due to The oxides of elements on extreme left are basic
involvement of electrons in outermost orbitals as and they react with water to form basic oxides.
er
non-transition elements and transition elements The oxides of elements on extreme right are acidic.
depend upon the electronic configurations of the For example, SO3, P2O5 are acidic and these react
atoms. with water as :
M
32. Nitrogen has positive electron gain enthalpy SO3 + H2O ⎯→ H2SO4
whereas oxygen has negative. However, oxygen has Sulphuric acid
©
35. How would you explain the fact that first explained in terms of net effect of the following
ionization enthalpy of sodium is lower than that factors :
of magnesium but its second ionization enthalpy (i) In going from top to bottom in a group, the
is higher than that of magnesium ? nuclear charge increases.
Ans. Refer conceptual Qs. 2 , Q. 9(Page 39). (ii) There is a gradual increase in atomic size
.
36. What do you understand by exothermic reaction due to an additional main energy shell (n).
ed
and endothermic reaction ? Give one example (iii) There is increase in shielding effect on the
of each type.
outermost electron due to increase in the
Ans. The chemical reactions which are accompanied by number of inner electrons.
rv
evolution of heat are called exothermic reactions.
The effect of increase in atomic size and the shielding
For example,
effect is much more than the effect of increase in
se
C(s) + O2 (g) ⎯→ CO2(g) ΔH = – 393.5 kJ
nuclear charge. As a result, the electron becomes
The chemical reactions which proceed by the less and less firmly held to the nucleus as we move
absorption of heat are called endothermic down the group. Hence, there is a gradual decrease
re
reactions. For example,
in the ionisation enthalpies in a group.
N2(g) + 3H2(g) ⎯→ 2NH3(g) ΔH = 92.4 kJ mol–1
40. How does the metallic and non-metallic
37. Arrange the elements N, P, O and S in the order
ts
character vary on moving from left to right in a
of period ?
(i) increasing first ionization enthalpy. Ans. On moving from left to right in a period, the number of
gh
(ii)increasing non-metallic character. electrons increases by one at each succeeding element
Give reason for the arrangement assigned. but the number of shell remains the same. Therefore,
ri
Ans. (i) S < P < O < N For reasons, refer Text, page 25. the effective nuclear charge increases and
(ii) P < S < N < O For reasons, refer Text, page 37. consequently the tendency of an atom to lose electron
38. Explain the deviation in ionization enthalpy ll decreases and hence metallic character decreases
of some elements from the general trend by when we move from left to right in a period. On the
.A
using Fig. 2. other hand, with increase in nuclear charge, the
2500 tendency of an atom to gain electron increases and
hence non-metallic character increases. Thus, metallic
(2080) Ne
Ionisation enthalpy
rs
he
O (1314) 41.
1000 (899) Be C (1086) atom. Give reason.
Ans. Refer NCERT Text Book Exercises Q. 13.
(520) Li B (801)
lis
moving from left to right in the periodic Element Atomic radius (pm)
table.
er
(i) Be (A) 74
(b) Ionization enthalpy decreases in a group
(ii) C (B) 88
from top to bottom ?
od
decrease in atomic size and increase in effective 44. Match the correct ionisation enthalpies and electron
nuclear charge. As a result of increase in effective gain enthalpies of the following elements :
nuclear charge, the attraction for the outer
©
45. Electronic configurations of some elements are given Column (I) Column (II)
in Column I and their electron gain enthalpies are given Electronic configuration Electron gain enthalpy/kJ mol–1
in Column II. Match the electronic configuration with
electron gain enthalpy. (i) 1s2 2s2 2p6 (A) – 53
(ii) 1s2 2s2 2p6 3s1 (B) – 328
(iii) 1s2 2s2 2p5 (C) – 141
.
1s2 2s2 2p4
ed
(iv) (D) + 48
ANSWERS / HINTS
rv
Matching Type
43. (i)–(C), (ii)–(D), (iii)–(E), (iv)–(B), (v)–(A) 45. (i)–(D), (ii)–(A), (iii)–(B), (iv)–(C)
se
As we move from left to right in a period, the atomic (i) This configuration corresponds to noble gas
radius decreases. Thus, Be > B > C > N > O. (neon). Since noble gases have positive ΔegH
re
44. (i)–(B), (ii)–(A), (iii)–(D), (iv)–(C) values, therefore, electronic configuration A
(i) Most reactive non-metals have high ΔiH1 and corresponds to element D.
ΔiH2 and negative ΔegH. Therefore, the element (ii) The electronic configuration corresponds to alkali
is B.
ts
metal (potassium). Since alkali metals have low
(ii) Most reactive metals have low ΔiH1 and high ΔiH2 ΔegH values, therefore, electronic configuration
(because the second electron has to be removed (ii) corresponds to element A.
gh
from noble gas configuration) and small ΔegH
(iii) This electronic configuration corresponds to
value. Therefore, the element is A.
halogen (fluorine). Since halogens have high
(iii) Noble gases are least reactive elements and
ri
negative ΔegH values, therefore, the electronic
have very high ΔiH1 and ΔiH2 and have positive
configuration (iii) corresponds to element B.
ΔegH. Therefore, the element is D.
(iv) Metals forming binary halides are rare earth
ll (iv) This electronic configuration corresponds to
metals. They have ΔiH1 and ΔiH2 values slightly oxygen family (oxygen). Since oxygen has ΔegH
.A
higher than those of most reactive metals (such value less negative than halogens, therefore, the
as A) and have less negative ΔegH values. electronic configuration (IV) corresponds to
Therefore, the element is C. element C.
rs
In the following questions a statement of Assertion is more shielded by the inner core of electrons than the
(A) followed by a statement of Reason (R) is given. 2s electrons.
(a) Assertion and reason both are correct statement
lis
shielding effect of inner core of electrons does not (d) Assertion and reason both are wrong statements.
increase very much to compensate for the increased 48. Assertion (A) : Electron gain enthalpy, in general,
n
attraction of the electron to the nucleus. becomes less negative as we go down a group.
(a) Assertion is correct statement and reason is
er
and reason is correct explanation of assertion. (a) Assertion and reason both are correct statements
(c) Assertion and reason both are correct but reason is not correct explanation for assertion.
statements.
M
ANSWERS / HINTS
Assertion Reason Type
46. (b) 47. (c) 48. (b)
.
table that led to its modification.
Ans. Refer Text. Page 28–29.
ed
Ans. Refer Text. Page 4.
50. Define ionisation enthalpy. Discuss the factors
54. In what manner is the long form of periodic
affecting ionisation enthalpy of the elements
rv
table better than Mendeleev's periodic table ?
and its trends in the periodic table.
Explain with examples.
Ans. Refer Text Page 23–25.
se
Ans. Refer Text. Page 4,6.
51. Justify the given statement with suitable
55. Discuss and compare the trend in ionisation
examples—“the properties of the elements are
enthalpy of the elements of group 1 with those
re
a periodic function of their atomic numbers.”
of group 17 elements.
Ans. Refer Text Page 6–7.
Ans. Refer Text. Page 23–25.
52. Write down the outermost electronic
ts
configuration of alkali metals. How will you
gh
ri
ll
.A
or false?
I. Read the following passage and answer
questions 1–5 that follow: II. Read the following passage and answer questions
6–10 that follow:
Ionization enthalpy and electron gain enthalpy are the
lis
fundamental properties which measure the tendency of Modern periodic table is essentially the consequence of
an atom to lose or gain an electron. In addition to size the periodic variation in electronic configurations of
ub
of the atom and charge on the nucleus, the electronic atoms, which determine the physical and chemical
configurations of atoms or ions also play significant role properties of the elements and compounds. In this
in determining their values. table, the elements are arranged in horizontal rows
P
periodic table?
2. The electronic configuration of three neutral atoms are:
er
value?
8. An element has the outer electronic configuration: 3s23p6.
3. Which out of F or Cl would have a higher negative
To which period and group does this element belong?
electron gain enthalpy?
M
of lithium vapours? (Ionization energy of lithium is 10. Five elements A, B, C, D and E have the atomic numbers
520 kJ mol–1). 6, 12, 32, 36 and 14 respectively. Which of these belong
to same group?
True or False Questions 10. The p-block has ……… columns and d-block has ……
columns in the periodic table.
Predict which of the following statements are true or
Assertion Reason Questions
false.
1. The elements cerium (Z = 58) and neptunium (Z = 93) Note : In the following questions a statement of assertion
.
belong to f-block elements. followed by a statement of reason is given. Choose
ed
the correct answer out of the following choices.
2. Metallic character increases from top to bottom in a (a) Assertion and reason both are correct statements and
group and non-metallic character increases from left reason is correct explanation for assertion.
rv
to right in a period. (b) Assertion and reason both are correct statements but
reason is not correct explanation for assertion.
3. The electron gain enthalpy of oxygen is more negative
(c) Assertion is correct statement but reason is wrong
se
than that of sulphur. statement.
4. Among oxides of group 15, N2O3 is acidic while Bi2O3 (d) Assertion is wrong statement but reason is correct
statement.
re
is basic.
5. The reactivity of non-metals increases down the group
from top to bottom. 1. Assertion : The first ionization enthalpy of
aluminium is lower than that of
ts
6. Chlorine is the most electronegative element in the
magnesium.
periodic table.
Reason : Atomic radius of aluminium is smaller
gh
7. The first ionization enthalpy of Mg is higher than that
than that of magnesium.
of Al.
2. Assertion : Both N 2 and NO + are diamagnetic
8. Cl–, Ar and Ca2+ are isoelectronic species.
ri
substances.
9. The first ionization enthalpy of O is lower than that of N. Reason : NO+ is isoelectronic with N2.
10. The effective nuclear charge increases in a period from 3. Assertion : F atom has less negative electron gain
ll
left to right. enthalpy than Cl atom.
.A
11. Among O, O– and O2–, oxygen has the largest size. Reason : Additional electrons are repelled more
12. Potassium has higher ionization enthalpy than sodium. effectively by 3p–electrons in Cl than by
13. Second electron gain enthalpy of oxygen is more 2p–electrons in F atom.
rs
negative than first electron gain enthalpy. 4. Assertion : The first ionization energy of Be is greater
14. The electron gain enthalpies of noble gases are positive. than that of B.
he
15. Among halogens, fluorine has the highest negative Reason : 2p-orbital is higher in energy than
electron gain enthalpy. 2s-orbital.
5. Assertion : The elements having 1s2 2s2 2p6 3s2 and
lis
3. An element of group 13 which forms basic oxide is Reason : These are isoelectronic species.
……….
n
of elements.
Reason : In isoelectronic ions, the size decreases
5. The formula of compound formed with element (M) with increase in atomic number.
od
block.
8. The IUPAC name of undiscovered element with negative as we go down a group.
atomic number 122 is ………. and its symbol is ………. Reason : Size of the atom increases on going down
9. The element having the configuration : (n – 1)d ns 2 2 the group and the added electron would
for n = 4 belongs to group ……… and period ………. be farther from the nucleus.
10. Assertion : Boron has a smaller first ionisation 19. Write the IUPAC name and symbol for an element
enthalpy than beryllium. having atomic number 113.
–
Reason : The penetration of a 2s electron to the 20. Name two species which are isoelectronic with Cl .
nucleus is more than the 2p electron hence
2p electron is more shielded by the inner
.
core of electrons than the 2s electrons.
ed
1. What is periodicity in elements ? What is its cause ?
Multiple Choice Questions Explain with one example.
2. What is the basic difference in the approach bet
rv
Refer TOPICWISE MULTIPLE CHOICE QUESTIONS, ween Mendeleev’s periodic law and the modern
COMPETITION FILE Page 3/60.
periodic law?
se
3. Why do elements in the same group have similar
One Word/Very Short Sentence Answers physical and chemical properties ?
re
4. What does the atomic radius and ionic radius really
1. How many groups are there in p-block elements and d-
mean to you ?
block elements ?
5. How does atomic radius vary in a period and in a group?
2. State modern periodic law.
ts
How do you explain the variation ?
3. How does ionisation enthalpy of the elements vary as
6. Explain why cations are smaller and anions larger in
gh
we move across the period from left to right ?
radii than their parent atoms ?
4. Noble gases have zero electron gain enthalpy values.
7. Account for the fact that the third period of the periodic
Explain.
table has eight and not eighteen elements.
ri
5. Which is large Na+ or K+ ? Why ? 8. Name the different blocks of elements in the periodic
6. What is meant by electron gain enthalpy ? ll table. Give the general electronic configuration of each
7. Write the electronic configuration of an element with block.
.A
atomic number 17. To which block does this element 9. What are transition elements ? Which of the following
belong ? are not transition elements ?
8. What are transition elements ? Name two transition Cu(Z = 29), S(Z = 16), Ga(Z = 31), Pd(Z = 46), U(Z = 92).
rs
and one negatively charged that will be isoelectronic following elements belong ?
with Ne. (i) 13Al (ii) 24Cr (iii) 29Cu (iv) 11Na.
ub
11. In terms of electronic configuration, what the elements 12. What is ionization enthalpy ? How does the ionization
of a given period and a group have in common ? enthalpy of the elements vary as we move down a group
12. What is similar about electronic structure of Li, Na and along a period ?
P
series ? 14. Among the elements Li, Na, Mg, S and Xe, which one has
er
16. Predict the position of the element in the periodic table (i) 1s2 2s2 2p6
satisfying the electronic configuration (n – 1) d1 ns2 for
(ii) 1s2 2s2 2p5
n = 4.
©
18. The first ionization enthalpy of C is 1086 kJ mol–1. (ii) Which of them has the highest negative electron
Would you expect the first ionization enthalpy of Si to gain enthalpy.
be greater or lesser than this amount? Explain. (iii) Which of them has positive electron gain enthalpy.
19. Describe the theory associated with the radius of atom 31. What do you understand by the term electron gain
as it enthalpy? Explain why do electron gain enthalpies of
.
ed
(a) gains an electron (b) loses an electron the atoms become more negative from left to right
20. What are the major differences between metals and along a period in the periodic table.
non-metals ? 32. There is a close relationship between the electronic
rv
21. Write the general outer electronic configurations of configuration and the chemical behaviour of elements.
s-, p-, d- and f- block elements. Justify the statement giving reasons supported by two
se
examples.
22. The increasing order of reactivity among group 1 elements
is Li < Na < K < Rb < Cs whereas that among the group 33. Account for the fact that the 4th period has eighteen
re
17 elements is F > Cl > Br > I. Explain. and not eight elements.
34. The valency of representative elements is either equal
23. Differentiate between ionization enthalpy and electron
to the number of valence electrons or eight minus this
gain enthalpy.
ts
number. What is the basis of this rule?
24. Define atomic radius. How do the atomic radii of
35. Some of the periods have different number of elements
elements vary when we move (i) down a group
gh
than others. Explain.
(ii) across a period? How would you account for this
conclusion? 36. For each of the following pairs, predict which one has
greater ionisation energy and greater electron affinity :
ri
25. The radii of positive ions are always smaller than the
radii of the corresponding atoms. Discuss. (a) I, I– (b) B, C (c) Li, Li+.
29. A, B and C are three elements. B is an inert gas other atom. Explain.
than helium. With this information complete the 41. In a period of the periodic table generally the size of an
ub
the valence element 42. Nitrogen has positive electron gain enthalpy whereas
shell belongs oxygen has negative electron gain enthalpy. However,
n
A Z–1 ................. ................. oxygen has lower ionization enthalpy than nitrogen.
er
Also explain the following : oxides. Explain each property by giving two examples
(i) Electron gain enthalpy of element is more negative and also write the reactions of these oxides with water.
than that of element C. 44. Explain the following :
M
(ii) Ionization enthalpy of element C is less than that (i) Electronegativity of elements increase on moving
of element A from left to right in the periodic table.
©
(iii) Electron affinity of B is zero. (ii) Ionization enthalpy decreases in a group from top
30. Write the electronic configurations of the elements with to bottom ?
atomic numbers 9, 11, 21 and 36. Stating reasons, 45. What do you understand by the terms
predict the following from these configurations :
(i) Ionization enthalpy
(i) Which of them has the lowest ionization enthalpy ?
(ii) Electron gain enthalpy?
.
(iii) Size of the cation is smaller than that of the 5. Which is an inert gas ?
ed
neutral atom while the size of the anion is more. 6. Which shows a valency of 3? Justify your answers.
47. Explain the term electron gain enthalpy. Discuss the 4. Describe the main characteristic properties of s, p, d
rv
factors which influence its value.
and f-block elements.
48. Why is ionization enthalpy of Be more than that of B?
5. Answer the following with reasons :
se
Explain.
49. How does the metallic and non-metallic character vary (a) Which element has higher value of negative
on moving from left to right in a period ? electron gain enthalpy, F or Cl ?
re
50. Among alkali metals which element do you expect to (b) Which element has larger atomic volume, Na or K?
be least electronegative and why ? (c) Which element is more metallic, Na or Cl ?
51. How would you explain the fact that first ionization
ts
(d) Which element has smaller size, O or F ?
enthalpy of sodium is lower than that of magnesium
but its second ionization enthalpy is higher than that (e) Which element has larger second ionization
gh
of magnesium ? enthalpy, Na or F ?
52. Arrange the elements N, P, O and S in the order of : 6. Among the elements of second period (Li to Ne) pick
ri
(i) increasing first ionization enthalpy. out the element :
(ii) increasing non-metallic character. (i) with the largest atomic radius
Give reason for the arrangement assigned.
ll (ii) with the highest ionization enthalpy
53. Why does the first ionization enthalpy increase as we
.A
(iii) with the highest negative electron gain enthalpy
go from left to right across a given period of the periodic (iv) is most reactive metal
table?
(v) is the most reactive non-metal.
rs
below :
than that of sodium. On the other hand, the second
ionization enthalpy of sodium is very much higher than Elements IE1 IE2
lis
58. Lanthanoids and actinoids are placed in separate rows at Which of the above elements is likely to be :
the bottom of the Periodic Table. Explain the reason for (a) a reactive metal
n
1. What is modern periodic law? Discuss the main features (d) a reactive non-metal.
of the long form of the periodic table. Give its important 8. Among the elements of the third period (Na to Ar),
advantages and disadvantages. pick out the element
M
2. What is ionization enthalpy ? On what factors does it (i) with the highest ionization enthalpy
depend? How does ionisation enthalpy of the elements (ii) with the largest atomic radius
©
vary as we move down a group and along a period? (iii) that is most reactive non metal
3. The electronic configurations of some elements are (iv) that is most reactive metal.
given below : 9. Discuss the following terms and account for the
(i) 1s2, 2s2 2p6, 3s2 (ii) 1s2, 2s2 2p6 variation in groups and periods of the Periodic Table :
(iii) 1s2, 2s2 2p2 (iv) 1s2, 2s2 2p6, 3s1 (i) Electron gain enthalpy
(v) 1s2, 2s2 2p5. (ii) Ionization enthalpy.
.
Passage Based Questions Fill in the Blanks Questions
ed
1. F 1. (n – 1)d1–10ns0–2 2. fluorine 3. thallium
2. R
4. f 5. MF4 6. supplied
rv
3. Cl 14 2 2
7. 5f 6d 7s , d 8. unbibium, Ubb
4. Li(g) + I.E ⎯⎯→ Li+ (g) + e– (g)
se
9. 4, 4 10. 6, 10
I.E. = 520 kJ
Assertion Reason Questions
7 × 10 −3
= 1 × 10–3 mol
re
Moles of Li vapours = 1. (b) 2. (a) 3. (c) 4. (a) 5. (a)
7
6. (c) 7. (a) 8. (c) 9. (d) 10. (a).
Amount of energy needed to ionize 1 mol of Li
ts
vapours = 520 kJ Very Short Answer Questions
–3
Amount of energy needed to ionize 1 × 10 mol 1. 6, 10
gh
of Li vapours 5. K+ is larger because size increases as we go down
= 520 × 1 × 10–3 kJ the group due to increase in principal shell.
7. p-block 9. C 10. Na+ , F–
ri
or = 520 J
11. In a period, number of shells is same.
5. True statement In a group, electrons in the outermost shell are
6. Periods = 7, Groups = 18
ll same.
7. According to Mendeleev's periodic law, the physical and 13. Ten 15. Ni, Fe, Cr
.A
chemical properties of the elements are periodic function 16. period 4, group 11. 17. K, Mg, Al, B
of their atomic weight. But according to Modern periodic 19. Ununtrium : Uut 20. Ar, K+
rs
Q.1.The first ionisation enthalpy of carbon atom is of B. After the removal of first electron, the second electron to
©
greater than that of boron whereas the reverse is true be removed from C atom is from 2p (2p1) whereas that from
for the second ionisation enthalpy. Explain. B atom is from 2s (2s2). Now, 2s orbital is more penetrating and
Ans. Electronic configuration of C is 1s2 2s22p2 and that hence is more strongly attracted by the nucleus. As a result,
of B is 1s22s22p1. The nuclear charge is more in C than in B. second I.E. of B is higher than that of C.
As a result, first ionisation enthalpy of C is higher than that
Q.2. Consider the elements : N, P, O and S. non-metallic element and P is the least non-metallic
Arrange them in order of element. Therefore, the increasing order of non-
metallic character is
(i) increasing size
P<S<N<O
(ii) increasing first ionization enthalpy
Q.3. What would be the atomic number of the next
.
(iii) increasing negative gain enthalpy
(i) alkali metal (ii) halogen and
ed
(iv) increasing non-metallic character. (iii) noble gas
Ans. These elements can be arranged in different periods if discovered in future.
rv
and groups as : Ans. (i) The next alkali metal, if discovered will have to
Group No. 15 16 be placed in the eighth period and hence, its outermost
se
2nd period N O electronic configuration will be 8s1. Therefore, its atomic
3rd period P S number will be 118 + 1 = 119.
(ii) The next halogen, if discovered will have the outermost
re
(i) The atomic radii increase from top to bottom in a
electronic configuration as 7s2 7p5. The filling of 7p orbitals
group. Therefore, N and O have smaller atomic size
will start after filling 6d orbitals completely. In accordance
than that of P and S. Further, atomic size decreases
with Aufbau principle, 6d orbitals will be completely filled at
ts
along a period with increase in atomic number and
atomic number 112. Therefore, the atomic number of next
therefore, the atomic size of N should be more than
halogen if discovered will be 112 + 5 = 117.
gh
that of O and atomic size of P should be more than
(iii) The next noble gas, if discovered will have the
that of S. However, when we move from N to O, the
electronic configuration 7s2 7p6 and hence its atomic number
nuclear charge increases by one. But at the same
ri
will be 112 + 6 = 118.
time, one of the p-orbital has two electrons which
repel each other. The interelectronic repulsions in O Q.4. Arrange the ions : Li2+, He+ and Be3+ in the
outweigh the effect of increased nuclear charge and increasing order of their ionic radii.
ll
hence atomic size increases from N to O. However, Ans. He+, Li2+ and Be3+ are isoelectronic ions. Among
.A
P has less atomic size than S as expected because isoelectronic ions, the ionic radius decreases as the positive
repulsions are not very strong in relatively larger 3p charge increases. Hence ionic radii decreases in the order :
orbital to outweigh the effect of increased nuclear He+ < Li2+ > Be3+.
rs
charge. Hence, the correct order of increasing atomic Q.5. Which of the elements Na, Mg, Si and P would
size is have the greater difference between the first and the
he
difficult to remove an electron from N than from O configuration i.e., (1s22s22p6). Therefore, its IE2 is expected to
even though O has higher nuclear charge. Similarly, be very high. Consequently, the difference in first and second
ionization enthalpies would be greatest in case of Na.
P
S<P<O<N enthalpy F or Cl ?
(iii) The electron gain enthalpies of P and S are more (c) Which has lower IE2 : Na or Mg ?
od
negative than those of N and O because adding an (d) Which has larger IE1: N or O ?
electron to smaller size 2p-subshell causes greater (e) Which is more metallic N or K?
repulsion than adding an electron to larger 3p-
M
low tendency to gain an electron. Therefore, their (b) Which is more acidic P2O5 or SiO2 ?
electron gain enthalpies are less negative. Thus, the (c) Which is less acidic H3PO3 or HClO
increasing order of negative electron gain enthalpy is (d) Which is more stable Sn2+ or Sn4+ ?
N<P<O<S (e) Which shows resemblances in behaviour with
(iv) Since non-metallic character decreases down a group Al, Mg or Be ?
and increases along a period, therefore, O is the most Ans. (a) Mg(OH)2 (b) P2O5 (c) H3PO3 (d) Sn4+ (e) Be
.
ed
rv
PERIODIC TRENDS
se
PERIODS
re
ATOMIC RADII Decreases along a period
ts
Ionization enthalpy
Electron gain enthalpy (-) Increases along a period
gh
GROUPS
Electronegativity
Increases down a Group
ri
Decreases along a period
Character
Elements having higher negative electron gain enthalpies are good oxidising agents.
Elements at the two extremes of a period are most reactive.
Fr is the largest element in the periodic table.
n
er
od
A1. (d)
A2. The maximum number of elements in 3rd period is : (b) Negative electron gain enthalpy
(a) 8 (b) 18 (c) Ionisation enthalpy
.
(c) 32 (d) between 8 and 18 (d) Atomic volume
ed
A3. Which pair of atomic number represents s-block A12. The correct order or radii of three species Ca, Ca+ and
elements ? Ca2+ is :
(a) 7,15 (b) 6, 12 (a) Ca > Ca+ > Ca2+ (b) Ca2+ > Ca+ > Ca
rv
(c) 9, 17 (d) 3, 12
(c) Ca+ > Ca > Ca2+ (d) Ca+ > Ca2+ > Ca
A4. Transition metals have the general electronic 3– 2– –
A13. In the P , S and Cl ions, the increasing order of size
se
configuration :
is :
(a) ns2nd1–10 (b) ns2np1 (n–1)d1–10
1,2 1–10 (a) Cl–, S2–, P3– (b) P3–, S2–, Cl–
(c) ns (n–1)d (d) ns2np6(n–1)d1–10
re
A5. The tendency towards complex formation is maximum (c) S2–, Cl–, P3– (d) S2–, P3–, Cl–
in A14. The property which regularly increases down the
(a) s-block elements (b) p-block elements group in the periodic table is :
ts
(c) d-block elements (d) none of these (a) Ionisation enthalpy (b) Electronegativity
A6. A element with atomic number 112 has recently been (c) Reducing nature (d) Electron gain enthalpy
gh
named as copernicium. It is named in honour of
A15. The ionisation gain enthalpy of nitrogen is more than
scientist astronomer Nicolaus Copernicus. Which of
that of oxygen because of :
the following statement about this element is not
(a) greater attraction of nucleus for the electrons
ri
correct?
(b) the extra stability of half filled p-orbitals
(a) It belongs to d-block
(b) It belongs to 12th group ll (c) smaller size of the nitrogen atom
(c) Its electronic configuration is [Rn] 5f146d107s2 (d) more penetrating effect.
.A
(d) It belongs to 6th period A16. The correct order of increasing radii of the elements
A7. The recently discovered element, Meitnerium (Z = 109) Na, Si, Al and P is :
belongs to : (a) Si, Al, P, Na (b) Al, Si, P, Na
rs
(a) s-block (b) p-block (c) P, Si, Al, Na (d) Al, P, Si, Na
(c) d-block (d) f-block A17. Which of the following electronic configurations
he
A8. The fourth period of the p-block contains : corresponds to elements with largest negative
(a) 6 elements (b) 8 elements electron gain enthalpy ?
(c) 10 elements (d) 18 elements (a) 1s2 2s2 2p5 (b) 1s2 2s2 2p6
lis
2 2 6 2 5
A9. Elements A, B, C, D and E have the following electronic (c) 1s 2s 2p 3s 3p (d) 1s2 2s2 2p6 3s1
configurations : A18. In which of the following pairs, the first atom or ion is
A : 1s22s22p1 B : 1s22s22p63s23p1 not larger than the second ?
ub
C : 1s 2s 2p 3s 3p
2 2 6 2 3
D : 1s22s22p63s23p5 (a) K, K+ (b) S, O
E : 1s 2s 2p 3s 3p 4s
2 2 6 2 6 2
(c) Br, Br –
(d) N, O
The elements belonging to same group in the periodic A19. The family of elements with the highest ionisation
P
(a) Aluminium (b) Silicon A20. Which of the following has largest negative electron
gain enthalpy ?
(c) Germanium (d) Sulphur
(a) F (b) Cl
M
A2. (a) A3. (d) A4. (c) A5. (c) A6. (d) A7. (c) A8. (a) A9. (c) A10. (b) A11. (a)
A12. (a) A13. (a) A14. (c) A15. (b) A16. (c) A17. (c) A18. (c) A19. (c) A20. (b) A21. (a)
A22. The ionic size of Na+, Mg2+, Al3+ and Si4+ follows the (a) Ne > F > O > N > C > B > Se > Li
order : (b) Ne > F > N > C > O > Be > B > Li
(a) Na+ < Mg2+ < Al3+ < Si4+ (c) Li > B > Be > C > O > N > F > Ne
.
(d) Ne > F > N > O > C > Be > B > Li
ed
(b) Na+ > Mg2+ > Al3+ > Si4+
(c) Na+ > Mg2+ < Al3+ < Si4+ A30. Which of the following configuration is expected to have
maximum difference in second and third ionisation
(d) Na+ < Mg2+ > Al3+ > Si4+
enthalpies ?
rv
A23. Amongst the following elements (where electronic
(a) 1s22s22p2 (b) 1s22s22p6 3s1
configurations are given below), the one having the 2 2 6 2
(c) 1s 2s 2p 3s (d) 1s22s22p1
highest ionisation enthalpy is :
se
A31. The process requiring absorption of energy is
(a) [Ne] 3s23p1 (b) [Ne] 3s23p3 (a) F → F
–
(b) O– → O2–
2 2
(c) [Ne] 3s 3p (b) [Ne] 3d104s24p2 (c) Cl → Cl –
(d) H → H–
A24. The penetration of the electrons in any principal shell A32. The incorrect statements among the following is
re
varies as : (a) The first ionisation enthalpy of Al is less than
(a) s > p > d > f (b) s < p < d < f the first ionisation enthalpy of Mg.
(c) s > p < d > f (d) s < p > d > f (b) The second ionisation enthalpy of Mg is
ts
A25. Which of the following oxide is most acidic ? greater than the second ionisation enthalpy
(a) BeO (b) MgO of Na.
gh
(c) CaO (d) BaO (c) The first ionisation enthalpy of Na is less than
A26. Which one of the following processes proceeds with the the first ionisation enthalpy of Mg.
absorption of energy ? (d) The third ionisation enthalpy of Mg is greater
than the third ionisation enthalpy of Al.
ri
(a) F(g) + e– ⎯⎯→ F– (g)
(b) O–(g) + e– ⎯⎯→ O2– (g) A33. The amount of energy released when one million
(c) O(g) + e– ⎯⎯→ O– (g) ll atoms of iodine in vapour state are converted to
(d) Cl(g) + e– ⎯⎯→ Cl– (g) I– ions is 4.9 × 10–13 J. The electron gain enthalpy
of iodine is
.A
A27. Which of the following statement is correct ?
(a) 1st and 2nd IE of nitrogen is more than 1st and 2nd (a) 4.9 × 1020 kJ mol–1 (b) 29.5 × 1010 kJ mol–1
IE of oxygen. (c) 2.95 kJ mol–1 (d) 295 kJ mol–1
(b) IE of oxygen is greater than IE of nitrogen. A34. The element with highest electron gain enthalpy
rs
A28. Ionization enthalpy of lithium is 520 kJ mol–1. How (c) period 3 group 17 (d) period 3 group 16
much enthalpy in joules must be needed to convert all A35. The five successive ionisation enthalpies of an
atoms of Li to Li+ ions present in 7 mg of Li vapours. element are 801, 2427, 3638, 25024, and 32824
(a) 74.3 kJ (b) 520 × 6.023 × 10–17 J kJ mol –1 respectively. The number of valence
lis
(c) 3 (d) 4
P
A22. (b) A23. (c) A24. (a) A25. (a) A26. (b) A27. (c) A28. (c) A29. (d) A30. (a) A31. (b)
A32. (b) A33. (d) A34. (c) A35. (c)
n
AIPMT, NEET & Other State Boards’ Select the correct answer using the codes given below :
Medical Entrance
(a) (i) < (ii) < (iv) < (iii)
B1. The electronic configuration of the atom having (b) (ii) < (i) < (iv) < (iii)
M
maximum difference in the first and the second (c) (i) < (iii) < (iv) < (ii)
ionization enthalpies is (d) (iii) < (iv) < (i) < (ii) (Kerala P.M.T. 2007)
©
(a) 1s22s22p63s1 (b) 1s22s22p63s2 B3. With which of the following electronic configuration
(c) 1s22s22p1 (d) 1s22s22p63s23p1 an atom has the lowest ionization ethalpy ?
(a) 1s22s22p6 (b) 1s22s22p5
(e) 1s22s22p3 (Kerala P.M.T. 2007) 2 2 3
(c) 1s 2s 2p (d) 1s22s22p53s1
(C.B.S.E. Med. 2007)
B4. Identify the correct order of size of the following : O(g) + e– → O– (g) ; Δf HΘ = – 141 kJ mol–1
(a) Ca2+ < K+ < Ar < Cl– < S2– O–(g) + e– → O2– (g) ; Δf HΘ = + 780 kJ mol–1
.
(b) Ar < Ca2+ < K+ < Cl– < S2– Thus process of formation of O2– in gas phase is
ed
(c) Ca2+ < Ar < K+ < Cl– < S2– unfavourable even though O2– is isoelectronic with
neon. It is due to the fact that,
(d) Ca2+ < K+ < Ar < S2– < Cl– (C.B.S.E. Med. 2007)
(a) oxygen is more electronegative
B5. The element with atomic number 117 has not been
rv
(b) addition of electron in oxygen results in larger
discovered yet. In which family would you place this
size of the ion.
element if discovered ?
se
(c) electron repulsion outweighs the stability gained
(a) Alkali metals (b) Alkaline earth metals
by achieving noble gas configuration.
(c) Halogens (d) Noble gases
(d) O– ion has comparatively smaller size than oxygen
(e) Coinage metals (Kerala Med. 2009)
re
atom (A.I.P.M.T. 2015)
B6. The correct order of the decreasing ionic radii in the
B13. The species Ar, K+ and Ca2+ contain the same number
following isoelectronic species is :
of electrons. In which order do their radii increase?
(a) S2– > Cl– > K+ > Ca2+ (b) K+ > Ca2+ > Cl– > S2–
ts
(a) Ca2+ < K+ < Ar (b) K+ < Ar < Ca2+
2+ + 2– –
(c) Ca > K > S > Cl (d) Cl– > S2– > Ca2+ < K+ (c) Ar < K+ < Ca2+ (d) Ca2+ < Ar < K+
(CBSE Med. 2010) (A.I.P.M.T. 2015)
gh
B7. Which of the following represents the correct order of B14. In which of the following options the order of
increasing electron gain enthalpy with negative sign arrangement does not agree with the variation of
for the elements O, S, F and Cl? property indicated against it?
ri
(a) F < S < O < Cl (b) S < O < Cl < F (a) I < Br < Cl < F (increasing electron gain enthalpy)
(b) Li < Na < K < Rb (increasing metallic radius)
(c) Cl < F < O < S (d) O < S < F < Cl ll (c) Al3+ < Mg2+ < Na+ < F– (increasing ionic size)
(CBSE Med. 2010) (d) B < C < N < O (increasing first ionization enthalpy)
.A
B8. Identify the wrong statement in the following : (NEET 2016)
(a) Amongst isoelectronic species, smaller the positive B15. The element Z = 114 has been discovered recently. It
charge on the cation, smaller is the ionic radius. will belong to which of the following family/group and
(b) Amongst isoelectronic species, greater the negative electronic configuration?
rs
charge on the anion, larger is the ionic radius. (a) Carbon family, [Rn] 5f14 6d10 7s2 7p2
(c) Atomic radius of the elements increases as one (b) Oxygen family, [Rn] 5f14 6d10 7s2 7p4
he
moves down the first group of the periodic table. (c) Nitrogen family, [Rn] 5f14 6d10 7s2 7p6
(d) Atomic radius of the elements decreases as one (d) Halogen family, [Rn] 5f14 6d10 7s2 7p5
moves across from left to right in the 2nd period of (NEET 2017)
lis
the periodic table. (A.I.P.M.T. 2012) B16. Magnesium reacts with an element (X) to form an ionic
B9. Which of the following pairs contain metalloid compound. If the ground state electronic configuration
elements in the periodic table? of (X) is 1s2 2s2 2p3, the simplest formula for this
ub
2– – +
(c) O > F > Na (d) Al3+ > Mg2+ > N3– (a) Li < Be < B < C < O < N < F < Ne
(AIPMT 2014, Karnataka CET 2018)
er
(a) Mg, Al3+ (b) Al3+, Mg (b) Li < B < Be < C < N < O < F < Ne (NEET 2019)
(c) Mg2+, Al (d) Al, Mg2+ JEE (Main) & Other State Boards’
(e) Mg2+, Al3+ (Kerala PMT 2015) Engineering Entrance
M
B12. The formation of the oxide ion, O2– (g), from oxygen B18. The electronic configuration of an element with
atom requires first an exothermic and then an maximum negative electron gain enthalpy is
endothermic step as shown below : (a) 1s22s22p3 (b) 1s22s22p5
©
2 2 6 2 5
(c) 1s 2s 2p 3s 3p (d) 1s22s22p63s23p3
2 2 6 1
(e) 1s 2s 2p 3s (Kerala P.E.T. 2008)
B4. (a) B5. (c) B6. (a) B7. (d) B8. (a) B9. (d) B10. (c) B11. (a) B12. (c)
B13. (a) B14. (a, d) B15. (a) B16. (d) B17. (c) B18. (c)
B19. In the graph below, the one which represents an alkali B25. Which of the following is the largest in size ?
metal with the higher atomic number is (a) S2– (b) Se2–
.
2–
(a) X (c) O (d) Te2– (Odisha JEE 2011)
First ionisation enthalpy
Z
ed
(b) Y B26. An element belongs to group 15 and third period of the
L
periodic table. Its electronic configuration will be
in arbitrary units
(c) Z
(a) 1s22s22p3 (b) 1s22s22p4
rv
(d) L Y 2 2 6 2 3
(c) 1s 2s 2p 3s 3p (d) 1s22s22p63s23p2
(e) M X (WB – JEE 2011)
M
se
B27. Which one of the following has the lowest ionisation
Atomic Number
(Kerala P.E.T. 2008) enthalpy ?
(a) 1s22s22p6 (b) 1s22s22p63s1
re
B20. The set representing the correct order of ionic radius is 2 2 5
(c) 1s 2s 2p (d) 1s22s22p3 (WB – JEE 2011)
(a) Li+ > Be2+ > Na+ > Mg2+
B28. The correct decreasing order of first ionisation
(b) Na+ > Li+ > Mg2+ > Be2+ enthalpies of five elements of the second period is
ts
(c) Li+ > Na+ > Mg2+ > Be2+ (a) Be > B > C > N > F (b) N > F > C > B > Be
(d) Mg2+ > Be2+ > Li+ > Na+ (A.I.E.E.E. 2009) (c) F > N > C > Be > B (d) N > F > B > C > Be
gh
B21. Which of the following set will have highest hydration (e) F > C > N > B > Be (Kerala PET 2011)
enthalpy and highest ionic radius? B29. The correct order of decreasing electronegativity values
(a) Na and Li (b) Li and Rb among the elements I-beryllium, II-oxygen, III-
ri
(c) K and Na (d) Cs and Na nitrogen and IV-magnesium is
(AMU Engg. 2010) (a) II > III > I > IV (b) III > IV > II > I
B22. The first (ΔiH1) and second (ΔiH2) ionization enthalpies ll (c) I > II > III > IV (d) I > II > IV > III
(in kJ mol–1) and the electron gain enthalpy (ΔegH) (in (e) II > III > IV > I (Kerala PET 2011)
.A
kJ mol–1) of the elements I, II, III, IV and V are given B30. Considering the elements B, C, N, F and Si, the correct
below: order of their non-metallic character is
Element Δ i H1 Δ i H2 Δeg H (a) B > C > Si > N > F
rs
III 1681 3374 – 328 (d) F > N > C > Si > B (A.M.U. Engg. 2011)
IV 1008 1846 – 295 B31. The correct order of electron gain enthalpy with
V 2372 5251 + 48 negative sign of F, Cl, Br and I, having atomic number
lis
The most reactive metal and the least reactive non- 9, 17, 35 and 53 respectively, is
metal of these are respectively (a) F > Cl > Br > I (b) Cl > F > Br > I
(a) I and V (b) V and II (c) Br > Cl > I > F (d) I > Br > Cl > F
ub
B23. The correct sequence which shows decreasing order of ionisation enthalpy is
the ionic radii of the elements is : (a) C (b) F
–
(a) Na+ > F > Mg2+ > O2– > Al3+ (c) Be (d) N
n
–
(b) O2– > F > Na+ > Mg2+ > Al3+ (e) Ne (Kerala P.E.T. 2012)
er
–
(c) Al3+ > Mg2+ > Na+ > F > O2– B33. The correct order of ionisation energy of C, N, O and F is
–
(d) Na+ > Mg2+ > Al3+ > O2– > F (A.I.E.E.E. 2010) (a) F < N < C < O (b) C < N < O < F
od
B24. The second ionization enthalpies of Li, Be, B and C (c) C < O < N < F (d) F < O < N < C
are in the order (Karnataka C.E.T. 2012)
(a) Li > C > B > Be (b) Li > B > C > Be B34. The correct order of electronegativities of N, O, F and
M
B19. (e) B20. (b) B21. (b) B22. (c) B23. (b) B24. (b) B25. (d) B26. (c) B27. (b)
B28. (c) B29. (a) B30. (d) B31. (b) B32. (e) B33. (c) B34. (c)
B35. The polarizing power of the following anions B43. The ionic radii (in Å) of N3–, O2– and F– are respectively
N3–, O2– and F–, follow the order (a) 1.71, 1.40 and 1.36 (b) 1.71, 1.36 and 1.40
(c) 1.36, 1.40 and 1.71 (d) 1.36, 1.71 and 1.40
.
(a) N3– > F– > O2– (b) O2– > N3– > F–
ed
(c) O2– > F– > N3– (d) N3– > O2– > F– (JEE Main 2015)
B44. An element X belongs to fourth period and fifteenth
(A.M.U. Engg. 2012)
group of the periodic table. Which of the following
B36. The increasing order of the ionic radii of the given
rv
statement is true?
isoelectronic species is (a) It has a completely filled s-orbital and a partially
(a) S2–, Cl–, Ca2+, K+ (b) Ca2+, K+, Cl–, S2– filled d-orbital.
se
(c) K+, S2–, Ca2+, Cl– (d) Cl–, Ca2+, K+, S2– (b) It has completely filled s-and p-orbitals and a
(A.I.E.E.E. 2012) partially filled d-orbital.
re
B37. Which one of the following sets of ions represents the (c) It has completely filled s-and p-orbitals and a half-
collection of isoelectronic species? filled d-orbitals.
(a) K+, Cl–, Mg2+, Sc3+ (d) It has a half-filled p-orbital and completely filled
s-and d-orbitals. (WB JEE 2016)
ts
(b) Na+, Ca2+, Sc3+, F–
B45. Which of the following atom has the highest first
(c) K+, Ca2+, Sc3+, Cl–
ionization enthalpy?
gh
(d) Na+, Mg2+, Al3+, Cl– (Karnataka C.E.T. 2013)
(a) Rb (b) Na
B38. Which of the following noble gases has the highest
(c) K (d) Sc (JEE Main 2016)
positive electron gain enthalpy value ?
ri
B46. Which of the following atom should have the highest
(a) Helium (b) Krypton
negative first electron gain enthalpy?
(c) Argon (d) Neon ll (a) F (b) O
(e) Xenon (Kerala P.E.T. 2013) (c) N (d) C (WB JEE 2017)
.A
B39. Which of the following represents the correct order of B47. The electronegativities of C, N, Si and P are in the
increasing first ionization enthalpy for Ca, Ba, S, Se order of
and Ar ?
(a) Si < P < C < N (b) Si < P < N < C
rs
(a) Ca < Ba < S < Se < Ar (c) P < Si < N < C (d) P < Si < C < N
(b) Ca < S < Ba < Se < Ar (Karnataka CET 2017)
he
(c) S < Se < Ca < Ba < Ar B48. The group having isoelectronic species is :
(d) Ba < Ca < Se < S < Ar (JEE Mains 2013) (a) O2–, F–, Na+, Mg2+ (b) O–, F–, Na, Mg+
B40. The first ionization potential of Na is 5.1 eV. The value (c) O2–, F–, Na, Mg2+ (d) O–, F–, Na+, Mg2+
lis
I.E.
577.5 1810 2750 11,580 14,820 radii in the following ?
er
The element is (a) Si4+ < P5+ < S6+ < Cl7+
(a) P (b) Mg
(b) P5+ < Si4+ < Cl7+ < S6+
(c) Si (d) Al
od
(Karnataka CET 2015) (c) Cl7+ < S6+ < P5+ < Si4+
B42. Amongst Be, B, Mg and Al, the second ionization (d) S6+ < P5+ < Cl7+ < Si4+ (J.K. C.E.T. 2018)
potential is maximum for B51. Which element has the highest first ionization
M
(c) He (d) H
(e) Li (Kerala P.E.T. 2018)
B35. (d) B36. (b) B37. (c) B38. (d) B39. (d) B40. (c) B41. (d) B42. (b) B43. (a) B44. (d)
B45. (d) B46. (a) B47. (a) B48. (a) B49. (b) B50. (c) B51. (c)
B52. Which statement(s) is (are) false for the periodic (a) unh (b) uun
classification of elements ? (c) une (d) uue (J.E.E. Main 2019)
(a) The properties of the elements are the periodic
.
B58. When the first electron gain enthalpy (ΔegH) of oxygen
function of their atomic numbers.
ed
is –141 kJ/mol, its second electron gain enthalpy is :
(b) Non-metallic elements are lesser in number than
the metallic elements. (a) almost the same as that of the first
(b) negative, but less negative than the first
rv
(c) The first ionization energies of the elements
along a period do not vary in a regular manner (c) a positive value
with increase in atomic number. (d) a more negative value than the first
se
(d) For transition elements, the d-electrons are filled
(J.E.E. Main 2019)
monotonically with increase in atomic number.
(e) Both (c) and (d). (Kerala PET 2018) B59. The size of the iso-electronic species Cl–, Ar and Ca2+
re
B53. The element with Z = 120 (not yet discovered) will be is affected by:
an/a (a) Azimuthal quantum number of valence shell
(a) Inner-transition metal (b) Electron-electron interaction in the outer
ts
(b) Transition metal orbitals
(c) Alkaline earth metal
(c) Principal quantum number of valence shell
(d) Alkali metal (JEE Main 2018)
gh
B54. The first ionization enthalpy of the following elements (d) Nuclear charge. (J.E.E. Main 2019)
are in the order : B60. In comparison to boron, beryllium has :
(a) C < N < Si < P (b) P < Si < N < C
ri
(a) lesser nuclear charge and greater first ionisation
(c) P < Si < C < N (d) Si < P < C < N enthalpy
(Karnataka CET 2019, Kerala PET 2019) ll (b) lesser nuclear charge and lesser first ionisation
B55. In general, the properties that decrease and increase enthalpy
.A
down a group in the periodic table, respectively, are : (c) greater nuclear charge and greater first
(a) electronegativity and electron gain enthalpy. ionisation enthalpy
(b) electronegativity and atomic radius.
(d) greater nuclear charge and lesser first ionisation
rs
(a) 16, 5 and 2 (b) 16, 6 and 3 B62. The isoelectronic set of ions is :
(c) 15, 5 and 3 (d) 15, 6 and 2 (a) N3–, Li+, Mg2+ and O2–
ub
B52. (d) B53. (c) B54. (d) B55. (b) B56. (c) B57. (d) B58. (c) B59. (d) B60. (a)
B61. (b) B62. (d)
od
C3. Which of the following order is not correct for first C6. Which of the following donot belong to d-block elements?
ionisation enthalpy ?
(a) [Xe] 5d16s2 (b) [Xe] 4f15d16s2
(a) C < N (b) O < S (c) N < O (d) Cl < F
.
1 1 2
(c) [Rn] 5f 6d 7s (d) [Ar] 3d104s1
ed
C4. Which of the following trend is not correct for size ?
(a) Na < K < Rb (b) I+ < I– < I C7. Which of the following property increases along a
2+ 3+
(c) Fe < Fe < Fe (d) F < O < N period (in general)
rv
(a) Atomic size
C5. In which of the following arrangements, the order is /
are not according to the property indicated : (b) Ionisation enthalpy
(c) Electron gain enthalpy (negative)
se
(a) C < N < O Ionization enthalpy
– + 2+ (d) Ionic size
(b) F < Na < Mg Increasing ionic size
C8. Which of the following are correct order of ionisation
(c) Br < F < Cl Increasing negative electron
re
enthalpy ?
gain enthalpy
(a) F < Cl < Br (b) C < O < N
(d) Na < K < Rb Increasing atomic radius (c) B < Al < Ga (d) O < S < Se
ts
gh
C3. (b, c) C4. (b, c) C5. (a,b) C6. (b, c) C7. (b, c) C8. (b, d)
Passage-II
In the modern periodic table, elements are arranged in
ri
order of increasing atomic numbers which is related to
the electronic configuration. Depending upon the type
ll
Passage - I of orbitals receiving the last electron, the elements in
.A
The properties of the elements such as atomic or ionic radii, the periodic table have been divided into four blocks,
ionisation ethalpy, electron gain enthalpy and viz, s, p, d and f. The modern periodic table consists of
electronegativity are directly or indirectly related to their 7 periods and 18 groups. Each period begins with the
electronic configuration and are called periodic properties. filling of a new energy shell. In accordance with the
rs
A part of the periodic table is given below : Aufbau principle, the seven periods (1 to 7) have 2, 8,
C N O F 8, 18, 18, 32 and 32 elements respectively. The seventh
he
Answer the following questions : bottom of the main body of the periodic table.
D1. Which of the following has highest ionisation enthalpy? Answer the following questions :
ub
(a) N (b) P (c) O (d) C D7. The last element of the p-block in 6th period is
D2. Bromine belongs to period represented by the outermost electronic
(a) third (b) fourth (c) fifth (d) second configuration.
P
D3. The highest negative electron gain enthalpy is of (a) 5f14 6d10 7s2 7p0 (b) 4f14 5d10 6s2 6p4
(a) F (b) N (c) S (d) Br 14 10
(c) 4f 5d 6s 6p 2 6
(d) 7s2 7p6
n
D6. Which of the following is not correct ? D9. The elements with atomic numbers 35, 53 and 85
(a) Electron gain enthalpy : Cl > F > Br are all
(b) Atomic size : N > O < F
(a) halogens (b) heavy metals
(c) Ionization enthalpy : N > O > F
M
(d) Ionic size : N3– > O2– > F– (c) light metals (d) noble gases
©
Passage - I D1. (a) D2. (b) D3. (a) D4. (d) D5. (c) D6. (b)
Passage-II D7. (c) D8. (b) D9. (a)
D10. The electronic configuration of the element which (c) 1s2 2s2 2p6 3s2 3p6 3d5 4s2 4p6
is just above the element with atomic number (d) 1s2 2s2 2p6 3s2 3p6 3d5 4s2
43 in the same group is
.
D11. Which of the elements whose atomic numbers
ed
(a) 1s2 2s2 2p6 3s2 3p6 3d7 4s2 are given below, cannot be accommodated in the
(b) 1s2 2s2 2p6 3s2 3p6 3d6 4s2 present set up of the long form of the periodic table?
rv
(a) 118 (b) 107 (c) 102 (d) 126
se
Passage-II D10. (d) D11. (d)
re
2. Match the property given in Column I with the property
Matrix Match Type Questions given in Column II
ts
Each question contains statements given in two columns, Column I Column II
which have to be matched. Statements in Column I are
labelled as A, B, C and D whereas statements in Column II (A) Ionisation enthalpy (p) decreases down the group
gh
are labelled as p, q, r and s. Match the entries of Column I (B) Negative electron (q) increases down the group
with appropriate entries of Column II. Each entry in Column gain enthalpy
(C) Metallic character (r) increases from left to right
I may have one or more than one correct option from Column
ri
in a period (in general)
II. The answers to these questions have to be appropriately (D) Atomic size (s) decreases from left to right
bubbled as illustrated in the following example. ll in a period (in general)
If the correct matches are A-q, A-r, B-p, B-s, C-r, C-s
.A
and D-q, then the correctly bubbled matrix will look like as 3. Electronic configuration of some elements is
given in Column I and their first ionization
shown: p q r S
enthalpies are given in Column II. Match the
A p q r S electronic configuration with first ionization
rs
enthalpy.
B p q r S
he
2 2 4
decreases (D) 1s 2s 2p (s) 2080
(C) O, O–, O2– (r) atomic size decreases
n
(1): (A) – (q) (B) – (p), (r), (s) (C) – (q) (D) – (q) (2) : (A) – (p), (r) (B) – (p), (r) (C) – (q), (s) (D) – (q), (s)
(3): (A) – (s) (B) – (p) (C) – (q) (D) – (r)
M
A6. (d) : The statements (a), (b) and (c) are correct but
A. Topicwise Multiple Choice Questions
statement (d) is incorrect because it belongs to
A3. (d) : 3(1s22s1), 12(1s2 2s2 2p6 3s2) have electron in 7th period (outermost shell).
outermost s– subshell. A7. (c) : Mt : [Rn] 5f 14 6d 7 7s 2 : d-block because the
outer most electron is in d-subshell.
A5. (c) : d-block elements have maximum tendency to form
complexes. A8. (a) : All p-block elements have 6 elements in a period.
A9. (c) : The elements having similar valence electronic B15. (a) : Electronic configuration of the element, Z = 114 is
configuation belong to same group of the periodic [Rn]86 5f14 6d10 7s2 7p2
table. Therefore, A and B having 3 electrons in the
.
It belongs to group 14 i.e. carbon family.
ed
valence shell belong to same group. B16. (d) : X having ground state configuration 1s22s22p3 is N
A13. (a) : P3–, S2– and Cl– have same number of electrons and its valency is –3.
(18) and size decreases as nuclear charge Formula: Mg+2X–3 or Mg3X2
increases.
rv
B18. (c) : The configuration corresponds to that of Cl, which
Cl– < S2– < P3– has the highest negative electron gain enthalpy.
A17. (c) : It corresponds to Cl and has largest negative
se
B19. (e) : Alkali metals have lowest value of ionisation
electron gain enthalpy. enthalpy in a period. On moving down the group
A28. (c) : Li (g) + I.E ⎯⎯ → Li+(g) + e–(g) from top to bottom, the ionisation enthalpy
re
decreases. Hence from the graph, M has least
7 × 10 –3
Moles of Li in 7 mg = = 1 × 10–3 mol ionisation enthalpy.
7 B20. (b) : Ionic radius increases from top to bottom and
Amount of energy needed to convert 1 × 10–3 mol of
ts
decreases on moving from left to right in a periodic
Li atoms
table.
520 × 103 J × 1 × 10–3 = 520 J
B22. (c) : The most reactive metal will be the one having
gh
A33. (d) : Amount of energy released for 1 × 106 atoms lowest ionisation enthalpy and the least reactive
= 4.9 × 10–13 J non-metal will be the one having lowest or positive
Amount of energy released for 6.022 × 1023 (1 mol) electron gain enthalpy.
ri
atoms B23. (b) : For isoelectronic ions, the ionic radius decreases
–13 with increase in nuclear charge.
4.9 × 10 ll
= × 6.022 × 1023 Correct order is :
1 × 106
.A
O2– (+8) > F– (+9) > Na+ (+11) > Mg2+ (+12) > Al3+ (+13)
= 29.5 × 104J = 295 kJ mol–1
B24. (b) : The configurations of atoms and ions after removing
A34. (c) : Chlorine has the highest negative electron gain one electron are :
enthalpy. It belongs to period 3 and group 17.
: 1s2 2s1 Li+ : 1s2
rs
Li
A35. (c) : The fourth ionisation enthalpy is abruptly Be 2
: 1s 2s 2
Be +
: 1s2 2s1
increased, which means that the electron is B 2 2
: 1s 2s 2p 1
B +
: 1s22s2
he
B5. (c) : 117 : [Rn] 5f146d107s27p5 : It will belong to halogen B26. (c) : Phosphorus belongs to group 15 and third period.
family. B35. (d) : Larger is the negative charge and size of anion,
higher is its polarising power.
ub
B10. (c) : For option (a), the order is H– > H > H+, for option B38. (d) : Among these neon has the highest positive electron
gain enthalpy.
(b) the order is O2– > F– > Na; for option (d), the order
B39. (d) : In general, ionization enthalpy decreases from top to
od
isoelectronic species.
Ba < Ca < Se < S < Ar
Ca2+ (Z = 20) < K+ (Z = 19) < Ar (Z = 18) B40. (c) : Electron gain enthalpy = – Ionisation potential
B14. (a, d) : The correct order of increasing –ve electron gain ∴ = – 5.1 eV
©
enthalpy value among halogens is B43. (a) : With increase in nuclear charge, size decreases in
I < Br < F < Cl option (a) is wrong. isoelectronic ions.
Nitrogen has the electronic configuration, 2s22p3 N3–(Z = 7) > O2–(Z = 8) < F–(Z = 9)
(i.e., half filled 2p configuration). Therefore, it has B44. (d) : The outer electronic configuration of the element ‘X’
higher first ionization enthalpy than oxygen. Option is : 3d10 4s2 4p3. It has half filled p-orbitals and
(d) is wrong. The correct order is : B < C < O < N. completely filled s-and d-orbitals.
B45. (d) : Sc has the highest ionization enthalpy. This is B57. (d) : 119: un (1) + un (1) + ennium = ununennium or
because, it is difficult to remove an electron from 4s uue
orbital of Sc as compared to 3s orbital of Na due to
B58. (c) : Second electron gain enthalpy is always positive
poor shielding of d-orbitals and high effective
for an element because energy is required to add
nuclear charge.
the second electron to negatively charged ion.
B46. (a) : F has the highest negative electron gain enthalpy.
.
B59. (d) : For isoelectronic ions, the size is affected by
ed
B48. (a) : Mg2+, Na+, O2– and F–, all have 10 electrons each and
are therefore, isoelectronic. nuclear charge.
B50. (c) : For isoelectronic ions, higher the positive charge, B60. (a) : In case of Be electron is to be removed from
rv
smaller will be the ionic radius. filled 2s subshell while in case of B, the electron
B51. (c) : Noble gases have high ionization potential. is to be removed from outer 2p subshell (2s2
B52. (d) : In general, electrons are added one at a time to the 2p1). Hence, Be has higher ionization enthalpy
se
(n–1)d orbitals in transition elements. However, than B though its nuclear charge is less.
because of extra stability of d5 and d10 configurations, B61. (b) : K has the electronic configuration : [Ar]4s1. After
re
two electrons appear to have entered the d-subshell
the removal of one electron, it acquires stable
due to shifting of one electron from ns to (n–1)d
noble gas configuration. Hence, second ionisation
orbital.
energy will be high and therefore, there is a large
B54. (d) : C N Si < C ; Si < P ; C < N ; P < N
ts
difference between first and second ionisation
2.5 3.0
Si P enthalpies.
gh
1.9 2.2
So, correct order is Si < P < C < N
B55. (b) : Electronegativity decreases as we go down the C1. (a,c) : (a) all have 10 electrons, (c) all have 18 electrons.
ri
group and atomic radius increases as we go down C2. (c,d) : 14 elements of 6th period are called lanthanoids,
the group. ll 4th period begins with potassium.
B56. (c) : Atomic no = 15 : 1s2 2s2 2p6 3s2 3p3 C3. (b,c) : O > S and N > O.
.A
Group no = 15, Valence electron = 2 + 3 = 5. C5. (a,b) : C < N > O, F– > Na+ > Mg2+
Valency = 8 – 5 = 3 C6. (b,c) : These belong to f-block
rs
he
lis
ub
P
n
er
od
M
©
.
ed
Time Allowed: 1½ Hr. Maximum Marks: 25
rv
Q. 1 and 2 are assertion reason questions.
In the following questions a statement of assertion followed by a statement of reason is given. Choose the
correct answer out of the following choices.
se
(a) Assertion and reason both are correct statements and reason is correct explanation for assertion.
(b) Assertion and reason both are correct statements but reason is not correct explanation for assertion.
(c) Assertion is correct statement but reason is wrong statement.
re
(d) Assertion is wrong statement but reason is correct statement.
1. Assertion: The electron gain enthalpy of F is less negative than that of Cl.
Reason: The atomic size of F is less than that of Cl. (1)
ts
2. Assertion: Nitrogen has higher ionization enthalpy than oxygen.
Reason: Oxygen has stable half filled configuration (1)
gh
3. Which of the following is correct set of ionisation enthalpy?
(a) Si < P < S (b) F < Cl > Br
ri
(c) B < C > N (d) N > O < F (1)
4. Which of the following electronic configuration has highest negative electron gain enthalpy?
ll
(a) 1s2 2s2 2p6 3s2 3p5 (b) 1s2 2s2 2p5
.A
(c) 1s2 2s2 2p6 (d) 1s2 2s2 2p6 3s2 (1)
5. The correct set of decreasing ionic size is
(a) Be2+ > Mg2+ > S2– > Cl– (b) S2– > Cl– > Na+ > Mg2+
rs
(c) Cl– > S2– > Mg2+ > Be2+ (d) S2– > Cl– > Mg2+ > Na+ (1)
he
of magnesium but its second ionization enthalpy is higher than that of magnesium? (2)
9. (a) Would you expect the second electron gain enthalpy of oxygen as positive, more negative or
ub
10. Among the elements of second and third period, predict the following :
n
(i) element (except noble gases) having largest ionization enthalpy and element having
lowest ionisation enthalpy.
er
(ii) element (except noble gases) having smallest atomic size and element having largest
atomic size.
od
A : 1s22s22p5 B : 1s22s22p4
C : 1s22s22p63s2 3p5 D : 1s22s22p63s1
E : 1s22s22p4
©
.
12. Explain the following :
ed
(a) Oxygen has lower ΔiH than N and F.
(b) Elements in the same group have similar physical and chemical properties.
rv
(c) Size of an anion is larger than the corresponding atom but the size of cation is smaller than
the corresponding atom. (3)
se
13. (a) What is the basic difference in approach between the Mendeleev’s periodic law and modern
periodic law ?
(b) Explain the variation in acidic and basic character of oxides of second row elements.
re
(c) Account for the variation in metallic or non-metallic character in third row elements. (5)
ts
gh
To check your performance, see HINTS and SOLUTIONS to some questions at the end of
Part I of the book.
ri
ll
.A
rs
he
lis
ub
P
n
er
od
M
©